FNP study1

Pataasin ang iyong marka sa homework at exams ngayon gamit ang Quizwiz!

The most frequently reported symptom of vulvar cancer is which one of the following? (check one) A. Longstanding pruritus B. Bleeding C. Pain D. Discharge E. Dysuria

A The most common symptom of vulvar cancer is longstanding pruritis. The other symptoms mentioned occur less frequently. Ref: Canavan TP, Cohen D: Vulvar cancer. Am Fam Physician 2002;66(7):1269-1274, 1276.

Contraindications to thrombolytic therapy in acute stroke include which one of the following? (check one) A. Age >80 B. Resolving transient ischemic attack C. Blood glucose >200 mg/dL D. Deficit present for >1 hour

B Thrombolysis is now an approved treatment for acute stroke. The critical time frame is 3 hours after the onset of the deficit. Beyond that time span, the use of thrombolytic agents is contraindicated. Advanced age per se is not a contraindication to thrombolytic therapy. Contraindications include blood glucose levels <50 mg/dL or >400 mg/dL, resolving transient ischemic attack, and hemorrhage visible on a CT scan. Ref: Rakel RE, Bope ET (eds): Conn's Current Therapy 2002. WB Saunders Co, 2002, pp 874-876.

A 2-year-old child stumbles, but his mother keeps him from falling by pulling up on his right hand. An hour later the child refuses to use his right arm and cries when his mother tries to move it. The most likely diagnosis is (check one) A. dislocation of the ulna B. dislocation of the olecranon epiphysis C. subluxation of the head of the radius D. subluxation of the head of the ulna E. anterior dislocation of the humeral head

C "Nursemaid's elbow" is one of the most common injuries in children under 5 years of age. It occurs when the child's hand is suddenly jerked up, forcing the elbow into extension and causing the radial head to slip out from the annular ligament. Ref: Kliegman RM, Stanton BF, Geme JW III, et al (eds): Nelson Textbook of Pediatrics, ed 19. Elsevier Saunders, 2011, p 2384.

Which one of the following should be avoided when treating pain in the elderly? (check one) A. Fentanyl (Sublimaze) B. Hydrocodone C. Meperidine (Demerol) D. Morphine E. Oxycodone (OxyContin)

C According to the Beers criteria, a list of drugs that should generally be avoided in older adults, meperidine should not be used in the elderly because its metabolite can accumulate and cause seizures. The other medications are not listed in the Beers criteria and are not contraindicated in the elderly. Ref: Pham CB, Dickman RL: Minimizing adverse drug events in older patients. Am Fam Physician 2007;76(12):1837-1844.

Which one of the following is consistent with spinal stenosis but not with a herniated vertebral disk? (check one) A. Numbness B. Muscle weakness C. Pain relieved by sitting D. Pain relieved by standing

C Causes of low back pain include vertebral disk herniation and spinal stenosis. Numbness and muscle weakness may be present in both. Pain from spinal stenosis is relieved by sitting and aggravated by standing, whereas the opposite is true for pain from a herniated disk.

Chronic excess thyroid hormone replacement over a number of years in postmenopausal women can lead to: (check one) A. Diffuse nontoxic goiter B. Osteoarthritis C. Osteoporosis D. Hyperparathyroidism

C Even mild chronic excess thyroid hormone replacement over many years can cause bone mineral resorption, increase serum calcium levels, and lead to osteoporosis. The elevated calcium decreases parathyroid hormone. Goiter is an indicator, not a cause, for hormone replacement. Osteoarthritis is not related to thyroid hormone replacement. Ref: Speroff L, Glass RH, Kase NG: Clinical Gynecologic Endocrinology and Infertility, ed 6. Lippincott Williams & Wilkins, 1999, p 817. 2) Cassel CK, Leipzig RM, Cohen HJ, et al (eds): Geriatric Medicine: An Evidence-Based Approach, ed 4. Springer, 2003, pp 621-650.

primigravida at 38 weeks gestation is concerned that her fetus is getting too large and wants to know what interventions could prevent complications from a large baby. On examination her uterine fundus measures 41 cm from the pubic symphysis. Ultrasonography is performed and an estimated fetal weight of 4000 g (8 lb 13 oz) is reported. Which one of the following management options is supported by the best evidence? (check one) A. Induction of labor B. Cesarean section C. Awaiting spontaneous labor D. Weekly ultrasonography to follow fetal growth

C This estimated fetal weight is at the 90th percentile for a term fetus. Unfortunately, the accuracy of fetal weight estimates declines as pregnancy proceeds, and the actual size may be as much as 15% different from the estimate. Delivery of a large infant results in shoulder dystocia more often than delivery of a smaller infant, but most large infants are delivered without complications. Intuitively, it would seem logical to induce labor when the fetus seems to be getting large, but this intervention has been studied in controlled trials and the only difference in outcome was an increase in the cesarean rate for women who underwent elective induction for this indication. Recently, there has been an increase in requests from patients to have an elective cesarean section near term to avoid the risks of labor, including pain, shoulder dystocia, and pelvic relaxation. The American Congress of Obstetricians and Gynecologists (ACOG) recommends consideration of cesarean delivery without a trial of labor if the estimated fetal weight is 4500 g in a mother with diabetes mellitus, or 5000 g in the absence of diabetes. Even at that size, there is not adequate data to show that cesarean section is preferable to a trial of labor. Frequent ultrasonography is often performed to reduce anxiety for both patient and physician, but the problem of accuracy of weight estimates remains an issue even with repeated scans at term. Ref: ACOG practice bulletin: Shoulder dystocia. American College of Obstetricians and Gynecologists, 2002, no 40. 2) Gherman RB, Chauhan S, Ouzounian JG, et al: Shoulder dystocia: The unpreventable obstetric emergency with empiric management guidelines. Am J Obstet Gynecol 2006;195(3):657-672. 3) Gabbe SG, Niebyl JR, Simpson JL (eds): Obstetrics: Normal and Problem Pregnancies, ed 5. Churchill Livingstone, 2007, pp 219-220, 851. 4) Mozurkewich E, Chilimigras J, Koepke E, et al: Indications for induction of labor: A best-evidence review. BJOG 2009;116(5):626-636.

Which one of the following fetal ultrasound measurements gives the most accurate estimate of gestational age in the first trimester (up to 14 weeks)? (check one) A. Femur length B. Biparietal diameter C. Abdominal circumference D. Crown-rump length E. Scapulo-sacral length

D Because the growth pattern of the fetus varies throughout pregnancy, the accuracy of measurements and their usefulness in determining gestational age and growth vary with each trimester. Crown-rump length is the distance from the top of the head to the bottom of the fetal spine. It is most accurate as a measure of gestational age at 7-14 weeks. After that, other measurements are more reliable. In the second trimester, biparietal diameter and femur length are used. During the third trimester, biparietal diameter, abdominal circumference, and femur length are best for estimating gestational age. Ref: Cunningham FG, Leveno KJ, Bloom SL, et al: Williams Obstetrics, ed 22. McGraw-Hill, 2005, p 392.

Which one of the following is most commonly implicated in interstitial nephritis? (check one) A. NSAIDs B. ACE inhibitors C. Diuretics D. Corticosteroids E. Antibiotics

E Antibiotics, especially penicillins, cephalosporins, and sulfonamides, are the most common drug-related cause of acute interstitial nephritis. Corticosteroids may be useful for treating this condition. The other drugs listed may cause renal injury, but not acute interstitial nephritis. Ref: Abdel-Kader K, Palevsky P: Acute kidney injury in the elderly. Clin Geriatr Med 2009;25(3):331-358.

Which one of the following drugs used to treat rheumatoid arthritis can delay the progression of the disease? (check one) A. Aspirin B. Ibuprofen C. Indomethacin (Indocin) D. Capsaicin (Zostrix) E. Hydroxychloroquine (Plaquenil)

E Hydroxychloroquine, originally developed as an antimalarial drug, is a well-known disease-modifying agent that can slow the progression of rheumatoid arthritis. Aspirin, indomethacin, and ibuprofen are anti-inflammatory agents. They relieve pain and improve mobility, but do not alter the progression of the disease. Capsaicin, a topical substance-P depleter, can relieve pain symptoms. Ref: Klippel JH, Stone JH, Crofford LJ, et al (eds): Primer on the Rheumatic Diseases, ed 13. Springer, 2008, pp 134-140.

A young woman in labor at term develops frank eclampsia. What is the best choice of anticonvulsant to treat her condition? (check one) A. Phenytoin (Dilantin) B. Diazepam (Valium) C. Topiramate (Topamax) D. Lamotrigine (Lamictal) E. Magnesium sulfate

E Intravenous magnesium sulfate reduces the risk of subsequent seizures in women with eclampsia compared with placebo, and with fewer adverse effects for the mother and baby compared with phenytoin or diazepam. The newer oral agents have no role in this emergency.

A 72-year-old white female is scheduled to undergo a total knee replacement for symptomatic osteoarthritis. She is otherwise healthy, with no history of vascular disease or deep vein thrombosis. She takes no routine medications. Which one of the following is most appropriate for prophylaxis against deep vein thrombosis? (check one) A. No prophylaxis if there are no surgical complications B. Aspirin, 325 mg daily C. Unfractionated heparin, 5000 U subcutaneously every 12 hours D. Thigh-high compression stockings E. Enoxaparin (Lovenox), 30 mg subcutaneously every 12 hours

E Prophylaxis is indicated with total knee or hip replacements. The two regimens recommended are low-molecular-weight heparin and adjusted-dose warfarin. These may be augmented by intermittent pneumatic compression. Ref: The use of oral anticoagulants (warfarin) in older people: American Geriatrics Society guideline. J Am Geriatr Soc 2002;50(8):1439-1445.

Which one of the following is associated with a history of sexual abuse in females? (check one) A. Lifelong functional gastrointestinal disorders B. Lifelong headache disorders C. Obesity D. Recurrent syncope

A A comprehensive, systematic literature review found an association of sexual abuse with a lifelong history of functional gastrointestinal disorders, irrespective of the age of the victim at the time of abuse. There was no statistically significant association with obesity, headache, or syncope.

Of the following, which one is the most effective treatment for bulimia nervosa? (check one) A. Fluoxetine (Prozac) B. Buspirone (BuSpar) C. Prochlorperazine (Compazine) D. Omeprazole (Prilosec) E. Metoclopramide (Reglan)

A number of placebo-controlled, double-blind trials have demonstrated the effectiveness of a variety of antidepressants in the treatment of bulimia nervosa. Fluoxetine has FDA approval for this indication. The other agents are not used for treating bulimia. Ref: Becker AE, Grinspoon SK, Klibanski A, et al: Eating disorders. N Engl J Med 1999;340(14):1092-1098. 2) Mehler PS: Clinical practice. Bulimia nervosa. N Engl J Med 2003;349(9):875-881.

A 74-year-old African-American female has moderately severe pain due to osteoarthritis. However, she is also on medication for a seizure disorder. When choosing medications to manage her chronic pain, which one of the following should be used with caution because of her history of seizures? (check one) A. Salsalate (Disalcid) B. Celecoxib (Celebrex) C. Hydrocodone (Lortab) D. Oxycodone (OxyContin) E. Tramadol (Ultram)

E via lowering of the seizure threshold

A 3-week-old male is brought to your office because of a sudden onset of bilious vomiting of several hours duration. He is irritable and refuses to breastfeed, but stools have been normal. He was delivered at term after a normal pregnancy, and has had no health problems to date. A physical examination shows a fussy child with a distended abdomen. Radiography of the abdomen shows a double bubble sign. Which one of the following is the most likely diagnosis? (check one) A. Infantile colic B. Necrotizing enterocolitis C. Hypertrophic pyloric stenosis D. Intussusception E. Midgut volvulus

E Volvulus may present in one of three ways: as a sudden onset of bilious vomiting and abdominal pain in a neonate; as a history of feeding problems with bilious vomiting that appears to be a bowel obstruction; or less commonly, as failure to thrive with severe feeding intolerance. The classic finding on abdominal plain films is the double bubble sign, which shows a paucity of gas (airless abdomen) with two air bubbles, one in the stomach and one in the duodenum. However, the plain film can be entirely normal. The upper gastrointestinal contrast study is considered the gold standard for diagnosing volvulus. Infantile colic usually begins during the second week of life and typically occurs in the evening. It is characterized by screaming episodes and a distended or tight abdomen. Its etiology has yet to be determined. There are no abnormalities on physical examination and ancillary studies, and symptoms usually resolve spontaneously around 12 weeks of age. Necrotizing enterocolitis is typically seen in the distressed neonate in the intensive-care nursery, but it may occasionally be seen in the healthy neonate within the first 2 weeks of life. The child will appear ill, with symptoms including irritability, poor feeding, a distended abdomen, and bloody stools. Abdominal plain films will show pneumatosis intestinalis, caused by gas in the intestinal wall, which is diagnostic of the condition. Hypertrophic pyloric stenosis is a narrowing of the pyloric canal caused by hypertrophy of the musculature. It usually presents during the third to fifth weeks of life. Projectile vomiting after feeding, weight loss, and dehydration are common. The vomitus is always nonbilious, because the obstruction is proximal to the duodenum. If a small olive-size mass cannot be felt in the right upper or middle quadrant, ultrasonography will confirm the diagnosis. Intussusception is seen most frequently between the ages of 3 months and 5 years, with 60% of cases occurring in the first year and a peak incidence at 6-11 months of age. The disorder occurs predominantly in males. The classic triad of intermittent colicky abdominal pain, vomiting, and bloody, mucous stools is encountered in only 20%-40% of cases. At least two of these findings will be present in approximately 60% of patients. The abdomen may be distended and tender, and there may be an elongated mass in the right upper or lower quadrants. Rectal examination may reveal either occult blood or frankly bloody, foul-smelling stool, classically described as currant jelly. An air enema using fluoroscopic guidance is useful for both diagnosis and treatment. Ref: McCollough M, Sharieff GQ: Abdominal pain in children. Pediatr Clin North Am 2006;53(1):107-137.

The only nonsexual behavior that is consistently and strongly correlated with cervical dysplasia and cervical cancer is: (check one) A. Alcohol consumption B. Caffeine consumption C. Cigarette smoking D. Cocaine use E. A high-fat diet

C Cigarette smoking is the only nonsexual behavior consistently and strongly correlated with cervical dysplasia and cancer, independently increasing the risk two- to fourfold. Ref: U.S. Preventive Services Task Force: Screening for cervical cancer: Recommendations and rationale. Am Fam Physician 2003;67(8):1759-1766.

You see a 90-year-old male with a 5-year history of progressive hearing loss. The most common type of hearing loss at this age affects: (check one) A. predominantly high frequencies B. predominantly mid frequencies C. predominantly low frequencies D. all frequencies roughly the same

A In the geriatric population, presbycusis is the most common cause of hearing loss. Patients typically have the most difficulty hearing higher-frequency sounds such as consonants. Lower-frequency sounds such as vowels are preserved.

Of the following, an 11-year-old who presents with knee pain is most likely to have: (check one) A. Gout B. Tibial apophysitis C. A popliteal cyst D. Inflammatory arthropathy E. Pes anserine bursitis

B tibial apophysitis AKA Osgood-Schlatter's: common cause of knee pain in patients whose bones are still growing; however, it is not really a disease.

A 60-year-old male indicates that he occasionally brings up what appears to be undigested food long after his meal. He also admits that he sometimes chokes on food, and that his wife says he has bad breath. The most likely diagnosis is: (check one) A. Achalasia B. Esophageal reflux C. Cancer of the esophagus D. Zenker's diverticulum E. Large cervical bone spur

D The combination of halitosis, late regurgitation of undigested food, and choking suggests Zenker's diverticulum. Patients may also have dysphagia and weight loss. The diagnosis is usually made with a barium swallow. The treatment is surgical. Ref: Woodson GE: Ear, Nose and Throat Disorders in Primary Care. WB Saunders Co, 2001, pp 222-224.

A 21-year-old white female presents to the emergency department with a history consistent with a lateral ankle sprain that occurred 2 hours ago while she was playing softball. She complains of pain over the distal anterior talofibular ligament, but is able to bear weight. There is mild swelling, mild black and blue discoloration, and moderate tenderness to palpation over the insertion of the anterior talofibular ligament, but the malleoli are nontender to palpation. Which one of the following statements is true regarding the management of this case? (check one) A. Anteroposterior, lateral, and 30 degrees internal oblique (mortise view) radiographs should be done to rule out fracture B. Stress radiographs will be needed to rule out a major partial or complete ligamentous tear C. The patient should use crutches and avoid weight bearing for 10-14 days D. Early range-of-motion exercises should be initiated to maintain flexibility E. For best results, functional rehabilitation should begin within the first 24 hours after injury

D This patient has an uncomplicated lateral ankle sprain and requires minimal intervention. The Ottawa ankle rules were developed to determine when radiographs are needed for ankle sprains. In summary, ankle radiographs should be done if the patient has pain at the medial or lateral malleolus and either bone tenderness at the back edge or tip of the lateral or medial malleolus, or an inability to bear weight immediately after the injury or in the emergency department, or both. If the patient complains of midfoot pain and/or bone tenderness at the base of the fifth metatarsal or navicular, or an inability to bear weight, radiographs should be ordered. Sprains can be differentiated from major partial or complete ligamentous tears by anteroposterior, lateral, and 30 degrees internal oblique (mortise view) radiographs. If the joint cleft between either malleolus and the talus is >4 mm, a major ligamentous tear is probable. Stress radiographs in forced inversion are sometimes helpful to demonstrate stability, but ankle instability can be present with a normal stress radiograph. Grade I and II ankle sprains are best treated with RICE (rest, ice, compression, elevation) and an air splint for ambulation. NSAIDs are used for control of pain and inflammation. Heat should not be applied. Early range-of-motion exercises should be initiated to maintain flexibility. Weight bearing is appropriate as tolerated and functional rehabilitation should be started when pain permits. Exercises on a balance board will help develop coordination. Ref: Rakel RE: Textbook of Family Practice, ed 6. WB Saunders Co, 2002, pp 922-924.

Which one of the following can contribute to serum calcium elevation? (check one) A. Furosemide (Lasix) B. Verapamil (Calan, Isoptin) C. Enalapril (Vasotec) D. Hydrochlorothiazide E. Allopurinol (Zyloprim)

D While thiazide diuretics do not cause hypercalcemia by themselves, they can exacerbate the hypercalcemia associated with primary hyperparathyroidism. Thiazides decrease the renal clearance of calcium by increasing distal tubular calcium reabsorption. Furosemide tends to lower serum calcium levels and is used in the treatment of hypercalcemia. None of the other medications would be expected to significantly affect the serum calcium level in this patient. Ref: Wilson JD, Foster DW, Kronenberg HM, et al: Williams Textbook of Endocrinology, ed 10. WB Saunders Co, 2003, p 1335.

The American Heart Association recommends a goal blood pressure of ≤130/80 mm Hg for patients with: (check one) A. Heart failure B. Pulmonary hypertension C. Atrial fibulation D. Angina pectoris E. Chronic kidney disease

E The American Heart Association recommends a goal blood pressure of 130/80 mm Hg or less for the treatment of hypertension in patients with diabetes mellitus, chronic kidney disease, or coronary artery disease. Ref: Pflieger M, Winslow BT, Mills K, Dauber IM: Medical management of stable coronary artery disease. Am Fam Physician 2011;83(7):819-826.

elines, which one of the following drugs should be added to his treatment regimen? (check one) A. Metronidazole (Flagyl) B. Amoxicillin/clavulanate (Augmentin) C. Ciprofloxacin (Cipro) D. Trimethoprim/sulfamethoxazole (Bactrim, Septra) E. Cefixime (Suprax)

A According to CDC guidelines, the initial workup for urethritis in men includes gonorrhea and Chlamydia testing of the penile discharge or urine, urinalysis with microscopy if no discharge is present, VDRL or RPR testing for syphilis, and HIV and hepatitis B testing. Empiric treatment for men with a purulent urethral discharge or a positive urine test (positive leukocyte esterase or ≥10 WBCs/hpf in the first-void urine sediment) includes azithromycin, 1 g orally as a single dose, OR doxycycline, 100 mg orally twice a day for 7 days, PLUS ceftriaxone, 125 mg intramuscularly, OR cefixime, 400 mg orally as a single dose. If the patient presents with the same complaint within 3 months, and does not have a new sexual partner, the tests obtained at his first visit should be repeated, and consideration should be given to obtaining cultures for Mycoplasma or Ureaplasma and Trichomonas from the urethra or urine. Treatment should include azithromycin, 500 mg orally once daily for 5 days, or doxycycline, 100 mg orally twice daily for 7 days, plus metronidazole, 2 g orally as a single dose.

Which one of the following is true regarding PPD testing for tuberculosis? (check one) A. Patients who have converted within the past year should be treated, regardless of age B. In patients who previously received a BCG vaccination, the threshold for a positive test is 25 mm of induration C. Patients who test positive only on the second step of a two-step PPD test, given 2 weeks after the first test, are at high risk for development of active disease D. PPD testing is contraindicated in patients who are HIV positive

A Because the risk of developing active disease is highest in patients within 2 years after conversion, recent converters should generally be treated regardless of age. BCG vaccination has a limited effect on PPD reactivity; tests should not be interpreted any differently in patients who have previously received BCG. The use of a two-step approach (i.e., retesting 1-4 weeks later in patients who initially test negative) is designed to decrease the false-negative rate of PPD testing. The significance of a positive result on either phase of the test is the same. Patients who are HIV positive are at higher risk for false-negative PPDs and active disease, but PPD testing is not contraindicated. Ref: Iseman M: A 52-year-old man with a positive PPD. JAMA 2001;286(16):2015-2022.

Treatment for Helicobacter pylori infection will reduce or improve which one of the following? (check one) A. The risk of peptic ulcer bleeding from chronic NSAID therapy B. The risk of developing gastric cancer in asymptomatic patients C. Symptoms of nonulcer dyspepsia D. Symptoms of gastroesophageal reflux disease

A Eradication of Helicobacter pylori significantly reduces the risk of ulcer recurrence and rebleeding in patients with duodenal ulcer, and reduces the risk of peptic ulcer development in patients on chronic NSAID therapy. Eradication has minimal or no effect on the symptoms of nonulcer dyspepsia and gastroesophageal reflux disease. Although H. pylori infection is associated with gastric cancer, no trials have shown that eradication of H. pylori purely to prevent gastric cancer is beneficial. Ref: Ables AZ, Simon I, Melton ER: Update on Helicobacter pylori treatment. Am Fam Physician 2007;75(3):351-358.

A 56-year-old female has been on combined continuous hormone therapy for 6 years. This is associated with a reduced risk for which one of the following? (check one) A. Bone fracture B. Myocardial infarction C. Stroke D. Breast cancer E. Venous thromboembolism

A Hormone replacement therapy that includes estrogen has been shown to decrease osteoporosis and bone fracture risk. The risk for colorectal cancer also is reduced after 5 years of estrogen use. The risk for myocardial infarction, stroke, breast cancer, and venous thromboembolism increases with long-term use. Ref: Farquhar CM, Marjoribanks J, Lethaby A, et al: Long-term hormone therapy for perimenopausal and postmenopausal women. Cochrane Database Syst Rev 2005;(3):CD004143. 2) Magee SR, Taylor JS: Cochrane for clinicians: Hormone therapy in postmenopausal and perimenopausal women. Am Fam Physician 2006;74(9):1501-1502.

Which one of the following most increases insulin sensitivity in an overweight patient with diabetes mellitus? (check one) A. Metformin (Glucophage) B. Acarbose (Precose) C. Glyburide (DiaBeta, Micronase) D. NPH insulin

A Metformin increases insulin sensitivity much more than sulfonylureas or insulin. This means lower insulin levels achieve the same level of glycemic control, and may be one reason that weight changes are less likely to be seen in diabetic patients on metformin. Acarbose is an α-glucosidase inhibitor that delays glucose absorption. Ref: Fauci AS, Braunwald E, Kasper DL, et al (eds): Harrisons Principles of Internal Medicine, ed 17. McGraw-Hill, 2008, pp 2299-2302.

Which one of the following side effects induced by traditional neuroleptic agents responds to treatment with beta-blockers? (check one) A. Akathisia B. Rigidity C. Dystonia D. Sialorrhea E. Stooped posture

A Rigidity, sialorrhea, and stooped posture are parkinsonian side effects of neuroleptic drugs. These are treated with anticholinergic drugs such as benztropine or amantadine. Dystonia, often manifested as an acute spasm of the muscles of the head and neck, also responds to anticholinergics. Akathisia (motor restlessness and an inability to sit still) can be treated with either anticholinergic drugs or beta-blockers. Ref: Sadock JB, Sadock VA (eds): Kaplan & Sadock's Synopsis of Psychiatry: Behavioral Sciences/Clinical Psychiatry, ed 9. Lippincott Williams & Wilkins, 2003, pp 1009, 1012-1015.

Which one of the following antidepressants is LEAST likely to cause sexual dysfunction? (check one) A. Bupropion (Wellbutrin) B. Sertraline (Zoloft) C. Fluoxetine (Prozac) D. Imipramine (Tofranil) E. Trazodone (Desyrel)

A Sexual dysfunction, including decreased libido, ejaculatory disturbance, and anorgasmia, is common with the SSRIs (e.g., sertraline and fluoxetine). Tricyclic antidepressants such as imipramine also cause sexual dysfunction. Trazodone can cause priapism. Only bupropion is relatively free of sexual side effects. Ref: Rakel RE, Bope ET (eds): Conn's Current Therapy 2005. Saunders, 2005, pp 1273-1279. 2) Ables AZ, Baughman OL III: Antidepressants: Update on new agents and indications. Am Fam Physician 2003;67(3):547-554.

A 5-year-old white male has an itchy lesion on his right foot. He often plays barefoot in a city park that is subject to frequent flooding. The lesion is located dorsally between the web of his right third and fourth toes, and extends toward the ankle. It measures approximately 3 cm in length, is erythematous, and has a serpiginous track. The remainder of his examination is within normal limits. Which one of the following is the most likely cause of these findings? (check one) A. Dog or cat hookworm (Ancylostoma species) B. Dog or other canid tapeworm (Echinococcus granulosus) C. Cat protozoa (Toxoplasma gondii) D. Dog or cat roundworm (Toxocara canis or T. mystax)

A This patient has cutaneous larva migrans, a common condition caused by dog and cat hookworms. Fecal matter deposited on soil or sand may contain hookworm eggs that hatch and release larvae, which are infective if they penetrate the skin. Walking barefoot on contaminated ground can lead to infection. Echinococcosis (hydatid disease) is caused by the cestodes (tapeworms) Echinococcus granulosus and Echinococcus multilocularis, found in dogs and other canids. It infects humans who ingest eggs that are shed in the animals feces and results in slow-growing cysts in the liver or lungs, and occasionally in the brain, bones, or heart. Toxoplasmosis is caused by the protozoa Toxoplasma gondii, found in cat feces. Humans can contract it from litter boxes or feces-contaminated soil, or by consuming infected undercooked meat. It can be asymptomatic, or it may cause cervical lymphadenopathy, a mononucleosis-like illness; it can also lead to a serious congenital infection if the mother is infected during pregnancy, especially during the first trimester. Toxocariasis due to Toxocara canis and Toxocara cati causes visceral or ocular larva migrans in children who ingest soil contaminated with animal feces that contains parasite eggs, often found in areas such as playgrounds and sandboxes. Ref: Rabinowitz PM, Gordon Z, Odofin L: Pet-related infections. Am Fam Physician 2007;76(9):1314-1322.

An 82-year-old white male suffers from chronic low back pain. He is on warfarin (Coumadin) for chronic atrial fibrillation, tamsulosin (Flomax) for benign prostatic hyperplasia, and famotidine (Pepcid) for gastroesophageal reflux disease. Which one of the following analgesic medications would have the least potential for adverse side effects? (check one) A. The lidocaine patch (Lidoderm) B. Hydrocodone/acetaminophen C. Nortriptyline (Pamelor) D. Duloxetine (Cymbalta) E. Celecoxib (Celebrex)

A Topical lidocaine produces very low serum levels of active drug, resulting in very few adverse effects (SOR C). Hydrocodone could produce any opiate-type effect. Nortriptyline and duloxetine could aggravate this patient's atrial arrhythmia and cause urinary retention. Celecoxib could aggravate his reflux problem. Ref: Malanga G, Paster Z: Update on managing chronic pain in the elderly. J Fam Pract 2007;56(12):S11-S16.

What is the most common cause of erythema multiforme, accounting for more than 50% of cases? (check one) A. Candida albicans B. Herpes simplex virus C. Mycoplasma pneumoniae D. Penicillin therapy E. Sulfonamide therapy

B Erythema multiforme usually occurs in adults 20-40 years of age, although it can occur in patients of all ages. Herpes simplex virus (HSV) is the most commonly identified cause of this hypersensitivity reaction, accounting for more than 50% of cases.

A 4-year-old white male is brought to your office in late August. His mother tells you that over the past few days he has developed a rash on his hands and sores in his mouth. On examination you note a vesicular exanthem on his hands, with lesions ranging from 3 to 6 mm in diameter. The oral lesions are shallow, whitish, 4- to 8-mm ulcerations distributed randomly over the hard palate, buccal mucosa, gingiva, tongue, lips, and pharynx. Except for a temperature of 37.4°C (99.3°F), the remainder of the examination is normal. The most likely diagnosis is (check one) A. herpangina B. hand, foot, and mouth disease C. aphthous stomatitis D. herpetic gingivostomatitis E. streptococcal pharyngitis

B Hand, foot, and mouth disease is a mild infection occurring in young children, and is caused by coxsackievirus A16, or occasionally by other strains of coxsackie- or enterovirus. In addition to the oral lesions, vesicular lesions may occur on the feet and nonvesicular lesions may occur on the buttocks. A low-grade fever may also develop. Herpangina is also caused by coxsackieviruses, but it is a more severe illness characterized by severe sore throat and vesiculo-ulcerative lesions limited to the tonsillar pillars, soft palate, and uvula, and occasionally the posterior oropharynx. Temperatures can range to as high as 41°C (106°F). The etiology of aphthous stomatitis is multifactorial, and it may be due to a number of conditions. Systemic signs, such as fever, are generally absent. Lesions are randomly distributed. Herpetic gingivostomatitis also causes randomly distributed oral ulcers, but it is a more severe illness, regularly accompanied by a higher fever, and is extremely painful. Streptococcal pharyngitis is rarely accompanied by ulceration except in agranulocytic patients. Ref: Kliegman RM, Behrman RE, Jenson HB, et al (eds): Nelson Textbook of Pediatrics, ed 18. Saunders, 2007, pp 1352, 1361-1362, 1752-1753, 2735.

The most appropriate initial treatment for scabies in an 8-year-old male is: (check one) A. 0.5% malathion lotion (Ovide) B. 5% permethrin cream (Elimite) C. 5% precipitated sulfur in petroleum D. trimethoprim/sulfamethoxazole (Bactrim, Septra) orally for 10 days

B In adults and children over 5 years of age, 5% permethrin cream is standard therapy for scabies. This agent is highly effective, minimally absorbed, and minimally toxic. Ref: Flinders DC, De Schweinitz P: Pediculosis and scabies. Am Fam Physician 2004;69(2):341-348.

A 54-year-old white female has been taking amoxicillin for 1 week for sinusitis. She has developed diarrhea and has had 6-8 stools per day for the past 2 days. Examination shows the patient to be well hydrated with normal vital signs and a normal physical examination. The stool is positive for occult blood, and a stool screen for Clostridium difficile toxin is positive. The most appropriate treatment at this time would be (check one) A. vancomycin (Vancocin) intravenously B. metronidazole (Flagyl) orally C. trimethoprim/sulfamethoxazole (Bactrim, Septra) orally D. ciprofloxacin (Cipro) orally

B Many antibiotics can induce pseudomembranous colitis. Although oral vancomycin was once the initial drug of choice for C. difficile, oral metronidazole is now the first-line agent because of cost considerations and because of concerns about the development of vancomycin-resistant organisms. If the patient has refractory symptoms despite treatment with oral metronidazole, then oral vancomycin would be appropriate. Vancomycin given orally is not absorbed, leading to high intraluminal levels of the drug. Ref: Goldman L, Ausiello D (eds): Cecil Medicine, ed 23. Saunders, 2008, p 2203.

A 55-year-old African-American male with osteoarthritis of the knees asks for advice on improving the function of his knees and controlling arthritis pain. Which one of the following would be appropriate advice? (check one) A. Topical capsaicin (Zostrix) applied twice daily will improve both pain and function B. Glucosamine will improve both pain and function C. A therapeutic exercise program will improve both pain and function D. An intra-articular corticosteroid injection will provide at least 6 months of pain relief E. NSAIDs will slow the progression of the disease

C A therapeutic exercise program will reduce both pain and disability in patients with osteoarthritis of the knee (SOR A). There is no evidence to support the use of capsaicin cream, but NSAIDs will reduce pain and there are proven therapies that will improve function of the patients knee. While intra-articular corticosteroids are effective in relieving pain in the short term (up to 4 weeks), there is no evidence for long-term efficacy. There is not good evidence to support the use of glucosamine for treating osteoarthritis of the knee. One systematic review found it no more effective than placebo. Ref: Scott D, Kowalcyzk A: Osteoarthritis of the knee. BMJ Clin Evid 2007;08:1121.

Which one of the following anticonvulsant medications is preferred for the treatment of mania or hypomania in patients with bipolar disorder? (check one) A. Phenytoin (Dilantin) B. Phenobarbital C. Valproic acid (Depakene) D. Gabapentin (Neurontin) E. Clonazepam (Klonopin)

C Anticonvulsant medications are used in the treatment of various psychiatric disorders. Valproic acid is FDA-approved for the treatment of manic episodes associated with bipolar disorder. It has been shown in controlled studies to be significantly more effective than placebo. The initial dosage is 750 mg daily given in divided doses, and most individuals require between 1000 and 2500 mg daily. Carbamazepine has also been used to treat mania and is an alternative for individuals who cannot tolerate lithium or valproic acid. Clonazepam is used in the treatment of panic attacks, and gabapentin is used to treat anxiety. Both phenytoin and gabapentin are also used to treat peripheral neuropathy. The primary use of phenobarbital is as an anticonvulsant. Ref: Valproate and other anticonvulsants for psychiatric disorders. Med Lett Drugs Ther 2000;42(1094):114-115. 2) Drug Facts and Comparisons, 2004 Edition. Facts and Comparisons, 2004, p 1205. 3) Kasper DL, Braunwald E, Fauci AS, et al (eds): Harrison's Principles of Internal Medicine, ed 16. McGraw-Hill, 2005, p 2557.

A 32-year-old white female at 16 weeks' gestation presents to your office with right lower quadrant pain. Which one of the following imaging studies would be most appropriate for initial evaluation of this patient? (check one) A. CT of the abdomen B. MRI of the abdomen C. Ultrasonography of the abdomen D. A small bowel series E. Intravenous pyelography

C CT has demonstrated superiority over transabdominal ultrasonography for identifying appendicitis, associated abscess, and alternative diagnoses. However, ultrasonography is indicated for the evaluation of women who are pregnant and women in whom there is a high degree of suspicion for gynecologic disease. Ref: Paulson EK, Kalady MF, Pappas TN: Suspected appendicitis. N Engl J Med 2003;348(3):236-241.

A 25-year-old male has a dental infection associated with facial swelling and lymphadenopathy. Which one of the following is the most appropriate antibiotic? (check one) A. Cephalexin (Keflex) B. Tetracycline C. Penicillin D. Ciprofloxacin (Cipro) E. Azithromycin (Zithromax)

C Dental infections complicated by the development of cellulitis should be treated with oral antibiotic therapy. The antibiotic of choice is penicillin. Clindamycin should be used if a patient is allergic to penicillin. Ref: Douglass AB, Douglass JM: Common dental emergencies. Am Fam Physician 2003;67(3):511-516.

A 56-year-old male with type 2 diabetes mellitus has normal cardiac and renal function but has failed to achieve adequate control of his diabetes with diet and multiple oral agents. His BMI is 30.1 kg/m2 and his hemoglobin A1c level is 9.1%. Which one of the following is most likely to be beneficial in combination with insulin and diet therapy in this patient? (check one) A. Acarbose (Precose) B. Glimepiride (Amaryl) C. Metformin (Glucophage) D. Pioglitazone (Actos) E. Repaglinide (Prandin)

C Metformin has been found to reduce cardiovascular risk in patients with type 2 diabetes mellitus. It also decreases the risk of weight gain, and unlike some oral agents it does not significantly increase the risk of hypoglycemia. It should be continued when insulin is initiated in patients with no renal impairment (SOR B). Ref: Unger J: Diagnosis and management of type 2 diabetes and prediabetes. Prim Care 2007;34(4):731-759. 2) Petznick A: Insulin management of type 2 diabetes mellitus. Am Fam Physician 2011;84(2):183-190.

Which one of the following is most typical of polymyalgia rheumatica? (check one) A. Headache and neck pain B. A normal erythrocyte sedimentation rate C. A dramatic response to corticosteroids D. A lack of systemic symptoms and signs

C Polymyalgia rheumatica is an inflammatory disorder that occurs in persons over the age of 50. White women of European ancestry are most commonly affected. The clinical hallmarks of polymyalgia rheumatica are pain and stiffness in the shoulder and pelvic girdle. One review found that 4%-13% of patients with clinical polymyalgia rheumatica have a normal erythrocyte sedimentation rate (ESR). As many as 5% of patients initially have a normal ESR that later rises. Polymyalgia rheumatica can have a variety of systemic symptoms. Fever is common, with temperatures as high as 39°C (102°F) along with night sweats. Additional symptoms include depression, fatigue, malaise, anorexia, and weight loss. Corticosteroids are the mainstay of therapy for polymyalgia rheumatica. Typically, a dramatic response is seen within 48-72 hours. Ref: Michet CJ, Matteson EL: Polymyalgia rheumatica. BMJ

The CAGE-AID questionnaire is a tool for screening for: (check one) A. depression B. bipolar illness C. substance abuse risk D. psychosis E. compatibility

C The CAGE-AID (CAGE Adapted to Include Drugs) questionnaire is a tool for assessing potential substance abuse risk. In one study it had a sensitivity of 70% and a specificity of 85% for drug abuse when two or more affirmative responses were defined as a positive result. It consists of the following four questions: Have you ever felt you ought to Cut down on your drinking or drug use? Have people Annoyed you by criticizing your drinking or drug use? Have you ever felt bad or Guilty about your drinking or drug use? Have you ever had a drink or used drugs first thing in the morning as an Eye opener to steady your nerves or to get rid of a hangover?

Which one of the following is most consistent with obsessive-compulsive disorder in adults? (check one) A. Impulses related to excessive worry about real-life problems B. A belief by the patient that obsessions are not produced by his or her own mind, but are "inserted" thoughts C. Recognition by the patient that the obsessions or compulsions are excessive or unreasonable D. Compulsions that bring relief to the patient rather than causing distress E. Full remission with treatment

C The DSM-IV criteria for obsessive-compulsive disorder (OCD) indicate that the patient at some point recognizes that the obsessions or compulsions are excessive or unreasonable. The impulses of OCD are not related to excessive worry about one's problems, and the patient recognizes that they are the product of his or her own mind. In addition, the patient experiences marked distress because of the impulses. Full remission is rare, but treatment can provide significant relief. Ref: Fenske JN, Schwenk TL: Obsessive-compulsive disorder: Diagnosis and management. Am Fam Physician 2009;80(3):239-245.

A previously healthy 27-year-old female has had a progressive decline in social and occupational functioning over the past year, along with a withdrawal from activities. In addition, her family notes that over the past 4 months she has had paranoid delusions, exhibited disorganized speech, and heard voices. She has not had any major depressive or manic episodes. A physical examination reveals a disheveled female with a flat affect, poor eye contact, and loosely-associated speech. A toxicology screen and basic laboratory analysis are unremarkable. She is not on any medications. Which one of the following is the most likely diagnosis? (check one) A. Brief psychotic disorder B. Delirium C. Schizophrenia D. Mood disorder with psychotic features E. Delusional disorder

C The diagnosis of schizophrenia requires two or more of the following characteristic symptoms (each present for a significant portion of time during a 1-month period): delusions, hallucinations, disorganized speech, grossly disorganized or catatonic behavior, and negative symptoms (i.e., affective flattening, alogia, or avolition). In addition, one or more major areas of functioning, such as work, interpersonal relationships, or self-care, should be markedly below the level seen prior to the onset of symptoms. Schizoaffective and mood disorders, substance abuse, medical illness or medication-induced disorders, and pervasive developmental disorders should be ruled out. Brief psychotic disorder is characterized by the presence of delusions, hallucinations, disorganized speech, or grossly disorganized or catatonic behavior lasting at least 1 day but less than 1 month. Delirium may present with psychotic symptoms but is the direct physiologic consequence of a general medical condition and usually has a much shorter course. Mood disorders with psychotic features can be ruled out if no major depressive, manic, or mixed episodes have occurred concurrently with the active-phase symptoms, or if the duration of mood disturbance is brief compared to the overall duration of active and residual symptoms. Delusional disorder does not cause bizarre delusions and also lacks other characteristic symptoms of schizophrenia such as hallucinations, disorganized speech or behavior, or prominent negative symptoms. Ref: American Psychiatric Association: Diagnostic and Statistical Manual of Mental Disorders, ed 4. American Psychiatric Association, 1994, pp 274-286. 2) Schultz SH, North SW, Shields CG: Schizophrenia: A review. Am Fam Physician 2007;75(12):1821-1829.

Because of safety concerns, which one of the following asthma medications should be used only as additive therapy and not as monotherapy? (check one) A. Inhaled corticosteroids B. Leukotriene-receptor antagonists C. Short-acting β2-agonists D. Long-acting β2-agonists E. Mast cell stabilizers

D Because of the risk of asthma exacerbation or asthma-related death, the FDA has added a warning against the use of long-acting β2-agonists as monotherapy. Inhaled corticosteroids, leukotriene-receptor antagonists, short-acting β2-agonists, and mast-cell stabilizers are approved and accepted for both monotherapy and combination therapy in the management of asthma (SOR A). Ref: Elward KS, Pollart SM: Medical therapy for asthma: Updates from the NAEPP guidelines. Am Fam Physician 2010;82 (10):1242-1251.

A 63-year-old white male sees you for an initial visit and is accompanied by his daughter, who is a patient of yours and scheduled the visit. The father recently relocated to be near the daughter after his wife died. He has well-controlled type 2 diabetes mellitus, but is otherwise healthy. Referring to the copy of the medical records they brought with them, the daughter notes that her father has received influenza vaccine in 3 of the past 5 years, but she can find no documentation that he ever had "the pneumonia vaccine." She asks if he should receive it at this visit. You advise them that he should receive pneumococcal vaccine: (check one) A. annually, along with influenza vaccine B. now and a repeat dose every 5 years C. every 5 years starting at age 65 D. now and a repeat dose once at age 68 E. only once, at age 65

D Both the CDC and the American Academy of Family Physicians recommend that all adults over the age of 65 receive a single dose of pneumococcal polysaccharide vaccine. Immunization before the age of 65 is recommended for certain subgroups of adults, including institutionalized individuals over the age of 50; those with chronic cardiac or pulmonary disease, diabetes mellitus, anatomic asplenia, chronic liver disease, or kidney failure; and health-care workers. It is recommended that those receiving the vaccine before the age of 65 receive an additional dose at age 65 or 5 years after the first dose, whichever is later.

Current thinking regarding infantile colic is that the cause is (check one) A. malabsorption B. overfeeding C. excessive air swallowing D. unknown E. parental anxiety

D Colic is a frustrating condition for parents and doctors alike. The parents would like an explanation and relief, and physicians would like to offer these things. At this time, however, in spite of numerous studies and theories, the cause of colic remains unknown. Ref: Miller-Loncar C, Bigsby R, High P, et al: Infant colic and feeding difficulties. Arch Dis Child 2004;89(10):908-912.

Which one of the following is a significant side effect of varenicline (Chantix)? (check one) A. Facial hirsutism B. Paroxysmal dysrhythmias C. Pleurisy-like symptoms D. Suicidal ideation E. Myalgias

D Education, support, and medications are all valuable tools in assisting patients with a smoking habit. Varenicline, a clinically effective smoking-cessation product, has been associated with patient mood changes following the initiation of therapy, including suicidal thoughts and aggressive and erratic behavior. The other problems listed have not been associated with varenicline use. Ref: Public health advisory: Important information on Chantix. US Food and Drug Administration, 2008.

Which one of the following is recommended for routine prenatal care? (check one) A. Hepatitis C antibody testing B. Parvovirus antibody testing C. Cystic fibrosis carrier testing D. HIV screening E. Examination of a vaginal smear for clue cells

D HIV screening is recommended as part of routine prenatal care, even in low-risk pregnancies. Counseling about cystic fibrosis carrier testing is recommended, but not routine testing. Hepatitis C and parvovirus antibodies are not part of routine prenatal screening. Routine screening for bacterial vaginosis with a vaginal smear for clue cells is not recommended.

A previously healthy 3-year-old male is brought to your office with a 4-hour history of abdominal pain followed by vomiting. Just after arriving at your office he passes bloody stool. A physical examination reveals normal vital signs, and guarding and tenderness in the right lower quadrant. A rectal examination shows blood on the examining finger. Which one of the following is the most likely diagnosis? (check one) A. Appendicitis B. Viral gastroenteritis C. Midgut volvulus D. Meckels diverticulum E. Necrotizing enterocolitis

D Meckels diverticulum is the most common congenital abnormality of the small intestine. It is prone to bleeding because it may contain heterotopic gastric mucosa. Abdominal pain, distention, and vomiting may develop if obstruction has occurred, and the presentation may mimic appendicitis. Children with appendicitis have right lower quadrant pain, abdominal tenderness, guarding, and vomiting, but not rectal bleeding. With acute viral gastroenteritis, vomiting usually precedes diarrhea (usually without blood) by several hours, and abdominal pain is typically mild and nonfocal with no localized tenderness. The incidence of midgut volvulus peaks during the first month of life, but it can present anytime in childhood. Volvulus may present in one of three ways: as a sudden onset of bilious vomiting and abdominal pain in the neonate; as a history of feeding problems with bilious vomiting that now appears to be due to bowel obstruction; or, less commonly, as a failure to thrive with severe feeding intolerance. Necrotizing enterocolitis is typically seen in the neonatal intensive-care unit, occurring in premature infants in their first few weeks of life. The infants are ill, and signs and symptoms include lethargy, irritability, decreased oral intake, abdominal distention, and bloody stools. A plain abdominal film showing pneumatosis intestinalis, caused by gas in the intestinal wall, is diagnostic of this disease. Ref: McCollough M, Sharieff GQ: Abdominal pain in children. Pediatr Clin North Am 2006;53(1):107-137.

A 36-year-old male presents with pain over the lumbar paraspinal muscles. He says the pain began suddenly while he was shoveling snow. Which one of the following is true regarding this patients injury? (check one) A. Systemic corticosteroids speed recovery B. Exercises specific to low back injuries speed recovery C. Opioids have significant advantages for symptom relief when compared with NSAIDs or acetaminophen D. Continued activity rather than bed rest helps speed recovery E. Trigger-point injections are superior to placebo in relieving acute back pain

D Multiple studies have demonstrated that bed rest is detrimental to recovery from low back pain. Patients should be encouraged to remain as active as possible. Exercises designed specifically for the treatment of low back pain have not been shown to be helpful. Neither opioids nor trigger-point injections have shown superiority over placebo, NSAIDs, or acetaminophen in relieving acute back pain. There is no good evidence to suggest that systemic corticosteroids are effective for low back pain with or without sciatica. Ref: Kinkade S: Evaluation and treatment of acute low back pain. Am Fam Physician 2007;75(8):1181-1188.

A 50-year-old female presents with right eye pain. On examination, you find no redness, but when you test her extraocular muscles she reports marked pain with eye movement. This finding suggests that her eye pain is caused by: (check one) A. an intracranial process B. an ocular condition C. a retinal problem D. an orbital problem E. an optic nerve problem

D Pain with eye movement suggests an orbital condition. Orbital inflammation, infection, or tumor invasion can lead to such eye pain. Other findings suggestive of an orbital cause of eye pain include diplopia or proptosis. If an orbital lesion is suspected, imaging studies should be performed. Ref: Fiore DC, Pasternak AV, Radwan RM: Pain in the quiet (not red) eye. Am Fam Physician 2010;82(1):69-73.

The sensitivity of a test is defined as: (check one) A. the probability of disease before a test is performed B. the probability of disease after a test is performed C. the percentage of patients with a positive test result who are confirmed to have the disease D. the percentage of patients with the disease who have a positive test result E. the percentage of patients without the disease who have a negative test result

D Sensitivity is the percentage of patients with a disease who have a positive test result. Specificity is the percentage of patients without the disease who have a negative test result. Pretest probability is the probability of disease before a test is performed. Posttest probability is the probability of disease after a test is performed. Positive predictive value is the percentage of patients with a positive test result who are confirmed to have the disease. Ref: Jaeschke R, Guyatt GH, Sackett DL: Users' guides to the medical literature. III. How to use an article about a diagnostic test. B. What are the results and will they help me in caring for my patients? The Evidence-Based Medicine Working Group. JAMA 1994;271(9):703-707. 2) Shaughnessy AF: Evaluating and understanding articles about treatment. Am Fam Physician 2009;79(8):668-670.

The most appropriate advice for a 50-year-old female who has passed six calcium oxalate stones over the past 4 years is to: (check one) A. restrict her calcium intake B. restrict her intake of yellow vegetables C. increase her sodium intake D. increase her dietary protein intake E. take potassium citrate with meals

E Calcium oxalate stones are the most common of all renal calculi. A low-sodium, restricted-protein diet with increased fluid intake reduces stone formation. A low-calcium diet has been shown to be ineffective. Oxalate restriction also reduces stone formation. Oxalate-containing foods include spinach, chocolate, tea, and nuts, but not yellow vegetables. Potassium citrate should be taken at mealtime to increase urinary pH and urinary citrate (SOR B).

Which one of the following medications has the best evidence for preventing hip fracture? (check one) A. Ibandronate (Boniva) B. Raloxifene (Evista) C. Denosumab (Prolia) D. Etidronate (Didronel) E. Alendronate (Fosamax)

E Ibandronate, raloxifene, denosumab, and etidronate have been shown to reduce new vertebral fractures, but are not proven to prevent hip fracture. Only zoledronic acid, risedronate, and alendronate have been confirmed in sufficiently powered studies to prevent hip fracture, and these are the anti-osteoporosis drugs of choice.

Which one of the following is a risk factor for osteoarthritis of the hip? (check one) A. Low bone mass B. Young age C. Participation in swimming D. Hyperthyroidism E. Obesity

E Risk factors for osteoarthritis of the hip include obesity, high bone mass, old age, participation in weight-bearing sports, and hypothyroidism. Ref: Lane NE: Osteoarthritis of the hip. N Engl J Med 2007;357(14):1413-1421.

In a child, which one of the following is most likely to improve adherence to a chronic medication regimen? (check one) A. Adding a favorite flavor to bitter liquid medications B. More frequent dosing of daily medication C. Having only one person from the health-care team discuss the medication regimen with the patient and his parents D. Advising the parents to avoid giving rewards for following the regimen E. Putting the parents in complete control of the dosing schedule

A A number of useful strategies for promoting adherence to a chronic medical regimen can be employed in children. Adding flavors to unpleasant tasting medicines is helpful (SOR B). Chocolate flavoring is especially useful for masking the taste of bitter medications. Using medications that are given only once or twice a day is associated with compliance rates of greater than 70% (SOR B). Consistent advice given by multiple members of the health-care team reinforces the importance of following a medication regimen. Parental use of rewards for children who take their medicine properly helps improve adherence. Involving children in decisions concerning their care gives them a sense of control and improves adherence. Other strategies for improving adherence include patient handouts, keeping financial costs in mind when prescribing, advising patients to incorporate dosing into daily routines such as meals, keeping tally sheets, and using visual reminders such as notes on the refrigerator. Ref: Gardiner P, Dvorkin L: Promoting medication adherence in children. Am Fam Physician 2006;74(5):793-798, 800.

A 15-year-old white male is being evaluated after a fall down one flight of stairs. He was transported by the local rescue squad with his cervical spine immobilized. He walked briefly at the scene and did not lose consciousness. His only complaint is a mild, generalized headache. One episode of vomiting occurred shortly after the accident. No weakness or numbness has been noted. Vital signs, mental status, and neurologic findings are normal. Radiologic evaluation of the cervical spine is remarkable only for an air-fluid level in the sphenoid sinus. Which one of the following abnormalities is most likely to be associated with this radiologic finding? (check one) A. A basilar skull fracture B. An orbital floor fracture C. An epidural hematoma D. A zygomatic arch fracture E. A mandible fracture

A A post-traumatic air-fluid level in the sphenoid sinus is associated with basilar skull fractures. This finding is frequently noted on cervical spine films. Orbital floor fractures may be associated with double vision, fluid in the maxillary sinus, an air-fluid level in the maxillary sinus, and diplopia. Epidural hematomas are more frequently associated with skull fractures in the area of the meningeal artery. Zygomatic arch fractures are more visible on Towne's view. Characteristic swelling and lateral orbital bruising are typically present. Mandible fractures may be associated with dental misalignment or bleeding. Panoramic views are often diagnostic. Ref: Barkin RM, Rosen P (eds): Emergency Pediatrics: A Guide to Ambulatory Care, ed 6. Mosby, 2003, pp 428, 433.

Which one of the following is a major advantage of second-generation (atypical) antipsychotics compared with first-generation antipsychotics? (check one) A. Less tardive dyskinesia B. Less monitoring for major side effects C. The availability of depot (intramuscular) formulations D. Lower cost E. Simpler dosing schedules

A A recent expert consensus panel endorsed the use of second-generation antipsychotics rather than first-generation drugs. Tardive dyskinesia is much less common with the use of second-generation antipsychotics. Several of the second-generation drugs require monitoring for major side effects, however. For example, clozapine, shown by studies to be the most efficacious of the new class, causes granulocytopenia or agranulocytosis, requiring weekly and later biweekly monitoring of blood counts. Both classes have depot formulations for intramuscular administration every 2-4 weeks. Oral dosing of drugs from both classes varies from 1 to 3 times daily. First-generation antipsychotics cost less than second-generation drugs. Ref: Kane JM, Leucht S, Carpenter D, et al: Expert consensus guideline series. Optimizing pharmacologic treatment of psychotic disorders. Introduction: Methods, commentary, and summary. J Clin Psychiatry 2003;64 supp 12:5-19. 2) Choice of an antipsychotic. Med Lett Drugs Ther 2003;45(1172):102-104.

A 60-year-old male is recovering from a non-Q-wave myocardial infarction. He has a 40-pack-year smoking history, currently smokes a pack of cigarettes per day, and has a strong family history of coronary artery disease. Studies ordered by the cardiologist showed no indication for any coronary artery procedures. His BMI is 27.5 kg/m 2 and his blood pressure is 130/70 mm Hg. Laboratory tests reveal a fasting blood glucose level of 85 mg/dL, a total cholesterol level of 195 mg/dL, and an LDL-cholesterol level of 95 mg/dL. Which one of the following secondary prevention measures would be LEAST likely to improve this patient's cardiovascular outcome? (check one) A. A weight reduction diet B. A β-blocker C. A statin D. An antiplatelet agent E. Smoking cessation

A Although dietary management may be appropriate, a weight reduction diet is not likely to improve this patient's cardiovascular outcome. In fact, even if this person were obese, there is insufficient evidence that weight reduction would decrease his cardiovascular mortality (SOR C). There is good evidence that the other options, even β-blockers in a patient with normal blood pressure, are indicated. All of these measures have evidence to support their usefulness for secondary prevention of coronary artery disease (SOR A).

A 6-year-old female presents with a 24-hour history of dry cough, malaise, and a temperature of 39.8°C (103.6°F). She received an influenza vaccination shot 7 days ago. A rapid influenza test is positive for influenza A. Which one of the following would be the best treatment option for this patient? (check one) A. Oseltamivir (Tamiflu) B. Amantadine (Symmetrel) C. Rimantadine (Flumadine) D. Zanamivir (Relenza)

A Amantadine and rimantadine are not recommended for the treatment of influenza A because of the development of resistance to these drugs. Resistance is not a problem with neuramidase inhibitors such as oseltamivir in immunocompetent patients. Zanamivir is not recommended for treatment of children under the age of 7. Although this child has recently received influenza vaccine, this is not a contraindication to drug therapy. Ref: Antiviral drugs for influenza. Med Lett Drugs Ther 2007;49(1272):85-86.

A 20-year-old female long-distance runner presents with a 3-month history of amenorrhea. A pregnancy test is negative, and other blood work is normal. She has no other medical problems and takes no medications. With respect to her amenorrhea, you advise her (check one) A. to increase her caloric intake B. that this is a normal response to training C. to begin an estrogen-containing oral contraceptive D. to stop running

A Amenorrhea is an indicator of inadequate calorie intake, which may be related to either reduced food consumption or increased energy use. This is not a normal response to training, and may be the first indication of a potential developing problem. Young athletes may develop a combination of conditions, including eating disorders, amenorrhea, and osteoporosis (the female athlete triad). Amenorrhea usually responds to increased calorie intake or a decrease in exercise intensity. It is not necessary for patients such as this one to stop running entirely, however. Ref: Master-Hunter T, Heiman DL: Amenorrhea: Evaluation and treatment. Am Fam Physician 2006;73(8):1374-1382.

A 20-year-old single white female who is a patient of yours was raped in her apartment at 7:00 a.m. today. She is brought to your office at 9:00 a.m. for assessment and treatment. Despite having occasional intercourse with her boyfriend, she has never used any type of contraceptive. They last had intercourse approximately 1 week ago, and the boyfriend has been out of town on business since then. The patient has a history of irregular periods, and her last normal period was approximately 2 and a half weeks ago. You note live sperm on a wet mount. In addition to many other issues that must be addressed at this visit, the patient asks about emergency contraception. Which one of the following would be accurate advice to the patient regarding this topic? (check one) A. Emergency contraception does not interfere with an established, post-implantation pregnancy B. The estrogen/progestin combination regimen appears to be more effective than the levonorgestrel-only regimen C. To be most effective, each dose of the 2-dose regimen should be administered at least 72 hours apart D. Fetal malformations have been reported as a result of the unsuccessful use of the high-dose emergency contraceptive regimen

A An FDA Advisory Committee has recommended over-the-counter marketing of Plan B, an emergency contraceptive package that contains two 0.75-mg tablets of levonorgestrel to be taken 12 hours apart. Plan B is one of the two FDA-approved products for this indication. The Preven emergency contraceptive kit includes four tablets, each containing 0.25 mg of levonorgestrel and 50 Μg of ethinyl estradiol; these are taken two at a time 12 hours apart. In a randomized, controlled trial comparing the single versus combined estrogen/progestin, the single-drug regimen was shown to be more effective. Pregnancy occurred in 11 of 976 women (1.1%) given levonorgestrel alone, and in 31 of 979 (3.2%) given ethinyl estradiol plus levonorgestrel. The proportion of pregnancies prevented, compared to the expected number without treatment, was 85% with levonorgestrel and 57% with the combination. In both regimens, the interval between individual doses is 12 hours. In this case, emergency contraception may be appropriate in the face of a possible pregnancy from previous consensual intercourse. Emergency contraception has not been found to interfere with an established post-implantation pregnancy. Furthermore, no fetal malformations have been reported as a result of the unsuccessful use of high-dose oral contraceptives for emergency contraception. Ref: Emergency contraception OTC. Med Lett Drugs Ther 2004;46(1175):10-11.

At a routine visit, a 50-year-old white female with a 10-year history of type 2 diabetes mellitus has a blood pressure of 145/90 mm Hg and significant microalbuminuria. Which one of the following would be an absolute contraindication to use of an ACE inhibitor in this patient? (check one) A. A previous history of angioneurotic edema B. Renal insufficiency C. Asthma D. A history of recent myocardial infarction E. A cardiac ejection fraction <25%

A Angioneurotic edema can be life-threatening, and ACE inhibitors should not be given to patients with a history of this condition from any cause. Elevated creatinine levels are not an absolute contraindication to ACE inhibitor therapy. Myocardial infarction and a reduced cardiac ejection fraction are indications for ACE inhibitor therapy. ACE inhibitors do not affect asthma. Ref: Bicket DP: Using ACE inhibitors appropriately. Am Fam Physician 2002;66(3):461-468, 473.

While evaluating a stroke patient, you ask him to stick out his tongue. At first he is unable to do this, but a few moments later he performs this movement spontaneously. This defect is known as: (check one) A. apraxia B. agnosia C. expressive (Broca's) aphasia D. astereognosis

A Apraxia is a transmission disturbance on the output side, which interferes with skilled movements. Even though the patient understands the request, he is unable to perform the task when asked, but may then perform it after a time delay. Agnosia is the inability to recognize previously familiar sensory input, and is a modality-bound deficit. For example, it results in a loss of ability to recognize objects. Aphasia is a language disorder, and expressive aphasia is a loss of the ability to express language. The ability to recognize objects by palpation in one hand but not the other is called astereognosis.

A 56-year-old African-American male with longstanding hypertension and a 30-pack-year smoking history has a 2-day history of dyspnea on exertion. Physical examination is unremarkable except for rare crackles at the bases. Which one of the following serologic tests would be most helpful for detecting left ventricular dysfunction? (check one) A. Beta-natriuretic peptide (BNP) B. Troponin-T C. C-reactive protein (CRP) D. D dimer E. Cardiac interleukin-2

A Beta-natriuretic peptide (BNP) is a 32-amino acid polypeptide secreted from the cardiac ventricles in response to ventricular volume expansion and pressure overload. The major source of BNP is the cardiac ventricles, and because of its minimal presence in storage granules, its release is directly proportional to ventricular dysfunction. It is a simple and rapid test that reliably predicts the presence or absence of left ventricular dysfunction on an echocardiogram. Ref: Krishnaswamy P, Lubien E, Clopton P, et al: Utility of beta-natriuretic peptide levels in identifying patients with left ventricular systolic or diastolic function. Am J Med 2001;111(4):274-279.

A 25-year-old primigravida presents with sharp, stabbing, left-sided pelvic pain that started yesterday, 45 days after her last menstrual period. Her past history is not remarkable, and a physical examination is normal except for moderate tenderness in the left adnexa on pelvic examination. A urinalysis is normal, as is a CBC. Her beta-hCG level is 1500 mIU/mL. Assuming no adnexal mass is seen, which one of the following transvaginal pelvic ultrasonography findings would be consistent with the highest likelihood of an ectopic pregnancy? (check one) A. Empty uterus: empty endometrial cavity with or without a thickened endometrium B. Abnormal gestational sac: anechoic intrauterine fluid collection either >10 mm in mean sac diameter or with a grossly irregular border C. Nonspecific fluid: anechoic intrauterine fluid collection <10 mm in mean sac diameter without an echogenic border D. Echogenic material: echogenic material within the endometrial cavity without a defined sac, or multiple discrete anechoic collections of various sizes divided by echogenic septations

A At this time in the patient's pregnancy, a gestational sac should be visible on ultrasonography. An empty uterus presents the highest risk (14%) for ectopic pregnancy, while nonspecific fluid and echogenic material are associated with a 5% and 4% risk, respectively. An abnormal or normal sac is associated with no risk, with the rare exception of multiple pregnancies with one being heterotopic. Ref: Dart R: First-trimester pregnancies: A practical approach to abdominal pain and vaginal bleeding in early pregnancy. Emerg Med Pract 2003;5(11):1-20.

Which one of the following reduces the incidence of atopic dermatitis in children? (check one) A. Exclusive breastfeeding until the infant is 4 months of age B. Prenatal ingestion of probiotics by the mother C. Delayed introduction of solid food until after 6 months of age D. Application of emollients E. Early exposure to dust mite

A Atopic dermatitis is a pruritic, inflammatory skin disorder affecting nearly 1 in 5 children residing in developed countries. The vast majority of those eventually afflicted experience the onset of symptoms by the age of 5 years, and more than half will present before the age of 1 year. The etiology is not fully understood, but it seems clear that environmental, immune, genetic, metabolic, infectious, and neuroendocrine factors all play a role. Environmental factors that may be involved include harsh detergents, abrasive clothing, Staphylococcus aureus skin infection, food allergens (cow's milk, eggs, peanuts, tree nuts, etc.), overheating, and psychological stress. Aeroallergens that are problematic for asthmatics, such as animal dander, dust mites, and pollen, have not been clearly linked to atopic dermatitis. Large, well-designed studies have found no evidence that delaying the introduction of solid foods until after 6 months of age reduces the likelihood of atopic dermatitis. Ingestion of probiotic agents during pregnancy has also not been shown to have any effect, and studies of probiotic use in breastfeeding mothers and their infants have yielded conflicting results. Exclusive breastfeeding for the first 4 months of life has been shown to reduce the cumulative incidence of atopic dermatitis in the first 2 years of life for infants at high risk of developing atopic disease; doing so beyond 4 months does not appear to provide additional benefit. Maternal dietary restriction during pregnancy and lactation has not been associated with significant benefit. Limited studies have demonstrated that emollients and moisturizers can reduce associated xerosis and are thought to be helpful treatments, but the data is not convincing.

A 45-year-old male sees you for a routine annual visit and is found to have atrial fibrillation, with a ventricular rate of 70-75 beats/min. He is otherwise healthy, and a laboratory workup and echocardiogram are normal. Which one of the following would be the most appropriate management? (check one) A. Aspirin, 325 mg daily B. Warfarin (Coumadin), with a target INR of 2.0-3.0 C. Clopidogrel (Plavix), 75 mg daily D. Amiodarone (Cordarone), 200 mg daily E. Observation only

A Atrial fibrillation is the most common arrhythmia, and its prevalence increases with age. The major risk with atrial fibrillation is stroke, and a patient's risk can be determined by the CHADS 2 score. CHADS stands for Congestive heart failure, Hypertension, Age >75, Diabetes mellitus, and previous Stroke or transient ischemic attack. Each of these is worth 1 point except for stroke, which is worth 2 points. A patient with 4 or more points is at high risk, and 2-3 points indicates moderate risk. Having ≤1 point indicates low risk, and this patient has 0 points. Low-risk patients should be treated with aspirin, 81-325 mg daily (SOR B). Moderate-or high-risk patients should be treated with warfarin. Amiodarone is used for rate control, and clopidogrel is used for vascular events not related to atrial fibrillation. Ref: Gutierrez C, Blanchard DG: Atrial fibrillation: Diagnosis and treatment. Am Fam Physician 2011;83(1):61-68.

An 8-year-old male presents with cervical lymphadenitis. He has a kitten at home and you are concerned about cat-scratch disease. Which one of the following antibiotics is most appropriate for treatment of Bartonella henselae infection? (check one) A. Azithromycin (Zithromax) B. Ceftriaxone (Rocephin) C. Amoxicillin/clavulanate (Augmentin) D. Doxycycline E. Clindamycin (Cleocin)

A Azithromycin has been shown to reduce the duration of lymphadenopathy in cat-scratch disease (SOR B). Other antibiotics that have been used include rifampin, ciprofloxacin, trimethoprim/sulfamethoxazole, and gentamicin. Ceftriaxone, amoxicillin/clavulanate, doxycycline, and clindamycin are not effective in the treatment of Bartonella infection.

A 56-year-old African-American male with long-standing hypertension and a 30-pack-year smoking history has a 2-day history of dyspnea on exertion. A physical examination is unremarkable except for rare crackles at the bases of the lungs. Which one of the following serologic tests would be most helpful for detecting left ventricular dysfunction? (check one) A. B-type natriuretic peptide B. Troponin T C. C-reactive protein (CRP D. D-dime E. Cardiac interleukin-2

A B-type natriuretic peptide (BNP) is a 32-amino acid polypeptide secreted from the cardiac ventricles in response to ventricular volume expansion and pressure overload. The major source of BNP is the cardiac ventricles, and because of the minimal presence of BNP in storage granules, its release is directly proportional to ventricular dysfunction. A BNP test is simple and time efficient, and reliably predicts the presence or absence of left ventricular dysfunction on an echocardiogram. Ref: Krishnaswamy P, Lubien E, Clopton P, et al: Utility of B-natriuretic peptide levels in identifying patients with left ventricular systolic or diastolic function. Am J Med 2001;111(4):274-279. 2) Bonow RO, Mann DL, Zipes DP, Libby P (eds): Braunwald's Heart Disease: A Textbook of Cardiovascular Medicine, ed 9. Elsevier Saunders, 2011, pp 505-516.

Repaglinide and nateglinide are nonsulfonylureas that act on a portion of the sulfonylurea receptor to stimulate insulin secretion. Pioglitazone is a thiazolidinedione, which reduces insulin resistance. It is believed that the mechanism for this is activation of PPAR-Y, a receptor that affects several insulin-responsive genes. Acarbose is a competitive inhibitor of α-glucosidases, enzymes that break down complex carbohydrates into monosaccharides. This delays the absorption of carbohydrates such as starch, sucrose, and maltose, but does not affect the absorption of glucose. Sitagliptin is a DPP-IV inhibitor, and this class of drugs inhibits the enzyme responsible for the breakdown of the incretins GLP-1 and GIP. Exenatide is an incretin mimetic that stimulates insulin secretion in a glucose-dependent fashion, slows gastric emptying, and may promote satiety.

A Behavioral interventions, especially those including telephone counseling and/or a dental examination, have been shown to be helpful for promoting smokeless tobacco cessation (SOR B). Studies examining mint snuff as a tobacco substitute, bupropion, and nicotine replacement in patch or gum form did not show any significant benefit.

A 30-year-old African-American female presents with a vaginal discharge. On examination the discharge is homogeneous with a pH of 5.5, a positive whiff test, and many clue cells. Which one of the following findings in this patient is most sensitive for the diagnosis of bacterial vaginosis? (check one) A. The pH of the discharge B. The presence of clue cells C. The character of the discharge D. The whiff test

A Patients must have 3 of 4 Amsel criteria to be diagnosed with bacterial vaginosis. These include a pH >4.5 (most sensitive), clue cells >20% (most specific), a homogeneous discharge, and a positive whiff test (amine odor with addition of KOH). Ref: Alfonsi GA, Shlay JC, Parker S, et al: Clinical inquiries. What is the best approach for managing recurrent bacterial vaginosis? J Fam Pract 2004;53(8):650-652.

A 48-year-old white female complains of anxiety and difficulty concentrating at home and at work. She reports that the symptoms have increased over the last 2 months because of her daughter's marital difficulties. She has had similar symptoms along with intermittent depression since she was a teenager. She admits to a loss of pleasure in work and recreational activities. Which one of the following is LEAST likely to help her coexistent depressive symptoms? (check one) A. Buspirone (BuSpar) B. Nortriptyline (Aventyl) C. Escitalopram (Lexapro) D. Venlafaxine (Effexor) E. Paroxetine (Paxil)

A Buspirone is indicated for the treatment of anxiety. Its advantages include the absence of addictive potential and sedation; like antidepressants, the onset of benefit is often delayed for several weeks. However, it is not an established antidepressant. Its use with other agents for depression has yielded questionable benefit, and used alone it is not an effective therapy for depression. The other agents listed have both antidepressant and antianxiety effects and were originally marketed for their antidepressant effect. Nortriptyline shares the risks of tricyclic agents, but historically it was one of the better tolerated tricyclics. The newer agents have serotonin-norepinephrine reuptake inhibition (SSRI activity). They have shown benefit in the treatment of anxiety as well as depression. Ref: Sadock BJ, Sadock VA: Kaplan & Sadock's Synopsis of Psychiatry, ed 9. Lippincott Williams & Wilkins, 2003, pp 1031-1033. 2) Fricchione G: Generalized anxiety disorder. N Engl J Med 2004;351(7):675-682.

A 70-year-old retired engineer who is an avid runner asks you about his slow, progressive decrease in exercise performance. He says he realizes he is getting older, but is in good health and is curious as to why this is happening. You tell him that there are multiple physiologic changes associated with aging that lower exercise performance, including a decrease in: (check one) A. cardiac output B. systolic blood pressure C. pulse pressure D. residual lung volume

A Cardiovascular changes associated with aging include decreased cardiac output, maximum heart rate, and stroke volume, as well as increased systolic and diastolic blood pressure. Respiratory changes include an increase in residual lung volume and a decrease in vital capacity. Other changes include decreases in nerve conduction, proprioception and balance, maximum O2 uptake, bone mass, muscle strength, and flexibility. Most of these changes, however, can be reduced in degree by a regular aerobic and resistance training program.

A 55-year-old white female presents with redness at the scar from a lumpectomy performed for stage I cancer of her right breast 4 months ago. The patient has completed radiation treatments to the breast. She is afebrile and there is no axillary adenopathy. There is no wound drainage, crepitance, or bullous lesions. Which one of the following organisms would be the most likely cause of cellulitis in this patient? (check one) A. Non-group A Streptococcus B. Pneumococcus pneumoniae C. Clostridium perfringens D. Escherichia coli E. Pasteurella multocida

A Cellulitis in patients after breast lumpectomy is thought to be related to lymphedema. Axillary dissection and radiation predispose to these infections. Non-group A hemolytic Streptococcus is the most common organism associated with this infection. The onset is often several weeks to several months after surgery. Pneumococcus is more frequently a cause of periorbital cellulitis. It is also seen in patients who have bacteremia with immunocompromised status. Immunocompromising conditions would include diabetes mellitus, alcoholism, lupus, nephritic syndrome, and some hematologic cancers. Clostridium and Escherichia coli are more frequently associated with crepitant cellulitis and tissue necrosis. Pasteurella multocida cellulitis is most frequently associated with animal bites, especially cat bites. Ref: Swartz MN: Cellulitis. N Engl J Med 2004;350(9):904-912.

Approximately 30%-50% of infants born to Chlamydia-positive mothers will have: (check one) A. Conjunctivitis B. Dermatitis C. Jaundice D. Pneumonia E. Urogenital infection

A Chlamydial genital infection is reported in 5%-30% of pregnant women, with vertical transmission to >50% of their infants at birth. An infant born to a mother with active chlamydial infection has a 50%-70% risk of acquiring infection at any anatomical site. Approximately 30%-50% of infants born to Chlamydia-positive mothers will develop conjunctivitis, and at least 50% of these children will also have nasopharyngeal infection. Infants born to women with chlamydial infection may also develop associated pneumonia, but this affects only 10%-20% of this population. Ref: Peipert JF: Genital chlamydial infections. N Engl J Med 2003;349(25):2424-2430. 2) Behrman RE, Kliegman RM, Jenson HB (eds): Nelson Textbook of Pediatrics, ed 17. Saunders, 2004, p 997.

A 40-year-old female with chronic plaque psoriasis requests topical treatment. Which one of the following topical therapies would be most effective and have the fewest adverse effects? (check one) A. High-potency corticosteroids B. Tazarotene (Tazorac) C. Coal tar polytherapy D. Anthralin

A Chronic plaque psoriasis is the most common type of psoriasis and is characterized by redness, thickness, and scaling. A variety of treatments were found to be more effective than placebo, but the best results were produced by topical vitamin D analogues and topical corticosteroids. Vitamin D and high-potency corticosteroids were equally effective when compared head to head, but the corticosteroids produced fewer local reactions (SOR A).

Which one of the following is the greatest risk factor for abdominal aortic aneurysm (AAA)? (check one) A. Cigarette smoking B. Diabetes mellitus C. Hypertension D. African-American race E. Female gender

A Cigarette smokers are five times more likely than nonsmokers to develop an abdominal aortic aneurysm (AAA). The risk is associated with the number of years the patient has smoked, and declines with cessation. Diabetes mellitus is protective, decreasing the risk of AAA by half. Women tend to develop AAA in their sixties, 10 years later than men. Whites are at greater risk than African-Americans. Hypertension is less of a risk factor than cigarette smoking (SOR A). Ref: Schermerhorn M: A 66-year-old man with an abdominal aortic aneurysm. JAMA 2009;302(18):2015-2022.

Cilostazol (Pletal) has been found to be a useful drug for the treatment of intermittent claudication. This drug is contraindicated in patients with: (check one) A. Congestive heart failure B. A past history of stroke C. Diabetes mellitus D. Third degree heart block E. Hyperlipidemia

A Cilostazol is a drug with phosphodiesterase inhibitor activity introduced for the symptomatic treatment of arterial occlusive disease and intermittent claudication. Cilostazol should be avoided in patients with congestive heart failure. There are no limitations on its use in patients with previous stroke or a history of diabetes. It has been found to have beneficial effects on HDL cholesterol levels and in the treatment of third degree heart block. Ref: Hiatt WMR: Drug therapy: Medical treatment of peripheral arterial disease and claudication. N Engl J Med 2001;344(2):1608-1621. 2) Chapman T, Goa KL: Cilostazol: A review of its use in intermittent claudication. Am J Cardiovasc Drugs 2003;3(2):117-138.

A 27-year-old white female sees you for the first time for a routine evaluation. A Papanicolaou test reveals atypical glandular cells of undetermined significance (AGUS). Of the following, which one is most commonly found in this situation? (check one) A. Cervical intraepithelial neoplasia B. Endometrial hyperplasia C. An endocervical polyp D. Endometrial cancer E. Ectopic decidua

A Clinical practice guidelines recommend that all patients with atypical glandular cells of undetermined significance (AGUS) be evaluated by colposcopy and endocervical curettage; endometrial sampling is recommended in women 35 years of age or older, and in those with AGUS favoring neoplasia or suggesting an endometrial source. Cervical intraepithelial neoplasia is the most common histologic diagnosis found in patients evaluated for AGUS. Ref: Valdini A, Vaccaro C, Pechinsky G, et al: Incidence and evaluation of an AGUS Papanicolaou smear in primary care. J Am Board Fam Pract 2001;14(3):172-177.

Which one of the following antipsychotic medications is most likely to cause agranulocytosis? (check one) A. Clozapine (Clozaril) B. Aripiprazole (Abilify) C. Risperidone (Risperdal) D. Olanzapine (Zyprexa)

A Clozapine was the first atypical antipsychotic drug, so designated because it has antipsychotic effects without the adverse effects on movement seen with first-generation agents, in addition to having enhanced therapeutic efficacy compared with first-generation drugs. Because of these advantages, it was introduced into clinical practice in the United States despite a serious known adverse effect: an increased incidence of agranulocytosis. Although only clozapine causes agranulocytosis in a substantial proportion of patients, many second-generation drugs produce clinically significant weight gain. Ref: Freedman R: Schizophrenia. N Engl J Med 2003;349(18):1738-1749.

A 17-year-old white female presents with new-onset left-sided lower abdominal pain. Color flow Doppler ultrasonography, in addition to pelvic ultrasonography, would be most useful for evaluating: (check one) A. Adnexal torsion B. Pelvic abscess C. Pelvic inflammatory disease D. Ruptured ovarian cyst

A Color Doppler flow studies are useful for evaluating blood flow to the ovary in possible cases of adnexal or ovarian torsion. Adnexal torsion is a surgical emergency. Pelvic ultrasonography, preferably with a vaginal probe, can be beneficial in the workup of ruptured ovarian cyst, pelvic abscess, and pelvic inflammatory disease without abscess. The Doppler flow study is not required with these condition. Ref: Hamilton GC, et al: Emergency Medicine: An Approach to Clinical Problem-Solving, ed 2. Saunders, 2003, pp 670-672.

You are helping a hospice program manage the symptoms of a 77-year-old male with end-stage colon cancer. He has required increasingly higher doses of his opioid medication to control symptoms of pain and dyspnea. In this situation, it should be kept in mind that which one of the following adverse effects of opioids does NOT diminish over time? (check one) A. Constipation B. Nausea C. Mental status changes D. Pruritus E. Sedation

A Constipation is one adverse effect of opioid treatment that does not diminish with time. Thus, this effect should be anticipated, and recommendations for prevention and treatment of constipation should be discussed when initiating opioids. Nausea and vomiting, mental status changes, sedation, and pruritus are also common with the initiation of opioid treatment, but these symptoms usually diminish with time, and can be managed expectantly.

A previously healthy 20-month-old female is brought to the urgent-care clinic during the evening with a barking cough. On examination her rectal temperature is 37.9°C (100.2°F), respiratory rate 18/min, heart rate 120 beats/min, and O2 saturation 94%. She has stridor, with mild substernal retractions only when her temperature was taken. Which one of the following would be most appropriate at this point? (check one) A. Dexamethasone, 0.6 mg/kg orally or intramuscularly as a single dose B. Guaifenesin/pseudoephedrine elixir orally until symptoms improve C. Azithromycin (Zithromax) orally for 5 days D. Observation in the clinic, and if there is improvement, a 5-day course of dexamethasone

A Croup is a syndrome most often caused by viruses, but can occasionally be of bacterial origin as in laryngotracheitis, laryngotracheobronchitis (LTB), laryngotracheobroncheopneumonia (LTBP), or laryngeal diphtheria. Mild croup is manifested by an occasional barking cough with no stridor at rest, and mild or absent intercostal retractions. Moderate croup presents with a more frequent barking cough, stridor with suprasternal and sternal retractions at rest, but no agitation. Severe croup includes more prominent inspiratory and expiratory stridor with agitation and distress. There is good evidence that corticosteroids produce significant improvement. The regimens studied most frequently have consisted of single-dose dexamethasone (0.6 mg/kg orally or intramuscularly), with some studies including up to four more doses over a 2-day period. Longer courses of corticosteroids have not proven to be more effective and may be harmful, leading to secondary infections. Racemic epinephrine by nebulization is indicated in severe croup. Antitussives and decongestants have not been studied and are not recommended. Antibiotics are indicated in LTB and LTBP, which can be diagnosed on the basis of crackles and wheezing on examination, or by an abnormal chest radiograph. Laryngotracheitis can sometimes be associated with a bacterial infection, but should be suspected only after a patient does not improve with corticosteroids and epinephrine. Ref: Cherry JD: Croup. N Engl J Med 2008;358(4):384-391.

A 2-week-old female is brought to the office for a well child visit. The physical examination is completely normal except for a clunking sensation and feeling of movement when adducting the hip and applying posterior pressure. Which one of the following would be the most appropriate next step? (check one) A. Referral for orthopedic consultation B. Reassurance that the problem resolves spontaneously in 90% of cases, and follow-up in 2 weeks C. Triple diapering and follow-up in 2 weeks D. A radiograph of the pelvis

A Developmental dysplasia of the hip encompasses both subluxation and dislocation of the newborn hip, as well as anatomic abnormalities. It is more common in firstborns, females, breech presentations, oligohydramnios, and patients with a family history of developmental dysplasia. Experts are divided with regard to whether hip subluxation can be merely observed during the newborn period, but if there is any question of a hip problem on examination by 2 weeks of age, the recommendation is to refer to a specialist for further testing and treatment. Studies show that these problems disappear by 1 week of age in 60% of cases, and by 2 months of age in 90% of cases. Triple diapering should not be used because it puts the hip joint in the wrong position and may aggravate the problem. Plain radiographs may be helpful after 4-6 months of age, but prior to that time the ossification centers are too immature to be seen. Because the condition can be difficult to diagnose, and can result in significant problems, the current recommendation is to treat all children with developmental dysplasia of the hip. Closed reduction and immobilization in a Pavlik harness, with ultrasonography of the hip to ensure proper positioning, is the treatment of choice until 6 months of age. The American Academy of Pediatrics recommends ultrasound screening at 6 weeks for breech girls, breech boys (optional), and girls with a positive family history of developmental dysplasia of the hip. Other countries have recommended universal screening, but a review of the literature has not shown that the benefits of early diagnosis through universal screening outweigh the risks and potential problems of overtreating. Ref: Storer SK, Skaggs DL: Developmental dysplasia of the hip. Am Fam Physician 2006;74(8):1310-1316. 2) Torchia MM, Phillips W: Developmental dysplasia of the hip. UpToDate, 2007.

A 55-year-old female who has hypertension, hyperlipidemia, and osteoarthritis of the knees develops acute gout and is found to have hyperuricemia. Discontinuation of which one of the following medications may improve her hyperuricemia? (check one) A. Hydrochlorothiazide B. Losartan (Cozaar) C. Metoprolol (Lopressor) D. Simvastatin (Zocor) E. Acetaminophen

A Diuretics such as hydrochlorothiazide are known to increase serum uric acid levels, but losartan has been shown to decrease uric acid. Metoprolol, simvastatin, and acetaminophen have no specific effect on serum uric acid levels.

A 64-year-old African-American male presents with persistent pleuritic pain. The patient does not feel well in general and has had a low-grade fever of around 100°F (38°C). His medications include simvastatin (Zocor), lisinopril (Prinivil, Zestril), low-dose aspirin, spironolactone (Aldactone), furosemide (Lasix), isosorbide mononitrate (Imdur), hydralazine, carvedilol (Coreg), and nitroglycerin as needed. A chest radiograph is normal and does not demonstrate a pneumothorax. Further evaluation rules out pulmonary embolus, pneumonia, and myocardial infarction. A diagnosis of pleurisy is made. Which one of the patient's medications could be related to this condition? (check one) A. Hydralazine B. Simvastatin C. Lisinopril D. Spironolactone E. Carvedilol

A Drug-induced pleuritis is one cause of pleurisy. Several drugs are associated with drug-induced pleural disease or drug-induced lupus pleuritis. Drugs that may cause lupus pleuritis include hydralazine, procainamide, and quinidine. Other drugs known to cause pleural disease include amiodarone, bleomycin, bromocriptine, cyclophosphamide, methotrexate, minoxidil, and mitomycin. Ref: Kass SM, Williams PM, Reamy BV: Pleurisy. Am Fam Physician 2007;75(9):1357-1364.

A 21-year-old married Hispanic female who is using no method of contraception presents to your office for evaluation of vaginal spotting 6 weeks after her last menstrual period. Her periods have previously been regular. She has had one previous episode of pelvic inflammatory disease. A home pregnancy test is positive. Which one of the following is true in this situation? (check one) A. Serum hCG levels should double every 2-3 days if the pregnancy is viable B. Painless bleeding excludes the diagnosis of ectopic pregnancy C. Laparoscopy should be performed to exclude ectopic pregnancy D. A serum progesterone level >25 ng/mL indicates that ectopic pregnancy is likely

A Early diagnosis of ectopic pregnancy requires a high index of suspicion. Risk factors include previous ectopic pregnancy, tubal sterilization, pelvic inflammatory disease, IUD use, and in utero exposure to diethylstilbestrol. The classic triad of missed menses, pain, and bleeding may not always be present. In early pregnancies of less than 5 weeks' gestation, serial hCG levels are helpful. Serum hCG levels double every 1.4-2 days. In a healthy pregnancy the level is expected to increase by at least 66% in 48 hours. Combining serial hCG levels with transvaginal ultrasonography is the best combination for evaluation of first-trimester problems. Serum hCG levels correlate well with sonographic landmarks. At 5 weeks' gestation in a normal pregnancy, serum hCG is >1000 mIU/mL and a gestational sac can be visualized in the uterus. Serum hCG is >2500 mIU/mL at 6 weeks and a yolk sac can be seen within the gestational sac. An hCG level of 5000 mIU/mL is compatible with visualization of a fetal pole. When the level is 17,000 mIU/mL, cardiac activity can be detected. Progesterone levels are also predictive of fetal outcome. A single level of 25 ng/mL or higher indicates a healthy pregnancy and excludes ectopic pregnancy with a sensitivity of 98%. If the level is <5 ng/mL, the pregnancy is nonviable. Assessment of fetal well-being is difficult if levels are in the intermediate range of 5-25 ng/mL. Ref: Gabbe SG, Niebyl JR, Simpson JL (eds): Obstetrics: Normal and Problem Pregnancies, ed 4. Churchill Livingstone, 2002, pp 743-747. 2) Choby BA: Pregnancy Care. AAFP Home Study Self-Assessment monograph series, 2003, no 292, pp 13-16.

A 76-year-old white male with a history of recurrent depression has recently become more depressed and developed psychotic features. His symptoms have not responded to antidepressants and antipsychotic agents, prescribed by his psychiatrist. The psychiatrist has recommended electroconvulsive therapy (ECT) for the patient. The patient's family visits you to ask for your opinion and recommendations regarding ECT in this individual. In your consultation with this family, which one of the following would be accurate advice regarding ECT? (check one) A. It is efficacious and safe B. There is evidence that it injures the brain C. It causes irreversible short-term memory loss D. There is evidence that it predisposes to the development of dementia E. It has a low response rate

A Electroconvulsive therapy (ECT) has a more than 60-year history of efficacy and safety for the treatment of severe depression. There has been no evidence of brain damage secondary to ECT. The most common side effect is reversible short-term memory loss. Dementia is not listed as a side effect. Response rates are generally in the 60%-90% range. Ref: Nierenberg AA, McColl RD: Management options for refractory depression. Am J Med 1996;101(suppl 6A):45S-52S. 2) Birrer RB, Vemuri SP: Depression in later life: A diagnostic and therapeutic challenge. Am Fam Physician 2004;69(10):2375-2382.

A 57-year-old male with severe renal disease presents with acute coronary syndrome. Which one of the following would most likely require a significant dosage adjustment from the standard protocol? (check one) A. Enoxaparin (Lovenox) B. Metoprolol (Lopressor, Toprol) C. Carvedilol (Coreg) D. Clopidogrel (Plavix) E. Tissue plasminogen activator (tPA)

A Enoxaparin is eliminated mostly by the kidneys. When it is used in patients with severe renal impairment the dosage must be significantly reduced. For some indications the dose normally given every 12 hours is given only every 24 hours. Although some β-blockers require a dosage adjustment, metoprolol and carvedilol are metabolized by the liver and do not require dosage adjustment in patients with renal failure. Clopidogrel is currently recommended at the standard dosage for patients with renal failure and acute coronary syndrome. Thrombolytics like tPA are given at the standard dosage in renal failure, although hemorrhagic complications are increased. Ref: O'Hanlon R, Reddan DN: Treatment of acute coronary syndromes in patients who have chronic kidney disease. Med Clin North Am 2005;89(3):563-585. 2) Field JM, Hazinski MF, Gilmore D (eds): Handbook of Emergency Cardiovascular Care for Healthcare Providers. American Heart Association, 2006, pp 45-63.

A 17-year-old white female has a history of anorexia nervosa, and weight loss has recently been a problem. The patient is an academically successful high-school student who lives with her parents and a younger sibling. Her BMI is 17.4 kg/m2 . Her serum electrolyte levels and an EKG are normal. Which one of the following interventions is most likely to be successful? (check one) A. Family-based treatment B. Adolescent-focused individual therapy C. Fluoxetine (Prozac) D. Phenelzine (Nardil) E. Desipramine (Norpramin)

A Family-based treatment for the adolescent with anorexia nervosa has been found to provide superior results when compared with individual adolescent-focused therapy (SOR B). Antidepressants have not been successful. They may be indicated for coexisting conditions, but this is more common with bulimia. Ref: Walsh BT, Kaplan AS, Attia E, et al: Fluoxetine after weight restoration in anorexia nervosa: A randomized controlled trial. JAMA 2006;295(22):2605-2612. 2) Goroll AH, Mulley AG (eds): Primary Care Medicine: Office Evaluation and Management of the Adult Patient, ed 6. Lippincott Williams & Wilkins, 2009, pp 1507-1513. 3) Rosen DS, American Academy of Pediatrics Committee on Adolescence: Identifying and treating eating disorders. Pediatrics 2010;126(6):1240-1253. 4) Lock J, Grange DL, Agras WS, et al: Randomized clinical trial comparing family-based treatment with adolescent-focused individual therapy for adolescents with anorexia nervosa. Arch Gen Psychiatry 2010;67(10):1025-1032.

The most common manifestation of uterine rupture during labor is: (check one) A. Fetal distress B. Sudden, tearing uterine pain C. Vaginal hemorrhage D. Cessation of uterine contractions E. Regression of the fetus

A Fetal distress with prolonged, variable, or late decelerations and bradycardia is the most common, and often only, sign of uterine rupture. The other signs listed are unreliable and often absent. Ref: Toppenberg KS, Block WA Jr: Uterine rupture: What family physicians need to know. Am Fam Physician 2002;66(5):823-828.

A previously healthy 82-year-old male is brought to your office by his daughter after a recent fall while getting up to go to the bathroom in the middle of the night. The patient denies any history of dizziness, chest pain, palpitations, or current injury. He has a history of bilateral dense cataracts. On examination, he is found to have an increased stance width and walks carefully and cautiously with his arms and legs abducted. A timed up-and-go test is performed, wherein the patient is asked to rise from a chair without using his arms, walk 3 meters, turn, return to his chair, and sit down. It takes the patient 25 seconds and he is noted to have an "en bloc" turn. Which one of the following is the most likely cause of this patient's gait and balance disorder? (check one) A. Visual impairment B. Cerebellar degeneration C. Frontal lobe degeneration D. Parkinson's disease E. Motor neuropathy

A Gait and balance disorders are one of the most common causes of falls in older adults. Correctly identifying gait and balance disorders helps guide management and may prevent consequences such as injury, disability, loss of independence, or decreased quality of life. The "Timed Up and Go" test is a reliable diagnostic tool for gait and balance disorders and is quick to administer. A time of <10 seconds is considered normal, a time of >14 seconds is associated with an increased risk of falls, and a time of >20 seconds usually suggests severe gait impairment. This patient has the cautious gait associated with visual impairment. It is characterized by abducted arms and legs; slow, careful, "walking on ice" movements; a wide-based stance; and "en bloc" turns. Patients with cerebellar degeneration have an ataxic gait that is wide-based and staggering. Frontal lobe degeneration is associated with gait apraxia that is described as "magnetic," with start and turn hesitation and freezing. Parkinson's disease patients have a typical gait that is short-stepped and shuffling, with hips, knees, and spine flexed, and may also exhibit festination and "en bloc" turns. Motor neuropathy causes a "steppage" gait resulting from foot drop with excessive flexion of the hips and knees when walking, short strides, a slapping quality, and frequent tripping. Ref: Salzman B: Gait and balance disorders in older adults. Am Fam Physician 2010;82(1):61-68.

Which one of the following has been shown to be effective for improving symptoms of varicose veins? (check one) A. Horse chestnut seed extract B. Vitamin B12 C. Ephedra D. Milk thistle E. St. John's wort

A Horse chestnut seed extract has been shown to have some effect when used orally for symptomatic treatment of chronic venous insufficiency, such as varicose veins. It may also be useful for relieving pain, tiredness, tension, and swelling in the legs. It contains a number of anti-inflammatory substances, including escin, which reduces edema and lowers fluid exudation by decreasing vascular permeability. Milk thistle may be effective for hepatic cirrhosis. Ephedra is considered unsafe, as it can cause severe life-threatening or disabling adverse effects in some people. St. John's wort may be effective for treating mild to moderate depression. Vitamin B12 is used to treat pernicious anemia. Ref: Jones RH, Carek PJ: Management of varicose veins. Am Fam Physician 2008;78(11):1289-1294. 2) Pittler MH, Ernst E: Horse chestnut seed extract for chronic venous insufficiency. Cochrane Database Syst Rev 2006;(1):CD003230.

A healthy 40-year-old female presents for her annual gynecologic examination. She tells you that she also needs a tuberculin screening test for her anticipated volunteer work at the local hospital. She has had no significant illness or exposures and has been your patient for nearly 20 years. You administer a PPD test which shows 10 mm of induration on the second day. The most appropriate next step in her evaluation would be: (check one) A. A repeat PPD in 2 weeks B. A chest radiograph (two views) C. Screening liver function tests D. Isoniazid (INH) for 9 months for treatment of her latent tuberculosis infection

A In 2000, the American Thoracic Society and the Centers for Disease Control and Prevention (CDC) advocated a shift in focus from screening the general population to testing only patients at increased risk for developing tuberculosis. In some persons PPD reactivity wanes with time but can be recalled by a second skin test administered 1 week or more after the first (i.e., two-step testing). For persons undergoing PPD skin testing, such as health-care workers, initial two-step testing may preclude misclassification of persons with boosted reactions as PPD converters. In those at low risk, such as this patient, a tuberculin skin test is now considered positive only if induration is at least 15 mm. Thus, this hospital volunteer would pose little risk to the hospital population since her 10-mm reaction falls within the guidelines of a negative test. She does not require diagnostic evaluation at this time, and isoniazid therapy is not indicated. Ref: Kasper DL, Braunwald E, Fauci AS, et al (eds): Harrison's Principles of Internal Medicine, ed 16. McGraw-Hill, 2005, pp 961-966.

Of the following, the INITIAL treatment of choice in the management of severe hypertension during pregnancy is: (check one) A. Labetalol (Trandate, Normodyne) intravenously B. Reserpine (Serpasil) intramuscularly C. Nifedipine (Procardia, Adalat) sublingually D. Enalapril (Vasotec) intravenously

A In pregnant women with severe hypertension, the primary objective of treatment is to prevent cerebral complications such as encephalopathy and hemorrhage. Intravenous hydralazine, intravenous labetalol, or oral nifedipine may be used. Sublingual nifedipine can cause severe hypotension, and reserpine is not indicated. Nitroprusside can be used for short intervals in patients with hypertensive encephalopathy, but fetal cyanide toxicity is a risk with infusions lasting more than 4 hours. ACE inhibitors are never indicated for hypertensive therapy during pregnancy. Ref: Cunningham FG, Gant NF, Leveno KJ, et al: Williams Obstetrics, ed 21. McGraw-Hill, 2001, pp 604. 2) Zamorski MA, Green LA: NHBPEP report on high blood pressure in pregnancy: A summary for family physicians. Am Fam Physician 2001;64(2):216, 263-270.

Which one of the following is consistent with terminology used in the 2001 Bethesda System for reporting cervical cytology? (check one) A. Atypical squamous cells—cannot exclude HSIL (ASC-H) B. Atypical squamous cells of unknown significance—favor neoplastic (ASCUS—favor neoplastic) C. Atypical squamous cells of unknown significance—favor reactive (ASCUS—favor reactive) D. Atypical glandular cells of unknown significance (AGUS)

A In the 2001 Bethesda System, atypical squamous cells of unknown significance (ASCUS) was replaced by atypical squamous cells (ASC). ASC is divided into atypical squamous cells-cannot exclude HSIL (ASC-H) and atypical squamous cells of unknown significance (ASC-US). ASCUS-favor reactive has been downgraded to negative in the 2001 system. Atypical glandular cells of unknown significance (AGUS) has been replaced by atypical glandular cells (AGC). Ref: Apgar B, Zoschnick L, Wright TC: The 2001 Bethesda System Terminology. Am Fam Physician 2003;68(8):1992-1998.

A 35-year-old African-American female with symptomatic uterine fibroids that are unresponsive to medical management prefers to avoid a hysterectomy. Which one of the following would be a reason for preferring myomectomy over fibroid embolization? (check one) A. A desire for future pregnancy B. Medical problems that increase general anesthesia risk C. Religious objections to blood transfusion D. The likelihood of a shorter hospital stay and recovery time E. The minimal risk of fibroid recurrence

A In the symptomatic patient with uterine fibroids unresponsive to medical therapy, myomectomy is recommended over fibroid embolization for patients who wish to become pregnant in the future. Uterine fibroid embolization requires a shorter hospitalization and less time off work. General anesthesia is not required, and a blood transfusion is unlikely to be needed. Uterine fibroids can recur or develop after either myomectomy or embolization. Ref: Schorge JO, Schaffer JI, Halvorsen LM, et al (eds): Williams Gynecology. McGraw-Hill, 2008, pp 205-207. 2) Bradley LD: Uterine fibroid embolization: A viable alternative to hysterectomy. Am J Obstet Gynecol 2009;201(2):127-135. 3) Goodwin SC, Spies JB: Uterine fibroid embolization. N Engl J Med 2009;361(7):690-697

A 22-year-old gravida 2 para 1 presents to your office with a 1-day history of vaginal bleeding and abdominal pain. Her last menstrual period was 10 weeks ago, and she had a positive home pregnancy test 6 weeks ago. She denies any passage of clots. On pelvic examination, you note blood in the vaginal vault. The internal cervical os is open. Which one of the following best describes the patient's current condition? (check one) A. Inevitable abortion B. Completed abortion C. Threatened abortion D. Incomplete abortion E. Missed abortion

A Inevitable abortion is defined by bleeding, an open os, and no passage of products of conception (POCs). Bleeding also occurs with completed abortion, but the os is closed and there is complete passage of POCs. Threatened abortion also is characterized by bleeding and a closed os, but there is no passage of POCs. With incomplete abortion there is bleeding and an open os, but POCs are visualized in the os or vaginal vault. There are no symptoms with missed abortion, but there is no embryo or fetus on ultrasonography. Ref: Cunningham FG, Leveno KJ, Bloom SL, et al: Williams Obstetrics, ed 22. McGraw-Hill, 2005, pp 232-241.

A pregnant patient is positive for hepatitis B surface antigen (HBsAg). Which one of the following would be most appropriate for her infant? (check one) A. Hepatitis B immune globulin (HBIG) and hepatitis B vaccine at birth B. Hepatitis B vaccine only, at birth C. HBIG only, at birth D. Testing for HBsAg before any immunization E. No immunization until 1 year of age

A Infants born to hepatitis B-positive mothers should receive both immune globulin and hepatitis B vaccine. They should receive the entire series of the vaccine, with testing for seroconversion only after completion of the vaccination series; the recommended age for testing is 9-12 months of age. Ref: Middleton DB, Zimmerman RK, Mitchell KB: Vaccine schedules and procedures, 2001. J Fam Pract 2001;50(10):S36-S47.

A 42-year-old male with well-controlled type 2 diabetes mellitus presents with a 24-hour history of influenza-like symptoms, including the sudden onset of headache, fever, myalgias, sore throat, and cough. It is December, and there have been a few documented cases of influenza recently in the community. The CDC recommends initiating treatment in this situation: (check one) A. on the basis of clinical symptoms alone B. only if rapid influenza testing is positive C. only if the diagnosis is confirmed by immunoassay testing D. only if the diagnosis is confirmed by reverse transcriptase polymerase chain reaction (PCR) assay

A Influenza is a highly contagious viral illness spread by airborne droplets. This patient's symptoms are highly suggestive of typical influenza: a sudden onset of malaise, myalgia, headache, fever, rhinitis, sore throat, and cough. While influenza is typically uncomplicated and self-limited, it can result in severe complications, including encephalitis, pneumonia, respiratory failure, and death. The effectiveness of treatment for influenza is dependent on how early in the course of the illness it is given. Because of the recent global H1N1 influenza outbreak that resulted in demand potentially outstripping the supply of antiviral medication, the Centers for Disease Control and Prevention has modified its recommendation as follows: Antiviral treatment is recommended as soon as possible for patients with confirmed or suspected influenza who have severe, complicated, or progressive illness or who require hospitalization. Antiviral treatment is recommended as soon as possible for outpatients with confirmed or suspected influenza who are at higher risk for influenza complications based on their age or underlying medical conditions. Clinical judgment should be an important component of outpatient treatment decisions. Antiviral treatment also may be considered on the basis of clinical judgment for any outpatient with confirmed or suspected influenza who does not have known risk factors for severe illness, if treatment can be initiated within 48 hours of illness onset. Many rapid influenza tests produce false-negative results, and more accurate assays can take more than 24 hours. Thus, treatment of patients with a clinical picture suggesting influenza is recommended, even if a rapid test is negative. Delaying treatment until further test results are available is not recommended. Ref: Fiore AE, Fry A, Shay D, et al; Centers for Disease Control and Prevention (CDC): Antiviral agents for the treatment and chemoprophylaxis of influenza—Recommendations of the Advisory Committee on Immunization Practices (ACIP). MMWR Recomm Rep 2011;60(1):1-24.

A 60-year-old male presents with profound weakness after 3 days of watery, frequent diarrhea. He has had no fever, bloody stool, or vomiting. His appetite has been poor. He has a history of hypertension treated with chlorthalidone, 25 mg daily, and potassium chloride, 20 mEq twice daily. Laboratory testing reveals a serum creatinine level of 2.0 mg/dL (N 0.6-1.5), a potassium level of 6.5 mmol/L (N 3.4-4.8), and a BUN of 50 mg/dL (N 8-25). Baseline values were normal. Which one of the following is most likely to lower the serum potassium within 1 hour? (check one) A. Regular insulin plus dextrose intravenously B. Calcium chloride, 10% solution intravenously C. Sodium polystyrene sulfonate (Kalexate) orally D. Sodium polystyrene sulfonate rectally

A Insulin and glucose intravenously will provide the fastest and most consistent early lowering of serum potassium (SOR C). Calcium is important for arrhythmia prevention, but does not lower the potassium level. Sodium polystyrene sulfonate given orally or rectally will only lower potassium in a delayed fashion.

A 3-year-old female is brought to your office for evaluation of mild intoeing. The child's patellae face forward, and her feet point slightly inward. Which one of the following would be most appropriate? (check one) A. Reassurance B. Foot stretching exercises C. Use of orthotics D. Use of night splints E. Surgery

A Intoeing, as described, is usually caused by internal tibial torsion. This problem is believed to be caused by sleeping in the prone position, and sitting on the feet. In 90% of cases, internal tibial torsion gradually resolves without intervention by the age of 8. Avoiding prone sleeping enhances resolution of the problem. Night splints, orthotics, and shoe wedges are ineffective. Surgery (osteotomy) has been associated with a high complication rate, and is therefore not recommended in mild cases before the age of 8. Ref: Sass P, Hassan G: Lower extremity abnormalities in children. Am Fam Physician 2003;68(3):461-468. 2) Behrman RE, Kliegman RM, Jenson HB (eds): Nelson Textbook of Pediatrics, ed 17. Saunders, 2004, pp 2252-2253.

A health-care worker repeatedly develops a rash on her hands after using latex gloves. The rash is papular and pruritic, with vesicles. Latex allergy is confirmed by skin patch testing. Which one of the following foods is most likely to provoke an allergic response in this patient? (check one) A. Avocados B. Walnuts C. Shellfish D. Strawberries E. Wheat

A Latex allergy management includes preventing exposure and treating reactions. Patients with latex allergy can reduce their risk of exposure by avoiding direct contact with common latex products. Additionally, they should be aware of foods with crossreactive proteins. Foods that have the highest association with latex allergy include avocados, bananas, chestnuts, and kiwi. Walnuts, shellfish, strawberries, and wheat have low or undetermined associations. Ref: Pollart SM, Warniment C, Mori T: Latex allergy. Am Fam Physician 2009;80(12):1413-1418.

Of the following antidepressants, which one is LEAST likely to cause drug interactions? (check one) A. Citalopram (Celexa) B. Fluoxetine (Prozac) C. Paroxetine (Paxil) D. Mirtazapine (Remeron)

A Like all drugs, SSRIs have significant side effects, including inhibition of the cytochrome P-450 system. However, citalopram is least likely to inhibit this system, making it a preferred SSRI for patients taking multiple medications for other illnesses. Ref: Sampson SM: Treating depression with selective serotonin reuptake inhibitors: A practical approach. Mayo Clin Proc 2001;76(7):739-744. 2) Ables AZ, Baughman OL III: Antidepressants: Update on new agents and indications. Am Fam Physician 2003;67(3):547-554.

A 49-year-old female who takes multiple medications has a chemistry profile as part of her routine monitoring. She is found to have an elevated calcium level. All other values on the profile are normal, and the patient is not currently symptomatic. Follow-up testing reveals a serum calcium level of 11.2 mg/dL (N 8.4-10.2) and an intact parathyroid hormone level of 80 pg/mL (N 10-65). Which one of the following should be discontinued for 3 months before repeat laboratory evaluation and treatment? (check one) A. Lithium B. Furosemide (Lasix) C. Raloxifene (Evista) D. Calcium carbonate E. Vitamin D

A Lithium therapy can elevate calcium levels by elevating parathyroid hormone secretion from the parathyroid gland. This duplicates the laboratory findings seen with mild primary hyperparathyroidism. If possible, lithium should be discontinued for 3 months before reevaluation (SOR C). This is most important for avoiding unnecessary parathyroid surgery. Vitamin D and calcium supplementation could contribute to hypercalcemia in rare instances, but they would not cause elevation of parathyroid hormone. Raloxifene has actually been shown to mildly reduce elevated calcium levels, and furosemide is used with saline infusions to lower significantly elevated calcium levels. Ref: Bilezikian JP, Silverberg SJ: Asymptomatic primary hyperparathyroidism. N Engl J Med 2004;350(17):1746-1751. 2) Javorsky BR, Dalkin AC: Hypercalcemia: A practical approach to a surprising condition. Emerg Med 2007;(Sept):13-20. 3) Kronenberg HM, Melmed S, Polonsky KS, et al (eds): Williams Textbook of Endocrinology, ed 11. Saunders Elsevier, 2008, pp 392-393.

A 76-year-old white male with heart failure is admitted to the hospital for the third time in a year. He responds to treatment with intravenous furosemide (Lasix), oxygen, and morphine. When he is discharged, his medications include carvedilol (Coreg), 25 mg twice daily; furosemide, 40 mg daily; and lisinopril (Prinivil, Zestril), 40 mg daily. He is also placed on a low-salt diet. Which one of the following is most likely to help prevent future admissions and decrease overall medical costs for this patient during the next year? (check one) A. Case management by a heart failure specialist nurse B. Nursing home admission C. Adding amiodarone (Cordarone) D. Increasing the dosage of lisinopril

A Many elderly patients with heart failure are hospitalized repeatedly at short intervals. As described in a 2012 Cochrane review of 25 randomized, controlled trials, there is now good evidence that case management interventions led by a heart failure specialist nurse reduce heart failure readmissions, all-cause readmissions, and all-cause mortality. Case management interventions include home care, telephone calls, patient education, self-management, and face-to-face visits. It is not possible to say which specific interventions were optimal, but telephone follow-up was a common component in most of these trials. Multidisciplinary interventions may also be effective. For patients treated with lisinopril, 40 mg/day is the maximum amount recommended for heart failure. While $-adrenergic blockers and aldosterone antagonists are used in the treatment of heart failure, prophylactic amiodarone would not be expected to help and may be harmful. Admission to a nursing home may reduce hospitalization, but it would also increase overall costs of care.

A 55-year-old white male sees you for a routine annual visit. His fasting blood glucose level is 187 mg/dL. Repeat testing 1 week later reveals a fasting glucose level of 155 mg/dL and an HbA1c of 9.4%. His BMI is 30 kg/m2. He does not seem to have any symptoms of diabetes mellitus. In addition to lifestyle changes, which one of the following would you prescribe initially? (check one) A. Metformin (Glucophage) B. Glyburide (DiaBeta, Micronase) C. Poiglitazone (Actos) D. Bedtime long-acting insulin (Lantus, Levamir) E. Bedtime long-acting insulin and rapid-acting insulin (NovoLog, Humalog) with each meal

A Metformin is widely accepted as the first-line drug for type 2 diabetes mellitus. It is relatively effective, safe, and inexpensive, and has been used widely for many years. Unlike other oral hypoglycemics and insulin, it does not cause weight gain. It should be started at the same time as lifestyle modifications, rather than waiting to see if a diet and exercise regimen alone will work. If metformin is not effective, a sulfonylurea, a thiazolidinedione, or insulin can be added, with the choice based on the severity of the hyperglycemia. Ref: Heine RJ, Diamant M, Mbanya JC, et al: Management of hyperglycemia in type 2 diabetes: The end of recurrent failure? BMJ 2006;333(7580):1200-1204.

A 33-year-old white female has a 12-year history of headache occurring 3-4 times per month, accompanied by nausea and vomiting. She takes over-the-counter analgesics, but relief is usually obtained only when she falls asleep. This is her first visit to you for this problem. You diagnose migraine without aura. Although the patient is willing to consider prescription drugs, she says that she would prefer "something that is natural and without side effects." Which one of the following would be the best recommendation? (check one) A. Biofeedback B. Ma huang C. Oxygen D. Epley canalith respositioning maneuver E. Phototherapy

A Of the listed options covering the realm of complementary and alternative medicine, only biofeedback has been shown to have a therapeutic effect on migraine. Specifically, the modality that seeks to control physiologic response to skin temperature and skin conductance appears to be the most successful. It is best performed in a medical office by caring, supportive staff members under physician supervision. Oxygen is used to treat cluster headaches. The Epley maneuver is used for managing benign positional vertigo, and phototherapy is useful in seasonal affective disorder. Ma huang, a Chinese herb, has ephedrine properties but is not useful in treating migraine headaches. Ref: Novey DW (ed): Clinician's Complete Reference to Complementary/Alternative Medicine. Mosby, 2000, pp 32-35. 2) Dambro MR (ed): Griffith's 5 Minute Clinical Consult. Lippincott Williams & Wilkins, 2002, pp 690-691. 3) Kasper DL, Braunwald E, Fauci AS, et al (eds): Harrison's Principles of Internal Medicine, ed 16. McGraw-Hill, 2005, pp 88-93.

A 55-year-old overweight male presents with a complaint of pain in the left big toe. He recently started jogging 2 miles a day to try to lose weight, but has not changed his diet and says he drinks 4 cans of beer every night. The pain has developed gradually over the last 2 weeks and is worse after running. An examination shows a normal foot with tenderness and swelling of the medial plantar aspect of the left first metatarsophalangeal joint. Passive dorsiflexion of the toe causes pain in that area. Plantar flexion produces no discomfort, and no numbness can be appreciated. Which one of the following is the most likely diagnosis? (check one) A. Sesamoid fracture B. Gout C. Morton's neuroma D. Cellulitis

A Pain involving the big toe is a common problem. The first metatarsophalangeal (MTP) joint has two sesamoid bones, and injuries to these bones account for 12% of big-toe injuries. Overuse, a sharp blow, and sudden dorsiflexion are the most common mechanisms of injury. Gout commonly involves the first MTP joint, but the onset is sudden, with warmth, redness, and swelling, and pain on movement of the joint is common. Morton's neuroma commonly occurs between the third and fourth toes, causes numbness involving the digital nerve in the area, and usually is caused by the nerve being pinched between metatarsal heads in the center of the foot. Cellulitis of the foot is common, and can result from inoculation through a subtle crack in the skin. However, there would be redness and swelling, and the process is usually more generalized. Sesamoiditis is often hard to differentiate from a true sesamoid fracture. Radiographs should be obtained, but at times they are nondiagnostic. Treatment, fortunately, is similar, unless the fracture is open or widely displaced. Limiting weight bearing and flexion to control discomfort is the first step. More complex treatments may be needed if the problem does not resolve in 4-6 weeks. Ref: Brukner P, Khan K: Clinical Sports Medicine, ed 3. McGraw Hill, 2006, pp 667-668, 861.

A 25-year-old male presents to your office with a 1-week history of neck pain with radiation to the left hand, along with intermittent numbness and tingling in the left arm. His history is negative for injury, fever, or lower extremity symptoms. Extension and rotation of the neck to the left while pressing down on the head (Spurling's maneuver) exacerbates the symptoms. His examination is otherwise normal. Cervical radiographs are negative. Which one of the following would be most appropriate at this point? (check one) A. NSAIDs for pain relief B. A trial of tricyclic antidepressants C. Cervical corticosteroid injection D. Cervical MRI E. Referral to a spine subspecialist

A Patients who present with acute cervical radiculopathy and normal radiographs can be treated conservatively. The vast majority of patients with cervical radiculopathy improve without surgery. Of the interventions listed, NSAIDs are the initial treatment of choice. Tricyclic antidepressants, as well as tramadol and venlafaxine, have been shown to help with chronic neuropathic pain. Cervical MRI is not indicated unless there are progressive neurologic defects or red flags such as fever or myelopathy. Likewise, referral to a subspecialist should be reserved for patients who have persistent pain after 6-8 weeks of conservative management and for those with signs of instability. Cervical corticosteroid injections have been found to be helpful in the management of cervical radiculopathy, but should not be administered before MRI is performed (SOR C).

Which one of the following is a risk factor for endometrial cancer? (check one) A. Polycystic ovary syndrome B. Multiparity C. Late menarche D. Use of an IUD E. Use of oral contraceptives

A Patients with persistent hyperestrogenic states are at heightened risk for the development of endometrial cancer. The chronic anovulation and consequent hyperstimulation of the endometrium seen with polycystic ovary syndrome predispose women to endometrial hyperplasia and carcinoma. Conversely, multiparity and late menarche are protective of the endometrium. Combination oral contraceptive use seems to decrease the risk for endometrial cancer. There is no evidence that IUD use leads to endometrial cancer, and it is thought that copper-containing IUDs may in fact provide some protection against endometrial cancer. Ref: Canavan TP, Doshi NR: Endometrial cancer. Am Fam Physician 1999;59(11):3069-3077. 2) Richardson MR: Current perspectives in polycystic ovary syndrome. Am Fam Physician 2003;68(4):697-704.

A 38-year-old day-care worker consults you for "a cold that won't go away." It began with a runny nose, malaise, and a slight temperature elevation up to 100°F (37.8°C). She notes that after 2 weeks she is now experiencing "coughing fits," which are sometimes so severe that she vomits. She has had no immunizations since her freshman year in college and does not smoke. On examination you note excessive lacrimation and conjunctival injection. Her lungs are clear. Which one of the following is the most likely diagnosis? (check one) A. Pertussis B. Rhinovirus infection C. Nonasthmatic eosinophilic bronchitis D. Cough-variant asthma E. Gastroesophageal reflux A 38-year-old day-care worker consults you for "a cold that won't go away." It began with a runny nose, malaise, and a slight temperature elevation up to 100°F (37.8°C). She notes that after 2 weeks she is now experiencing "coughing fits," which are sometimes so severe that she vomits. She has had no immunizations since her freshman year in college and does not smoke. On examination you note excessive lacrimation and conjunctival injection. Her lungs are clear. Which one of the following is the most likely diagnosis? (check one) A. Pertussis B. Rhinovirus infection C. Nonasthmatic eosinophilic bronchitis D. Cough-variant asthma E. Gastroesophageal reflux

A Pertussis, once a common disease in infants, declined to around 1000 cases in 1976 as a result of widespread vaccination. The incidence began to rise again in the 1980s, possibly because the immunity from vaccination rarely lasts more than 12 years. The disease is characterized by a prodromal phase that lasts 1-2 weeks and is indistinguishable from a viral upper respiratory infection. It progresses to a more severe cough after the second week. The cough is paroxysmal and may be severe enough to cause vomiting or fracture ribs. Patients are rarely febrile, but may have increased lacrimation and conjunctival injection. The incubation period is long compared to a viral infection, usually 7-10 days. Nonasthmatic eosinophilic bronchitis, cough-variant asthma, and gastroesophageal reflux disease cause a severe cough not associated with a catarrhal phase. A rhinovirus infection would probably be resolving within 2-3 weeks.

When prescribing an inhaled corticosteroid for control of asthma, the risk of oral candidiasis can be decreased by: (check one) A. using a valved holding chamber B. limiting use of the inhaled corticosteroid to once daily C. adding nasal fluticasone propionate (Flonase) D. adding montelukast (Singulair) E. adding salmeterol (Serevent)

A Pharyngeal and laryngeal side effects of inhaled corticosteroids include sore throat, coughing on inhalation of the medication, a weak or hoarse voice, and oral candidiasis. Rinsing the mouth after each administration of the medication and using a valved holding chamber when it is delivered with a metered-dose inhaler can minimize the risk of oral candidiasis. Ref: Fanta CH: Asthma. N Engl J Med 2009;360(10):1002-1014.

A 7-year-old male presents with a 3-day history of sore throat, hoarseness, fever to 100 degrees (38 degrees C), and cough. Examination reveals injection of his tonsils, no exudates, and no abnormal breath sounds. Which one of the following would be most appropriate? (check one) A. Recommend symptomatic treatment B. Perform a rapid antigen test for streptococcal pharyngitis C. Treat empirically for streptococcal pharyngitis D. Perform a throat culture for streptococcal pharyngitis E. Perform an office test for mononucleosis

A Pharyngitis is a common complaint, and usually has a viral cause. The key factors in diagnosing streptococcal pharyngitis are a fever over 100.4 degrees F, tonsillar exudates, anterior cervical lymphadenopathy, and absence of cough. Age plays a role also, with those <15 years of age more likely to have streptococcal infection, and those 10-25 years of age more likely to have mononucleosis. The scenario described is consistent with a viral infection, with no risk factors to make streptococcal infection likely; therefore, this patient should be offered symptomatic treatment for likely viral infection. Testing for other infections is not indicated unless the patient worsens or does not improve. Ref: Vincent M, Celestin N, Hussain A: Pharyngitis. Am Fam Physician 2004;69(6):1465-1470.

Which one of the following is true regarding the schedule for administering rotavirus vaccine? (check one) A. The first dose should be given prior to 12 weeks of age B. The vaccine should be given at 2 months, 6 months, and 12 months of age C. A catch-up vaccination can be administered up to 3 years of age D. The minimum time between doses is 2 weeks

A The rotavirus vaccine is an oral vaccine recommended for infants in a 3-dose schedule at ages 2, 4, and 6 months. The first dose should be given between 6 and 12 weeks of age, with additional doses given at 4 to 10-week intervals. The vaccine cannot be initiated after 12 weeks of age and should not be administered after 32 weeks of age. The first rotavirus vaccine was taken off the market in 1999 after an increased risk of intussusception was noted in infants when the first dose of the vaccine was given after 12 weeks of age. Because of this, the two vaccines licensed in 2006 carry the recommendation that the vaccine not be initiated in infants over 12 weeks of age. There is no data regarding safety and efficacy after this age. American Academy of Pediatrics Committee on Infectious Diseases: Recommended immunization schedules for children and adolescents-United States, 2007. Pediatrics 2007;119(1):207-208.

A 60-year-old male with a right-sided pleural effusion undergoes thoracentesis. Analysis of the pleural fluid reveals a protein level of 2.0 g/dL and an LDH level of 70 U/L. His serum protein level is 7.0 g/dL (N 6.0-8.3) and his serum LDH level is 200 U/L (N 100-105). Based on these findings, which one of the following is the most likely diagnosis? (check one) A. Heart failure B. Pulmonary embolism C. Tuberculous pleurisy D. Malignancy E. Bacterial pneumonia

A Pleural effusions may be exudates or transudates. The distinction is important for an accurate diagnosis and to help determine what further evaluations may be necessary. Lights criteria use ratios of fluid/serum values for protein and LDH. Pleural fluid/serum ratios greater than 0.6 for LDH and 0.5 for protein are indicative of exudates. In the scenario presented, both ratios are approximately 0.3; therefore, the fluid is a transudate. The list of causes for transudates is much shorter than for exudates. The vast majority of transudates are due to heart failure, with cirrhosis being the next most common cause. Once there is reasonable certainty that the fluid is a transudate, additional studies usually are not necessary. The other conditions listed result in exudative pleural effusions. Ref: Porcel JM, Light RT: Diagnostic approach to pleural effusion in adults. Am Fam Physician 2006;73(7):1211-1220.

You see a 17-year-old white female who has recently become sexually active. She requests oral contraceptives and you perform a brief evaluation, including blood pressure measurement. A pregnancy test is negative. She is resistant to further evaluation unless it is necessary. In addition to appropriate counseling, which one of the following should be done before prescribing oral contraceptives? (check one) A. No further evaluation at this visit unless indicated by history B. A pelvic examination and Papanicolaou test C. Screening for sexually transmitted diseases D. A breast examination

A Policy statements from major organizations based on reviews of relevant medical literature support the practice of prescribing initial hormonal contraception after performing only a careful review of the medical history plus measurement of blood pressure. Requiring that patients undergo pelvic and breast examinations leads many young women to avoid this most reliable method of contraception, resulting in a much higher rate of unwanted pregnancy. Follow-up blood pressure measurements are important. Often, younger women will be willing to undergo Papanicolaou (Pap) tests and STD screening later, and periodic follow-up must be scheduled. Sexually active adolescents should have annual screening for cervical cancer and sexually transmitted diseases, but these are not necessary before prescribing oral contraceptives. The longest period of time a prescription should be given without a Pap test is 1 year, but this restriction is under study. Obviously, any history indicative of high risk would modify this approach. Ref: Stewart FH, Harper CC, Ellertson CE, et al: Clinical breast and pelvic examination requirements for hormonal contraception: Current practice vs evidence. JAMA 2001;285(17):2232-2239. 2) Rimsza ME: Counseling the adolescent about contraception. Pediatr Rev 2003;24(5):162-170.

A 62-year-old white male complains of fatigue and proximal extremity discomfort without any localized joint pain. Which one of the following conditions is associated with a consistently normal creatine kinase enzyme level at all phases of disease? (check one) A. Polymyalgia rheumatica B. Polymyositis C. Dermatomyositis D. Drug-induced myopathy E. Hypothyroid endocrinopathy

A Polymyalgia rheumatica is a disease of the middle-aged and elderly. Discomfort is common in the neck, shoulders, and hip girdle areas. There is an absence of objective joint swelling, and findings tend to be symmetric. Characteristically, the erythrocyte sedimentation rate and C-reactive protein levels are significantly elevated; however, these tests are nonspecific. Occasionally there are mild elevations of liver enzymes, but muscle enzymes, including creatine kinase, are not elevated in this disorder. Elevation of muscle enzymes strongly suggests another diagnosis. Polymyositis and dermatomyositis are associated with variable levels of muscle enzyme elevations during the active phases of the disease. Drug-induced myopathies such as those seen with the cholesterol-lowering statin medications tend to produce some elevation of muscle enzymes during the course of the disorder. Hypothyroidism is associated with creatine kinase elevation. It should be strongly considered in the patient with unexplained, otherwise asymptomatic creatine kinase elevation found on a routine chemistry profile. Hyperthyroidism may cause muscle disease and loss of muscle, but it is not associated with creatine kinase elevation. Ref: Wallach J: Interpretation of Diagnostic Tests, ed 5. Little Brown & Co, 1992, pp 240, 245, 259. 2) Rendt K: Inflammatory myopathies: Narrowing the differential diagnosis. Cleve Clin J Med 2001;68(6):505-519. 3) Dale DC, Federman DD (eds): ACP Medicine, 2004-2005 ed. American College of Physicians, 2004, pp 1348-1391. 4) Dambro MR (ed): Griffith's 5-Minute Clinical Consult, 2005. Lippincott Williams & Wilkins, 2004, p 872. 5) Needham M, Mastaglia FL: Inclusion body myositis: Current pathogenetic concepts and diagnostic and therapeutic approaches. Lancet Neurol 2007;6(7):620-631.

A 67-year-old smoker with a history of pulmonary tuberculosis at 22 years of age presents with a 6-month history of increasing shortness of breath. On office spirometry his FVC is 60% of predicted, his FEV1 is 80% of predicted, and his FEV1/FVC ratio is 0.8. Which one of the following would be the most appropriate next step in his evaluation? (check one) A. Refer to a pulmonary laboratory for static lung volume measurement and diffusion studies B. Perform a bronchodilator challenge test C. Investigate for nonpulmonary causes for the patient's symptoms D. Order a PPD skin test

A Pulmonary function tests are usually classified as normal, compatible with a restrictive defect, or consistent with obstructive airway disease. In restrictive ventilatory processes, the FVC is decreased, the FEV1 is decreased or normal, and the absolute FEV1/FVC is >0.7. In obstructive airway problems, findings include a normal or decreased FVC, a decreased FEV1, and an absolute FEV1/FVC <0.7. When simple spirometry suggests a restrictive ventilatory problem, the patient should undergo full pulmonary function testing for static lung volume measurements and diffusing capacity of the lung for carbon monoxide. If spirometry suggests an obstructive problem, it should be repeated after administering an inhaled bronchodilator. Because this patient has a history of pulmonary tuberculosis, a TB skin test is inappropriate and would not provide any useful information. Ref: Barreiro TJ, Perillo I: An approach to interpreting spirometry. Am Fam Physician 2004;69(5):1107-1114.

You have just diagnosed post-traumatic stress disorder in a 32-year-old male. You immediately begin a program of patient education for him and his family, and connect them with a support group. Since his symptoms are quite severe you decide to begin pharmacotherapy before initiating trauma-focused psychotherapy. Based on available evidence, which one of the following medications is the best INITIAL treatment choice? (check one) A. Sertraline (Zoloft) B. Amitriptyline C. Phenelzine (Nardil) D. Alprazolam (Xanax) E. Haloperidol (Haldol)

A Selective serotonin reuptake inhibitors (SSRIs) such as sertraline have the broadest range of efficacy in treating posttraumatic stress disorder (PTSD) since they are able to reduce all three clusters of PTSD symptoms. Studies on the effectiveness of tricyclic antidepressants such as amitriptyline demonstrate modest lessening of the symptoms of reexperiencing, with minimal or no effect on avoidance or arousal symptoms. Patients treated with monoamine oxidase inhibitors such as phenelzine have shown moderate to good improvement in reexperiencing and avoidance symptoms, but little improvement in hyperarousal. Benzodiazepines such as alprazolam have been used to treat PTSD, but their efficacy against the major symptoms has not been proven in controlled studies.

Which one of the following is appropriate treatment for asymptomatic chlamydial infection during the second trimester of pregnancy? (check one) A. Azithromycin (Zithromax) B. Doxycycline C. Metronidazole (Flagyl) D. Levofloxacin (Levaquin)

A Several clinical trials suggest that 7-day regimens of erythromycin or amoxicillin, and single-dose regimens of azithromycin, are effective for treating chlamydial infections during pregnancy. Doxycycline and levofloxacin are contraindicated during pregnancy due to potential ill effects on the fetus, and metronidazole is not effective for the treatment of chlamydial infections. Ref: Peipert JF: Genital chlamydial infections. N Engl J Med 2003;349(25):2424-2430.

A 75-year-old Hispanic male presents with dyspnea on exertion which has worsened over the last several months. He denies chest pain and syncope, and was fairly active until the shortness of breath slowed him down recently. You hear a grade 3/6 systolic ejection murmur at the right upper sternal border which radiates into the neck. Echocardiography reveals aortic stenosis, with a mean transvalvular gradient of 55 mm Hg and a calculated valve area of 0.6 cm2. Left ventricular function is normal. Which one of the following is appropriate management for this patient? (check one) A. Aortic valve replacement B. Aortic balloon valvotomy C. Medical management with beta-blockers and nitrates D. Watchful waiting until the gradient is severe enough for treatment E. Deferring the decision pending results of an exercise stress test

A Since this patient's mean aortic-valve gradient exceeds 50 mm Hg and the aortic-valve area is not larger than 1 cm2, it is likely that his symptoms are due to aortic stenosis. As patients with symptomatic aortic stenosis have a dismal prognosis without treatment, prompt correction of his mechanical obstruction with aortic valve replacement is indicated. Medical management is not effective, and balloon valvotomy only temporarily relieves the symptoms and does not prolong survival. Patients who present with dyspnea have only a 50% chance of being alive in 2 years unless the valve is promptly replaced. Exercise testing is unwarranted and dangerous in patients with symptomatic aortic stenosis. Ref: Carabello BA: Aortic stenosis. N Engl J Med 2002;346(9):677-682.

A 28-year-old white female presents with painful genital ulcers. She has not had any previous episodes of similar outbreaks. She is single, but has had several heterosexual relationships. She has been with her current partner for 3 years. A culture confirms a herpes simplex virus (HSV) infection. Which one of the following is true regarding her situation? (check one) A. Suppressive therapy can reduce the risk of transmission to her partner B. In the genital area, HSV type 1 infection can be differentiated clinically from HSV type 2 infection C. This outbreak is conclusive evidence of infidelity in her partner D. An HSV vaccine is available for her partner to reduce his risk of infection

A Suppressive therapy with acyclovir, valacyclovir, or famciclovir reduces, but does not eliminate, the risk of transmission of HSV to sexual partners. HSV type 1 and HSV type 2 infections in the genital area are clinically identical. Psychological issues, including anger, guilt, low self-esteem, anxiety, and depression are common after first receiving a diagnosis of genital HSV infection. Initial clinical outbreaks of genital HSV infections are often recurrences of previous infection. Either of the partners may have had an asymptomatic infection acquired in a previous relationship. An experimental HSV type 2 vaccine has been developed, but it is ineffective in men. Ref: Kimberlin DW, Rouse DJ: Genital herpes. N Engl J Med 2004;350(19):1970-1977.

Which one of the following historical features is most suggestive of congestive heart failure in a 6-month-old white male presenting with tachypnea? (check one) A. Diaphoresis with feeding B. Fever C. Nasal congestion D. Noisy respiration or wheezing E. Staccato cough

A Symptoms of congestive heart failure in infants are often related to feedings. Only small feedings may be tolerated, and dyspnea may develop with feedings. Profuse perspiration with feedings, is characteristic, and related to adrenergic drive. Older children may have symptoms more similar to adults, but the infant's greatest exertion is related to feeding. Fever and nasal congestion are more suggestive of infectious problems. Noisy respiration or wheezing does not distinguish between congestive heart failure, asthma, and infectious processes. A staccato cough is more suggestive of an infectious process, including pertussis. Ref: Strange GR, Ahrens WR, Lelyveld S, et al (eds): Pediatric Emergency Medicine: A Comprehensive Study Guide, ed 2. American College of Emergency Physicians, 2002, pp 216, 226, 246. 2) Behrman RE, Kliegman RM, Jenson HB (eds): Nelson Textbook of Pediatrics, ed 17. Saunders, 2004, p 1583.

In addition to calcium and vitamin D supplementation, patients who are beginning long-term treatment with prednisone (≥3 months at a dosage ≥5 mg/day) should also receive (check one) A. bisphosphonate therapy B. calcitonin C. estrogen replacement therapy D. recombinant human parathyroid hormone E. raloxifene (Evista)

A The American College of Rheumatology recommends that patients who are beginning long-term treatment with prednisone (≥3 months at a dosage ≥5 mg/day), or an equivalent, receive bisphosphonate therapy in addition to calcium and vitamin D supplementation, regardless of their DEXA-scan T score. The other treatments are not recommended for prevention of glucocorticoid-induced osteoporosis. Ref: Zizic TM: Pharmacologic prevention of osteoporotic fractures. Am Fam Physician 2004;70(7):1293-1300.

According to the Beers criteria, a list of drugs that should be avoided in geriatric patients, which one of the following NSAIDs should be avoided in older patients due to its higher rate of adverse central nervous system effects? (check one) A. Indomethacin B. Ibuprofen C. Diclofenac sodium D. Etodolac E. Celecoxib (Celebrex)

A The Beers criteria, a list of drugs that should generally be avoided by older patients, was developed by expert consensus, and was last updated in 2002. Indomethacin is on the list due to its propensity to produce more central nervous system adverse effects than other NSAIDs. Ref: Pham CB, Dickman RL: Minimizing adverse drug events in older patients. Am Fam Physician 2007;76(12):1837-1844.

The mother of a 3-year-old male is concerned that he doesn't like being held, doesn't interact much with other children, and rarely smiles. Of the following, which feature would be most helpful in distinguishing Asperger's syndrome from autism in this patient? (check one) A. Normal language development B. Delayed gross motor development C. Repetitive fine motor mannerisms D. Preoccupation with parts of objects E. Focused patterns of intense interest

A The DSM-IV categorizes Asperger's syndrome and autism as pervasive developmental disorders. In both conditions, children have significant difficulties with social interactions, although the impairment is more severe and sustained in autism. Both Asperger's and autism may be associated with symptoms of repetitive motor mannerisms, restricted patterns of interest (which are abnormal in focus or intensity), or preoccupation with parts of objects. However, unlike children with Asperger's syndrome, autistic children have serious problems with communication skills, either in the development of speech itself or in the ability to carry on a conversation. Normal, age-appropriate language skills in a 3-year-old would rule out a diagnosis of autism. It is an important distinction to make, as the prognosis for independent functioning in children with Asperger's syndrome is significantly better than in children with autism. Ref: Sadock BJ, Sadock VA (eds): Kaplan & Sadock's Comprehensive Textbook of Psychiatry, ed 8. Lippincott Williams & Wilkins, 2005, pp 3164-3182.

A 50-year-old male who is a heavy smoker asks you about vitamin supplementation to prevent cancer and cardiovascular disease. The patient is unwilling to stop smoking. According to the 2003 recommendations of the U.S. Preventive Services Task Force, which one of the following is true regarding vitamin supplementation in adults who are middle-age or older? (check one) A. Large supplemental doses of ~17beta-carotene may increase the risk of lung cancer in heavy smokers B. Beta-carotene supplementation decreases the risk of cardiovascular disease and cancer in nonsmokers C. Supplementation with vitamins A, C, and E plus folic acid decreases the risk of cardiovascular disease D. Supplementation with antioxidant combination vitamins plus folic acid decreases the risk of cancer

A The U.S. Preventive Services Task Force found that beta-carotene supplementation provides no benefit in the prevention of cancer in middle-aged and older adults. In two trials limited to heavy smokers, supplementation with beta-carotene was associated with a higher incidence of lung cancer and all-cause mortality. In general, little evidence was found to determine whether supplementation of any of the mentioned vitamins reduces the risk of cardiovascular disease or cancer. Ref: U.S. Preventive Services Task Force: Routine vitamin supplementation to prevent cancer and cardiovascular disease: Recommendations and rationale. Ann Intern Med 2003;139(1):51-55.

A 56-year-old white male presents with a 2-week history of intermittent pain in his left leg. The pain usually occurs while he is walking and is primarily in the calf muscle or Achilles region. Sometimes he will awaken at night with cramps in the affected leg. He has no known risk factors for atherosclerosis. Which one of the following would be the best initial test for peripheral vascular occlusive disease? (check one) A. Ankle-brachial index B. Arterial Doppler ultrasonography C. Arteriography D. Magnetic resonance angiography (MRA) E. Venous ultrasonography

A The ankle-brachial index (ABI) is an inexpensive, sensitive screening tool and is the most appropriate first test for peripheral vascular occlusive disease (PVOD) in this patient. The ABI is the ratio of systolic blood pressure measured in the ankle to systolic pressure using the standard brachial measurement. A ratio of 0.9-1.2 is considered normal. Severe disease is defined as a ratio <0.50. More invasive and expensive testing using Doppler ultrasonography, arteriography, or magnetic resonance angiography may be useful if the ABI suggests an abnormality. Venous ultrasonography would not detect PVOD, but it could rule out deep venous thrombosis, which is another common etiology for calf pain. Ref: Sontheimer DL: Peripheral vascular disease: Diagnosis and treatment. Am Fam Physician 2006;73(11):1971-1976.

In a 34-year-old primigravida at 35 weeks' gestation, which one of the following supports a diagnosis of MILD preeclampsia rather than severe preeclampsia? (check one) A. A blood pressure of 150/100 mm Hg B. A 24-hr protein level of 6 g C. A platelet count <100,000/mm3 D. Liver enzyme elevation with epigastric tenderness E. Altered mental status

A The criteria for severe preeclampsia specify a blood pressure of 160/110 mm Hg or above on two occasions, 6 hours apart. Other criteria include proteinuria above 5 g/24 hr, thrombocytopenia with a platelet count <100,000/mm3, liver enzyme abnormalities, epigastric or right upper quadrant pain, and alteration of mental status. Ref: Sibai BM: Diagnosis and management of gestational hypertension and preeclampsia. Obstet Gynecol 2003;102(1):181-192. 2) DeCherney AH, Nathan L (eds): Current Obstetric & Gynecologic Diagnosis & Treatment, ed 9. Lange Medical Books/McGraw-Hill, 2003, p 338.

A 72-year-old African-American male comes to your office for surgical clearance to undergo elective hemicolectomy for recurrent diverticulitis. The patient suffered an uncomplicated acute anterior-wall myocardial infarction approximately 18 months ago. A stress test was normal 2 months after he was discharged from the hospital. Currently, the patient feels well, walks while playing nine holes of golf three times per week, and is able to walk up a flight of stairs without chest pain or significant dyspnea. Findings are normal on a physical examination. Which one of the following would be most appropriate for this patient prior to surgery? (check one) A. A 12-lead resting EKG B. A graded exercise stress test C. A stress echocardiogram D. A persantine stressed nuclear tracer study (technetium or thallium) E. Coronary angiography

A The current recommendations from the American College of Cardiology and the American Heart Association on preoperative clearance for noncardiac surgery state that preoperative intervention is rarely needed to lower surgical risk. Patients who are not currently experiencing unstable coronary syndrome, severe valvular disease, uncompensated congestive heart failure, or a significant arrhythmia are not considered at high risk, and should be evaluated for most surgery primarily on the basis of their functional status. If these patients are capable of moderate activity (greater than 4 METs) without cardiac symptoms, they can be cleared with no stress testing or coronary angiography for an elective minor or intermediate-risk operation such as the one this patient is to undergo. A resting 12-lead EKG is recommended for males over 45, females over 55, and patients with diabetes, symptoms of chest pain, or a previous history of cardiac disease. Ref: ACC/AHA guideline update for perioperative cardiovascular evaluation for noncardiac surgery--Executive summary: A report of the American College of Cardiology/American Heart Association Task Force on Practice Guidelines (Committee to Update the 1996 Guidelines on Perioperative Cardiovascular Evaluation for Noncardiac Surgery). Circulation 2002;105(10):1257-1267.

The husband and daughter of a 65-year-old female report recent changes in her behavior, including decreased energy, lack of motivation, difficulty making decisions, decreased appetite, and insomnia of 4 weeks' duration. The patient is not on any new medications, and has no previous medical problems. Over the past 2 days, she has become concerned about memory loss for both recent and remote events. This patient most likely has: (check one) A. Depression B. Dementia C. A brain tumor D. Hypoglycemia E. Myocardial infarction

A The diagnosis of depression requires the presence of at least five of the following: depressed mood, sleep disturbance, lack of interest or pleasure in activities, guilt and feelings of worthlessness, lack of energy, loss of concentration and difficulty making decisions, anorexia or weight loss, psychomotor agitation or retardation, and suicidal ideation. The symptoms must be present nearly every day during a 2-week period. Because dementia may cause similar symptoms, distinguishing between the two is important. Dementia is insidious, with a long duration of symptoms including fluctuating mood and memory impairment for recent events. Memory loss often precedes mood changes. Organic problems such as brain tumor, hypoglycemia, and myocardial infarction may cause similar symptoms, but are far less likely to be the cause. Ref: Birrer RB, Vemuri SP: Depression in later life: A diagnostic and therapeutic challenge. Am Fam Physician 2004;69(10):2375-2382.

Which one of the following is true regarding the use of a diaphragm for contraception? (check one) A. It must be refitted if the patient gains more than 15 lb B. Use of nonoxynol-9 will prevent HIV C. Diaphragms are made only of latex D. Diaphragms are recommended for women with a history of toxic shock syndrome E. The diaphragm should be removed immediately after intercourse

A The diaphragm is an effective method of contraception if used correctly. A weight change of more than 15 lb, pregnancy, or pelvic surgery may necessitate refitting. If used with nonoxynol-9, a diaphragm may actually increase the risk of HIV transmission. Diaphragms are made of latex, but a wide seal rim model made of silicone is available for those who are latex sensitive. Diaphragm use is contraindicated in women with a history of toxic shock syndrome. The diaphragm should remain in place for 6-24 hours after intercourse. Ref: Allen R: Diaphragm fitting. Am Fam Physician 2004;69(1):97-100.

A 22-year-old female presents with lower right leg pain. She reports that it hurts when she presses her shin. She has been training for a marathon over the past 4 months and has increased her running frequency and distance. She now runs almost every day and is averaging approximately 40 miles per week. She has little pain while at rest, but the pain intensifies with weight bearing and ambulation. She initially thought the pain was from shin splints, but it has intensified this week and she has had to shorten her usual running distances due to worsening pain. On examination you note tenderness to palpation over the anterior aspect of her mid-tibia. She also has trace edema localized to the area of tenderness. Which one of the following imaging studies should be performed first? (check one) A. Plain radiographs B. CT C. MRI D. Ultrasonography E. Bone scintigraphy

A The findings in this patient are consistent with a stress fracture. Plain radiographs should be the initial imaging modality because of availability and low cost (SOR C). These are usually negative initially, but are more likely to be positive over time. If the initial films are negative and the diagnosis is not urgently needed, a second plain radiograph can be performed in 2-3 weeks. Although CT is useful for evaluation of bone pathology, it is not commonly used as even second-line imaging for stress fractures, due to lower sensitivity and higher radiation exposure than other modalities. Triple-phase bone scintigraphy has a high sensitivity and was previously used as a second-line modality; however, MRI has equal or better sensitivity than scintigraphy and higher specificity. MRI is now recommended as the second-line imaging modality when plain radiographs are negative and clinical suspicion of stress fracture persists (SOR C). Musculoskeletal ultrasonography has the advantage of low cost with no radiation exposure, but additional studies are needed before it can be recommended as a standard imaging modality. Ref: Patel DS, Roth M, Kapil N: Stress fractures: Diagnosis, treatment, and prevention. Am Fam Physician 2011;83(1):39-46.

The intranasal live, attenuated influenza vaccine would be appropriate for which one of the following? (check one) A. A 5-year-old female who is otherwise healthy B. A 12-year-old male who has a history of severe persistent asthma C. A 21-year-old female who has a history of Guillain-Barré syndrome D. A 24-year-old female who is 24 weeks pregnant E. A 55-year-old healthy male who requests influenza vaccine

A The live, attenuated influenza vaccine is an option for vaccinating healthy, nonpregnant individuals age 5-49 years. The vaccine is administered intranasally. It is not indicated in patients with underlying medical conditions, such as chronic pulmonary or cardiovascular disease, or in patients with a history of Guillain-Barré syndrome, pregnant patients, or children and adolescents who receive long-term aspirin or salicylate therapy. Patients with a history of hypersensitivity to eggs should not receive this vaccine. Ref: Prevention and control of influenza: Recommendations of the Advisory Committee on Immunization Practices (ACIP)-2007. MMWR 2007;56(RR-6):1-54.

In adults, which one of the following is the most likely cause of chronic, unilateral nasal obstruction? (check one) A. Nasal septal deviation B. Foreign body impaction C. Allergic rhinitis D. Adenoidal hypertrophy

A The most common cause of nasal obstruction in all age groups is the common cold, which is classified as mucosal disease. Anatomic abnormalities, however, are the most frequent cause of constant unilateral obstruction. Of these, septal deviation is the most common. Foreign body impaction is an important, but infrequent, cause of unilateral obstruction and purulent rhinorrhea. Mucosal disease is usually bilateral and intermittent. Adenoidal hypertrophy is the most common tumor or growth to cause nasal obstruction, followed by nasal polyps, but both are less frequent than true anatomic causes of constant obstruction. Ref: Fornadley JA: The stuffy nose and rhinitis. Med Clin North Am 1999;83(1):1-12. 2) Rakel RE: Textbook of Family Practice, ed 6. WB Saunders Co, 2002, pp 447-448.

A 28-year-old female presents for evaluation of a persistent thin discharge, with a "fishy" odor particularly noticeable after intercourse. She has no dyspareunia or dysuria, is in a monogamous relationship, and has used oral contraceptives for many years. Physical examination reveals no vulvar, vaginal, or cervical erythema. There is a homogenous white discharge that coats the vaginal walls. The vaginal pH is 7.0 and on microscopy you note stippled epithelial cells but no hyphae or trichomonads. Which one of the following is true regarding this patient? (check one) A. The treatment of choice may interact with alcohol B. The patient's partner needs to be treated simultaneously C. The diagnosis should be confirmed with a culture D. Oral contraceptives contribute to the risk for this condition

A The patient has the typical symptoms and signs of bacterial vaginosis. There is no need for confirmatory testing. The treatment of choice is oral metronidazole, which may cause a disulfiram-like interaction with alcohol. Treatment of the partner has not been shown to improve the outcome. Ref: Branch WT (ed): Office Practice of Medicine, ed 4. Saunders, 2003, pp 581-585.

A 7-year-old male is hospitalized after sustaining abdominal trauma in an accident. The child is conscious. His pulse rate is 150 beats/min, his systolic blood pressure is palpated at 60 mm Hg, and his respiratory rate is 40/min. His hemoglobin level is 4.0 g/dL because of trauma-related blood loss. His clinical condition is deteriorating despite an infusion of intravenous volume expanders, but the parents are Jehovah's Witnesses and refuse to consent to a blood transfusion because of their religious convictions. Your prognosis is that without a blood transfusion the patient will die. According to medical-legal precedent, which one of the following is correct? (check one) A. The patient should receive the transfusion regardless of the parents' wishes B. The patient can be transfused regardless of the parents' wishes once he becomes asystolic C. The parents may refuse the transfusion if they are in agreement D. The parents may refuse the transfusion if the patient identifies himself as a Jehovah's Witness E. The parents may refuse the transfusion if there is a legally executed advance directive

A The refusal to accept any medical intervention, including life-saving blood transfusions, has been well established for adults who have the ability to definitively communicate their wishes. Also, parents have the power to give or withhold consent to medical treatment on behalf of their children. However, Western courts have deemed that parents cannot refuse emergency, life-saving treatment to children based on these principles: (1) the child's interests and those of the state outweigh parental rights to refuse medical treatment; (2) parental rights do not give parents life and death authority over their children; and (3) parents do not have an absolute right to refuse medical treatment for their children, if that refusal is regarded as unreasonable. Ref: Woolley S: Jehovah's Witnesses in the emergency department: What are their rights? Emerg Med J 2005;22(12):869-871.

Which one of the following unimmunized patients should receive two doses of influenza vaccine? (check one) A. A 5-year-old with asthma B. A 10-year-old with cystic fibrosis C. A 15-year-old with sickle cell anemia D. A 30-year-old with HIV infection E. A 65-year-old with bullous emphysema

A Two doses of influenza vaccine are recommended for children under the age of 9 years unless they have been vaccinated previously. Children 3-8 years of age should receive one or two 0.5-mL doses of split-virus vaccine intramuscularly. Ref: Couch RB: Prevention and treatment of influenza. N Engl J Med 2000;343(24):1778-1787.

The sudden onset of severe abdominal pain, vomiting, and diarrhea in a patient with a cardiac source of emboli and evidence of a separate embolic event makes superior mesenteric artery embolization likely. In this case, evidence of a brachial artery embolus and a cardiac rhythm indicating atrial fibrillation suggest the diagnosis. Some patients may have a surprisingly normal abdominal examination in spite of severe pain. Microscopic hematuria and blood in the stool may both occur with embolization, and severe leukocytosis is present in more than two-thirds of patients with this problem. Diagnostic confirmation by angiography is recommended. Immediate embolectomy with removal of the propagated clot can then be accomplished and a decision made regarding whether or not the intestine should be resected. A second procedure may be scheduled to reevaluate intestinal viability.

A The threshold for prophylactic platelet transfusion is 10,000/μL (SOR A). Platelet transfusion decreases the risk of spontaneous bleeding in such patients. A count below 50,000/μL is an indication for platelet transfusion in patients undergoing an invasive procedure.

A 35-year-old female is planning a second pregnancy. Her last pregnancy was complicated by placental abruption caused by a large fibroid tumor of the uterus, which is still present. Which one of the following would be the most appropriate treatment for the fibroid tumor? (check one) A. Myomectomy B. Myolysis with endometrial ablation C. Uterine artery embolization D. Observation

A There are numerous options for the treatment of uterine fibroids. When pregnancy is desired, myomectomy offers the best chance for a successful pregnancy when prior pregnancies have been marked by fibroid-related complications. Endometrial ablation eliminates fertility, and there is a lack of long-term data on fertility after uterine artery embolization. Observation without treatment would not remove the risk for recurrent complications during subsequent pregnancies. Ref: Evans P, Brunsell S: Uterine fibroid tumors: Diagnosis and treatment. Am Fam Physician 2007;75(10):1503-1508.

When a woman less than 50 years of age develops vulvar cancer, which one of the following associated conditions is most frequently present? (check one) A. Human papillomavirus B. Lichen sclerosus C. Diabetes mellitus D. Syphilis E. Lymphogranuloma venereum

A There has been an increase in vulvar cancer in women 35-65 years of age over the last decade. This increase is associated with human papillomavirus infection, particularly involving subtypes 16 and 18. Lichen sclerosus is associated with vulvar cancer in older women. Hypertension, diabetes mellitus, and obesity may coexist, but are not felt to be independent risk factors. Syphilis and other granulomatous diseases have been associated with vulvar cancer in the past; they are not currently considered to be significant risk factors, but are considered markers for sexual behavior associated with increased risk. Ref: Canavan TP, Cohen D: Vulvar cancer. Am Fam Physician 2002;66(7):1269-1276.

You see a 5-year-old white female with in-toeing due to excessive femoral anteversion. She is otherwise normal and healthy, and her mobility is unimpaired. Her parents are greatly concerned with the cosmetic appearance and possible future disability, and request that she be treated. You recommend which one of the following? (check one) A. Observation B. Medial shoe wedges C. Torque heels D. Sleeping in a Denis Browne splint for 6 months E. Derotational osteotomy of the femur

A There is little evidence that femoral anteversion causes long-term functional problems. Studies have shown that shoe wedges, torque heels, and twister cable splints are not effective. Surgery should be reserved for children 8-10 years of age who still have cosmetically unacceptable, dysfunctional gaits. Major complications of surgery occur in approximately 15% of cases, and can include residual in-toeing, out-toeing, avascular necrosis of the femoral head, osteomyelitis, fracture, valgus deformity, and loss of position. Thus, observation alone is appropriate treatment for a 5-year-old with uncomplicated anteversion. Ref: Sass P, Hassan G: Lower extremity abnormalities in children. Am Fam Physician 2003;68(3):461-468. 2) Kliegman RM, Behrman RE, Jenson HB, et al (eds): Nelson Textbook of Pediatrics, ed 18. Saunders, 2007, pp 2784-2785, 2786.

A 70-year-old female consults you about osteoporosis treatment. Two years ago her DEXA scan T score was -2.6, and she began taking risedronate (Actonel), 35 mg/week. Her BMI is 24 kg/m2, she takes appropriate doses of calcium and vitamin D, and she takes walks almost every day. Her current T score is -2.5, and she is concerned about the minimal change in spite of therapy. She has never had a fracture, but asks if more could be done to reduce her fracture risk. Which one of the following would be the most appropriate recommendation? (check one) A. Continue current treatment B. Stop risedronate and start alendronate (Fosamax) C. Stop risedronate and start teriparatide (Forteo) D. Add raloxifene (Evista) E. Order a bone biopsy to evaluate bone architecture

A There is not a linear correlation between bone mineral density and fracture risk. Bone architecture may be changed by bisphosphonate therapy, which may result in a decreased fracture risk. This patient has not had a fracture and is on adequate medical therapy that should be continued. Ref: Boonen S: Bisphosphonate efficacy and clinical trials for postmenopausal osteoporosis: Similarities and differences. Bone 2007;3(2):S26-S31.

A 60-year-old African-American male who has a 15-year history of diabetes mellitus reports a 1-week history of weakness of the lower left leg, giving way of the knee, and discomfort in the anterior thigh. He has no history of recent trauma. A physical examination reveals decreased sensation to pinprick and light touch over the left anterior thigh, and reduced motor strength on hip flexion and knee extension. The straight leg raising test is normal. The most likely cause of this condition is: (check one) A. femoral neuropathy B. diabetic polyneuropathy C. meralgia paresthetica D. spinal stenosis E. iliofemoral atherosclerosis

A These findings are typical of femoral neuropathy, a mononeuropathy commonly associated with diabetes mellitus, although it has been found to be secondary to a number of conditions that are common in diabetics and not to the diabetes itself. Diabetic polyneuropathy is characterized by symmetric and distal limb sensory and motor deficits. Meralgia paresthetica, or lateral femoral cutaneous neuropathy, may be secondary to diabetes mellitus, but is manifested by numbness and paresthesia over the anterolateral thigh with no motor dysfunction. Spinal stenosis causes pain in the legs, but is not associated with the neurologic signs seen in this patient, nor with knee problems. Iliofemoral atherosclerosis, a relatively common complication of diabetes mellitus, may produce intermittent claudication involving one or both calf muscles but would not produce the motor weakness noted in this patient. Ref: Bradley WG, Daroff RB, Fenichel GM, Jankovic J (eds): Neurology in Clinical Practice, ed 5. Butterworth Heinemann Elsevier, 2008, pp 442-443. 2) Ropper AH, Samuels MA: Adams and Victor's Principles of Neurology, ed 9. McGraw-Hill, 2009, p 1318. 3) Melmed S, Polonsky KS, Larsen PR, Kronenberg HM (eds): Williams Textbook of Endocrinology, ed 12. Elsevier Saunders, 2011, pp 1503-1505.

A male infant is delivered by cesarean section because of dystocia due to macrosomia. Apgar scores are 8 at 1 minute and 10 at 5 minutes. However, at about 1 hour of age he begins to have tachypnea without hypoxemia. A chest radiograph shows diffuse parenchymal infiltrates and fluid in the pulmonary fissures. The symptoms resolve without treatment within 24 hours. The most likely diagnosis is (check one) A. transient tachypnea of the newborn B. intracranial hemorrhage C. laryngotracheomalacia D. meconium aspiration syndrome E. hyaline membrane disease

A This child had transient tachypnea of the newborn, the most common cause of neonatal respiratory distress. It is a benign condition due to residual pulmonary fluid remaining in the lungs after delivery. Risk factors include cesarean delivery, macrosomia, male gender, and maternal asthma and/or diabetes mellitus. The other conditions listed cause neonatal respiratory distress, but do not resolve spontaneously. They also cause additional significant abnormal findings on physical examination and/or ancillary studies such as imaging and laboratory studies. Ref: Hermansen CL, Lorah KN: Respiratory distress in the newborn. Am Fam Physician 2007;76(7):987-994.

You see a 9-month-old male with a 1-day history of cough and wheezing. He has previously been healthy and was born after an uncomplicated term pregnancy. He is up to date on his immunizations. On examination his temperature is 38.6°C (101.5°F) and his respiratory rate is 30/min. He has diffuse wheezing and his oxygen saturation on room air is 94%. Because it is midwinter, you obtain a swab for influenza, which is negative. A chest radiograph shows peribronchiolar edema. Appropriate management would include which one of the following? (check one) A. Supportive care only B. Inhaled corticosteroids C. Ribavirin (Rebetol) D. Palivizumab (Synagis) E. Supplemental oxygen

A This child has a respiratory syncytial virus (RSV) infection. Supportive care is the mainstay of therapy. If the child can take in fluids by mouth and tolerate room air, outpatient management with close physician contact as needed is reasonable, especially in the absence of significant underlying risk factors. Routine use of corticosteroids is not recommended (SOR B). Although up to 60% of infants hospitalized for bronchiolitis receive corticosteroid therapy, studies have not provided sufficient evidence to support their use. Inhaled corticosteroids have not been shown to be beneficial, and the safety of high doses in infants is unclear. Supplemental oxygen should be administered if functional oxygen saturation (SpO2) persistently falls below 90% and can be discontinued when an adequate level returns (SOR C). Antiviral therapy for RSV bronchiolitis is controversial because of its marginal benefit, cumbersome delivery, potential risk to caregivers, and high cost (SOR B). Studies of ribavirin in patients with bronchiolitis have produced inconsistent findings. Palivizumab is a preventive measure, and is not used for treatment of the active disease. It may be considered in select infants and children with prematurity, chronic lung disease of prematurity, or congenital heart disease (SOR A). If used, it should be administered intramuscularly in five monthly doses of 15 mg/kg, usually beginning in November or December (SOR C). Ref: American Academy of Pediatrics Subcommittee on Diagnosis and Management of Bronchiolitis: Diagnosis and management of bronchiolitis. Pediatrics 2006;118(4):1774-1793. 2) Krilov LR: Respiratory syncytial virus (RSV) infection. eMedicine 2006. Available at http://www.emedicine.com/ped/topic2706.htm.

An 87-year-old African-American female is admitted to your hospital with a hip fracture. She lives alone and has been self-sufficient. She has been able to drive, go to the grocery, and balance her own checkbook. She does well in the hospital until the second postoperative day, when she develops agitated behavior, tremor, and disorientation. She attempts to remove her Foley catheter repeatedly. She exhibits alternating periods of somnolence and agitation, and describes seeing things in the room that are not there. Which one of the following is the most likely diagnosis? (check one) A. Delirium B. Alzheimer's disease C. Senile dementia D. Schizophrenia E. Psychosis

A This individual is exhibiting symptoms of delirium. Diagnostic criteria for delirium, according to the Diagnostic and Statistical Manual of Mental Disorders (DSM-IV-TR), include the following: A. Disturbance of consciousness (i.e., reduced clarity of awareness about the environment) with reduced ability to focus, sustain, or shift awareness. B. A change in cognition (e.g., memory deficit, disorientation, language disturbance) or development of a perceptual disturbance that is not better accounted for by a preexisting, established, or evolving dementia. C. Development over a short period of time (usually hours to days) with a tendency to fluctuate during the course of a day. D. Evidence from the history, physical examination, or laboratory findings that indicates the disturbance is caused by direct physiologic consequences of a general medical condition. In the case described, the patient's history does not indicate preexisting problems and she had a relatively abrupt onset of disturbance of consciousness and change in cognition, related to the hospitalization for hip fracture. Ref: American Psychiatric Association: Diagnostic and Statistical Manual of Mental Disorders, ed 4. American Psychiatric Association, 1994, pp 132-133. 2) Gleason OC: Delirium. Am Fam Physician 2003;67(5):1025-1034.

A 72-year-old male slipped on a rug in his kitchen and struck his right side against a counter. He presents several days after the fall with a complaint of ongoing pain in his flank. He has a history of chronic atrial fibrillation, which is treated with warfarin (Coumadin). His vital signs are normal. A physical examination reveals tenderness to palpation along the posterior-lateral chest wall and decreased breath sounds in the right base. Radiographs reveal two fractured ribs on the right side and a moderately large pleural effusion in the right hemithorax. Laboratory test results include a hemoglobin of 10.5 mg/dL (baseline 11.0-12.0 mg/dL) and a prothrombin time of 33.5 seconds with an INR of 3.5. Which one of the following would be the most appropriate management at this time? (check one) A. Evacuation of the pleural space B. Prophylactic antibiotics C. Open fixation of the ribs with control of bleeding D. Symptomatic treatment and close follow-up E. Use of a rib binder for 2-3 weeks

A This patient has been clinically stable despite losing what appears to be a fair amount of blood into his pleural space after fracturing two ribs, a condition referred to as hemothorax. The treatment of choice in this condition is to remove the bloody fluid and re-expand the associated lung. This therapy is felt to decrease any ongoing blood loss by having the lung pleura put a direct barrier over the site that is bleeding. It also prevents the development of empyema or fibrosis, which could occur if the blood were to remain. Ref: Feliciano DV, Mattox KL, Moore EE, et al: Trauma, ed 6. McGraw-Hill, 2007, chap 26.

A 75-year-old male is brought to your office by his wife, who states that he has had mental difficulties in recent months, such as not being able to balance their checkbook or plan for his annual visit with the accountant. He was able to capably perform these activities in the past. She also tells you that he has reported seeing animals in the room with him that he can describe vividly. He naps for 3 or more hours each day, and stares blankly for long periods of time. He seems almost normal at times, but appears very confused at other times. This confusion seems to come and go randomly. He also has been dreaming a lot, and has fallen more than once recently. His only medication is aspirin, 81 mg/day. On examination the patient walks slowly with a somewhat stooped posture and almost falls when turning around. He has only minimal facial expressiveness. No tremor is noted, and the remainder of the examination is normal. He is able to recall three words out of three, but clock drawing is abnormal. Laboratory studies are normal, and a CT of the brain shows changes of aging. What type of dementia does this patient most likely have? (check one) A. Dementia with Lewy bodies B. Dementia of Parkinson's disease C. Alzheimers disease D. Frontotemporal dementia E. Vascular dementia

A This patient has dementia with Lewy bodies, which is the second most common histopathologic type of dementia after Alzheimer's disease. He demonstrates typical symptoms and signs of dementia with Lewy bodies, including well-formed hallucinations, vivid dreams, fluctuating cognition, sleep disorder with periods of daytime sleeping, frequent falls, deficits in visuospatial ability (abnormal clock drawing), and REM sleep disorder (vivid dreams). In Alzheimer's disease the predominant early symptom is memory impairment, without the other symptoms found in this patient. In dementia of Parkinson's disease, extrapyramidal symptoms such as tremor, bradykinesia, and rigidity precede the onset of memory impairment by more than a year. Patients with vascular dementia have risk factors and symptoms of stroke. Frontotemporal dementia presents with behavioral changes, including disinhibition, or language problems such as various types of aphasia. Ref: Neef D, Walling AD: Dementia with Lewy bodies: An emerging disease. Am Fam Physician 2006;73(7):1223-1229, 1230.

A 34-year-old female who delivered a healthy infant 18 months ago complains of a milky discharge from both nipples. She reports that normal periods have resumed since cessation of breastfeeding 6 months ago. She takes ethinyl estradiol/norgestimate (Ortho Tri-Cyclen) for birth control. A complete review of systems is otherwise negative. The most likely cause of the discharge is (check one) A. a medication side effect B. breast cancer C. a hypothalamic tumor D. hypothyroidism

A This patient has galactorrhea, which is defined as a milk-like discharge from the breast in the absence of pregnancy in a non-breastfeeding patient who is more than 6 months post partum. It is more common in women ages 20-35 and in women who are previously parous. It also can occur in men. Medication side effect is the most common etiology. The most common pharmacologic cause of galactorrhea is oral contraceptives. Oral contraceptives that contain estrogen can both suppress prolactin inhibitory factor and stimulate the pituitary directly, both of which can cause galactorrhea. Other medications that can cause galactorrhea include metoclopramide, cimetidine, risperidone, methyldopa, codeine, morphine, verapamil, SSRIs, butyrophenones, dopamine-receptor blockers, tricyclics, phenothiazines, and thioxanthenes. Breast cancer is unlikely to present with a bilateral milky discharge. The nipple discharge associated with cancer is usually unilateral and bloody. Pituitary tumors are a pathologic cause of galactorrhea due to the hyperprolactinemia that is caused by the blockage of dopamine from the hypothalamus, or by the direct production of prolactin. However, patients often have symptoms such as headache, visual disturbances, temperature intolerance, seizures, disordered appetite, polyuria, and polydipsia. Patients with prolactinomas often have associated amenorrhea. These tumors are associated with marked levels of serum prolactin, often >200 ng/mL. Hypothalamic lesions such as craniopharyngioma, primary hypothalamic tumor, metastatic tumor, histiocytosis X, tuberculosis, sarcoidosis, and empty sella syndrome are significant but infrequent causes of galactorrhea, and generally cause symptoms similar to those of pituitary tumors, particularly headache and visual disturbances. It is rare for primary hypothyroidism to cause galactorrhea in adults. Symptoms that would be a clue to this diagnosis include fatigue, constipation, menstrual irregularity, weight changes, and cold intolerance. Ref: Leung AK, Pacaud D: Diagnosis and management of galactorrhea. Am Fam Physician 2004;70(3):543-550.

A 12-year-old female has a cough and slight shortness of breath on a daily basis. She is awakened by the cough at least 3 nights per week. Which one of the following would be the most appropriate treatment for this patient? (check one) A. Inhaled corticosteroids daily B. An oral leukotriene inhibitor as needed C. Oral prednisone daily D. A short-acting β-agonist daily E. A long-acting β-agonist daily

A This patient has moderate persistent asthma. The preferred and most effective treatment is daily inhaled corticosteroids. A leukotriene inhibitor would be less effective. Oral prednisone daily is not recommended because of the risk of inducing adrenal insufficiency. Short- and long-acting β-agonists are not recommended as daily therapy because either can cause tachyphylaxis. They are considered rescue medications rather than preventive treatments.Ref: Scow DT, Luttermoser GK, Dickerson KS: Leukotriene inhibitors in the treatment of allergy and asthma. Am Fam Physician 2007;75(1):65-70. 2) National Asthma Education and Prevention Program: Expert Panel Report 3: Guidelines for the Diagnosis and Management of Asthma. National Heart, Lung, and Blood Institute, 2007. Available at

A 25-year-old male presents to your office for evaluation of pain in the right index finger that has been present for the past 4 days. The pain has been getting progressively worse. On examination the finger is swollen and held in a flexed position. The pain increases with passive extension of the finger, and there is tenderness to palpation from the tip of the finger into the palm. Which one of the following is the most appropriate management of this patient? (check one) A. Surgical drainage and antibiotics B. Antiviral medication C. Oral antibiotics and splinting D. Needle aspiration E. Corticosteroid injection

A This patient has pyogenic tenosynovitis. When early tenosynovitis (within 48 hours of onset) is suspected, treatment with antibiotics and splinting may prevent the spread of the infection. However, this patient's infection is no longer in the early stages and is more severe, so it requires surgical drainage and antibiotics. A delay in treatment of these infections can lead to ischemia of the tendons and damage to the flexor tendon and sheath. This can lead to impaired function of the finger. Needle aspiration would not adequately drain the infection. Antiviral medication would not be appropriate, as this is a bacterial infection. Corticosteroid injections are contraindicated in the presence of infection.

A 60-year-old male presents with an acute onset of pain and swelling in the right big toe. He can recall no mechanism of injury. He has hypertension which is well controlled with hydrochlorothiazide. On examination the area around the base of the toe is reddened, slightly warm, and very tender on palpation. Which one of the following should be AVOIDED in this patient at this time? (check one) A. Allopurinol (Zyloprim) B. Colchicine C. NSAIDs D. Prednisone E. Aspiration of the joint

A This patient likely has gout. Aspiration should be attempted to get a specific diagnosis. The initial treatment for gout is NSAIDs, colchicine, or cortisone injections (SOR B). Allopurinol should be avoided until the episode of gout is controlled, because it may cause temporary worsening. In addition to medication, recommended management includes addressing risk factors such as obesity, diuretic use, high-purine diet, and alcohol intake (SOR B). Ref: Eggebeen AT: Gout: An update. Am Fam Physician 2007;76(6):801-808.

A 27-year-old white male presents to the emergency department 2 hours after being bitten by a rattlesnake. He complains of weakness, abdominal cramping, left leg pain, and left leg swelling. His speech is slurred, and his breath smells of alcohol. Physical Findings Temperature 37.0° C (98.6° F) Blood pressure 100/60 mm Hg Pulse 122 beats/min Respirations 24/min Skin diaphoretic; ecchymoses on both forearms; bite puncture site just above left lateral malleolus Lungs clear to auscultation Cardiac normal heart tones, 1+ posterior tibial pulses Abdomen flat; hypoactive bowel sounds; no masses or guarding Extremities visible swelling of left leg and thigh; skin tightness of left leg Neurologic decreased sensation to light touch and sharp sensation in left foot Which one of the following therapeutic interventions is indicated? (check one) A. Antivenin administration B. Venom extractor use C. Tourniquet application at the upper thigh D. Surgical consultation for decompression fasciotomy E. Administration of platelets and fresh frozen plasma

A This patient presents with a history of snakebite, swelling of an entire extremity, weakness, and ecchymosis. This is consistent with a grade III envenomation and merits antivenin therapy. Production of equine-derived antivenin has stopped, but may still be indicated where available. The ovine product, CroFab, is less allergenic but still scarce due to limited production. Venom extractors are thought to be useful only in the first few minutes after a bite. Two hours is too late to be of any use. Tourniquets are thought to be contraindicated when used to compress an artery. Low-pressure constriction of lymphatic and venous vessels is controversial. Fasciotomy has not proved useful. Antivenin is indicated before any consideration of compartment syndrome. Pressure measurements would be required because of the clinical similarities between envenomation injury and compartment syndrome. Coagulation factors and blood products are rapidly inactivated. They are indicated only in the presence of exsanguination. Ref: Walter FG, Bilden EF, Gibly RL: Envenomations. Crit Care Clin 1999;15(2):353-386. 2) Juckett G, Hancox JG: Venomous snakebites in the United States: Management review and update. Am Fam Physician 2002;65(7):1367-1374, 1377. 3) Marx JA (ed): Rosen's Emergency Medicine, ed 5. Mosby Inc, 2002, pp 786-793. 4) Rakel RE, Bope ET (eds): Conn's Current Therapy 2004. Saunders, 2004, pp 1192-1193.

A 57-year-old female is noted to have a serum calcium level of 11.1 mg/dL (N 8.9-10.5) on a chemistry profile obtained at the time of a routine annual visit. The remainder of the chemistry profile is unremarkable, including normal BUN and creatinine levels. She is otherwise healthy, and is on no medications. On follow-up testing her calcium level is unchanged, a vitamin D level is normal, and her parathyroid hormone level is elevated. Which one of the following is the most likely cause of her hypercalcemia? (check one) A. Primary hyperparathyroidism B. Secondary hyperparathyroidism C. Tertiary hyperparathyroidism D. Cancer metastatic to bone E. Humoral hypercalcemia of malignancy

A This woman most likely has primary hyperparathyroidism due to a parathyroid adenoma or hyperplasia. Secondary hyperparathyroidism is unlikely with normal renal function, a normal vitamin D level, and hypercalcemia. Likewise, tertiary hyperparathyroidism is unlikely with normal renal function. The parathyroid hormone level is suppressed with hypercalcemia associated with bone metastases. Parathyroid hormone-related protein, produced by cancer cells in humoral hypercalcemia of malignancy, is not detected by the assay for parathyroid hormone.

A 50-year-old female with a history of paroxysmal atrial fibrillation has been successfully treated for depression with sertraline (Zoloft). However, she has persistent insomnia, and a 10-day trial of zolpidem (Ambien) has yielded minimal improvement. The most appropriate addition to her current medication would be: (check one) A. Trazodone (Desyrel) each evening B. Lorazepam (Ativan) daily C. Zaleplon (Sonata) at night D. Amitriptyline (Elavil) at night

A Trazodone has been found useful for its sedative and hypnotic effects, and is often used in combination with another antidepressant. Benzodiazepines are not recommended for long-term use. Zaleplon is more short-acting than zolpidem and therefore would not be more effective. Amitriptyline could be used for its antidepressant and sedative effects, but its chronotropic side effects make it less preferable for someone with a disposition to cardiac arrhythmia. Ref: Kaplan HI, Sadock BJ: Kaplan & Sadock's Synopsis of Psychiatry: Behavioral Sciences/Clinical Psychiatry, ed 8. Lippincott Williams & Wilkins, 1998, pp 1098-1099. 2) Dording CM, Mischoulon D, Petersen TJ, et al: The pharmacologic management of SSRI-induced side effects: A survey of psychiatrists. Ann Clin Psychiatry 2002;14(3):143-147.

Question 8 of 10 An 18-year-old white female presents with small, localized warts on the vulva and lower vaginal mucosa. She wants to avoid injections and surgical treatment if possible. Which one of the following is an acceptable topical agent for treating these vaginal lesions? (check one) A. Trichloroacetic acid B. Podofilox gel (Condylox) C. Imiquimod cream (Aldara) D. Interferon E. Podophyllin 25% solution in alcohol (Podocon-25, Podofin)

A Trichloroacetic acid is acceptable for use on vaginal mucosa. It is also acceptable for use when pregnancy is a possibility. Professional application is necessary. Podofilox and podophyllin in alcohol are not safe for use on mucosa. Imiquimod cream is also not approved for mucosal use. Interferon requires injection. Ref: Mead PB, Hager WD, Faro S: Protocols for Infectious Diseases in Obstetrics and Gynecology, ed 2. Blackwell Science, 2000, pp 380-389. 2) Havens CS, Sullivan ND (eds): Manual of Outpatient Gynecology, ed 4. Lippincott Williams & Wilkins, 2002, pp 39-42.

You have been treating a 43-year-old male for unipolar depression for 4 years. He has developed treatment-resistant depression, and despite having a good initial response to an SSRI, his symptoms are worsening. He has failed to improve despite escalated doses of multiple SSRIs and SNRIs. He is currently taking citalopram (Celexa), 60 mg daily. Of the following, the most effective adjunctive therapy would be augmentation with: (check one) A. lithium bicarbonate B. high-dose triiodothyronine C. an atypical antipsychotic, such as olanzapine (Zyprexa) D. an anticonvulsant, such as gabapentin (Neurontin)

A Up to one-third of patients with unipolar depression will fail to respond to treatment with a single antidepressant, despite adequate dosing and an appropriate treatment interval. Lithium, triiodothyronine (T3 ), and atypical antipsychotics can all provide clinical improvement when used in conjunction with the ineffective antidepressant. The American Psychiatric Association and the Institute for Clinical Systems Improvement both recommend a trial of lithium or low-dose T 3 for patients who have an incomplete response to antidepressant therapy. A meta-analysis showed that a serum lithium level ≥0.5 mEq/L and a treatment duration of 2 weeks or greater resulted in a good response (SOR A). While thyroid supplementation as adjunctive therapy is effective, the recommended dosage is no higher than 50 μg/day (SOR B). Atypical antipsychotics can be used as add-on therapy, but are not as effective as lithium or T3 (SOR B). Anticonvulsant medications such as gabapentin have been shown to be effective in the management of bipolar affective disorder, but not as adjunctive therapy in the treatment of unipolar depression resistant to single-agent antidepressants.

A 47-year-old female presents to your office complaining of hot flashes and cold sweats of several months' duration. She is premenopausal. Which one of the following is accurate advice for this patient regarding vasomotor symptoms? (check one) A. They usually peak around the time of menopause, then decline after menopause B. Without treatment, they usually get worse each year after menopause C. They are always caused by estrogen deficiency D. Estrogen alone is recommended for therapy

A Vasomotor symptoms slowly increase until perimenopause, at which time they peak. The symptoms then tend to diminish after menopause. Numerous other pathologic and functional vasomotor etiologies may mimic hot flashes. Estrogen is effective in treating hot flashes but generally should not be given alone, as it increases the risk for endometrial cancer. Ref: Kaunitz AM: Gynecologic problems of the perimenopause: Evaluation and treatment. Obstet Gynecol Clin North Am 2002;29(3):455-473.

Which one of the following is the most common risk factor for retinal detachment? (check one) A. Posterior detachment of the vitreous B. Hyphema C. Glaucoma D. Cataract surgery E. Diabetic retinopathy

A Vitreous detachment is very common after age 60 and occurs frequently in younger persons with myopia. The separation of the posterior aspect of the vitreous from the retina exerts traction on the retina, with the attendant risks of a retinal tear and detachment. Symptoms of retinal detachment may include light flashes (photopsia), a sudden appearance or increase in "floaters," or peripheral visual field loss, any of which should prompt an ophthalmology referral. Cataract surgery can result in premature shrinkage of the vitreous and thereby poses an increased risk, but vitreous detachment resulting from other processes is more common. Hyphema, glaucoma, and diabetic retinopathy are not specific risk factors for retinal detachment. Ref: Gariano RE, Kim CH: Evaluation and management of suspected retinal detachment. Am Fam Physician 2004;69(7):1691-1697.

In a patient with a solitary thyroid nodule, which one of the following is associated with a higher incidence of malignancy? (check one) A. Hoarseness B. Hyperthyroidism C. Female gender D. A nodule size of 2 cm

A When evaluating a patient with a solitary thyroid nodule, red flags indicating possible thyroid cancer include male gender; age <20 years or >65 years; rapid growth of the nodule; symptoms of local invasion such as dysphagia, neck pain, and hoarseness; a history of head or neck radiation; a family history of thyroid cancer; a hard, fixed nodule >4 cm; and cervical lymphadenopathy. Ref: Welker MJ, Orlov D: Thyroid nodules. Am Fam Physician 2003;67(3):559-566.

A positive flexion abduction external rotation (FABER) test that elicits posterior pain indicates involvement of which joint? (check one) A. Sacroiliac B. Shoulder C. Ankle D. Wrist E. Knee

A When the flexion abduction external rotation (FABER) test elicits pain posteriorly, it indicates sacroiliac involvement. Anterior pain indicates hip involvement. Ref: Margo K, Drezner J, Motzkin D: Evaluation and management of hip pain: An algorithmic approach. J Fam Pract 2003;52(8):607-616.

A 40-year-old female comes to your office for a routine examination. She has been in good health and has no complaints other than obesity. Her mother is diabetic and the patient has had a child that weighed 9 lb at birth. Her examination is negative except for her obesity. A fasting glucose level is 128 mg/dL, and when repeated 2 days later it is 135 mg/dL. Which one of the following would be most appropriate at this point? (check one) A. Diagnose type 2 diabetes mellitus and begin diet and exercise therapy B. Begin an oral hypoglycemic agent C. Order a glucose tolerance test D. Tell the patient that she has impaired glucose homeostasis but is not diabetic

A iteria for diagnosing diabetes mellitus include any one of the following: symptoms of diabetes (polyuria, polydipsia, weight loss) plus a casual glucose level ≥200 mg/dL; a fasting plasma glucose level ≥126 mg/dL; or a 2-hour postprandial glucose level ≥200 mg/dL after a 75 gram glucose load. In the absence of unequivocal hyperglycemia the test must be repeated on a different day. The criteria for impaired glucose homeostasis include either a fasting glucose level of 100-125 mg/dL (impaired fasting glucose) or a 2-hour glucose level of 140-199 mg/dL on an oral glucose tolerance test. Normal values are now considered <100 mg/dL for fasting glucose and <140 mg/dL for the 2-hour glucose level on an oral glucose tolerance test. Ref: American Diabetes Association: Diagnosis and classification of diabetes mellitus. Diabetes Care 2008;31(Suppl 1):S55-S60.

A 67-year-old white male with hypertension and chronic kidney disease presents with the recent onset of excessive thirst, frequent urination, and blurred vision. Laboratory testing reveals a fasting blood glucose level of 270 mg/dL, a hemoglobin A 1c of 8.5%, a BUN level of 32 mg/dL, and a serum creatinine level of 2.3 mg/dL. His calculated glomerular filtration rate is 28 mL/min. Which one of the following medications should you start at this time? (check one) A. Glipizide (Glucotrol) B. Metformin (Glucophage) C. Glyburide (DiaBeta) D. Acarbose (Precose)

A t is recommended that metformin be avoided in patients with a creatinine level >1.5 mg/dL for men or >1.4 mg/dL for women. Glyburide has an active metabolite that is eliminated renally. This metabolite can accumulate in patients with chronic kidney disease, resulting in prolonged hypoglycemia. Acarbose should be avoided in patients with chronic kidney disease, as it has not been evaluated in these patients. Glipizide does not have an active metabolite, and is safe in patients with chronic renal disease. Ref: Cavanaugh KL: Diabetes management issues for patients with chronic kidney disease. Clinical Diabetes 2007;25(3):90- 97.

12-year-old white male is brought to your office after accidentally cutting his left hand with a pocketknife. On examination you find a deep 2-cm laceration at the base of the thenar eminence. To test for motor injury to the median nerve you would have the patient: (check one) A. Extend the thumb and fingers B. Oppose the thumb and little finger C. Flex the wrist D. Abduct the thumb and index finger

B The ability to touch the tip of the thumb to the tip of the little finger indicates normal motor function of the median nerve. The radial nerve controls extension of the thumb and fingers. The median nerve partially controls flexion of the wrist, but the site of innervation is proximal to the wound site at the base of the thumb. Abduction of the thumb is a function of the radial nerve. Finger abduction is a function of the ulnar nerve. Ref: Marx JA (ed): Rosen's Emergency Medicine, ed 5. Mosby Inc, 2002, p 505.

A 46-year-old white female complains of a 3-month history of hoarseness and nocturnal wheezing. On further questioning, she tells you that she has to clear her throat repeatedly and feels like she has something stuck in her throat. These symptoms are most likely related to: (check one) A. thyroid disease B. gastroesophageal reflux disease C. sinusitis D. tracheal stenosis

B Acid laryngitis is a group of respiratory symptoms related to gastroesophageal reflux disease. The symptoms of hoarseness (especially in the morning), a repeated need to clear the throat, and nocturnal or early morning wheezing may occur singly or in varying combinations, and are believed to be caused by gastric contents irritating the larynx and hypopharynx. Thyroid disease, sinusitis, and tracheal stenosis can produce one or more of the symptoms described, but not all of them. Ref: Feldman M, Friedman LS, Brandt LJ: Sleisenger and Fordtran's Gastrointestinal and Liver Disease, ed 9. Saunders, 2010, pp 179-181.

An outbreak of pediatric diarrhea has swept your community. You evaluate a 30-month-old male who developed diarrhea yesterday. He is still breastfed. He is alert, his mucous membranes are moist, and his skin turgor is good. He passes a liquid stool in your office. Which one of the following would be the best advice with regard to his diet? (check one) A. The mother should withhold breastfeeding B. He should consume a normal age-appropriate diet, and continue breastfeeding C. Fasting will promote intestinal mucosal recovery D. Oral intake should be limited to clear fluids, bananas, rice, applesauce, and toast (BRAT diet)

B He should consume a normal age-appropriate diet, and continue breastfeeding. Insufficient evidence to suggest efficacy of BRAT diet.

The American College of Obstetricians and Gynecologists and the American Academy of Pediatrics support the advance provision of drugs and instructions for emergency contraception to sexually active women, so that they have ready access to them if they are needed. The evidence shows that advance provision of emergency contraception (check one) A. decreases pregnancy rates on a population level B. decreases the time from unprotected sex to use of emergency contraception C. decreases contraception use by the patient prior to sexual activity D. increases rates of sexually transmitted infection E. increases rates of unprotected intercourse

B A Cochrane review including randomized, controlled trials (RCTs) compared standard access to emergency contraception (EC) with advance provision. The review found eight trials, five of which were conducted in the U.S. Two of the RCTs were sufficiently powered to show a difference in pregnancy rates. No study showed that giving advance EC reduced pregnancy rates on a population level. However, women who were provided with advance EC took the pills an average of approximately 15 hours sooner than women without advance access. Five studies that reported on contraception use did not show a difference in type or frequency of regular contraception use among women who were provided advance EC. Women randomized to the advance EC groups were 2.5 times more likely to use EC once, and 4 times more likely to use it 2 or more times, compared to those without advance access. Three studies reported rates of sexually transmitted infection and none found differences between the advance and standard access EC groups. Six studies reported rates of unprotected sexual intercourse and found no difference. The Cochrane review concludes that advance access to EC appears to be safe, but does not reduce pregnancy on a population level. However, advance provision might be beneficial because it increases the speed and frequency of EC use. Ref: Emergency contraception. American College of Obstetricians and Gynecologists, Practice Bulletin no 69, 2005. 2) American Academy of Pediatrics Committee on Adolescence: Emergency contraception. Pediatrics 2005;116(4):1026-1035. 3) Cripke C: Advance provision for emergency oral contraception. Am Fam Physician 2007;76(5):654.

A 48-year-old white female comes to see you because of abnormal vaginal bleeding. Her periods are lasting 3-5 days longer than usual, bleeding is heavier, and she has experienced some intermenstrual bleeding. Her physical examination is unremarkable, except for a parous cervix with dark blood at the os and in the vagina. She has no orthostatic hypotension, and her hemoglobin level is 11.5 g/dL. A pregnancy test is negative. Which one of the following is the most important next step in management? (check one) A. Laboratory tests to rule out thyroid dysfunction B. An endometrial biopsy C. Oral contraceptives, 4 times a day for 5-7 days D. Cyclic combination therapy with conjugated estrogens (Premarin) and medroxy-progesterone (Provera) each month E. Administration of a gonadotropin-releasing hormone analog such as leuprolide acetate (Eligard Lupron Depot)

B A patient over the age of 35 who experiences abnormal vaginal bleeding must have an endometrial assessment to exclude endometrial hyperplasia or cancer. An endometrial biopsy is currently the preferred method for identifying endometrial disease. A laboratory evaluation for thyroid dysfunction or hemorrhagic diathesis is appropriate if no cancer is present on an endometrial biopsy and medical therapy fails to halt the bleeding. The other options listed can be used as medical therapy to control the bleeding once the histopathologic diagnosis has been made.

A 59-year-old male reports decreases in sexual desire and spontaneous erections, as well as reduced beard growth. The most appropriate test to screen for late-onset male hypogonadism is: (check one) A. free testosterone B. total testosterone C. sex hormone-binding globulin D. LH E. FSH

B A serum total testosterone level is recommended as the initial screening test for late-onset male hypogonadism. Due to its high cost, a free testosterone level is recommended only if the total testosterone level is borderline and abnormalities in sex hormone-binding globulin are suspected. Follow-up LH and FSH levels help to distinguish primary from secondary hypogonadism. Ref: Bhasin S, Cunningham GR, Hayes FJ, et al: Testosterone therapy in men with androgen deficiency syndromes: An Endocrine Society clinical practice guideline. J Clin Endocrinol Metab 2010;95(6):2536-2559.

Which one of the following is considered a contraindication to the use of beta-blockers for congestive heart failure? (check one) A. Mild asthma B. Symptomatic heart block C. New York Heart Association (NYHA) Class III heart failure D. NYHA Class I heart failure in a patient with a history of a previous myocardial infarction E. An ejection fraction <30%

B According to several randomized, controlled trials, mortality rates are improved in patients with heart failure who receive beta-blockers in addition to diuretics, ACE inhibitors, and occasionally, digoxin. Contraindications to beta-blocker use include hemodynamic instability, heart block, bradycardia, and severe asthma. Beta-blockers may be tried in patients with mild asthma or COPD as long as they are monitored for potential exacerbations. Beta-blocker use has been shown to be effective in patients with NYHA Class II or III heart failure. There is no absolute threshold ejection fraction. Beta-blockers have also been shown to decrease mortality in patients with a previous history of myocardial infarction, regardless of their NYHA classification. Ref: Chavey WE II: The importance of beta blockers in the treatment of heart failure. Am Fam Physician 2000;62(11):2453-2462.

A 70-year-old male with a history of hypertension and type 2 diabetes mellitus presents with a 2-month history of increasing paroxysmal nocturnal dyspnea and shortness of breath with minimal exertion. An echocardiogram shows an ejection fraction of 25%. Which one of the patients current medications should be discontinued? (check one) A. Lisinopril (Zestril) B. Pioglitazone (Actos) C. Glipizide (Glucotrol) D. Metoprolol (Toprol-XL) E. Repaglinide (Prandin)

B According to the American Diabetes Association guidelines, thiazolidinediones (TZDs) are associated with fluid retention, and their use can be complicated by the development of heart failure. Caution is necessary when prescribing TZDs in patients with known heart failure or other heart diseases, those with preexisting edema, and those on concurrent insulin therapy (SOR C). Older patients can be treated with the same drug regimens as younger patients, but special care is required when prescribing and monitoring drug therapy. Metformin is often contraindicated because of renal insufficiency or heart failure. Sulfonylureas and other insulin secretagogues can cause hypoglycemia. Insulin can also cause hypoglycemia, and injecting it requires good visual and motor skills and cognitive ability on the part of the patient or a caregiver. TZDs should not be used in patients with New York Heart Association class III or IV heart failure. Ref: American Diabetes Association: Standards of medical care in diabetes-2007. Diabetes Care 2007;30(Suppl 1):S4-S41.

A 20-year-old white female presents with painful and frequent urination that has had a gradual onset over the past week. She has never had a urinary tract infection. There is no associated hematuria, flank pain, suprapubic pain, or fever. She says she has not noted any itching or vaginal discharge. A midstream urine specimen taken earlier in the week showed significant pyuria but a culture was reported as no growth. She has taken an antibiotic for 2 days without relief. Her only other medication is an oral contraceptive agent. Which one of the following is the most likely infectious agent? (check one) A. Escherichia coli B. Chlamydia trachomatis C. Candida albicans D. Staphylococcus saprophyticus

B Women who present with symptoms of acute dysuria, frequency, and pyuria do not always have bacterial cystitis. In fact, up to 30% will show either no growth or insignificant bacterial growth on a midstream urine culture. Most commonly these patients represent cases of sexually transmitted urethritis caused by Chlamydia trachomatis, Neisseria gonorrhoeae, or herpes simplex virus. In this case, the gradual onset, absence of hematuria, and week-long duration of symptoms suggest a sexually transmitted disease. A history of a new sexual partner or a finding of mucopurulent cervicitis would confirm the diagnosis. Empiric treatment with a tetracycline and a search for other sexually transmitted diseases would then be indicated. Another possible diagnosis is urinary tract infection with Escherichia coli or Staphylococcus species; however, the onset of these infections is usually abrupt and accompanied by other signs, such as suprapubic pain or hematuria. Candida is unlikely because there is no accompanying discharge or itching, and the patient's symptoms predate the use of antibiotics. Ref: Berek JS (ed): Berek & Novak's Gynecology, ed 14. Lippincott Williams & Wilkins, 2007, pp 556-557. 2) Rakel RE, Bope ET, Kellerman R (eds): Conn's Current Therapy 2010. Saunders Elsevier, 2010, pp 1251-1255.

A 52-year-old white male is being considered for pharmacologic treatment of hyperlipidemia because of an LDL cholesterol level of 180 mg/dL. Before beginning medication for his hyperlipidemia, he should be screened for: (check one) A. Hyperthyroidism B. Hypothyroidism C. Addison's disease D. Cushing's disease E. Pernicious anemia

B According to the Summary of the National Cholesterol Education Program (NCEP) Adult Treatment Panel III Report of 2001, any person with elevated LDL cholesterol or any other form of hyperlipidemia should undergo clinical or laboratory assessment to rule out secondary dyslipidemia before initiation of lipid-lowering therapy. Causes of secondary dyslipidemia include diabetes mellitus, hypothyroidism, obstructive liver disease, chronic renal failure, and some medications. Ref: Executive summary of the Third Report of the National Cholesterol Education Program (NCEP) Expert Panel on Detection, Evaluation, and Treatment of High Blood Cholesterol in Adults (Adult Treatment Panel III). JAMA 2001;285(19):2486-2509

As a single measurement, which one of the following provides the most accurate estimate of gestational age by ultrasound determination during the second trimester? (check one) A. Transabdominal diameter B. Biparietal diameter C. Femur length D. Crown-rump length

B All of the options listed can be assessed by ultrasonography. Crown-rump length is a very accurate parameter in the first trimester, but the biparietal diameter is the most accurate parameter during the second trimester. Both have a 95% confidence level of being within 5-10 days of the actual gestational age when used at the proper time. Ref: Cunningham FG, Leveno KJ, Bloom SL, et al: Williams Obstetrics, ed 22. McGraw-Hill, 2005, p 392.

A 55-year-old female is concerned about variations in her heartbeat. She describes "fluttering," "flip-flopping," and sometimes "pounding" sensations in her chest, with occasional delays between beats. Her symptoms are episodic, and have been occurring for several months. They have not been present for the past week. The patient's family history is negative for thyroid disease, but she recalls some "heart trouble" in several family members that was accompanied by fainting spells, and at least one relative died suddenly. She takes no medications, has a negative psychiatric review of systems, and has a normal physical examination. Which one of the following would be most appropriate at this point? (check one) A. Reassurance that her symptoms are associated with a benign condition B. A standard 12-lead EKG C. Echocardiography D. Intermittent event (loop) cardiac monitoring

B All patients who present with palpitations should be evaluated for a cardiac cause, since this is the etiology in 43% of cases. A standard 12-lead EKG is the initial test of choice and, along with a history and physical examination, can determine the cause in 40% of cases. A normal resting EKG does not exclude a cardiac arrhythmia. Therefore, if the EKG is normal, palpitations of suspected arrhythmic etiology may require further investigation with ambulatory EKG monitoring. Echocardiography is helpful in evaluating patients for structural heart disease and should be performed when the initial history, physical examination, and EKG are unrevealing, or in patients with a history of cardiac disease or more complex signs and symptoms. This patient's family history, along with the fact that she takes no medications, suggests the possibility of familial long QT syndrome, which often can be diagnosed from a resting EKG. Diagnosing long QT syndrome is important, since it is associated with an increased risk of sudden cardiac death. Based on the patient's clinical presentation and evaluation, hospital admission is not warranted prior to obtaining a standard 12-lead EKG. Ref: Wexler RK, Pleister A, Raman S: Outpatient approach to palpitations. Am Fam Physician 2011;84(1):63-69.

A 23-year-old male returns from a Florida beach vacation, where he sustained a cut to his foot while wading. The cut wasn't treated when it happened, and it is healing, but he says that it feels like something in the wound is "poking" him. Of the following, which one would most likely be easily visible on plain film radiography? (check one) A. A wood splinter B. A glass splinter C. A plastic splinter D. A sea urchin spine

B Almost all glass is visible on radiographs if it is 2 mm or larger, and contrary to popular belief, it doesn't have to contain lead to be visible on plain films. Many common or highly reactive materials, such as wood, thorns, cactus spines, some fish bones, other organic matter, and most plastics, are not visible on plain films. Alternative techniques such as ultrasonography or CT scanning may be effective and necessary in those cases. Sea urchin spines, like many animal parts, have not been found to be easily detected by plain radiography. Ref: Tintinalli JE (ed): Emergency Medicine: A Comprehensive Study Guide, ed 6. McGraw-Hill, 2004, pp 318-324.

Which one of the following benzodiazepines has the shortest half-life? (check one) A. Flurazepam (Dalmane) B. Alprazolam (Xanax) C. Clorazepate (Tranxene) D. Diazepam (Valium) E. Clonazepam (Klonopin)

B Alprazolam (Xanax) has a half-life of about 12 hours, versus 25 hours for clonazepam and 50 hours for flurazepam, clorazepate, and diazepam. Ref: Sadock BJ, Sadock VA (eds): Kaplan & Sadock's Comprehensive Textbook of Psychiatry, ed 7. Lippincott Williams & Wilkins, 2000, p 1013. 2) Drug Facts and Comparisons, 2003 Edition. Facts and Comparisons, 2004, pp 1011-1013.

A mother brings in her 2-month-old infant for a routine checkup. The baby is exclusively breastfed, and the mother has no concerns or questions. Which one of the following would you recommend at this time in addition to continued breastfeeding? (check one) A. Iron supplementation B. Vitamin D supplementation C. A multivitamin D. 8 oz of water daily E. 4 oz of cereal daily

B Although breast milk is the ideal source of nutrition for healthy term infants, supplementation with 200 IU/day of vitamin D is recommended beginning at 2 months of age and continuing until the child is consuming at least 500 mL/day of formula or milk containing vitamin D (SOR B). The purpose of supplementation is to prevent rickets. Unless the baby is anemic or has other deficiencies, neither iron nor a multivitamin is necessary. Parents often mistakenly think babies need additional water, which can be harmful because it decreases milk intake and can cause electrolyte disturbances. Cereal should not be started until 4 to 6 months of age. Ref: Burke BL, Hall RW: Newborn care: 12 beliefs that shape practice (but should they?). J Fam Pract 2007;56(10):802-807.

In adults, the most common cause of right heart failure is: (check one) A. myocarditis B. left heart failure C. pulmonic stenosis D. ventricular septal defect

B Although myocarditis, pulmonic stenosis, and ventricular septal defects can be causes of right heart failure, left heart failure is the most common cause of right heart failure in adults. Ref: Goldman L, Ausiello D (eds): Cecil Medicine, ed 23. Saunders, 2008, pp 349-350.

A 62-year-old male has been taking omeprazole (Prilosec) for over a year for gastroesophageal reflux disease. He is asymptomatic and has had no problems tolerating the drug, but asks you about potential side effects, as well as the benefits of continuing therapy. It would be most accurate to tell him that omeprazole therapy is associated with which one of the following? (check one) A. A decreased rate of hip fracture B. Decreased vitamin B12 absorption C. A reduced likelihood of pneumonia D. A reduced likelihood of Clostridium difficile colitis E. An increased likelihood of iron deficiency anemia

B Although proton pump inhibitors are the most effective treatment for patients with asymptomatic gastroesophageal reflux disease, there are several potential problems with prolonged therapy. Omeprazole is associated with an increased risk of community-acquired pneumonia and Clostridium difficile colitis. Omeprazole has also been shown to acutely decrease the absorption of vitamin B 12 , and it decreases calcium absorption, leading to an increased risk of hip fracture. The risk for Clostridium difficile colitis is also increased.

Which one of the following is the first-line antibiotic treatment for uncomplicated acute otitis media? (check one) A. Ceftriaxone (Rocephin) B. Amoxicillin C. Azithromycin (Zithromax) D. Cefuroxime (Ceftin) E. Trimethoprim/sulfamethoxazole (Bactrim, Septra)

B Amoxicillin remains the recommended first-line treatment for uncomplicated acute otitis media. Various other antimicrobial agents have not proved to be more efficacious, and are associated with more frequent side effects. Ref: Weber SM, Grundfast KM: Modern management of acute otitis media. Pediatr Clin North Am 2003;50(2):399-411.

An enlarged tongue is associated with which one of the following? (check one) A. Pellagra B. Amyloidosis C. Pernicious anemia D. Xerostomia E. Syphilis

B An enlarged tongue (macroglossia) may be part of a syndrome found in developmental conditions such as Down syndrome, or may be caused by a tumor (hemangioma or lymphangioma), metabolic diseases such as primary amyloidosis, or endocrine disturbances such as acromegaly or cretinism. A "bald" tongue may be associated with xerostomia, pernicious anemia, iron deficiency anemia, pellagra, or syphilis. Ref: Fauci AS, Braunwald E, Kasper DL, et al (eds): Harrison's Principles of Internal Medicine, ed 17. McGraw-Hill, 2008, p 219.

You see a 22-year-old female who sustained a right knee injury in a recent college soccer game.She is a defender and executed a sudden cutting maneuver. With her right foot planted and her ankle locked, she attempted to shift the position of her body to stop an oncoming ball and felt her knee pop. She has had a moderate amount of pain and swelling, which began within 2 hours of the injury, but she is most concerned about the loss of knee hyperextension. Which one of the following tests is most likely to be abnormal in this patient? (check one) A. Anterior drawer B. Lachman C. McMurray D. Pivot shift

B Anterior cruciate ligament (ACL) tears occur more commonly in women than in men. The intensity of play is also a factor, with a much greater risk of ACL injuries occurring during games than during practices. The most accurate maneuver for detecting an ACL tear is the Lachman test (sensitivity 60%-100%, mean 84%), followed by the anterior drawer test (sensitivity 9%-93%, mean 62%) and the pivot shift test (sensitivity 27%-95%, mean 62%) (SOR C). McMurray's test is used to detect meniscal tears. Ref: Solomon DH, Simel DL, Bates DW, et al: The rational clinical examination. Does this patient have a torn meniscus or ligament of the knee? Value of the physical examination. JAMA 2001;286(13):1610-1620. 2) Cimino F, Volk BS, Setter D: Anterior cruciate ligament injury: Diagnosis, management, and prevention. Am Fam Physician 2010;82(8):917-922.

A 40-year-old male respiratory therapist presents for a health examination prior to hospital employment. His history indicates that as a child he lived on a farm in Iowa, and his examination is unremarkable, but a chest radiograph shows that both lung fields have BB-sized calcifications in a miliary pattern. No other findings are noted. A PPD skin test is negative. The findings in this patient are most likely a result of (check one) A. HIV infection B. histoplasmosis C. coccidioidomycosis D. tuberculosis E. cryptococcosis

B Asymptomatic patients in excellent health often present with this characteristic chest radiograph pattern, which is usually due to histoplasmosis infection, especially if the patient has been in the midwestern United States. Exposure to bird or bat excrement is a common cause, and treatment is usually not needed. This pattern is not characteristic of the other infections listed, although miliary tuberculosis is a remote possibility in spite of the negative PPD skin test. Ref: Fauci AS, Braunwald E, Kasper DL, et al (eds): Harrison's Principles of Internal Medicine, ed 17. McGraw-Hill, 2008, pp 1244-1246.

You are the team physician for the local high-school swim team. Over the past week, seven members of the team have developed both folliculitis and outer ear infections. You suspect bacterial contamination of the swimming pool. Which one of the following is the most likely cause? (check one) A. Streptococcus pneumoniae B. Pseudomonas aeruginosa C. Corynebacterium ulcerans D. Staphylococcus epidermidis E. Escherichia coli

B Athletes, including swimmers, are susceptible to a number of skin infections. The pH of the external ear is normally acidic. Continued water exposure raises the pH, creating conditions for bacterial overgrowth, most often caused by either Pseudomonas aeruginosa or Staphylococcus aureus. Swimming pool folliculitis is most often attributed to colonization of water with P. aeruginosa. Ref: Fiorillo L, Zucker M, Sawyer D, et al: The pseudomonas hot-foot syndrome. N Engl J Med 2001;345(5):335-338. 2) Adams BB: Dermatologic disorders of the athlete. Sports Med 2002;32(5):309-321. 3) Levy JA: Common bacterial dermatoses: Protecting competitive athletes. Physician Sportsmed 2004;32(6):33-39.

A 30-year-old female at 36 weeks gestation has a positive culture for group B Streptococcus. Her past medical history is significant for the development of a nonurticarial rash in response to penicillin. Which one of the following is most appropriate for intrapartum antibiotic prophylaxis in this patient? (check one) A. Azithromycin (Zithromax) B. Clindamycin (Cleocin) C. Vancomycin (Vancocin) D. Ampicillin E. Cefazolin

B Cefazolin is appropriate for intrapartum prophylaxis against group B Streptococcus (GBS) in penicillin-allergic patients who do not have a history of anaphylaxis, urticaria, angioedema, or respiratory distress. Depending on the antibiotic sensitivity of the GBS organism, either vancomycin or clindamycin is recommended for patients at higher risk for anaphylaxis. Ref: Verani JR, McGee L, Schrag SJ, et al: Prevention of perinatal group B streptococcal disease—Revised guidelines from CDC, 2010. MMWR Recomm Rep 2010;59(RR-10):1-36.

An 81-year-old African-American female complains of increasing fatigue over the past several months. She has also noticed that her skin and hair feel dry and that she often feels cold. She also complains of intermittent swallowing difficulties. Her past medical history is significant for long-standing coronary artery disease, for which she takes metoprolol (Lopressor). Her physical examination is normal except for a resting pulse rate of 56 beats/min, dry skin, brittle hair, and a slow relaxation phase of the deep tendon reflexes. Her serum TSH level is 63.2 μU/mL (N 0.5-5.0). Which one of the following should you do now? (check one) A. Stop the metoprolol B. Start levothyroxine (Synthroid) C. Start liothyronine (Cytomel) D. Start propylthiouracil E. Refer for radioactive iodine ablation

B Autoimmune hypothyroidism is common in elderly women. Symptoms often include fatigue, bradycardia, dry skin, brittle hair, and a prolonged relaxation phase of the deep tendon reflexes. While replacement therapy with levothyroxine is indicated, care must be taken in the elderly, particularly in those with coronary artery disease, to replace the deficit slowly. Levothyroxine replacement should begin at 25μg daily for 6 weeks, with the dosage increased in 25-μg increments as needed, based on TSH levels. Rapid replacement of thyroid hormone can increase the metabolic rate, and therefore myocardial oxygen demand, too quickly. This can precipitate complications of coronary artery disease such as atrial fibrillation, angina, and myocardial infarction. Stopping a β-blocker in this setting is likely to increase the risk. Radioactive iodine ablation is indicated for some cases of hyperthyroidism.

A 27-year-old white male has been in rehabilitation for C6 complete quadriplegia. His health had been good prior to a diving accident 2 months ago which caused his paralysis. The patient has been catheterized since admission and his recovery has been steady. His vital signs have been normal and stable. The nurse calls and tells you that for the past hour the patient has experienced sweating, rhinorrhea, and a pounding headache. His heart rate is 55/min and his blood pressure is 220/115 mm Hg. His temperature and respirations are reported as normal. There has been no vomiting and his neurologic examination is unchanged. The most likely diagnosis is: (check one) A. Cluster headache B. Autonomic hyperreflexia C. Sepsis D. Intracranial hemorrhage E. Progression of the spinal cord lesion

B Autonomic hyperreflexia is characterized by the sudden onset of headache and hypertension in a patient with a lesion above the T6 level. There may be associated bradycardia, sweating, dilated pupils, blurred vision, nasal stuffiness, flushing, or piloerection. It usually occurs several months after the injury and has an incidence as high as 85% in quadriplegic patients. Frequently, it subsides within 3 years of injury, but it can recur at any time. Bowel and bladder distention are common causes. Hypertension is the major concern because of associated seizures and cerebral hemorrhage. Cluster headaches have a constant unilateral orbital localization. The pain is steady (non-throbbing) and lacrimation and rhinorrhea may be part of the syndrome. Sepsis is usually manifested by chills, fever, nausea, and vomiting. Common signs include tachycardia and hypotension rather than bradycardia and hypertension. Signs and symptoms of intracranial hemorrhage vary depending upon the site of the hemorrhage, but the unchanged neurologic status and the lack of a history of hypertension decrease the likelihood of this diagnosis. There are no neurologic findings or history which suggest progression of the patient's lesion at C6. Ref: Braunwald E, Fauci AS, Kasper DL, et al (eds): Harrison's Principles of Internal Medicine, ed 15. McGraw-Hill, 2001, pp 70, 78, 799, 2385, 2419.

A 55-year-old male presents with a 2-year history of persistent, worsening neck stiffness. Over the past month, the stiffness has been associated with left thumb tingling. After completing a thorough history and physical examination, which one of the following studies would be the most appropriate next step in further evaluating the patient's complaints? (check one) A. Lateral neck radiography B. A cervical spine series C. Neck MRI D. CT myelography E. Diskography

B Based on the American College of Radiology's Appropriateness Criteria for chronic neck pain, a complete cervical spine series that includes five views is the correct study in a patient of any age with chronic neck pain and no history of trauma, malignancy, or surgery. If the radiographs are normal and the patient has neurologic signs or symptoms, the next step would be MRI. If MRI is contraindicated, CT myelography should be offered (SOR B). A single lateral radiograph is not sufficient. Diskography is not recommended in patients with chronic neck pain (SOR C). Ref: Daffner RH: Radiologic evaluation of chronic neck pain. Am Fam Physician 2010;82(8):959-964.

Which one of the following therapeutic agents is most appropriate for daily use in the prevention of migraine headache? (check one) A. Dihydroergotamine (D.H.E. 45) B. Amitriptyline (Elavil) C. Sumatriptan (Imitrex) D. Aspirin/caffeine/butalbital (Fiorinal) E. Acetaminophen/hydrocodone bitartrate (Vicodin)

B Beta-adrenergic blockers, antidepressants, anticonvulsants, calcium channel blockers, NSAIDs, and serotonin antagonists are the major classes of drugs used for preventive migraine therapy. All of these medications result in about a 50% reduction in the frequency of headaches. The other drugs listed are useful for the treatment of acute migraine, but not for prevention. Ref: Noble SL, Moore KL: Drug treatment of migraine: Part II. Preventive therapy. Am Fam Physician 1997;56(9):2279-2286. 2) Goadsby PJ, Lipton RB, Ferrari MD: Migraine—Current understanding and treatment. N Engl J Med 2002;346(4):257-270.

Which one of the following has been shown to decrease mortality late after a myocardial infarction? (check one) A. Nitrates B. Beta-blockers C. Digoxin D. Thiazide diuretics E. Calcium channel antagonists

B Beta-blockers and ACE inhibitors have been found to decrease mortality late after myocardial infarction. Aspirin has been shown to decrease nonfatal myocardial infarction, nonfatal stroke, and vascular events. Nitrates, digoxin, thiazide diuretics, and calcium channel antagonists have not been found to reduce mortality after myocardial infarction. Ref: Foody JM (ed): Preventive Cardiology: Strategies for the Prevention and Treatment of Coronary Artery Disease. Humana Press, 2001, pp 244-247.

Black cohosh is: (check one) A. A form of herbal licorice with gastrointestinal effects B. A botanical medicine used to alleviate menopausal symptoms C. A type of toxic hallucinogenic mushroom D. A variety of Cannabis sativa E. A form of dried hashish

B Black cohosh is an herbal preparation widely used in the treatment of menopausal symptoms and menstrual dysfunction. Studies have demonstrated that this botanic medicine appears to be effective in alleviating menopausal symptoms. It has not been proven effective in randomized controlled trials and should not be used to prevent osteoporosis. Questions as to its stimulating effect on endometrial tissue are as yet unanswered. Ref: Kligler B: Black cohosh. Am Fam Physician 2003;68(1):114-116.

You prescribe enalapril (Vasotec) for a 68-year-old male with heart failure. At a follow-up visit 6 weeks later the patient's serum creatinine level is 2.5 mg/dL (N 0.6-1.5) and his serum potassium level is 5.7 mEq/L (N 3.4-4.8). His baseline values were normal. Which one of the following is a side effect of ACE inhibitors that is the most likely cause of these changes in renal function? (check one) A. Toxicity to the proximal renal tubules B. Impaired autoregulation of glomerular blood flow C. Microangiopathic arteriolar thrombosis D. Rhabdomyolysis E. Interstitial nephritis

B Blood flow to the kidney is autoregulated so as to sustain pressure within the glomerulus. This is influenced by angiotensin II-related vasoconstriction. ACE inhibitors can impair the kidney's autoregulatory function, resulting in a decreased glomerular filtration rate and possibly acute renal injury. This is usually reversible if it is recognized and the offending agent stopped. NSAIDs can exert a similar effect, but they can also cause glomerulonephritis and interstitial nephritis. Statins, haloperidol, and drugs of abuse (cocaine, heroin) can cause rhabdomyolysis with the release of myoglobin, which causes acute renal injury. Thrombotic microangiopathy is a rare mechanism of injury to the kidney, and may be caused by clopidogrel, quinine, or certain chemotherapeutic agents. Ref: Naughton CA: Drug-induced nephrotoxicity. Am Fam Physician 2008;78(6):743-750.

In a patient with a sudden onset of dyspnea, which one of the following makes a pulmonary embolus more likely? (check one) A. Fever >38.0°C (100.4°F) B. Chest pain C. Orthopnea D. Wheezes E. Rhonchi

B Chest pain is common in patients with pulmonary embolism (PE). When evaluating a patient for possible PE, the presence of orthopnea suggests heart failure, fever suggests an infectious process, wheezing suggests asthma or COPD, and rhonchi suggest heart failure, interstitial lung disease, or infection. These generalizations are supported by a 2008 study designed to improve the diagnosis of PE based on the history, physical examination, EKG, and chest radiograph. Ref: Miniati M, Bottai M, Monti S, et al: Simple and accurate prediction of the clinical probability of pulmonary embolism. Am J Respir Crit Care Med 2008:178(3):290-294.

A 21-year-old African-American female has been confused and delirious for 2 days. She has no significant past medical history, and she is taking no medications. She recently returned from a missionary trip to Southeast Asia. During your initial examination in the emergency department, she has several convulsions and rapidly becomes comatose. Her temperature is 37.9°C (100.3°F) and her blood pressure is 80/50 mm Hg. A neurologic examination shows no signs of meningeal irritation and a cranial nerve evaluation is normal. There is a mild, bilateral, symmetric increase in deep tendon reflexes. All other physical examination findings are normal. Laboratory Findings Hemoglobin........................... 7.0 g/dL (N 12.0-16.0) Hematocrit............................ 20% (N 36-46) WBCs.. . . . . . . . . . . . . . . . . . . . . . . . . . . . . . . 6500/mm3 (N 4300-10,800) Platelets. ............................. 450,000/mm3 (N 150,000-350,000) Serum bilirubin Total............................... 5.0 mg/dL (N 0.3-1.1) Direct.............................. 1.0 mg/dL (N 0.1-0.4) The urine is dark red and positive for hemoglobin. CT of the brain shows neither bleeding nor infarction. The most likely diagnosis is: (check one) A. vitamin B12 deficiency B. malaria C. ehrlichiosis D. sickle cell anemia

B Clinical clues to the diagnosis of malaria in this case include an appropriately targeted recent travel history, a prodrome of delirium or erratic behavior, unarousable coma following a generalized convulsion, fever, and a lack of focal neurologic signs in the presence of a diffuse, symmetric encephalopathy. The peripheral blood smear shows normochromic, normocytic anemia with Plasmodium falciparum trophozoites and schizonts involving erythrocytes, diagnostic of cerebral malaria. Treatment of this true medical emergency is intravenous quinidine gluconate. Vitamin B 12 deficiency is a predominantly peripheral neuropathy seen in older adults. Ehrlichiosis causes thrombocytopenia but not hemolytic anemia. Sickle cell disease presents with painful vaso-occlusive crises in multiple organs. Coma is rare.

You see a 32-year-old white female for her first visit. She presents with numerous complaints which do not conform to patterns seen in organic disease. She states that she has seen several physicians and describes a changing set of symptoms. Although she appears to be well, she claims to have been "sickly" for years. From her affect, you suspect that she is depressed. The most likely diagnosis is: (check one) A. Conversion reaction B. Chronic somatization disorder C. Schizophrenia with multiple somatic delusions D. Histrionic personality E. Primary hypochondriasis

B Conversion disorder usually involves a single symptom which is neurologic or pain-related. Symptoms of chronic somatization differ from psychoses in that the symptoms of the psychotic patient are bizarre and more vivid, persist over time, are unaltered by reasoned argument, and are not congruent with the patient's social or cultural background. The delusional nature of psychotic somatic symptoms usually unfolds as the patient talks. The essential feature of the histrionic (hysterical) personality is a pervasive pattern of excessive emotionality and attention seeking. People with this disorder constantly seek to be the center of attention. Emotions are often expressed with inappropriate exaggeration. People with this disorder tend to be very self-centered and have little tolerance for delayed gratification. These people are typically attractive and seductive, often to the point of looking flamboyant and acting inappropriately. Features of primary hypochondriasis include the patient's fixed conviction that he or she is ill, the interpretation of all somatic changes as confirmation of this, and a relentless pursuit of medical assistance despite persistent dissatisfaction with the results. The patient's symptoms remain consistent for years. Physicians frequently feel overwhelmed when initially presented with a patient with somatization disorder. This disorder begins before age 30 and is rarely seen in males. The patient complains of multiple symptoms which involve many organ systems and do not readily conform to patterns seen in organic diseases. The patient skips back and forth from symptom to symptom during the interview. Anxiety and depressed mood are frequent in this disorder, and suicide attempts are common. Ref: Hales RE, Yudofsky SC (ed): Textbook of Clinical Psychiatry, ed 4. American Psychiatric Publishing, 2003, pp 660-665, 818-820.

A 66-year-old white male is brought to your office for evaluation of progressive memory loss over the last several months. The problem seems to wax and wane significantly over the course of days and weeks. At times when he is more confused, he tends to have visual and auditory hallucinations that he is back fighting in Vietnam, thinking a ringing telephone is calling in fighter jets. He has also been falling occasionally. On physical examination, he has a resting tremor in his left leg, and rigidity of his upper body and face. A full medical workup, including standard blood work and a CT scan, shows no abnormalities that suggest delirium, stroke, or other primary etiologies. Which one of the following is the most likely diagnosis? (check one) A. Alzheimer's disease B. Dementia with Lewy bodies C. Fronto-temporal dementia D. Multi-infarct dementia E. Pseudodementia

B Dementia with Lewy bodies is currently considered one of the most common etiologies of dementia in elderly patients, representing up to 20%-30% of those with significant memory loss. The clinical presentation consists of parkinsonian symptoms (rigidity, tremor), fluctuating levels of alertness and cognitive abilities, and behavior sometimes mimicking acute delirium. Significant visual hallucinations are common, and delusions and auditory hallucinations are seen to a lesser degree. On pathologic examination, Lewy bodies (seen in the substantia nigra in patients with Parkinson's disease) are present diffusely in the cortex. There is currently no specific treatment. Ref: Galvin JE: Dementia with Lewy bodies. Arch Neurol 2003;60(9):1332-1335. 2) Kasper DL, Braunwald E, Fauci AS, et al (eds): Harrison's Principles of Internal Medicine, ed 16. McGraw-Hill, 2005, pp 2402-2403.

You discover a 10-cm enlarging hematoma adjacent to the episiotomy site in a patient whose baby you delivered 6 hours ago. The best management at this time is: (check one) A. A perineal pad and cold compresses B. Removal of the sutures and clots, and reclosure C. Hypogastric artery ligation D. Needle aspiration of the hematoma

B Enlarging postpartum hematomas adjacent to an episiotomy are best treated by removing the sutures and ligating the specific bleeding sites. A perineal pad and cold compresses are inadequate for an enlarging lesion, and hypogastric artery ligation and hysterectomy are indicated only with supravaginal hematomas. Ref: Cunningham FG, Leveno KJ, Bloom SL, et al: Williams Obstetrics, ed 22. McGraw-Hill, 2005, pp 435-438, 719-720, 836-837.

You are evaluating a 45-year-old male construction worker with regard to his skin and sun exposure history. Which one of the following lesions should be considered premalignant? (check one) A. Sebaceous hyperplasia B. Actinic keratosis C. Seborrheic keratosis D. A de Morgan spot E. A halo nevus

B Family physicians should advise patients of the dangers of sun exposure especially those with a fair complexion who work outdoors. Although malignant melanoma is the most serious condition of those listed, actinic keratosis may lead to squamous cell carcinoma with significant morbidity. Ref: Habif TP: Clinical Dermatology: A Color Guide to Diagnosis and Therapy, ed 4. Mosby Inc, 2004, p 736.

A 67-year-old female is admitted to the hospital with severe community-acquired pneumonia. Her urine should be tested for which one of the following antigens? (check one) A. Chlamydia B. Mycoplasma C. Legionella D. Haemophilus influenzae E. Pseudomonas

C In patients with severe pneumonia, the urine should be tested for antigens to Legionella and pneumococcus. Two blood cultures should also be drawn, but these are positive in only 10%-20% of all patients with community-acquired pneumonia. Ref: Niederman M: In the clinic. Community-acquired pneumonia. Ann Intern Med 2009;151(7):ITC4-2-ITC4-14

A 28-year-old female sees you with a complaint of irregular menses. She has not had a menstrual period for 6 months. She is also concerned about weight gain, worsening acne, and dark hair on her upper lip, chin, and periareolar region. She is also interested in becoming pregnant soon. The patient tells you she has started an exercise program, which has helped with weight loss, but she continues to have amenorrhea. She has a negative urine β-hCG test, a mild elevation in free testosterone levels, and glucose intolerance. Which one of the following would you consider initially for inducing ovulation? (check one) A. Insulin B. Metformin (Glucophage) C. Ethinyl estradiol/norgestimate (Ortho Tri-Cyclen) D. Glipizide (Glucotrol) E. Spironolactone (Aldactone)

B First-line agents for ovulation induction and treatment of infertility in patients with polycystic ovary syndrome (PCOS) include metformin and clomiphene, alone or in combination, as well as rosiglitazone (SOR A). In one study of nonobese women with PCOS, metformin was found to be more effective than clomiphene for improving the rate of conception (level of evidence 1b). However, the treatment of infertile women with PCOS remains controversial. One recent group of experts recommended that metformin use for ovulation induction in PCOS be restricted to women with glucose intolerance (SOR C). Oral contraceptives are commonly used to treat menstrual irregularities in women with PCOS; however, there are few studies supporting their use, and they would not be appropriate for ovulation induction. Spironolactone is a first-line agent for treatment of hirsutism (SOR A) and has shown promise in treating menstrual irregularities, but is not commonly recommended for ovulation induction. There is a high prevalence of insulin resistance in women with PCOS, as measured by glucose intolerance; insulin-sensitizing agents are therefore indicated, but not insulin or sulfonylurea medications.

A small child with failure to thrive is found to have a bone age that is markedly delayed relative to height age and chronologic age. The most likely etiology is: (check one) A. Cystic fibrosis B. Hypothyroidism C. Down syndrome D. Fetal alcohol syndrome E. Gonadal dysgenesis

B Hypothyroidism is associated with markedly delayed bone age relative to height age and chronologic age. In cystic fibrosis, bone age and height age are equivalent, but both lag behind chronologic age. Children with chromosomal anomalies such as trisomy 21 (Down syndrome) or XO have a height age which is delayed relative to bone age. This pattern is also seen as a result of maternal substance abuse. Ref: Rudolph CD, Rudolph AM (eds): Rudolph's Pediatrics, ed 21. McGraw-Hill, 2003, p 2021.

A 65-year-old male has recently undergone coronary artery bypass graft (CABG) surgery. Generally, he has recovered well from his surgery. However, his cardiac surgeon referred him back to you because of symptoms suggestive of depression. Which one of the following is true in this situation? (check one) A. Patients with chronic cardiac symptoms prior to surgery are more likely to develop postoperative depression B. Postoperative depression increases the risk for subsequent cardiovascular events C. Treatment of postoperative depression with antidepressants decreases the rate of subsequent cardiovascular events D. Enrollment in a cardiac rehabilitation program often worsens depression

B In patients who are depressed after coronary artery bypass graft (CABG) surgery, impaired memory and cognition are seen more frequently than other depressive symptoms. Patients with rapid progression of cardiac symptoms before surgery are at particular risk of depressive symptoms after surgery. Newly depressed patients are at higher risk than non-depressed patients for long-term cardiovascular events and death from cardiovascular causes. The Sertraline AntiDepressant Heart Attack Randomized Trial (SADHART) showed that antidepressant use was associated with a slight, but not significant, reduction in the rates of cardiovascular events. The Enhancing Recovery in Coronary Heart Disease (ENRICHD) trial showed that although it did not reduce the risk of cardiac events, participation in a cardiac rehabilitation program reduced depressive symptoms and increased social ties. Ref: Charlson ME, Isom OW: Care after coronary-artery bypass surgery. N Engl J Med 2003;348(15):1456-1463.

In a patient who presents with symptoms of acute myocardial infarction, which one of the following would be an indication for thrombolytic therapy? (check one) A. New-onset ST-segment depression B. New-onset left bundle branch block C. New-onset first degree atrioventricular block D. New-onset Wenckebach second degree heart block E. Frequent unifocal ventricular ectopic beats

B In patients with ischemic chest pain, the EKG is important for determining the need for fibrinolytic therapy. Myocardial infarction is diagnosed by ST elevation ≥1 mm in two or more limb leads and ≥2 mm in two or more contiguous precordial leads. In a patient with an MI, new left bundle branch block suggests occlusion of the left anterior descending artery, placing a significant portion of the left ventricle in jeopardy. Thrombolytic therapy could be harmful in patients with ischemia but not infarction - they will show ST-segment depression only. Frequent unifocal ventricular ectopy may warrant antiarrhythmic therapy, but not thrombolytic therapy. Ref: Marx JA (ed): Rosens Emergency Medicine: Concepts and Clinical Practice, ed 6. Mosby Elsevier, 2006, pp 1189-1192.

Which one of the following is true regarding the use of opiates in terminally ill patients? (check one) A. They are frequently addictive B. They are indicated for relieving dyspnea C. A medication contract is required by law D. Respiratory depression is the first sign of excessive dosage E. Gastrointestinal hypermotility is a common side effect

B In terminally ill patients, the most common physical symptoms are pain, fatigue, and dyspnea. Opiates are useful for controlling pain and relieving dyspnea as well. Even small doses of a weaker opiate can reduce the sensation of shortness of breath in cancer patients and in those with heart failure or chronic obstructive lung disease. Addiction is rare in terminally ill patients who are being treated with opiates for pain and/or dyspnea. A medication contract between physician and patient is not required by law and generally is not necessary in this situation, unless diversion of the medication from the patient by the caregivers is suspected. Constipation due to decreased gastrointestinal motility is a very common, if not universal, side effect. Respiratory depression is a late, not early, sign of excessive opiate dosage. Another sign of opiate excess, pinpoint pupils, occurs before respiratory depression and is therefore a useful parameter for monitoring these patients. Ref: Fauci AS, Braunwald E, Kasper DL, et al (eds): Harrisons Principles of Internal Medicine, ed 17. McGraw-Hill, 2008, pp 70-71.

Routine blood tests frequently reveal elevated calcium levels. When this elevation is associated with elevated parathyroid hormone levels, which one of the following is an indication for parathyroid surgery? (check one) A. Age >50 B. Kidney stones C. Serum calcium 0.5 mg/dL above the upper limit of normal D. Concurrent hyperthyroidism E. Increased bone density

B Indications for parathyroid surgery include kidney stones, age less than 50, a serum calcium level greater than 1 mg/dL above the upper limit of normal, and reduced bone density. Hyperthyroidism is not a factor in deciding to perform parathyroid surgery. Ref: Bilezikian JP, Silverburg SJ: Asymptomatic primary hyperparathyroidism. N Engl J Med 2004;350(17):1746-1751.

Compared to children with attention-deficit/hyperactivity disorder (ADHD), adults with ADHD: (check one) A. Tend to be more hyperactive B. Tend to be less impulsive C. Are less likely to have corroboration of symptoms by family members D. Are less likely to complain of inattention difficulties

B Longitudinal studies of young people diagnosed with attention-deficit/hyperactivity disorder (ADHD) show that symptoms of hyperactivity and impulsivity may decrease with age, but inattention tends to persist. Studies of clinically referred adults with ADHD show that about half have clinically important levels of hyperactivity and impulsivity and up to 90% have prominent attentional symptoms. Like some youth with ADHD, adults with ADHD tend to have additional cognitive deficits, specifically executive function deficits, which include problems encoding and manipulating information and difficulties with organization and time management. Research shows that using retrospective self-reports of adults is a valid method of diagnosing ADHD. Studies have shown that the consistent reporting of childhood ADHD symptoms by both adults and their parents is highly correlated. Research has also found strong agreement between the self-reports of adults and their partners regarding ADHD symptoms. Ref: Wilens TE, Faraone SV, Biederman J: Attention-deficit/hyperactivity disorder in adults. JAMA 2004;292(5):619-623.

A 32-year-old white male teacher is seen for a paroxysmal cough of 5 days duration. He tells you that a student in his class was diagnosed with pertussis 3 weeks ago. Which one of the following would be the best treatment? (check one) A. Amoxicillin B. Azithromycin (Zithromax) C. Cephalexin (Keflex) D. Ciprofloxacin (Cipro) E. Doxycycline

B Macrolides are considered first-line therapy for Bordetella pertussis infection. Trimethoprim/sulfamethoxazole is considered second-line therapy. Ref: Wenzel RP, Fowler AA: Acute bronchitis. N Engl J Med 2006;355(20):2125-2130.

Which one of the following would be most appropriate to treat a dental infection requiring antibiotic therapy? (check one) A. Erythromycin B. Penicillin C. Dicloxacillin (Dynapen) D. Tetracycline E. Cefixime (Suprax)

B Many dental conditions causing inflammation do not require antibiotic therapy. Dental caries, reversible pulpitis, gingivitis, periodontitis, and periapical abscesses usually are treated with local procedures without antibiotics. Cellulitis, however, requires either outpatient antibiotics or inpatient antibiotic treatment if the cellulitis spreads to the deeper spaces of the head and neck. The antibiotic of choice, especially for outpatient treatment, is oral penicillin G, 500 mg 3 times daily. Ref: Douglass AB, Douglass JM: Common dental emergencies. Am Fam Physician 2003;67(3):5

For which one of the following respiratory infections should antibiotic therapy be initiated immediately upon diagnosis? (check one) A. Bronchitis B. Epiglottitis C. Laryngitis D. Rhinosinusitis E. Tracheitis

B Many infections of the respiratory tract have a viral etiology, and when this is the case early antibiotic treatment offers little to no benefit. Once the clinical course of a respiratory illness exceeds the expected length for a viral illness, it may be proper to initiate antibiotic treatment for a suspected atypical or secondary bacterial infection. Epiglottitis is one exception to this approach because of the possibility of a bacterial infection, particularly with Haemophilus influenzae type b, that can produce a rapidly worsening, potentially fatal airway compromise. When epiglottitis is suspected based on findings such as hoarseness, dysphagia, stridor, drooling, fever, chills, and respiratory distress, intravenous antibiotic treatment should be instituted immediately, ideally with a $-lactam drug that exhibits activity against methicillin-resistant Staphylococcus aureus.

Which one of the following is more likely to occur with glipizide (Glucotrol) than with metformin (Glucophage)? (check one) A. Lactic acidosis B. Hypoglycemia C. Weight loss D. Gastrointestinal distress

B Metformin is a biguanide used as an oral antidiabetic agent. One of its main advantages over some other oral agents is that it does not cause hypoglycemia. Lactic acidosis, while rare, can occur in patients with renal impairment. In contrast to most other agents for the control of elevated glucose, which often cause weight gain, metformin reduces insulin levels and more frequently has a weight-maintaining or even a weight loss effect. Gastrointestinal distress is a common side-effect of metformin, particularly early in therapy. Ref: Rakel RE, Bope ET (eds): Conn's Current Therapy 2002. WB Saunders Co, 2003, pp 624-625.

A 53-year-old male presents with a 3-month history of despondency, insomnia, and irritability with family and co-workers. During your interview you also discover that he is drinking heavily at times and has several firearms at home. He thinks his life is "useless," noting that he "would be better off dead." The most appropriate action at this time would be to: (check one) A. Prescribe an SSRI B. Arrange immediate hospitalization C. Have the patient agree to a suicide prevention contract D. Avoid direct questions regarding suicidal thoughts

B More than 50% of suicides are associated with a major depressive episode and 25% are associated with a substance abuse disorder. Suicide rates increase with age and are higher among men. Increased suicide rates also occur in patients with significant medical illnesses. Because discussing suicidal ideation may relieve the patient's anxiety, the physician should directly ask depressed patients about any suicidal thoughts. There are no known reliable tools for assessing suicide risk, so the assessment is subjective. The initial management of suicidal ideation should establish safety, often by hospitalization. The suicide prevention contract is of unproven clinical and legal usefulness. Antidepressant medication has not been shown to reduce suicide rates, especially on a short-term basis. Ref: Stovall J, Domino FJ: Approaching the suicidal patient. Am Fam Physician 2003;68(9):1814-1818.

Which one of the following community health programs best fits the definition of secondary prevention? (check one) A. An antismoking education program at a local middle school B. Blood pressure screening at a local church C. A condom distribution program D. Screening diabetic patients for microalbuminuria

B Prevention traditionally has been divided into three categories: primary, secondary, and tertiary. Primary prevention targets individuals who may be at risk to develop a medical condition and intervenes to prevent the onset of that condition (e.g., childhood vaccination programs, water fluoridation, antismoking programs, and education about safe sex). Secondary prevention targets individuals who have developed an asymptomatic disease and institutes treatment to prevent complications (e.g., routine Papanicolaou tests; screening for hypertension, diabetes, or hyperlipidemia). Tertiary prevention targets individuals with a known disease, with the goal of limiting or preventing future complications (e.g., screening diabetics for microalbuminuria, rigorous treatment of diabetes mellitus, and post-myocardial infarction prophylaxis with β-blockers and aspirin).

A 14-year-old African-American female presents for a routine evaluation. On examination, you note a rubbery, well-defined, nontender breast mass approximately 2 cm in diameter. The patient denies any history of breast tenderness, nipple discharge, or skin changes. The most likely diagnosis is: (check one) A. Fibrocystic breast disease B. Fibroadenoma C. Benign breast cyst D. Cystosarcoma phyllodes E. Intraductal papilloma

B Most breast masses in adolescent girls are benign. Fibroadenoma is the most common, accounting for approximately two-thirds of all adolescent breast masses. It is characterized by a slow growing, nontender, rubbery, well-defined mass, most commonly located in the upper, outer quadrant. Size varies, and is most commonly in the range of 2-3 cm. Fibrocystic disease is found in older adolescents and is characterized by bilateral nodularity and cyclic tenderness. Benign breast cysts are characterized by a spongy, tender mass with symptoms exacerbated by menses. Cysts are frequently multiple, and spontaneous regression occurs in 50% of patients. Cystosarcoma phyllodes is a rare tumor with malignant potential, although most are benign. It presents as a firm, rubbery mass that may enlarge rapidly. Skin necrosis is usually associated with the tumor. Intraductal papillomas are usually benign but do have malignant potential. They are commonly subareolar and are associated with nipple discharge. These tumors are rare in the adolescent population. Ref: Hay WH (ed): Current Pediatric Diagnosis and Treatment, ed 16. McGraw-Hill, 2003, pp 122-123.

Patients with obstructive sleep apnea have an increased risk for (check one) A. chronic renal failure B. hypertension C. hypokalemia D. hypothyroidism E. sepsis

B Obstructive sleep apnea-hypopnea syndrome is defined as the presence of at least five obstructive events per hour with associated daytime sleepiness. It is present in 2%-4% of the population. The prevalence in men is almost three times that seen in premenopausal women and twice that of postmenopausal women. Other factors associated with an increased prevalence are obesity, older age, and systemic hypertension. Ref: Basner RC: Continuous positive airway pressure for obstructive sleep apnea. N Engl J Med 2007;356(17):1751-1758.

You are evaluating a 28-year-old primigravida for an abnormal Papanicolaou (Pap) test. Which one of the following procedures would be contraindicated? (check one) A. Colposcopy B. Endocervical curettage C. Human papillomavirus (HPV) testing D. Cervical staining E. A cervical biopsy

B Of the choices listed, only endocervical curettage is contraindicated in pregnancy. Colposcopy, cervical biopsy, cervical staining, and HPV testing can all be safely performed during pregnancy. Ref: Apgar BS, Brotzman G: Management of cervical cytologic abnormalities. Am Fam Physician 2004;70(10):1905-1916. 2) Wright TC Jr, Cox JT, Massad LS, et al: 2001 Consensus Guidelines for the management of women with cervical cytological abnormalities. JAMA 2002;287(16):2120-2129.

A 40-year-old female comes to your office with a 1-month history of right heel pain that she describes as sharp, searing, and severe. The pain is worst when she first bears weight on the foot after prolonged sitting and when she gets out of bed in the morning. It gets better with continued walking, but worsens at the end of the day. She does not exercise except for being on her feet all day in the hospital where she works as a floor nurse. She denies any history of trauma. An examination reveals point tenderness to palpation on the plantar surface of the heel at the medial calcaneal tuberosity. Which one of the following should you recommend as first-line treatment? (check one) A. Taping/strapping B. Over-the-counter heel inserts C. Extracorporeal shock wave therapy D. A corticosteroid injection E. A fiberglass walking cast

B Plantar fasciitis is a common cause of heel pain. It may be unilateral or bilateral, and the etiology is unknown, although it is thought to be due to cumulative overload stress. While it may be associated with obesity or overuse, it may also occur in active or inactive patients of all ages. Typically the pain is located in the plantar surface of the heel and is worst when the patient first stands up when getting out of bed in the morning (first step phenomenon) or after prolonged sitting. The pain may then improve after the patient walks around, only to worsen after prolonged walking. The diagnosis is made by history and physical examination. Typical findings include point tenderness to palpation on the plantar surface of the heel at the medial calcaneal tuberosity where the calcaneal aponeurosis inserts. Radiographs are not necessary unless there is a history of trauma or if the diagnosis is unclear. The condition may last for months or years, and resolves in most patients over time with or without specific therapy. One long-term follow-up study showed that 80% of patients had complete resolution of their pain after 4 years. Treatments with limited (level 2) evidence of effectiveness include off-the-shelf insoles, custom-made insoles, stretching of the plantar fascia, corticosteroid iontophoresis, custom-made night splints, and surgery (for those who have failed conservative therapy). NSAIDs and ice, although not independently studied for plantar fasciitis, are included in most studies of other treatments, and are reasonable adjuncts to first-line therapy. Magnetic insoles and extracorporeal shockwave therapy are ineffective in treating plantar fasciitis. Due to their expense, custom-made insoles, custom-made night splints, and corticosteroid iontophoresis should be reserved as second-line treatments for patients who fail first-line treatment. Surgery may be offered if more conservative therapies fail. Corticosteroid injection may have a short-term benefit at 1 month, but is no better than other treatments at 6 months and carries a risk of plantar fascia rupture. Ref: Landorf KB, Keenan AM, Herbert RD: Effectiveness of foot orthoses to treat plantar fasciitis: A randomized trial. Arch Intern Med 2006;166(12):1305-1310.

A 79-year-old male is admitted to the hospital because of a sudden inability to ambulate. He has a past history of gout. On examination his temperature is 38.2°C (100.8°F) and he has bilateral knee effusions. His WBC count is 14,000/mm3 with 82% segs. His serum uric acid level is 8.5 mg/dL (N <6.5). Which one of the following would be most appropriate at this point? (check one) A. 24-hour urine collection for uric acid B. Arthrocentesis C. Initiation of allopurinol D. Initiation of antibiotics E. Initiation of furosemide (Lasix)

B Polyarticular arthritis often presents with fever, knee and other joint effusions, and leukocytosis. A 24-hour urine collection is not routine, is difficult for the patient, and typically does not change therapy. Especially in cases where a joint effusion is accompanied by fever, diagnostic arthrocentesis should be performed to help guide therapy. Allopurinol should not be initiated during an acute gouty attack, but may be started after a patient has recovered. Diuretics increase uric acid levels. Ref: Eggebeen AT: Gout: An update. Am Fam Physician 2007;76(6):801-808.

A nurse who completed a hepatitis B vaccine series a year ago is accidentally stuck by a needle that has just been used on a dialysis patient. The patient is known to be HBsAg-positive. Your first response should be to: (check one) A. Provide reassurance only B. Test the nurse for hepatitis B antibody C. Repeat the hepatitis B vaccine series D. Administer hepatitis B immune globulin (HBIG) only E. Administer HBIG plus a booster of hepatitis B vaccine

B Postexposure prophylaxis after hepatitis B exposure via the percutaneous route depends upon the source of the exposure and the vaccination status of the exposed person. In the case described, a vaccinated person has been exposed to a known positive individual. The exposed person should be tested for hepatitis B antibodies; if antibody levels are inadequate (<10 IU/L by radioimmunoassay, negative by enzyme immunoassay) HBIG should be administered immediately, as well as a hepatitis B vaccine booster dose. An unvaccinated individual in this same setting should receive HBIG immediately (preferably within 24 hours after exposure) followed by the hepatitis B vaccine series (injection in 1 week or less, followed by a second dose in 1 month and a third dose in 6 months). Ref: Updated U.S. Public Health Service guidelines for the management of occupational exposures to HBV, HCV, and HIV and recommendations for postexposure prophylaxis. MMWR 2001;50(RR-11):22.

Outbreaks of dermatitis and folliculitis associated with swimming pools and hot tubs are often caused by which one of the following? (check one) A. Listeria B. Pseudomonas C. Streptococcus D. Shigella E. Staphylococcus

B Pseudomonas organisms have been associated with outbreaks of otitis externa, dermatitis, and folliculitis in persons using swimming pools and hot tubs. Ref: Goldman L, Ausiello D (eds): Cecil Medicine, ed 23. Saunders, 2008, p 2956. 2) Habif TP: Clinical Dermatology: A Color Guide to Diagnosis and Therapy, ed 5. Mosby Elsevier, 2010, pp 363-365.

A 37-year-old female presents with concerns about difficulty initiating and maintaining sleep for the past 3-4 months. She is irritable and feels fatigued and sleepy during the day. After further evaluation, she is diagnosed with chronic insomnia. She asks about alternatives to hypnotic drug treatments. Which one of the following management options is best supported by current evidence? (check one) A. Diphenhydramine (Benadryl) B. Cognitive behavior therapy C. St. Johns wort D. 4 oz of red wine 30 minutes before bedtime E. Vigorous aerobic exercise 30-45 minutes before bedtime

B Routine use of over-the-counter antihistamines should be discouraged because they are only minimally effective in inducing sleep, may reduce sleep quality, and can cause residual drowsiness. Cognitive-behavioral therapy helps change incorrect beliefs and attitudes about sleep (e.g., unrealistic expectations, misconceptions, amplifying consequences of sleeplessness). Techniques include reattribution training (goal setting and planning coping responses), decatastrophizing (balancing anxious automatic thoughts), reappraisal, and attention shifting. Cognitive-behavioral therapy is recommended as an effective, nonpharmacologic treatment for chronic insomnia (SOR A). Many herbs and dietary supplements have been promoted as sleep aids. However, with the exceptions of melatonin and valerian, there is insufficient evidence of benefit. Alcohol acts directly on GABA-gated channels, reducing sleep-onset latency, but it increases wakefulness after sleep onset and suppresses rapid eye movement (REM) sleep. It also has the potential for abuse and should not be used as a sleep aid. Moderate-intensity exercise can improve sleep, but exercising just before bedtime can delay sleep onset. Ref: Ramakrishnan K, Scheid DC: Treatment options for insomnia. Am Fam Physician 2007;76(4):517-526.

Osteoporotic bone loss can be caused or accelerated by prolonged use of which one of the following medications? (check one) A. Hydrochlorothiazide B. Phenytoin C. Raloxifene (Evista) D. Diazepam (Valium) E. Fluoxetine (Prozac)

B Secondary osteoporosis can result from a variety of endocrine, nutritional, or genetic disorders, as well as from prolonged use of certain medications. Anticonvulsants such as phenytoin increase the hepatic metabolism of vitamin D, thereby reducing intestinal calcium absorption. Other medications that adversely affect bone mineral density include glucocorticoids, cyclosporine, phenobarbital, and heparin. Thiazide diuretics reduce urinary calcium loss and are believed to preserve bone density with long-term use. Benzodiazepines and SSRIs have not been associated with increases in bone loss or in hip fractures. Raloxifene, a selective estrogen receptor modulator, is indicated for the prevention and treatment of osteoporosis in postmenopausal women. Ref: Kronenberg H, Melmed S, Polonsky K, et al: Williams Textbook of Endocrinology, ed 11. Saunders, 2008, p 1288.

A 50-year-old Hispanic male has a solitary 5-mm pulmonary nodule on a chest radiograph. His only medical problem is severe osteoarthritis. He quit smoking 10 years ago. Which one of the following would be the most appropriate follow-up for the pulmonary nodule? (check one) A. Positron emission tomography (PET) B. Chest CT C. A repeat chest radiograph in 6 weeks D. A repeat chest radiograph in 6 months

B Solitary pulmonary nodules are common radiologic findings, and the differential diagnosis includes both benign and malignant causes. The American College of Chest Physicians guidelines for evaluation of pulmonary nodules are based on size and patient risk factors for cancer. Lesions ≥8 mm in diameter with a "ground-glass" appearance, an irregular border, and a doubling time of 1 month to 1 year suggest malignancy, but smaller lesions should also be evaluated, especially in a patient with a history of smoking. CT is the imaging modality of choice to reevaluate pulmonary nodules seen on a radiograph (SOR C). PET is an appropriate next step when the cancer pretest probability and imaging results are discordant (SOR C). Patients with notable nodule growth during follow-up should undergo a biopsy (SOR C).

Which one of the following is associated with the use of epidural anesthesia during labor and delivery? (check one) A. A shorter first stage of labor B. A longer second stage of labor C. An increased rate of cesarean delivery D. An increased likelihood of postpartum urinary incontinence

B Studies have shown that epidural analgesia increases the length of both the first and second stage of labor. Although there is an increase in the rate of instrument-assisted delivery and fourth degree laceration, an increase in the rate of cesarean sections has not been shown. An increase in the rate of urinary incontinence also has not been shown. Ref: Leeman L, Fontaine P, King V, et al: The nature and management of labor pain: Part II. Pharmacologic pain relief. Am Fam Physician 2003;68(6):1121-1122.

Which one of the following would suggest that the sudden and unexpected death of a healthy infant resulted from deliberate suffocation rather than sudden infant death syndrome? (check one) A. No previous history of apneic episodes B. An age of 9 months C. Mottled skin D. Clenched fists E. Blood-tinged froth in the mouth

B Sudden infant death syndrome (SIDS) is the most common cause of death during the first 6 months of life in the United States, with a peak incidence at 2-4 months of age and a quick dropoff by the age of 6 months. The cause of death is a retrospective diagnosis of exclusion, and is supported by a history of quiet death during sleep in a previously healthy infant younger than 6 months of age. Evidence of terminal activity may be present, such as clenched fists or a serosanguineous, blood-tinged, or mucoid discharge from the mouth or nose. Lividity and mottling are frequently present in dependent areas. The reported history and autopsy findings of deliberate suffocation may mirror the findings of SIDS, but suffocation should be considered when there is documentation of any of the following: infant age older than 6 months, previous similar sibling deaths, simultaneous twin deaths, or evidence of pulmonary hemorrhage. A history of recurrent apnea or cyanosis has not been causally linked to SIDS; when such reported events have only been witnessed by one caretaker, deliberate suffocation should be suspected. Ref: American Academy of Pediatrics, Hymel KP; Committee on Child Abuse and Neglect; National Association of Medical Examiners: Distinguishing sudden infant death syndrome from child abuse fatalities. Pediatrics 2006;118(1):421-427.

A patient with chronic kidney disease presents with chronic normocytic anemia with a hemoglobin level of 7.8 g/dL. The best outcome is predicted if you raise the hemoglobin level to: (check one) A. 8-10 g/dL B. 10-12 g/dL C. 12-14 g/dL D. >14 g/dL

B The Cardiovascular risk Reduction by Early Anemia Treatment with Epoetin Beta (CREATE) trial, the Correction of Hemoglobin and Outcomes in Renal insufficiency (CHOIR) trial, and the Trial to Reduce Cardiovascular Events with Aranesp Therapy (TREAT) have shown that patients who had hemoglobin levels targeted to normal ranges did worse than patients who had hemoglobin levels of 10-12 g/dL. The incidence of stroke, heart failure, and death increased in patients targeted to normal hemoglobin levels, and there was no demonstrable decrease in cardiovascular events (SOR A). Ref: Taliercio JJ: Anemia and chronic disease: What's the connection? J Fam Pract 2010;59(1):14-18.

In order to be eligible for Medicare hospice benefits, a patient must be entitled to Medicare Part A and: (check one) A. be essentially bedridden B. have a life expectancy of 6 months or less C. have a hematologic or a solid tumor malignancy D. have a caregiver in the home who is present at least 50% of the time E. have documentation of a do-not-resuscitate (DNR) order

B The Medicare Hospice Benefit reimburses hospice providers for the care of terminally ill patients. In order to be eligible for this benefit, patients must be entitled to Medicare Part A and be certified by both the personal physician and the hospice medical director as having a life expectancy of 6 months or less. Services covered include physician services; nursing services; social services; counseling services; physical, occupational, and speech therapy; diagnostic testing; home health aides; homemaker services; and medical supplies. These services may be provided in the patient's home or in the hospital setting. Malignancy, ambulatory status, caregiver availability, and do-not-resuscitate orders are not specifically related to eligibility requirements for this benefit. Ref: Centers for Medicare and Medicaid Services: Medicare Benefit Policy Manual (Internet only). US Dept of Health and Human Services, 2011, rev 141, pub 100-02, chap 9.

A 6-year-old male is brought in for evaluation by his mother, who is concerned that he may have asthma. She reports that he coughs about 3 days out of the week and has a nighttime cough approximately 1 night per week. There is a family history of eczema and allergic rhinitis. Which one of the following would be the preferred initial treatment for this patient? (check one) A. A leukotriene receptor antagonist such as montelukast (Singulair) B. A low-dose inhaled corticosteroid such as budesonide (Pulmicort Turbuhaler) C. A long-acting beta-agonist such as salmeterol (Serevent) D. A mast-cell stabilizer such as cromolyn sodium (Intal)

B The National Asthma Education and Prevention Program (NAEPP) updated its recommendations for the treatment of asthma in 2002. Treatment is based on asthma classification. This child meets the criteria for mild persistent asthma: symptoms more than 2 times per week but less than once a day, symptoms less than 2 nights per month, peak expiratory flow (PEF) or FEV1 >80% of predicted, and a PEF variability of 20%-30%. Asthma controller medications are recommended for all patients with persistent asthma, and the preferred long-term controller treatment in mild persistent asthma is a low-dose inhaled corticosteroid. Cromolyn, leukotriene modifiers, nedocromil, and sustained-release theophylline are alternatives, but are not preferred initial agents. Quick-acting, quick-relief agents such as short-acting beta-agonists are appropriate for prompt reversal of acute airflow obstruction. Ref: Redding GJ, Stoloff SW: Changes in recommended treatments for mild and moderate asthma. J Fam Pract 2004;53(9):692-700.

Which one of the following patients should be advised to take aspirin, 81 mg daily, for the primary prevention of stroke? (check one) A. A 42-year-old male with a history of hypertension B. A 72-year-old female with no chronic medical conditions C. An 80-year-old male with a history of depression D. An 87-year-old female with a history of peptic ulcer disease

B The U.S. Preventive Services Task Force (USPSTF) has summarized the evidence for the use of aspirin in the primary prevention of cardiovascular disease as follows: The USPSTF recommends the use of aspirin for men 45-79 years of age when the potential benefit from a reduction in myocardial infarctions outweighs the potential harm from an increase in gastrointestinal hemorrhage (Grade A recommendation) The USPSTF recommends the use of aspirin for women 55-79 years of age when the potential benefit of a reduction in ischemic strokes outweighs the potential harm of an increase in gastrointestinal hemorrhage (Grade A recommendation) The USPSTF concludes that the current evidence is insufficient to assess the balance of benefits and harms of aspirin for cardiovascular disease prevention in men and women 80 years of age or older (Grade I statement) The USPSTF recommends against the use of aspirin for stroke prevention in women younger than 55 and for myocardial infarction prevention in men younger than 45 (Grade D recommendation) In summary, consistent evidence from randomized clinical trials indicates that aspirin use reduces the risk for cardiovascular disease events in adults without a history of cardiovascular disease. It reduces the risk for myocardial infarction in men, and ischemic stroke in women. Consistent evidence shows that aspirin use increases the risk for gastrointestinal bleeding, and limited evidence shows that aspirin use increases the risk for hemorrhagic strokes. The overall benefit in the reduction of cardiovascular disease events with aspirin use depends on baseline risk and the risk for gastrointestinal bleeding. Ref: US Preventive Services Task Force: Aspirin for the primary prevention of cardiovascular events: An update of the evidence. AHRQ pub no 09-05129-EF-4, 2009. 2) Aspirin for the Prevention of Cardiovascular Disease. US Preventive Services Task Force, 2009.

Current U.S. Preventive Services Task Force recommendations for preventing dental caries in preschool-aged children include which one of the following? (check one) A. Primary care clinicians should prescribe oral fluoride at currently recommended doses to all preschool-aged children B. Oral fluoride is not necessary if the primary water source contains adequate fluoride C. Oral fluoride supplementation should begin at birth when indicated D. Evidence for fluoride supplementation is insufficient to recommend for or against its use

B The U.S. Preventive Services Task Force (USPSTF) recommends that primary care clinicians prescribe oral fluoride supplementation at currently recommended dosages to preschool-aged children older than 6 months of age whose primary water source is deficient in fluoride. The USPSTF concluded that the benefits of caries prevention outweigh the potential harms of dental fluorosis. The USPSTF also concluded that there is insufficient evidence to recommend for or against routine risk assessment of preschool-aged children for dental disease prevention. Ref: U.S. Preventive Services Task Force: Prevention of dental caries in preschool children: Recommendations and rationale. Am Fam Physician 2004;70(8):1529-1532.

According to the U.S. Preventive Services Task Force, which one of the following patients should be screened for an abdominal aortic aneurysm? (check one) A. A 52-year-old male with type 2 diabetes mellitus B. An asymptomatic 67-year-old male smoker with no chronic illness C. A 72-year-old male with a history of chronic renal failure D. A 69-year-old female with a history of coronary artery disease E. A 75-year-old female with hypertension and hypothyroidism

B The U.S. Preventive Services Task Force has released a statement summarizing recommendations for screening for abdominal aortic aneurysm (AAA). The guideline recommends one-time screening with ultrasonography for AAA in men 65-75 years of age who have ever smoked. No recommendation was made for or against screening women. Men with a strong family history of AAA should be counseled about the risks and benefits of screening as they approach 65 years of age. Ref: Upchurch GR, Schaub TA: Abdominal aortic aneurysm. Am Fam Physician 2006;73(7):1198-1204.

A 21-year-old sexually active female presents with acute pelvic pain of several days' duration. A pelvic examination reveals right-sided tenderness and a general fullness in that area. In addition to laboratory testing, you decide to order an imaging study. Which one of the following is the best choice at this time? (check one) A. Transabdominal ultrasonography B. Transvaginal ultrasonography C. Contrast CT of the abdomen and pelvis D. Hysteroscopy E. Hysterosalpingography

B The best initial imaging study for acute pelvic pain in women is transvaginal ultrasonography (SOR C). This provides the greatest level of detail regarding the uterus and adnexae, superior to transcutaneous ultrasonography. CT of the abdomen/pelvis and hysterosalpingography may be indicated eventually in some patients with pelvic pain, but they are not the initial studies of choice. Hysteroscopy is not routinely used in the evaluation of pelvic pain.

A 15 and a half year-old female is brought to your office by her mother. They are concerned because she has not started her periods. She has been healthy, and has grown several inches in the last year. Her height is now 152 cm (60 in) and she weighs 44 kg (98 lb). She started to develop breast buds about a year ago and has scant pubic hair. She denies sexual activity. The mother's menarche occurred at age 15. Which one of the following is true in this case? (check one) A. The patient has delayed puberty and should have her hormone levels evaluated B. The patient will likely start her periods within a year C. Oral contraceptives will be needed to trigger menarche D. A pregnancy test should be performed E. The daughter's age of menarche is unrelated to her mother's age of menarche

B The changes associated with puberty occur in an orderly sequence over a definite time frame. Any deviation from this sequence or time frame should be regarded as abnormal. In girls, pubertal development typically requires 4.5 years. Although generally the first sign of puberty is accelerated growth, breast budding is usually the first recognized pubertal change, followed by the presence of pubic hair, peak growth velocity, and menarche. Girls must have adequate nutrition and reach a critical body weight and body fat percentage before menarche occurs. There is a concordance in the age of menarche in mother-daughter pairs and between sisters. Delayed or interrupted puberty is defined as failure to develop any secondary sex characteristics by age 13, to have menarche by age 16, or to have menarche 5 or more years after the onset of pubertal development. Ref: Berek JS (ed): Novak's Gynecology, ed 13. Lippincott Williams & Wilkins, 2002, pp 805-813.

You are caring for an 88-year-old female nursing-home resident with multiple comorbidities and advanced Alzheimer's disease. The patient has never completed advance directives and no longer has the ability to make decisions. The family inquires about hospice services for this patient. Which one of the following is true regarding this patient and hospice? (check one) A. Nursing-home residents are not eligible for hospice B. The decision to enter hospice care is reversible C. End-stage Alzheimer's disease is not a qualifying diagnosis for hospice D. Failure to complete advance directives by this patient prevents participation in hospice E. The patient must have a life expectancy of less than 4 months to qualify for hospice services

B The decision to utilize the Medicare hospice benefit is reversible, and patients may elect to return to Medicare Part A. Individuals who reside in nursing homes and assisted-living facilities are eligible for the Medicare hospice benefit. Patients with end-stage Alzheimer's disease are eligible for the Medicare hospice benefit if they meet criteria for hospice. If the patient lacks decision-making capacity, a family member or guardian may elect the Medicare hospice benefit for the patient. The patient must be certified by the hospice medical director and primary physician to have a life expectancy of less than 6 months to qualify for hospice services. This requirement is the same whether or not the patient resides in a nursing home.

A 22-year-old male presents to your office with a 2-hour history of a painful right scrotal mass. The physical examination raises concerns that the patient may have testicular torsion. The imaging study of choice would be (check one) A. a plain film B. color duplex Doppler ultrasonography C. CT D. MRI E. a nuclear scan

B The history and physical examination are critical for making a diagnosis in patients with scrotal pain. Transillumination may also be performed as part of the clinical assessment. If the diagnosis is uncertain, ultrasonography with color Doppler imaging has become the accepted standard for evaluation of the acutely swollen scrotum (SOR B). Ultrasonography alone can confirm the diagnosis in a number of conditions, such as hydrocele, spermatocele, and varicocele. For other conditions such as orchitis, carcinoma, or torsion, color Doppler ultrasonography is essential because it will show increased flow in orchitis, normal or increased flow in carcinoma, and decreased blood flow in testicular torsion. For testicular torsion, color Doppler ultrasonography has a sensitivity of 86%-88% and a specificity of 90%-100%. When testicular torsion is strongly suspected, emergent surgical consultation should be obtained before ultrasonography is performed, because surgical exploration as soon as possible is critical to salvaging the testis and should not be delayed for imaging unless the diagnosis is in doubt. While radionuclide imaging would be accurate for diagnosing testicular torsion, it is not used for this purpose because of time limits and lack of easy availability. CT or MRI may be appropriate if ultrasonography indicates a possibility of carcinoma. Plain films are not useful in assessing scrotal swelling or masses.

A 52-year-old male has had a chronic course of multiple vague and exaggerated symptoms for which no cause has been found despite extensive testing. Which one of the following is the most effective management approach for this patient? (check one) A. Reassure the patient that his symptoms are not real B. Schedule the patient for regular appointments every 2-4 weeks C. Prescribe opioids for the pain D. Order additional diagnostic tests E. Advise the patient to go to the emergency department if the symptoms occur after office hours

B The management of somatizing patients can be difficult. One strategy that has been shown to be effective is to schedule regular office visits so that the patient does not need to develop new symptoms in order to receive medical attention. Regular visits have been shown to significantly reduce the cost and chaos of caring for patients with somatization disorder and to help progressively diminish emergency visits and telephone calls. In addition, it is important to describe the patient's diagnosis with compassion and avoid suggesting that it's "all in your head." Continued diagnostic testing and referrals in the absence of new symptoms or findings is unwarranted. Visits to the emergency department often result in inconsistent care and mixed messages from physicians who are seeing the patient for the first time, and unnecessary and often repetitive tests may be ordered. Opiates have significant side effects such as constipation, sedation, impaired cognition, and risk of addiction. Ref: Servan-Schreiber D, Tabas G, Kolb R: Somatizing patients: Part II. Practical management. Am Fam Physician 2000;61(5):1423-1428, 1431-1432. 2) Hull SK, Broquet K: How to manage difficult patient encounters. Fam Pract Manag 2007;14(6):30- 34.

The probability of pregnancy after unprotected intercourse is the highest at which one of the following times? (check one) A. 3 days before ovulation B. 1 day before ovulation C. The day of ovulation D. 1 day after ovulation E. 3 days after ovulation

B There is a 30% probability of pregnancy resulting from unprotected intercourse 1 or 2 days before ovulation, 15% 3 days before, 12% the day of ovulation, and essentially 0% 1-2 days after ovulation. Knowing the time of ovulation therefore has implications not only for "natural" family planning, but also for decisions regarding postcoital contraception. Ref: Emergency contraception OTC. Med Lett Drugs Ther 2004;46(1175):10.

A 36-year-old member of the National Guard who has just returned from Iraq consults you because of several "boils" on the back of his neck that have failed to heal over the last 6 months, despite two week-long courses of cephalexin (Keflex). You observe three 1- to 2-cm raised minimally tender lesions with central ulceration and crust formation. He denies any fever or systemic symptoms. The most likely cause of these lesions is: (check one) A. Pyogenic granuloma B. Leishmaniasis C. Atypical mycobacterial infection D. Squamous cell carcinoma E. Epidermal inclusion cysts

B The most likely diagnosis is cutaneous leishmaniasis, caused by an intracellular parasite transmitted by the bite of small sandflies. Lesions develop gradually, and are often misdiagnosed as folliculitis or as infected epidermal inclusion cysts, but they fail to respond to usual skin antibiotics. Hundreds of cases have been diagnosed in troops returning from Iraq, most due to Leishmania major. Treatment is not always required, as most lesions will resolve over several months; however, scarring is frequent. U.S. military medical facilities and the CDC are coordinating treatment when indicated with sodium stibogluconate. Family physicians can play a key role in correctly identifying these lesions. Ref: Markle WH, Makhoul K: Cutaneous leishmaniasis: Recognition and treatment. Am Fam Physician 2004;69(6):1455-1460. 2) Update: Cutaneous leishmaniasis in U.S. military personnel—Southwest/Central Asia, 2002-2004. MMWR 2004;53(12):264-265.

In patients with breast cancer, the most reliable predictor of survival is (check one) A. estrogen receptor status B. cancer stage at the time of diagnosis C. tumor grade D. histologic type E. lymphatic or blood vessel involvement

B The most reliable predictor of survival in breast cancer is the stage at the time of diagnosis. Tumor size and lymph node involvement are the main factors to take into account. Other prognostic parameters (tumor grade, histologic type, and lymphatic or blood vessel involvement) have been proposed as important variables, but most microscopic findings other than lymph node involvement correlate poorly with prognosis. Estrogen receptor (ER) status may also predict survival, with ER-positive tumors appearing to be less aggressive than ER-negative tumors. Ref: Abeloff MD, Armitage JO, Niederhuber JE, et al (eds): Clinical Oncology, ed 3. Elsevier Churchill Livingstone, 2004, pp 2399-2401.

The best drug treatment for symptomatic mitral valve prolapse is: (check one) A. quinidine B. propranolol (Inderal) C. digoxin D. procainamide E. phenytoin (Dilantin)

B The primary treatment for symptomatic mitral valve prolapse is β-blockers. Quinidine and digoxin were used to treat this problem in the past, especially if sinus bradycardia or cardiac arrest occurred with administration of propranolol. Procainamide and phenytoin have not been used to treat this syndrome. Asymptomatic patients require only routine monitoring, while those with significant mitral regurgitation may require surgery. Some patients with palpitations can be managed with lifestyle changes such as elimination of caffeine and alcohol. Orthostatic hypotension can often be managed with volume expansion, such as by increasing salt intake.

The Current Procedural Terminology (CPT) code to document a patient encounter can be determined by the amount of face-to-face time spent with the patient when (check one) A. the office visit exceeds 1 hour B. counseling or coordinating care accounts for more than 50% of the face-to-face time spent with the patient C. the intensity of the visit supports a higher code than is supported by the level of history, examination, and medical decision making D. a comprehensive history and examination or complex medical decision making is conducted E. considerable time is spent outside the office visit coordinating prior authorizations or referrals on behalf of the patient

B The times published for Current Procedural Terminology codes are typical for each level of office visit, but there is no requirement associated with them unless counseling or coordination of care accounts for more than 50% of the face-to-face time of the encounter. In such cases physicians are able to code on the basis of time. Ref: Hughes C: Coding and documentation. Fam Pract Manage 2006;13(10):27.

A 5-month-old female is brought in with a 1-day history of an axillary temperature of 100.6°F and mild irritability. Findings are normal on examination except for a runny nose and a moderately distorted, immobile, red right eardrum. There is no history of recent illness or otitis in the past. The most appropriate management would be: (check one) A. azithromycin (Zithromax) for 5 days B. amoxicillin for 10 days C. amoxicillin for 5 days D. oral decongestants E. observation and a repeat examination in 2 weeks

B The treatment for otitis media is evolving. Recommendations by the American Academy of Family Physicians and the American Academy of Pediatrics advocate a 10-day course of antibiotics for children under the age of 2 years if the diagnosis is certain. If the diagnosis is not certain and the illness is not severe, there is an option of observation with follow-up. For children over the age of 2 years, the recommendation is still to treat if the diagnosis is certain, but there is an option of observation and follow-up if the illness is not severe and follow-up can be guaranteed. Amoxicillin is the first-line therapy; the recommended dosage is 80-90 mg/kg/day in two divided doses, which increases the concentration of amoxicillin in the middle ear fluid to help with resistant Pneumococcus. Azithromycin, because of a broader spectrum and potential for causing resistance, is not considered the treatment of first choice. Treatment regimens ranging from 5 to 7 days are appropriate for selected children over the age of 5 years. Oral decongestants and antihistamines are not recommended for children with acute otitis media.

A 35-year-old African-American female has just returned home from a vacation in Hawaii. She presents to your office with a swollen left lower extremity. She has no previous history of similar problems. Homan's sign is positive, and ultrasonography reveals a noncompressible vein in the left popliteal fossa extending distally. Which one of the following is true in this situation? (check one) A. Monotherapy with an initial 10-mg loading dose of warfarin (Coumadin) would be appropriate B. Enoxaparin (Lovenox) should be administered at a dosage of 1 mg/kg subcutaneously twice a day C. The incidence of thrombocytopenia is the same with low-molecular-weight heparin as with unfractionated heparin D. The dosage of warfarin should be adjusted to maintain the INR at 2.5-3.5 E. Anticoagulant therapy should be started as soon as possible and maintained for 1 year to prevent deep vein thrombosis (DVT) recurrence

B The use of low-molecular-weight heparin allows patients with acute deep vein thrombosis (DVT) to be managed as outpatients. The dosage is 1 mg/kg subcutaneously twice daily. Patients chosen for outpatient care should have good cardiopulmonary reserve, normal renal function, and no risk for excessive bleeding. Oral anticoagulation with warfarin can be initiated on the first day of treatment after heparin loading is completed. Monotherapy with warfarin is inappropriate. The incidence of thrombocytopenia with low-molecular-weight heparin is lower than with conventional heparin. The INR should be maintained at 2.0-3.0 in this patient. The 2.5-3.5 range is used for patients with mechanical heart valves. The therapeutic INR should be maintained for 3-6 months in a patient with a first DVT related to travel. Ref: Weismantel D: Treatment of the patient with deep vein thrombosis. J Fam Pract 2001;50(3):249-256. 2) Kasper DL, Braunwald E, Fauci AS, et al (eds): Harrison's Principles of Internal Medicine, ed 16. McGraw-Hill, 2005, pp 570, 1491-1429.

A 3-year-old male is brought to your office by his parents because they are concerned about three "spells" he has had in the past month. In each case, the child started crying when he was prevented by a parent from doing something he wished to do. While crying, he suddenly stopped breathing and his face and lips began to turn blue. After 30-45 seconds he resumed crying, his color returned to normal, and he showed no evidence of impairment. A physical examination today is normal and the child is developmentally appropriate for his age. A recent hemoglobin level was in the normal range. Which one of the following should you do now? (check one) A. Teach the parents age-appropriate disciplinary procedures to implement when the child behaves in this manner B. Reassure the parents that this is a benign condition and will resolve as the child gets older C. Order an EEG D. Obtain appropriate laboratory studies to confirm the most likely diagnosis E. Initiate treatment with valproic acid (Depakene)

B This child is experiencing simple breath-holding spells, a relatively common and benign condition that usually begins in children between the ages of 6 months and 6 years. The cause is uncertain but seems to be related to overactivity of the autonomic nervous system in association with emotions such as fear, anger, and frustration. The episodes are self-limited and may be associated with pallor, cyanosis, and loss of conciousness if prolonged. There may be an association with iron deficiency anemia, but this child had a recent normal hemoglobin level. These events are not volitional, so disciplinary methods are neither effective nor warranted. While children may experience a loss of consciousness and even exhibit some twitching behavior, the episodes are not seizures so neither EEG evaluation nor anticonvulsant therapy is indicated. No additional laboratory studies are indicated. Parents should be reassured that the episodes are benign and will resolve without treatment. Ref: Rudolph CD, Rudolph AM, Lister G, et al (eds): Rudolph's Pediatrics, ed 22. McGraw-Hill, 2011, p 2223.

A 20-year-old white male presents to your office after a fall on an outstretched hand while skateboarding. He has pain at the anatomic snuffbox with no abrasion. Radiographs are negative. Which one of the following would be the most appropriate management? (check one) A. A long arm cast for 8 weeks B. A thumb spica splint and follow-up radiographs in 2 weeks C. A sugar tong splint and follow-up radiographs in 2 weeks D. An Ace bandage and follow-up radiographs in 2 weeks E. An Ace bandage and follow-up in 2 weeks if the patient is still

B This is a classic presentation of a possible scaphoid fracture. This fracture is important to diagnose and treat appropriately because of a high rate of non-union. If radiographs are negative, the patient should be placed in a thumb spica splint and have repeat radiographs in 2 weeks, because initial studies may be negative. An Ace bandage or a sugar tong splint would be inappropriate because they do not immobilize the thumb. A long arm cast for 8 weeks would immobilize the thumb, but could lead to loss of function, and may overtreat the injury if it is not truly a scaphoid fracture. Ref: Daniels J, Zook E, Lynch J: Hand and wrist injuries. Part I. Nonemergent evaluation. Am Fam Physician 2003;69(8):1941-1948.

A 30-year-old white male complains of several weeks of nasal stuffiness, purulent nasal discharge, and facial pain. He does not respond to a 3-day course of trimethoprim/sulfamethoxazole (Bactrim, Septra). Follow-up treatment with 2 weeks of amoxicillin/clavulanate (Augmentin) is similarly ineffective. Of the following diagnostic options, which one is most appropriate at this time? (check one) A. Pulmonary function testing B. Coronal CT of the sinuses C. Culture and sensitivity testing of the discharge D. Erythrocyte sedimentation rate

B This patient has a clinical presentation consistent with acute sinusitis. Failure to respond to adequate antibiotic therapy suggests either a complication, progression to chronic sinusitis, or a different, confounding diagnosis. The diagnostic procedure of choice in this situation is coronal CT of the sinuses, due to its increased sensitivity and competitive cost when compared with standard radiographs. Cultures of the nasal discharge give unreliable results because of bacterial contamination from the resident flora of the nose. The other options listed do not contribute to the diagnosis and treatment of sinusitis. Ref: Mandell GL, Bennett JE, Dolin R (eds): Mandell, Douglas, and Bennett's Principles and Practice of Infectious Diseases, ed 6. Churchill Livingstone, 2005, pp 772-782.

A 60-year-old female is admitted to the hospital with pneumonia 1 week after her discharge following elective colorectal surgery. Her initial stay was 5 days and she had no complications. She had no signs of infection until 2 days ago when she developed a temperature of 39.1°C (102.4°F), a cough with yellow sputum, and hypoxia. She has no abdominal pain or diarrhea. Her pulse rate is slightly elevated to 96 beats/min, and her blood pressure is unchanged from baseline. A chest radiograph confirms a left lingular infiltrate. Methicillin-resistant Staphylococcus pneumonia is rare in this institution. Of the following antibiotic regimens, which one would be the best initial treatment for this patient? (check one) A. Ampicillin/sulbactam (Unasyn) B. Ceftazidime sodium (Fortaz) and gentamicin C. Ceftriaxone (Rocephin) and azithromycin (Zithromax) D. Clarithromycin (Biaxin) E. Levofloxacin (Levaquin)

B This patient has a significant pneumonia that requires the initiation of empiric antibiotics. It is important to remember that because this patient was recently in the hospital, the usual coverage for community-acquired pneumonia is not adequate. Health care-associated pneumonia is more likely to involve severe pathogens such as Pseudomonas aeruginosa, Klebsiella pneumoniae, and Acinetobacter species. Methicillin-resistant Staphylococcus aureus also is a consideration, depending on local prevalence. Of the antibiotic regimens listed, ceftazidime and gentamicin is the only choice that covers these organisms. Ref: American Thoracic Society, Infectious Diseases Society of America: Guidelines for the management of adults with hospital-acquired, ventilator-associated, and healthcare-associated pneumonia. Am J Respir Crit Care Med 2005;171(4):388-416.

A 67-year-old female presents with progressive weakness, dry skin, lethargy, slow speech, and eyelid edema. Of the following medications currently taken by this patient, which one is most likely to be causing her symptoms? (check one) A. Donepezil (Aricept) B. Lithium C. Lisinopril (Prinivil, Zestril) D. Alendronate (Fosamax) E. Glyburide (DiaBeta, Micronase)

B This patient has classic signs of hypothyroidism. Of the drugs listed, only lithium is associated with the development of hypothyroidism. In patients taking lithium, it is recommended that in addition to regular serum lithium levels, thyroid function tests including total free T4, and TSH be obtained yearly. ref: Griswold KS, Pessar LF: Management of bipolar disorder. Am Fam Physician 2000;62(6):1343-1353, 1357-1358. 2) Kronenberg HM, Melmed S, Polonsky KS, et al(eds): Williams Textbook of Endocrinology, ed 11. Saunders Elsevier, 2008, pp 392-393.

A 70-year-old African-American male undergoes routine sigmoidoscopy. He has a long history of constipation, hypertension, and diet-controlled type 2 diabetes mellitus. The examination reveals brown to black leopard spotting of the colonic mucosa. You would now: (check one) A. perform a metastatic workup B. review his medications C. prescribe oral corticosteroids D. prescribe antibiotics and a proton pump inhibitor E. check his stool for Clostridium difficile

B This patient has melanosis coli, which is a benign condition resulting from abuse of anthraquinone laxatives such as cascara, senna, or aloe. The condition resolves with discontinuation of the medication. Ref: Rakel RE, Bope ET, Kellerman R (eds): Conn's Current Therapy 2010. Saunders Elsevier, 2010, p 19.

A 42-year-old female presents with a 2-day history of chest pain. She describes the pain as sharp, located in the right upper chest, and worsened by deep breathing or coughing. She also complains of shortness of breath. She was previously healthy and has no recent history of travel. Her vital signs are normal. A pleural friction rub is noted on auscultation of the lungs. The remainder of the examination is normal. An EKG, cardiac enzymes, oxygen saturation, and a D-dimer level are all normal. Which one of the following would be most appropriate at this point? (check one) A. No further testing B. A chest radiograph C. An antinuclear antibody test D. Echocardiography E. Pulmonary angiography

B This patient has pleurisy. Patients presenting with pleuritic chest pain may have life-threatening disorders, and pulmonary embolism, acute myocardial infarction, and pneumothorax should be excluded. While 5%-20% of patients with pulmonary embolism present with pleuritic chest pain, this patient has no risks for pulmonary embolism and the normal D-dimer level obviates the need for further evaluation. Moderate- to high-risk patients may need a helical CT scan or other diagnostic testing. An EKG and chest radiograph are recommended in the evaluation of acute/subacute pleuritic chest pain. The chest radiograph will exclude pneumothorax, pleural effusion, or pneumonia. An echocardiogram would not be indicated if the cardiac examination and EKG are normal. An antinuclear antibody level could be considered in recurrent pleurisy or if other symptoms or signs of lupus were present, but it would not be indicated in this patient. Most cases of acute pleurisy are viral and should be treated with NSAIDs unless the workup indicates another problem. Ref: Kass SM, Williams PM, Reamy BV: Pleurisy. Am Fam Physician 2007;75(9):1357-1364.

A 70-year-old male sees you because of slowly increasing problems with COPD. He has had frequent exacerbations requiring emergency department visits. He currently uses a tiotropium (Spiriva) inhaler once a day, as well as an albuterol (Proventil) inhaler, 2 puffs 4 times a day as needed. An examination shows decreased breath sounds throughout, and an oxygen saturation of 92%. Spirometry shows he has severe COPD (stage III); his FEV1/FVC ratio is 65% of predicted and his FEV1 is 45% of predicted. The most reasonable change in treatment would be to add (check one) A. oxygen, 2 L/min while sleeping B. inhaled fluticasone (Flovent), 2 puffs twice daily C. oral low-dose prednisone daily D. oral theophylline (TheoDur, Uniphyl) twice daily

B This patient is suffering from severe COPD (stage III) and has a history of frequent exacerbations. The addition of a corticosteroid inhaler for patients with severe disease has been found to significantly decrease the number of exacerbations, but has no effect on overall mortality. Side effects of oral candidiasis and easy bruising of the skin are increased. Continuous oxygen has been shown to improve overall mortality and endurance in patients with an oxygen saturation of 88% or less, but has not been shown to improve quality of life in those with mild hypoxemia or if used only at night. Oral prednisone has been shown to be effective when used to treat acute exacerbations, but when used on a chronic basis it is no more effective than corticosteroid inhalers. Chronic oral prednisone is also associated with significant side effects, and therefore is not generally recommended. Oral theophylline has not been shown to be of benefit in either preventing exacerbations or improving quality of life, and has significant side effects of gastrointestinal toxicity, seizures, and arrhythmias. It should be reserved for carefully selected patients only. Ref: Grimes GC, Manning JL, Patel P, et al: Medications for COPD: A review of effectiveness. Am Fam Physician 2007;76(8):1141-1148. 2) Global Initiative for Chronic Obstructive Lung Disease, Global Strategy for the Diagnosis, Management, and Prevention of Chronic Obstructive Pulmonary Disease, 2007.

A 31-year-old healthy female is admitted to the hospital from the emergency department after presenting with aching in her right shoulder and swelling in the ipsilateral forearm and hand. The only precipitating event that she can recall is digging strenuously in the back yard to put in a new garden. Ultrasonography is remarkable for a thrombus in the axillosubclavian vein. She has no prior history of clotting, takes no medications, and has no previous history of medical or surgical procedures involving this extremity. The most likely etiology for this patient's condition is (check one) A. a hypercoagulable state B. a compressive anomaly in the thoracic outlet C. use of injection drugs D. Budd-Chiari syndrome

B Thrombosis of the upper extremity accounts for about 10% of all venous thromboembolism (VTE) cases. However, axillosubclavian vein thrombosis (ASVT) is becoming more frequent with the increased use of indwelling subclavian vein catheters. Spontaneous ASVT (not catheter related) is seen most commonly in young, healthy individuals. The most common associated etiologic factor is the presence of a compressive anomaly in the thoracic outlet. These anomalies are often bilateral, and the other upper extremity at similar risk for thrombosis. While a hypercoagulable state also may contribute to the thrombosis, it is much less common. Budd-Chiari syndrome refers to thrombosis in the intrahepatic, suprahepatic, or hepatic veins. It is not commonly associated with spontaneous upper-extremity thrombosis. Ref: Joffe HV, Goldhaber SZ: Upper-extremity deep vein thrombosis. Circulation 2002;106(14):1874-1880. 2) Marx JA (ed): Rosens Emergency Medicine: Concepts and Clinical Practice, ed 6. Mosby Elsevier, 2006, pp 1358-1360.

A 75-year-old male consults you after his family expresses concern about his loss of interest in his usual activities. They believe he has become increasingly withdrawn since the death of his wife 8 months earlier. You note he has lost 8 kg (18 lb) since his last office visit 6 months earlier. He does not drink alcohol. His physical examination is unremarkable for his age except for a blood pressure of 105/70 mm Hg. Detailed laboratory studies, including thyroid function tests, are all within normal limits. He tells you he would be fine if he could just get some sleep. His Mini-Mental State Examination is normal, but he is obviously clinically depressed. The most appropriate medication for his depression would be: (check one) A. trazodone (Oleptro) B. mirtazapine (Remeron) C. bupropion (Wellbutrin) D. amitriptyline E. nortriptyline (Pamelor)

B Trazodone may be useful for insomnia, but is not recommended as a primary antidepressant because it causes sedation and orthostatic hypotension at therapeutic doses. Bupropion would aggravate this patient's insomnia. Tricyclic antidepressants may be effective, but are no longer considered first-line treatments because of side effects and because they can be cardiotoxic. Mirtazapine has serotonergic and noradrenergic properties and is associated with increased appetite and weight gain. It may be particularly useful for patients with insomnia and weight loss.

Two doses of varicella vaccine are recommended for: (check one) A. adults under 60 years of age who develop shingles B. all children with normal immune status C. only immunocompromised individuals D. only children between 12 months and 13 years of age

B Two doses of varicella vaccine are recommended for all children unless they are immunocompromised, in which case they should not be immunized against varicella, or with other live-virus vaccines. Shingles is evidence of prior varicella infection and is a reason not to vaccinate with varicella vaccine.

A 28-year-old primigravida is at 20 weeks' gestation by dates but her fundal height is consistent with a 26-week gestation. She has had episodes of vomiting during the pregnancy that were more severe than the physiologic vomiting typically seen in pregnancy. A sonogram performed at about 5 weeks' gestation for vaginal bleeding was normal and showed a single fetus. Which one of the following would be most appropriate at this point? (check one) A. A serum hCG level B. A repeat sonogram C. MRI of the pelvis D. Expectant management

B Ultrasonography is the initial test of choice for evaluating the possibility of multiple gestation. It should be done if uterine size is larger than expected, or if pregnancy-associated symptoms are excessive. It should also be done in women who received fertility treatment. An initial sonogram that shows a single pregnancy does not rule out multiple gestation. In one study, 30 of 220 twin pregnancies had an original sonogram which showed a single pregnancy. Serum hCG and MRI would not be indicated at this stage in the evaluation. Ref: Egan JFX, Borgida AF: Multiple gestations: The importance of ultrasound. Obstet Gynecol Clin North Am 2004;31(1):141-158.

Good evidence supports which one of the following measures for reducing the risk of postoperative pulmonary complications after major abdominal surgery? (check one) A. Avoidance of preoperative systemic corticosteroids B. Use of an incentive spirometer C. Antibiotic prophylaxis for 48 hours D. Postoperative total parenteral nutrition

B Use of an incentive spirometer and similar lung expansion techniques such as chest physiotherapy have been shown to have significant benefit for reducing postoperative complications. A course of preoperative corticosteroids has been shown to be beneficial for patients with COPD. Prolonged antibiotic prophylaxis and postoperative total parenteral nutrition have not been shown to have any benefit. Ref: Smetana GW: A 68-year-old man with COPD contemplating colon cancer surgery. JAMA 2007;297(19):2121-2130.

Uterine rupture is a potential complication of attempted vaginal birth after cesarean (VBAC). The most reliable indication that uterine rupture may have occurred is: (check one) A. Cessation of uterine contractions during active labor B. Fetal bradycardia C. Vaginal bleeding D. Sudden lower abdominal pain E. Maternal hypotension

B Uterine rupture occurs in 0.2%-1.0% of women in labor after one previous low transverse cesarean section. Obviously, this can have devastating consequences for the mother and baby, so vigilance during labor is paramount. Uterine pain, cessation of contractions, vaginal bleeding, failure of labor to progress, or fetal regression may occur, but none of these are as consistent as fetal bradycardia in cases of uterine rupture during labor for VBAC patients. Ref: Toppenberg KS, Block WA Jr: Uterine rupture: What family physicians need to know. Am Fam Physician 2002;66(5):823-828.

A 25-year-old female sees you in the office for follow-up after a visit to the emergency department for respiratory distress. She complains of several episodes of an acute onset of shortness of breath, wheezing, coughing, and a choking sensation, without any obvious precipitant. She has been on inhaled corticosteroids for 2 months without any improvement in her symptoms. Albuterol (Proventil, Ventolin) does not consistently relieve her symptoms. She is asymptomatic today. Spirometry shows a normal FEV1 , a normal FVC and FEV1 /FVC ratio, and a flattened inspiratory loop. The most likely diagnosis is: (check one) A. globus hystericus B. vocal cord dysfunction C. asthma D. anaphylaxis E. COPD

B Vocal cord dysfunction is an idiopathic disorder commonly seen in patients in their twenties and thirties in which the vocal cords partially collapse or close on inspiration. It mimics, and is commonly mistaken for, asthma. Symptoms include episodic tightness of the throat, a choking sensation, shortness of breath, and coughing. A careful history and examination reveal that the symptoms are worse with inspiration than with exhalation, and inspiratory stridor during the episode may be mistaken for the wheezing of asthma. The sensation of throat tightening or choking also helps to differentiate it from asthma. Pulmonary function tests (PFTs) are normal, with the exception of flattening of the inspiratory loop, which is diagnostic of extra-thoracic airway compression. Fiberoptic laryngoscopy shows paradoxical inspiratory and/or expiratory partial closure of the vocal cords. Vocal cord dysfunction is treated with speech therapy, breathing techniques, reassurance, and breathing a helium-oxygen mixture (heliox). PFTs in patients with asthma are normal between exacerbations, but when symptoms are present the FEV1 /FVC ratio is reduced, as with COPD. With anaphylaxis, there will typically be itching or urticaria and signs of angioedema, such as lip or tongue swelling, in response to a trigger such as food or medication; PFTs are normal when anaphylaxis symptoms are absent. Globus hystericus is a type of conversion disorder in which emotional stress causes a subjective sensation of pain or tightness in the throat, and/or dysphagia; diagnostic tests such as spirometry and laryngoscopy are normal. Ref: Deckert J, Deckert L: Vocal cord dysfunction. Am Fam Physician 2010;81(2):156-159.

Which one of the following is associated with the use of percutaneous endoscopic gastrostomy (PEG) tubes? (check one) A. A reduced risk of aspiration pneumonia in patients with dysphagia B. Increased use of restraints C. Improved nutritional status in nursing-home residents with dementia D. Improved quality of life for patients with dementia

B When a patient or nursing-home resident is losing weight or has suffered an acute change in the ability to perform activities of daily living, a decision must be made as to whether or not to place a PEG tube to provide artificial nutrition. Studies have shown that PEG tubes do not improve nutritional status or quality of life for residents with dementia, nor do they decrease the risk of aspiration pneumonia, although aspiration risk may possibly be decreased if the feeding tube is placed below the gastroduodenal junction (SOR B). Feeding tubes can also cause discomfort and agitation, leading to an increased use of restraints (SOR B). Ref: Unwin BK, Porvaznik M: Nursing home care: Part II. Clinical aspects. Am Fam Physician 2010;81(10):1229-1237.

A 4-year-old is brought to the emergency department with abdominal pain and is noted to have 3+ proteinuria on a dipstick. Three days later the pain has resolved spontaneously, and a repeat urinalysis in your office shows 2+ proteinuria with normal findings on microscopic examination. A metabolic panel, including creatinine and total protein, is also normal. Which one of the following would be most appropriate at this point? (check one) A. Renal ultrasonography B. A spot first morning urine protein/creatinine ratio C. An antinuclear antibody and complement panel D. Referral to a nephrologist

B When proteinuria is noted on a dipstick and the history, examination, full urinalysis, and serum studies suggest no obvious underlying problem or renal insufficiency, a urine protein/creatinine ratio is recommended. This test correlates well with 24-hour urine protein, which is particularly difficult to collect in a younger patient. Renal ultrasonography is appropriate once renal insufficiency or nephritis is established. If pathogenic proteinuria is confirmed, an antinuclear antibody and/or complement panel may be indicated. A nephrology referral is not necessary until the presence of kidney disease or proteinuria from a cause other than benign postural proteinuria is confirmed.

A 19-year-old white female presents for an initial family planning evaluation. Specifically, she is interested in oral contraception. She is not presently sexually active, but has a steady boyfriend. She has no contraindications to oral contraceptive use. She has mild acne vulgaris. You discuss possible side effects and benefits of combined oral contraceptives, including improvement of her acne. Which one of the following is also associated with oral contraceptive use? (check one) A. Increased risk of ovarian cancer B. Decreased risk of ovarian cysts C. Increased risk for ectopic pregnancy D. Increased incidence of dysmenorrhea

B Women who take combination oral contraceptives have a reduced risk of both ovarian and endometrial cancer. This benefit is detectable within a year of use and appears to persist for years after discontinuation. Other benefits include a reduction in dysfunctional uterine bleeding and dysmenorrhea; a lower incidence of ovarian cysts, ectopic pregnancy and benign breast disease; and an increase in hemoglobin concentration. Many women also benefit from the convenience of menstrual regularity. All combination oral contraceptives raise sex hormone-binding globulin and decrease free testosterone concentrations, which can lead to improvement in acne. Ref: Choice of contraceptives. Treat Guidel Med Lett 2004;2(24):55-62.

A 36-hour-old male is noted to have jaundice extending to the abdomen. He is breastfeeding well, 10 times a day, and is voiding and passing meconium-stained stool. He was born by normal spontaneous vaginal delivery at 38 weeks gestation after an uncomplicated pregnancy. The mother's blood type is A positive with a negative antibody screen. The infants total serum bilirubin is 13.0 mg/dL. Which one of the following would be the most appropriate management of this infants jaundice? (check one) A. Continue breastfeeding and supplement with water or dextrose in water to prevent dehydration B. Continue breastfeeding, evaluate for risk factors, and initiate phototherapy if at risk C. Discontinue breastfeeding and supplement with formula until the jaundice resolves D. Discontinue breastfeeding and supplement with formula until total serum bilirubin levels begin to decrease

B n 2004 the American Academy of Pediatrics published updated clinical practice guidelines on the management of hyperbilirubinemia in the newborn infant at 35 or more weeks gestation. These guidelines focus on frequent clinical assessment of jaundice, and treatment based on the total serum bilirubin level, the infants age in hours, and risk factors. Phototherapy should not be started based solely on the total serum bilirubin level. The guidelines encourage breastfeeding 8-12 times daily in the first few days of life to prevent dehydration. There is no evidence to support supplementation with water or dextrose in water in a nondehydrated breastfeeding infant. This infant is not dehydrated and is getting an adequate number of feedings, and there is no reason to discontinue breastfeeding at this time. Ref: American Academy of Pediatrics Subcommittee on Hyperbilirubinemia: Management of hyperbilirubinemia in the newborn infant 35 or more weeks of gestation. Pediatrics 2004;114(1):297-316.

A 62-year-old male presents for surgical clearance prior to transurethral resection of the prostate. His past history is significant for a pulmonary embolus after a cholecystectomy 15 years ago. His examination is unremarkable except that he is 23 kg (50 lb) overweight. The most appropriate recommendation to the urologist would be to: (check one) A. Cancel the surgery indefinitely B. Place the patient on 650 mg of aspirin daily prior to surgery C. Start the patient on subcutaneous enoxaparin (Lovenox), 40 mg 1-2 hr prior to surgery and once a day after surgery D. Start warfarin (Coumadin) after surgery with a goal INR of 1.5 E. Start intravenous heparin according to a weight-based protocol 24 hours after surgery

C A patient with a past history of postoperative venous thromboembolism is at risk for similar events with subsequent major operations. The most appropriate treatment of the choices listed would be subcutaneous enoxaparin. Aspirin is ineffective for prophylaxis of venous thromboembolism. Warfarin is effective at an INR of 2.0-3.0. Full anticoagulation with heparin is unnecessary for prophylaxis and can result in a higher rate of postoperative hemorrhage. Ref: Ramzi DW, Leeper KV: DVT and pulmonary embolism: Part II. Treatment and prevention. Am Fam Physician 2004;69(12):2841-2848.

A 75-year-old white male suffers an anteroseptal myocardial infarction. Four hours after admission to the hospital his blood pressure is 65/40 mm Hg. A Swan-Ganz catheter is inserted into the pulmonary artery, and the pulmonary capillary wedge pressure is found to be 8 mm Hg. The best therapy in this instance is: (check one) A. infusion of dopamine B. infusion of 5% dextrose C. infusion of normal saline D. digoxin E. furosemide (Lasix)

C A pulmonary capillary wedge pressure of 8 mm Hg suggests hypovolemia. Normal saline should be given because 5% dextrose is not a reliable volume expander.

An 83-year-old female presents with pain, swelling, and erythema of her left knee. She first noticed this problem last night before going to bed. She is generally healthy and takes no medications. She has not been sexually active since being widowed 15 years ago, and she currently lives with her sister. She states that she developed pain and swelling in her left ankle 2 years ago that lasted only a couple of days and resolved spontaneously. Blood testing shows a very elevated erythrocyte sedimentation rate but a normal rheumatoid factor and uric acid level. Which one of the following is the most likely diagnosis? (check one) A. Gonococcal arthritis B. Gout C. Pseudogout (calcium pyrophosphate disease) D. Rheumatoid arthritis E. Rupture of the anterior cruciate ligament

C Acute monoarthritis in adults is most commonly caused by infection, trauma, or crystal deposition. Rheumatoid arthritis seldom presents as monoarthritis, and more often has a subacute course with multiple, symmetric joints involved. Although osteoporosis may result in a fracture of the knee joint without trauma at this age, there is no reason to believe that this patient has a torn anterior cruciate ligament. Gonococcal arthritis is one of the most common causes of septic arthritis, but is highly unlikely in this elderly, sexually inactive patient. Nongonococcal septic arthritis (especially due to staphylococcal and streptococcal bacteria) is still a consideration and should be ruled out by aspiration of fluid to be sent for culture. This patient's presentation is most consistent with pseudogout. Having a normal uric acid level suggests against gout, but does not rule it out. Also, gout is seven times more likely to be seen in males, whereas pseudogout is 1.5 times more frequent in females. Pseudogout most often affects the elderly, and usually affects the knee, wrist, and ankle. Gout presents most commonly in the first metatarsophalangeal joint and insteps of the feet, but also can occur in the knee, wrist, finger, and olecranon bursa. Differentiating between gout and pseudogout can be difficult and is best done by analysis of joint fluid. In patients with gout, this fluid contains highly negative birefringent, needle-shaped urate crystals, whereas in pseudogout the fluid contains rhomboid-shaped, weakly positive birefringent calcium pyrophosphate crystals. Ref: Siva C, Velazquez C, Mody A, et al: Diagnosing acute monoarthritis in adults: A practical approach for the family physician. Am Fam Physician 2003;68(1):83-90. 2) Schlesinger N: Diagnosis of gout: Clinical, laboratory, and radiologic findings. Am J Manag Care 2005;11(15 Suppl):S443-S450.

A 30-year-old male presents to the emergency department with a sensation of a racing heart. His history is significant for known Wolff-Parkinson-White syndrome (WPW). On examination he is alert and in no severe distress. His blood pressure is 130/70 mm Hg, pulse rate 220 beats/min, and oxygen saturation 96%. An EKG reveals a regular, wide-complex tachycardia with a rate of 220 beats/min. You determine that he is stable, the EKG is consistent with WPW, and pharmacologic conversion is a safe initial therapy. Which one of the following would be the treatment of choice? (check one) A. Verapamil (Calan) B. Adenosine (Adenocard) C. Procainamide D. Digoxin

C Adenosine, digoxin, and calcium channel antagonists act by blocking conduction through the atrioventricular (AV) node, which may increase the ventricular rate paradoxically, initiating ventricular fibrillation. These agents should be avoided in Wolff-Parkinson-White syndrome. Procainamide is usually the treatment of choice in these situations, although amiodarone may also be used. Ref: Blomström-Lundqvist C, Scheinman MM, Aliot EM, et al: ACC/AHA/ESC guidelines for the management of patients with supraventricular arrhythmias—Executive summary. A report of the American College of Cardiology/American Heart Association Task Force on Practice Guidelines and the European Society of Cardiology Committee for Practice Guidelines (Writing Committee to Develop Guidelines for the Management of Patients with Supraventricular Arrhythmias) developed in collaboration with NASPE-Heart Rhythm Society. J Am Coll Cardiol 2003;42(8):1493-1531. 2) Bonow RO, Mann DL, Zipes DP, Libby P (eds): Braunwald's Heart Disease: A Textbook of Cardiovascular Medicine, ed 9. Elsevier Saunders, 2011, pp 787, 795, 798-801.

A 60-year-old type 2 diabetic requires urgent appendectomy. Which one of the following should be withheld until normal kidney function is documented at 24 and 48 hours after the surgery? (check one) A. Acarbose (Precose) B. Glimepiride (Amaryl) C. Metformin (Glucophage) D. Nateglinide (Starlix)

C Administration of general anesthesia may cause hypotension, which leads to renal hypoperfusion and peripheral tissue hypoxia, with subsequent lactate accumulation. Therefore, if administration of radiocontrast material is required or urgent surgery is needed, metformin should be withheld and hydration maintained until preserved kidney function is documented at 24 and 48 hours after the intervention. Ref: Kirpichnikov D, McFarlane SI, Sowers JR: Metformin: An update. Ann Intern Med 2002;137(1):25-33.

You have just diagnosed mild persistent asthma in a 13-year-old African-American female. Along with patient education, your initial medical management should be: (check one) A. a short-acting inhaled β-agonist to be used only as needed B. a long-acting inhaled β-agonist daily C. a low-dose inhaled corticosteroid daily, along with a short-acting inhaled β-agonist as needed D. a low-dose inhaled corticosteroid daily, along with a long-acting inhaled β-agonist daily E. montelukast (Singulair) daily

C Inhaled corticosteroids improve asthma control in adults and children more effectively than any other single long-term controller medication, and all patients should also receive a prescription for a short-acting β-agonist (SOR A). Ref: Pollart SM, Elward KS: Overview of changes to asthma guidelines: Diagnosis and screening. Am Fam Physician 2009;79(9):761-767.

During a routine physical examination of a 35-year-old Asian female, you note a right adnexal fullness. She has had no symptoms of pain or bloating and has been menstruating normally. Her menses occur approximately every 30 days and her next period is expected to occur in 1 week. Pelvic ultrasonography reveals a thin-walled simple cyst 5 cm in diameter. No other abnormalities are seen in the pelvic structures. Which one of the following is the best course of management for this condition? (check one) A. Reassurance only B. Checking for any increase in adnexal fullness at her next annual physical examination C. Repeat ultrasonography in 2-3 months to confirm resolution of the cyst D. Referral for ultrasound-guided aspiration of the cyst E. Referral for laparoscopic removal of the cyst

C Adnexal masses in women under 45 years of age are benign in 80%-85% of cases. The specific findings of this case also strongly suggest a benign etiology, namely a thin-walled, simple cyst, a lesion that is less than 8 cm in size, and a patient of relatively young age. No aggressive means are indicated in these situations unless there are significant clinical symptoms such as pain, abdominal pressure, urinary symptoms, or gastrointestinal symptoms. Most experts currently recommend a conservative approach with repeat ultrasonography in at least 2 months, during which time the vast majority of benign cysts resolve spontaneously. Ref: Berek JS (ed): Novak's Gynecology, ed 13. Lippincott Williams & Wilkins, 2002, pp 382-387, 396-398.

Which one of the following is the leading cause of blindness in individuals over age 65 in the U.S.? (check one) A. Open angle glaucoma B. Narrow angle glaucoma C. Macular degeneration D. Diabetic retinopathy E. Ophthalmic artery occlusion

C Age-related macular degeneration (AMD) is the leading cause of blindness in the U.S. in individuals over age 65. AMD currently affects more than 1.75 million individuals in the U.S. Due to the rapid aging of the population, this number will increase to almost 3 million by 2020. Ref: Eye Diseases Prevalence Research Group: Prevalence of age-related macular degeneration in the U.S. Arch Ophthalmol 2004;122(4):564-572. 2) Bressler NM: Age-related macular degeneration is the leading cause of blindness. JAMA 2004;291(15):1900-1901.

Which one of the following is the most common cause of hypertension in children under 6 years of age? (check one) A. Essential hypertension B. Pheochromocytoma C. Renal parenchymal disease D. Hyperthyroidism E. Excessive caffeine use

C Although essential hypertension is most common in adolescents and adults, it is rarely found in children less than 10 years old and should be a diagnosis of exclusion. The most common cause of hypertension is renal parenchymal disease, and a urinalysis, urine culture, and renal ultrasonography should be ordered for all children presenting with hypertension. Other secondary causes, such as pheochromocytoma, hyperthyroidism, and excessive caffeine use, are less common, and further testing and/or investigation should be ordered as clinically indicated. Ref: Luma GB, Spiotta RT: Hypertension in children and adolescents. Am Fam Physician 2006;73(9):1558-1566.

A 54-year-old female takes levothyroxine (Synthroid), 0.125 mg/day, for central hypothyroidism secondary to a pituitary adenoma. The nurse practitioner in your office orders a TSH level, which is found to be 0.1 mIU/mL (N 0.5-5.0). Which one of the following would you recommend? (check one) A. Decrease the dosage of levothyroxine B. Increase the dosage of levothyroxine C. Order a free T4 level D. Order a TRH stimulation test E. Repeat the TSH level in 3 months

C Although uncommon, pituitary disease can cause secondary hypothyroidism. The characteristic laboratory findings are a low serum free T4 and a low TSH. A free T4 level is needed to evaluate the proper dosage of replacement therapy in secondary hypothyroidism. The TSH level is not useful for determining the adequacy of thyroid replacement in secondary hypothyroidism since the pituitary is malfunctioning. In the initial evaluation of secondary hypothyroidism, a TRH stimulation test would be useful if TSH failed to rise in response to stimulation. It is not necessary in this case, since the diagnosis has already been made. Ref: Devdhar M, Ousman YH, Burman KD: Hypothyroidism. Endocrinol Metab Clin North Am 2007;36(3):595-615.

Treatment with which one of the following antihypertensive medications may mimic the effects of primary hyperparathyroidism? (check one) A. Amlodipine (Norvasc) B. Doxazosin (Cardura) C. Hydrochlorothiazide D. Lisinopril (Prinivil, Zestril) E. Metoprolol (Lopressor, Toprol-XL)

C An elevated level of parathyroid hormone (or a level that is in an unexpected "normal" range) in a patient with an elevated calcium level generally indicates a diagnosis of primary hyperparathyroidism. However, these laboratory findings may also occur with lithium or thiazide use, tertiary hyperparathyroidism associated with end-stage renal failure, or familial hypocalciuric hypercalcemia, and a medical and family history should be obtained to assess these possibilities. The other medications listed do not cause hypercalcemia. Ref: Marcocci C, Cetani F: Primary hyperparathyroidism. N Engl J Med 2011;365(25):2389-2397.

The most serious complication of a slipped capital femoral epiphysis is: (check one) A. Osteomyelitis B. Pathologic fracture C. Avascular necrosis D. Chondrolysis

C Avascular necrosis is the most serious complication of a slipped capital femoral epiphysis, and leads to more rapid arthritic deterioration. It may require hip fusion and total hip replacement early in adulthood. Ref: Loder RT: Slipped capital femoral epiphysis. Am Fam Physician 1998;57(9):2135-2142.

A 70-year-old white male has a slowly enlarging, asymptomatic abdominal aortic aneurysm. You should usually recommend surgical intervention when the diameter of the aneurysm approaches: (check one) A. 3.5 cm B. 4.5 cm C. 5.5 cm D. 6.5 cm E. 7.5 cm

C Based on recent clinical trials, the most common recommendation for surgical repair is when the aneurysm approaches 5.5 cm in diameter. Two large studies, the Aneurysm Detection and Management (ADAM) Veteran Affairs Cooperative Study, and the United Kingdom Small Aneurysm Trial, failed to show any benefit from early surgery for men with aneurysms less than 5.5 cm in diameter. The risks of aneurysm rupture were 1% or less in both studies, with 6-year cumulative survivals of 74% and 64%, respectively. Interestingly, the risk for aneurysm rupture was four times greater in women, indicating that 5.5 cm may be too high, but a new evidence-based threshold has not yet been defined. Ref: Lederle FA, Wilson SE, Johnson GR, et al: Immediate repair compared with surveillance of small abdominal aortic aneurysms. N Engl J Med 2002;346(19):1437-1444. 2) United Kingdom Small Aneurysm Trial Participants: Long-term outcomes of immediate repair compared with surveillance of small abdominal aortic aneurysms. N Engl J Med 2002;346(19):1445-1452. 3) Powell JT, Greenhalgh RM: Small abdominal aortic aneurysms. N Engl J Med 2003;348(19):1895-1901.

A 68-year-old African-American female with primary hypothyroidism is taking levothyroxine (Synthroid), 125 μg/day. Her TSH level is 0.2μU/mL (N 0.5-5.0). She has no symptoms of either hypothyroidism or hyperthyroidism. Which one of the following would be most appropriate at this point? (check one) A. Continuing levothyroxine at the same dosage B. Increasing the levothyroxine dosage C. Decreasing the levothyroxine dosage D. Discontinuing levothyroxine E. Ordering a free T 4

C Because of the precise relationship between circulating thyroid hormone and pituitary TSH secretion, measurement of serum TSH is essential in the management of patients receiving levothyroxine therapy. Immunoassays can reliably distinguish between normal and suppressed concentrations of TSH. In a patient receiving levothyroxine, a low TSH level usually indicates overreplacement. If this occurs, the dosage should be reduced slightly and the TSH level repeated in 2-3 months' time. There is no need to discontinue therapy in this situation, and repeating the TSH level in 2 weeks would not be helpful. A free T4 level would also be unnecessary, since it is not as sensitive as a TSH level for detecting mild states of excess thyroid hormone. Ref: Hennessey JV, Scherger JE: Evaluating and treating the patient with hypothyroid disease. J Fam Pract 2007;56(8 Suppl Hot Topics):S31-S39. 2) Woeber KA: The year in review: The thyroid. Ann Intern Med 1999;131(12):959-962. 3) Fauci AS, Braunwald E, Kasper DL, et al (eds): Harrison's Principles of Internal Medicine, ed 17. McGraw-Hill, 2008, pp 2228-2233.

A 16-year-old male is brought to your office by his mother for "stomachaches." On review of systems, he also complains of headaches, occasional bedwetting, and trouble sleeping. His examination is within normal limits. His mother says that he is often in the nurse's office at school, and doesn't seem to have any friends. After some questions from you, he admits to being called names and teased at school. Which one of the following would be most appropriate? (check one) A. Explain that he must try to conform to be more popular B. Explain that these symptoms are a stress reaction and will lessen with time C. Explore whether his school counselor has a process to address this problem D. Order a TSH level

C Childhood bullying has potentially serious implications for bullies and their targets. The target children are typically quiet and sensitive, and may be perceived to be weak and different. Children who say they are being bullied must be believed and reassured that they have done the right thing in acknowledging the problem. Parents should be advised to discuss the situation with school personnel. Bullying is extremely difficult to resolve. Confronting bullies and expecting victims to conform are not successful approaches. The presenting symptoms are not temporary, and in fact can progress to serious issues such as suicide, substance abuse, and victim-to-bully transformation. These are not signs or symptoms of thyroid disease. The Olweus Bullying Program developed in Norway is a well documented, effective program for reducing bullying among elementary and junior-high-school students by altering social norms and by changing school responses to bullying incidents, including efforts to protect and support victims. Students who have been bullied regularly are most likely to carry weapons to school, be in frequent fights, and eventually be injured. Ref: Lyznicki JM, McCaffree MA, Robinowitz CB: Childhood bullying: Implications for physicians. Am Fam Physician 2004;70(9):1723-1728.

A middle-aged hairdresser presents with a complaint of soreness of the proximal nail folds of several fingers on either hand, which has slowly worsened over the last 6 months. The nails appear thickened and distorted. Otherwise she is healthy and has no evidence of systemic disease. Which one of the following would be the most effective initial treatment? (check one) A. Soaking in a dilute iodine solution twice daily to cleanse and sterilize the nail beds B. Oral amoxicillin/clavulanate (Augmentin) for up to 4-6 weeks C. Topical betamethasone dipropionate (Diprolene) applied twice daily to the nail folds for 3-4 weeks D. Evaluation for HIV, hepatitis C, psoriasis, and rheumatoid arthritis

C Chronic paronychia is a common condition in workers whose hands are exposed to chemical irritants or are wet for long periods of time. This patient is an otherwise healthy hairdresser, with frequent exposure to irritants. The patient should be advised to avoid exposure to harsh chemicals and water. In addition, the use of strong topical corticosteroids over several weeks can greatly reduce the inflammation, allowing the nail folds to return to normal and helping the cuticles recover their natural barrier to infection. Soaking in iodine solution would kill bacteria, but would also perpetuate the chronic irritation. Because the condition is related to chemical and water irritation, a prolonged course of antibiotics should not be the first treatment step, and could have serious side effects. There is no need to explore less likely autoimmune causes for nail changes at this time. Ref: Rigopoulos D, Larios G, Gregoriou S, et al: Acute and chronic paronychia. Am Fam Physician 2008;77(3):339-346.

Patients treated with which one of the following require regular hematologic monitoring for the development of granulocytopenia? (check one) A. Olanzapine (Zyprexa) B. Haloperidol (Haldol) C. Clozapine (Clozaril) D. Fluphenazine (Prolixin) E. Risperidone (Risperdal)

C Clozapine is one of the so-called second-generation antipsychotics, which are believed to be less likely to cause extrapyramidal side effects than the first-generation drugs such as haloperidol or the phenothiazines (e.g., fluphenazine). A 2003 meta-analysis concluded that clozapine was the most efficacious second-generation antipsychotic, followed by risperidone and olanzapine. However, clozapine use is associated with an approximately 1% incidence of granulocytopenia or agranulocytosis. Early detection by monitoring blood counts every 1-2 weeks has led to a reduction in agranulocytosis-related death, but clozapine is generally considered second-line therapy, to be used in cases unresponsive to other drugs. Ref: Choice of an antipsychotic. Med Lett Drugs Ther 2003;45:102-104.

Which one of the following tinea infections in children always requires systemic antifungal therapy? (check one) A. Tinea cruris B. Tinea corporis C. Tinea capitis D. Tinea pedis E. Tinea versicolor

C Dermatophyte infections caused by aerobic fungi produce infections in many areas. Tinea capitis requires systemic therapy to penetrate the affected hair shafts. Tinea cruris and tinea pedis rarely require systemic therapy. Extensive outbreaks of tinea corporis and tinea versicolor benefit from both oral and topical treatment (SOR A), but more localized infections require only topical treatment.

A 60-year-old male presents to the urgent care center with a fever and a productive cough. He has a 40 pack/year history of cigarette smoking. In addition to lobar pneumonia on a chest radiograph, there is an incidental finding of bilateral pleural plaques. Which one of the following is the most likely cause of this finding? (check one) A. Coal dust B. Silicon dust C. Asbestos D. Vinyl chloride E. Radon

C Development of pleural plaques is the most common pathologic pulmonary response to asbestos inhalation. Over time, collagen is deposited in the pleura and may calcify. Most plaques are asymptomatic, and there is no evidence that plaques transform into malignant lesions. Plaques occur in approximately 50% of persons with heavy and prolonged exposure to asbestos and, therefore, are a marker of asbestos exposure. This should alert the physician to follow the patient for development of more serious asbestos-related diseases (e.g., lung cancer and mesothelioma). Although the other substances listed are associated with pulmonary diseases (coal dust and silicon dust with pneumoconiosis, and vinyl chloride and radon with lung cancer), none is associated with pleural plaques as found in this patient. Ref: O'Reilly KM, McLaughlin AM, Beckett WS, et al: Asbestos-related lung disease. Am Fam Physician 2007;75(5):683-688. 2) Collins LG, Haines C, Perkel R, et al: Lung cancer: Diagnosis and management. Am Fam Physician 2007;75(1):56-63.

For 2 weeks, a 62-year-old male with biopsy-documented cirrhosis and ascites has had diffuse abdominal discomfort, fever, and night sweats. His current medications are furosemide (Lasix) and spironolactone (Aldactone). On examination, his temperature is 38.0° C (100.4° F), blood pressure 100/60 mm Hg, heart rate 92 beats/min and regular. The heart and lung examination is normal. The abdomen is soft with vague tenderness in all quadrants. There is no rebound or guarding. The presence of ascites is easily verified. Bowel sounds are quiet. The rectal examination is normal, and the stool is negative for occult blood. You perform diagnostic paracentesis and send a sample of fluid for analysis. Which one of the following findings would best establish the suspected diagnosis of spontaneous bacterial peritonitis? (check one) A. pH <7.2 B. Bloody appearance C. Neutrophil count >300/mL D. Positive cytology E. Total protein >1 g/dL

C Diagnostic paracentesis is recommended for patients with ascites of recent onset, as well as for those with chronic ascites who present with new clinical findings such as fever or abdominal pain. A neutrophil count >250/mL is diagnostic for peritonitis. Once peritonitis is diagnosed, antibiotic therapy should be started immediately without waiting for culture results. Bloody ascites with abnormal cytology may be seen with hepatoma, but is not typical of peritonitis. The ascitic fluid pH does not become abnormal until well after the neutrophil count has risen, so it is a less reliable finding for treatment purposes. A protein level >1 g/dL is actually evidence against spontaneous bacterial peritonitis. Ref: Braunwald E, Fauci AS, Kasper DL, et al (eds): Harrison's Principles of Internal Medicine, ed 15. McGraw-Hill, 2001, p 829. 2) Tierney LM, McPhee SJ, Papadakis MA (eds): Current Medical Diagnosis & Treatment 2002, ed 41. Lange Medical Books/McGraw-Hill, 2002, pp 594-596.

A 27-year-old female presents to the emergency department with a complaint of bloody diarrhea and abdominal cramping. A few days ago she ate a rare hamburger at a birthday party for her 4-year-old son. He ate hot dogs instead, and has not been ill. A stool specimen is positive for Escherichia coli O:157. Which one of the following should you do next? (check one) A. Provide levofloxacin (Levaquin) prophylaxis to her close contacts B. Monitor her liver enzymes C. Monitor her renal function D. Reassure her that her son is not at risk of illness

C Escherichia coli O:157 is an increasingly common cause of serious gastrointestinal illness. The usual source is undercooked beef. The child is at risk, since at least 20% of cases result from secondary spread. Transmission is frequent in children's day-care facilities and nurseries. Some cases are asymptomatic, but the great majority are symptomatic, and patients present with bloody diarrhea. Levofloxacin is not useful for prophylaxis in contacts. This patient has a 10%-15% risk of developing hemolytic uremic syndrome secondary to her E. coli O:157 infection, making close monitoring of renal function essential.

The FDA issued a boxed warning describing an increased risk of tendinopathy and tendon rupture associated with the use of which class of antibiotics? (check one) A. Macrolides B. Aminoglycosides C. Fluoroquinolones D. Tetracyclines E. Polypeptides

C Fluoroquinolones are associated with an increased risk of tendinopathy and tendon rupture. About 1/6000 prescriptions will cause an Achilles tendon rupture. The risk is higher in those also taking corticosteroids or over the age of 60.

The best management of localized, well-differentiated prostate cancer in men older than 65 is: (check one) A. radiation implants B. external beam radiation therapy C. watchful waiting D. primary androgen deprivation therapy E. robot-assisted prostatectomy

C For men older than 65 years of age with small-volume, low-grade disease and a 10- to 15-year life expectancy, the risk of complications from treatment outweighs any decreased risk of dying from prostate cancer. Radiation, androgen deprivation therapy, and surgical approaches have not been shown to improve disease-free survival (SOR A).

Which one of the following should be avoided in the treatment and prophylaxis of migraine during early pregnancy? (check one) A. Calcium channel blockers B. Beta-blockers C. Triptans D. NSAIDS

C Headaches, and migraines in particular, are very common in women of childbearing age. Migraine sufferers usually have improvement of symptoms in pregnancy and many have complete remission. Most medications used for prophylaxis and abortive treatment of migraines in the nonpregnant patient can also be used in pregnant patients. Most beta-blockers and calcium channel blockers are safe. Acetaminophen and narcotics can be used for acute pain. Ibuprofen can also be used but should be avoided late in pregnancy because it is associated with premature closure of the ductus arteriosus and oligohydramnios. Ergotamines should be avoided as they are uterotonic and have abortifacient properties. They have also been associated with case reports of fetal birth defects. Triptans have the potential to cause vasoconstriction of the placental and uterine vessels and should be used only if the benefit clearly outweighs the harm. Ref: Gabbe SG, Niebyl JR, Simpson JL (eds): Obstetrics: Normal and Problem Pregnancies, ed 4. Churchill Livingstone, 2002, pp 1244-1246.

A mother brings in her 2-week-old infant for a well child check. She reports that she is primarily breastfeeding him, with occasional formula supplementation. Which one of the following should you advise her regarding vitamin D intake for her baby? (check one) A. Breastfed infants do not need supplemental vitamin D B. As long as the baby is taking at least 16 oz of formula per day, he does not need supplemental vitamin D C. The baby should be given 400 IU of supplemental vitamin D daily D. Intake of vitamin D in excess of 200 IU/day is potentially toxic E. Vitamin D supplementation should not be started until he is at least 6 months old

C In 2008, the American Academy of Pediatrics increased its recommended daily intake of vitamin D in infants, children, and adolescents to 400 IU/day (SOR C). Breastfeeding does not provide adequate levels of vitamin D. Exclusive formula feeding probably provides adequate levels of vitamin D, but infants who consume less than 1 liter of formula per day need supplementation with 400 IU of vitamin D daily. Vitamin D supplementation should be started within the first 2 months of birth.

For normal term infants, current practice is to introduce solid foods into the diet at what age? (check one) A. 2-4 weeks B. 2-3 months C. 4-6 months D. 7-9 months E. 1 year

C In normal term infants, there is little evidence that solid foods contribute to well-being before the age of 4-6 months. In addition, the extrusion reflex (pushing foreign material out of the mouth with the tongue) makes feeding of solids difficult and often forced. This reflex disappears around the age of 4 months, making feeding easier. The introduction of solids at this age helps supply calories, iron, and vitamins, and may prepare the infant for later dietary diversity and healthy dietary habits. Ref: Kliegman RM, Stanton BF, Geme JW III, et al (eds): Nelson Textbook of Pediatrics, ed 19. Elsevier Saunders, 2011, pp 164-165.

In prescribing an exercise program for elderly, community-dwelling patients, it is important to note that: (check one) A. Graded exercise stress testing should be done before beginning the program B. Target heart rates should be 80% of the predicted maximum C. The initial routines can be as short as 6 minutes repeated throughout the day and still be beneficial D. Treadmill walking is especially beneficial to patients with peripheral neuropathy

C Initial exercise routines for the elderly can be as short as 6 minutes in duration. Even 30 minutes per week of exercise has been shown to be beneficial. Graded exercise testing need not be done, especially if low-level exercise is planned. A target heart rate of 60%-75% of the predicted maximum should be set as a ceiling. Patients with peripheral neuropathy should not perform treadmill walking or step aerobics because of the risk of damage to their feet. Ref: Heath JM, Stuart MR: Prescribing exercise for frail elders. J Am Board Fam Pract 2002;15(3):218-228.

A 68-year-old female is being monitored in the hospital after elective surgery. On her third postoperative day she suddenly develops hypoxia, fever, tachycardia, and hypotension. You institute high-rate intravenous fluids and empiric antibiotics. However, approximately 2 hours into this therapy, her blood pressure remains at 80 mm Hg systolic with sluggish urine output. Which one of the following hormones should be assessed at this time? (check one) A. Aldosterone B. Catecholamines C. Cortisol D. Renin E. TSH

C It has been recognized that patients suffering from a critical illness with an exaggerated inflammatory response often have a relative cortisol deficiency. Clinically, this can cause hypotension that is resistant to intravenous fluid resuscitation, and evidence is mounting that survival is increased if these patients are treated with intravenous corticosteroids during acute management. Cortisol levels can be assessed with a single serum reading, or by the change in the cortisol level after stimulation with cosyntropin (referred to as Δcortisol). The other hormones listed are not important for the acute management of a critically ill patient.

A 25-year-old female at 36 weeks gestation presents for a routine prenatal visit. Her blood pressure is 118/78 mm Hg and her urine has no signs of protein or glucose. Her fundal height shows appropriate fetal size and she says that she feels well. On palpation of her legs, you note 2+ pitting edema bilaterally. Which one of the following is true regarding this patient's condition? (check one) A. You should order a 24-hr urine for protein B. A workup for possible cardiac abnormalities is necessary C. Her leg swelling requires no further evaluation D. She most likely has preeclampsia E. She most likely has deep venous thrombosis

C Lower-extremity edema is common in the last trimester of normal pregnancies and can be treated symptomatically with compression stockings. Edema has been associated with preeclampsia, but the majority of women who have lower-extremity edema with no signs of elevated blood pressure will not develop preeclampsia or eclampsia. For this reason, edema has recently been removed from the diagnostic criteria for preeclampsia. Disproportionate swelling in one leg versus another, especially associated with leg pain, should prompt a workup for deep venous thrombosis but is unlikely given this patient's presentation, as are cardiac or renal conditions. Ref: Wagner LK: Diagnosis and management of preeclampsia. Am Fam Physician 2004;70(12):2317-2324. 2) Cunningham FG, Leveno KJ, Bloom SL, et al: Williams Obstetrics, ed 22. McGraw-Hill, 2005, pp 126-127.

A 23-year-old Hispanic female at 18 weeks' gestation presents with a 4-week history of a new facial rash. She has noticed worsening with sun exposure. Her past medical history and review of systems is normal. On examination, you note symmetric, hyperpigmented patches on her cheeks and upper lip. The remainder of her examination is normal. The most likely diagnosis is: (check one) A. Lupus erythematosus B. Pemphigoid gestationis (herpes gestationis) C. Melasma (chloasma) D. Prurigo gestationis

C Melasma or chloasma is common in pregnancy, with approximately 70% of pregnant women affected. It is an acquired hypermelanosis of the face, with symmetric distribution usually on the cheeks, nose, eyebrows, chin, and/or upper lip. The pathogenesis is not known. UV sunscreen is important, as sun exposure worsens the condition. Melasma often resolves or improves post partum. Persistent melasma can be treated with hydroquinone cream, retinoic acid, and/or chemical peels performed post partum by a dermatologist. The facial rash of lupus is usually more erythematous, and lupus is relatively rare. Pemphigoid gestationis is a rare autoimmune disease with extremely pruritic, bullous skin lesions that usually spare the face. Prurigo gestationis involves pruritic papules on the extensor surfaces and is usually associated with significant excoriation by the uncomfortable patient. Ref: Gabbe SG, Niebyl JR, Simpson JL (eds): Obstetrics: Normal and Problem Pregnancies, ed 4. Churchill Livingstone, 2002, pp 1283-1284.

A 57-year-old previously healthy menopausal female presents to your office with a 1-year history of palpitations and an unintentional 10-lb weight loss. A review of systems is negative for tremors or visual changes. Vital signs include a blood pressure of 129/85 mm Hg and a heart rate of 110 beats/min. A physical examination is otherwise unremarkable except for a nontender, diffusely enlarged thyroid with no distinct nodules, and mild proptosis. Laboratory studies are significant for a TSH level<0.01 :U/mL (N 0.60-3.30), a free T3 level of 14.51 pg/mL (N 2.0-3.5), and a free T4 level of 4.52 ng/dL (N 0.71-1.40). A thyroid-stimulating immunoglobulin test is positive. In addition to a β-blocker, which one of the following is the most appropriate initial management? (check one) A. Radioactive iodine ablation B. Thyroidectomy C. Methimazole (Tapazole) D. Propylthiouracil (PTU)

C Methimazole and propylthiouracil (PTU) are the two oral antithyroid medications available. However, because of reports of severe hepatocellular damage, methimazole should be used instead of PTU unless it is contraindicated. Radioactive iodine treatment (131I) is an option, especially for patients who do not achieve remission with antithyroid medications. However, worsening of preexisting orbitopathy is a well-recognized potential complication of 131I treatment, as well as a transient increase in thyroid hormone levels that can precipitate thyroid storm. Thus, patients with elevated free T3 or free T4 levels should be treated with methimazole prior to 131I administration. Thyroidectomy is most often recommended for patients with thyroid nodules and those who are suspected of having cancer or who do not tolerate or refuse alternative forms of therapy. However, antithyroid medication should be given to achieve a euthyroid state prior to surgery in most patients.

Your patient is in the second stage of labor, and you determine that the fetus is in face presentation, mentum anterior. Progress has been rapid and fetal heart tones are normal. You would now: (check one) A. Perform an immediate cesarean delivery B. Proceed with midforceps delivery C. Anticipate vaginal delivery with close fetal monitoring D. Manually convert to vertex presentation

C Most infants with face presentation, mentum anterior, can be delivered vaginally, either spontaneously or with low forceps. Cesarean section is indicated for fetal distress and failure to progress. Midforceps delivery is not indicated. If fetal electrodes are attached, the chin is the preferred location. Ref: Scott JR, Gibbs RS, Karlan BY, et al (eds): Danforth's Obstetrics and Gynecology, ed 9. Lippincott Williams & Wilkins, 2003, pp 388-390. 2) Cunningham FG, Leveno KJ, Bloom SL, et al: Williams Obstetrics, ed 22. McGraw-Hill, 2005, pp 506-507, 549.

After fitting a 30-year-old gravida 2 para 2 for a diaphragm, you advise her not to leave the diaphragm in place for longer than 24 hours because of the risk of which one of the following? (check one) A. Loss of contraceptive effectiveness B. Chlamydia infection C. Toxic shock syndrome D. Human papillomavirus (HPV) infection E. Adhesions

C Much like with tampons, leaving diaphragms in place for more than 24 hours is associated with toxic shock syndrome. Ref: Allen RE: Diaphragm fitting. Am Fam Physician 2004;69(1):97-100.

As a member of the local emergency response management team you are asked about the treatment of nerve gas (e.g., sarin) poisoning. Which one of the following is most effective in reversing the symptoms of nerve gas toxicity? (check one) A. Albuterol (Proventil, Ventolin) via inhalation B. Ciprofloxacin (Cipro) C. Atropine D. Parenteral verapamil (Calan, Isoptin) E. Parenteral corticosteroids

C Nerve gas agents such as sarin resemble organophosphate insecticides and inactivate anticholinesterase, leading to the accumulation of acetylcholine at nerve endings. Respiratory symptoms include rhinorrhea, bronchorrhea, bronchospasm, and respiratory muscle paralysis. Gastrointestinal symptoms include nausea, vomiting, and diarrhea. Central nervous system symptoms include headache, vertigo, agitation, seizures, and coma. Exposed patients benefit from treatment with atropine, which competitively inhibits acetylcholine. Pralidoxine chloride and diazepam are also beneficial. Although beta-agonists and corticosteroids are beneficial in the general treatment of bronchospasm, atropine is preferred in this situation. Verapamil and ciprofloxacin have no role in the treatment of nerve gas exposure. Ref: Prevention and treatment of injury from chemical warfare agents. Med Lett Drugs Ther 2002;44:1-4.

A 38-year-old widow consults you 2 years after her husband's accidental death. She is planning to remarry and asks about the possibility of resuming the low-dose oral contraceptives she took before she was widowed. Which one of the following may contraindicate resumption of oral contraceptives? (check one) A. Her 42-year-old sister has breast cancer B. Her blood pressure is 135/88 mm Hg C. She smokes a pack of cigarettes each day D. She has a history of migraines resistant to triptans E. Her LDL/HDL ratio is 2.8

C Oral contraceptives increase the risk of venous thromboembolic phenomena. The combination of oral contraceptives and smoking substantially increases the risk of cardiovascular disease. Caution should be exercised in prescribing oral contraceptives for women older than 35 years of age who smoke. In general, oral contraceptive use is considered absolutely contraindicated in women older than 35 who are heavy smokers. Women who smoke fewer than 15 cigarettes a day and patients with mildly elevated blood pressure and elevated lipid levels are not at increased risk for cardiovascular disease when oral contraceptives are used. Ref: Seibert C, Barbouche E, Fagan J, et al: Prescribing oral contraceptives for women older than 35 years of age. Ann Intern Med 2003;138:54-64.

A 57-year-old male executive sees you because of "shaky hands." His tremor is most noticeable when he is holding something or writing, and is more prominent in his hand than in his shoulder. He has noticed that it seems better "after a beer or two" at social gatherings. He has no other health problems. On examination you note a very definite tremor when he unbuttons his shirt. His gait is normal and there is no resting tremor. He has no other health problems. On examination you note a very definite tremor when he unbuttons his shirt. His gait is normal and there is no resting tremor. He has a previous history of intolerance to β-blockers. Of the following, which medication would be the best choice for this patient? (check one) A. Levodopa/carbidopa (Sinemet) B. Amantadine (Symmetrel) C. Primidone (Mysoline) D. Lithium carbonate

C Parkinson's disease and essential tremor are the primary concerns in a person of this age who presents with a new tremor. A coarse, resting, pill-rolling tremor is characteristic of Parkinson's disease. Essential tremor is primarily an action tremor and is a common movement disorder, occurring in members of the same family with a high degree of frequency. Alcohol intake will temporarily cause marked reduction in the tremor. β-Adrenergic blockers have been the mainstay of treatment for these tremors, but this patient is intolerant to these drugs. Primidone has been effective in the treatment of essential tremor, and in head-to-head studies with propranolol has been shown to be superior after 1 year. Levodopa in combination with carbidopa is useful in the treatment of parkinsonian tremor but not essential tremor.

A 30-year-old female asks you whether she should have a colonoscopy, as her father was diagnosed with colon cancer at the age of 58. There are no other family members with a history of colon polyps or cancer. You recommend that she have her first screening colonoscopy: (check one) A. now and every 5 years if normal B. now and every 10 years if normal C. at age 40 and then every 5 years if normal D. at age 40 and then every 10 years if normal E. at age 50 and then every 5 years if normal

C Patients should be risk-stratified according to their family history. Patients who have one first degree relative diagnosed with colorectal cancer or adenomatous polyps before age 60, or at least two second degree relatives with colorectal cancer, are in the highest risk group. They should start colon cancer screening at age 40, or 10 years before the earliest age at which an affected relative was diagnosed (whichever comes first) and be rescreened every 5 years. Colonoscopy is the preferred screening method for this highest-risk group, as high-risk patients are more likely to have right-sided colon lesions that would not be detected with sigmoidoscopy. Ref: Wiltz SA, Nelson RM: What's the most effective way to screen patients with a family history of colon cancer? J Fam Pract 2010;59(3):176-178.

The definition of post-term pregnancy is a pregnancy that has reached: (check one) A. 40 weeks' gestation B. 41 weeks' gestation C. 42 weeks' gestation D. 43 weeks' gestation

C Postdate and post-term pregnancy are terms that are used interchangeably. The postdate pregnancy is defined as a pregnancy that has reached 42 weeks of amenorrhea. This is important because perinatal mortality doubles at 42 weeks gestational age. The diagnosis of postdate pregnancy depends heavily on accurate dating methods. Ref: Gabbe SG (ed): Obstetrics: Normal and Problem Pregnancies, ed 3. Churchill Livingstone, 2002, pp 887-888.

A 22-year-old white female comes to your office complaining of dizziness. She was in her usual good health until about 2 weeks before this visit, when she developed a case of gastroenteritis that other members of her family have also had. Since that time she has been lightheaded when standing, feels her heart race, and gets headaches or blurred vision if she does not sit or lie down. She has not passed out but has been unable to work due to these symptoms. She is otherwise healthy and takes no regular medications. A physical examination is normal except for her heart rate, which rises from 72 beats/min when she is lying or sitting to 112 beats/min when she stands. Her blood pressure remains unchanged with changes of position. Routine laboratory tests and an EKG are normal. What is the most likely cause of this patient's condition? (check one) A. Myocarditis B. A seizure disorder C. Postural orthostatic tachycardia syndrome (POTS) D. Systemic lupus erythematosus E. Somatization disorder

C Postural orthostatic tachycardia syndrome (POTS) is manifested by a rise in heart rate >30 beats/min or by a heart rate >120 beats/min within 10 minutes of being in the upright position. Symptoms usually include position-dependent headaches, abdominal pain, lightheadedness, palpitations, sweating, and nausea. Most patients will not actually pass out, but some will if they are unable to lie down quickly enough. This condition is most prevalent in white females between the ages of 15 and 50 years old. Often these patients are hardworking, athletic, and otherwise in good health. There is a high clinical correlation between POTS and chronic fatigue syndrome. Although no single etiology for POTS has been found, the condition is thought to have a genetic predisposition, is often incited after a prolonged viral illness, and has a component of deconditioning. The recommended initial management is encouraging adequate fluid and salt intake, followed by the initiation of regular aerobic exercise combined with lower-extremity strength training, and then the use of β-blockers.

Which one of the following is true regarding the risk of physical spouse abuse? (check one) A. It decreases during pregnancy B. It decreases when a woman exits an abusive relationship C. It increases with alcohol and substance abuse D. It increases as socioeconomic status rises E. It is higher among patients from racial minorities

C Pregnancy represents a time of heightened risk for battery, and prompts a shift in the area of physical abuse from the head and neck to the breasts and abdomen. Although some studies have shown a higher prevalence of partner abuse among minority women, a National Crime Victimization Survey concluded that women of all races and ethnic backgrounds are equally likely to be abused by an intimate. Alcohol and/or substance abuse is a significant risk factor associated with the occurrence of partner abuse and family violence in general. Less education and lower occupational status or income (particularly when total family income is less than $10,000 per year) have been found to increase the risk of violence. Women who have separated from their spouses are 3 times more likely to be victimized than are those who are already divorced and 25 times more likely than women who are married. A woman's danger substantially increases at the point of exiting an abusive relationship. Ref: Rakel RE: Textbook of Family Practice, ed 6. WB Saunders Co, 2002, pp 74-76.

Which one of the following is the most correct recommendation regarding seat belt use for a woman at 38 weeks' gestation? (check one) A. The seat belt should be positioned over the dome of the uterus and the shoulder harness should be positioned between the breasts B. The seat belt should be positioned under the abdomen over both the anterior superior iliac spines and the pubic symphysis; the belt should be applied with some slack C. The seat belt should be positioned under the abdomen over both the anterior superior iliac spines and the pubic symphysis; the shoulder harness should be positioned between the breasts; the belt should be applied as snugly as comfort will allow D. Seat belts, should not be used in the later stages of pregnancy

C Pregnant women can and should always wear a seat belt when driving or riding in a car. The seat belt should be positioned under the pregnant woman's abdomen over both the anterior superior iliac spines and the pubic symphysis. The shoulder harness should be positioned between the breasts. Ref: American College of Obstetricians and Gynecologists: ACOG Educational Bulletin. ACOG, 1998, no 251, pp 195-201. 2) Scott JR, Gibbs RS, Karlan BY, et al (eds): Danforth's Obstetrics and Gynecology, ed 9. Lippincott Williams & Wilkins, 2003, p 19.

A 36-year-old male presents to the emergency department with disorientation, tachycardia, diaphoresis, and hypertension. According to his family, he has been consuming up to a fifth of vodka daily but abruptly discontinued alcohol consumption 2 days ago. There is no history of additional substance abuse and a urine drug screen is negative. Which one of the following is most indicated in the management of this patient? (check one) A. An anticonvulsant B. A typical antipsychotic C. A benzodiazepine D. A centrally-acting α2-agonist E. Baclofen

C Psychomotor agitation is experienced by most patients during alcohol withdrawal. Benzodiazepines are clearly the drug class of choice. Providing medication on an as-needed basis rather than on a fixed schedule is generally preferred. Antipsychotics and butyrophenones (including haloperidol) lower the seizure threshold and should not be used. For short-term management of status epilepticus, anticonvulsants may be used in conjunction with benzodiazepines. The vast majority of seizures from withdrawal are self-limited and do not require anticonvulsant treatment. Clonidine and other α2-agonists do reduce minor symptoms of withdrawal, but have not been shown to prevent seizures. The effectiveness of baclofen in acute alcohol withdrawal is unknown. Ref: Kosten TR, O'Connor PG: Management of drug and alcohol withdrawal. N Engl J Med 2003;348(18):1786-1795. 2) Bayard M, McIntyre J, Hill KR, Woodside J Jr: Alcohol withdrawal syndrome. Am Fam Physician 2004;69(6):1443-1450. 3) Ricks J, Replogle WH, Cook NJ: FPIN's Clinical Inquiries. Management of alcohol withdrawal syndrome. Am Fam Physician 2010;82(4):344-347.

A 32-year-old meat cutter comes to your office with persistent symptoms of nausea, vomiting, and diarrhea which began about 36 hours ago on the last day of a 5-day Caribbean cruise. His wife was sick during the first 2 days of the cruise with similar symptoms. On the ship, they both ate the "usual foods" in addition to oysters. Findings on examination are negative, and a stool specimen is negative for white cells. Which one of the following is the most likely cause of his illness? (check one) A. Escherichia coli B. Rotavirus C. Norwalk virus D. Hepatitis A E. Giardia species

C Recent reports of epidemics of gastroenteritis on cruise ships are consistent with Norwalk virus infections due to waterborne or foodborne spread. In the United States, these viruses are responsible for about 90% of all epidemics of nonbacterial gastroenteritis. The Norwalk-like viruses are common causes of waterborne epidemics of gastroenteritis, and have been shown to be responsible for outbreaks in nursing homes, on cruise ships, at summer camps, and in schools. Symptomatic treatment is usually appropriate. Ref: Braunwald E, Fauci AS, Kasper DL, et al (eds): Harrison's Principles of Internal Medicine, ed 15. McGraw-Hill, 2001, p 1138.

As the medical review officer for a local business, you are required to interpret urine drug tests. Assuming the sample was properly collected and handled, which one of the following test results is consistent with the history provided and should be reported as a negative test? (check one) A. Diazepam (Valium) identified in an employee taking oxazepam prescribed by a physician B. Morphine identified in an employee undergoing a prescribed methadone pain management program C. Morphine identified in an employee taking a prescribed cough medicine containing codeine D. Tetrahydrocannabinol above the threshold value in an employee who reports secondary exposure to marijuana E. Tetrahydrocannabinol identified in an employee taking prescribed tramadol (Ultram)

C Results of urine drug test panels obtained in the workplace are reported by a Medical Review Officer (MRO) as positive, negative, dilute, refusal to test, or test canceled; the drug/metabolite for which the test is positive or the reason for refusal (e.g., the presence of an adulterant) or cancellation is also included in the final report. The MRO interpretation is based on consideration of many factors, including the confirmed patient medical history, specimen collection process, acceptability of the specimen submitted, and qualified laboratory measurement of drugs or metabolites in excess of the accepted thresholds. These thresholds are set to preclude the possibility that secondary contact with smoke, ingestion of poppy seeds, or similar exposures will result in an undeserved positive urine drug screen report. Other findings, such as the presence of behavioral or physical evidence of unauthorized use of opiates, may also factor into the final report. When a properly collected, acceptable specimen is found to contain drugs or metabolites that would be expected based on a review of confirmed prescribed use of medications, the test is reported as negative. Morphine is a metabolite of codeine that may be found in the urine of someone taking a codeine-containing medication; morphine is not a metabolite of methadone. Oxazepam is a metabolite of diazepam but the reverse is not true. Tetrahydrocannabinol would not be found in the urine as a result of tramadol use. Ref: Procedures for Transportation Workplace Drug and Alcohol Testing Programs. Office of Drug & Alcohol Policy & Compliance, US Dept of Transportation, 2010, sections 40.139, 40.141, 40.163. 2) Standridge JB, Adams SM, Zotos AP: Urine drug screening: A valuable office procedure. Am Fam Physician 2010;81(5):635-640.

The mother of an 4-week-old male asks about the viral gastroenteritis vaccine. You advise that it is (check one) A. routinely given at the 12-month visit B. associated with an increased risk for intussusception C. initiated at 6-12 weeks of age D. indicated only for immunocompromised children E. indicated only for children attending day care

C Rotavirus vaccine (RotaTeq) was licensed in February 2006 to protect against viral gastroenteritis. The Advisory Committee on Immunization Practices recommends the routine vaccination of infants with three doses to be given at 2, 4, and 6 months of age. The first dose should be given between 6 and 12 weeks of age, and subsequent doses should be given at 4- to 10-week intervals, but all three doses should be administered by 32 weeks of age. Unlike the vaccine RotaShield, which was marketed in 1999, RotaTeq is not known to increase the risk for intussusception. Ref: Parashar UD, Alexander JP, Glass RI, et al: Prevention of rotavirus gastroenteritis among infants and children: Recommendations of the Advisory Committee on Immunization Practices. MMWR 2006;55(RR-12):1-13.

Metformin (Glucophage) should be stopped prior to which one of the following, and withheld until 48 hours after completion of the test? (check one) A. An upper GI series B. Abdominal ultrasonography C. CT angiography D. MRI of the brain E. Colonoscopy

C Since even a temporary reduction in renal function, such as occurs after pyelography or angiography, can cause lactic acidosis in patients taking metformin, the drug should be discontinued 48 hours before such procedures (SOR C) and restarted 48 hours after the procedure if renal function is normal. The other procedures listed are not indications for stopping metformin. Ref: Stacul F, Adam A, Becker CR, et al: Strategies to reduce the risk of contrast-induced nephropathy. Am J Cardiol 2006;98(suppl 6A):59k-77k.

A mother meets you in the emergency department with her 3-week-old infant. The infant was delivered at term, with an uneventful prenatal and postnatal course to this point. The mother reports that the infant stopped breathing for 20-25 seconds, and that his lips and tongue appeared bluish. There was no coughing, choking, or congestion, but the child seemed "limp." The episode ended when the mother vigorously stimulated her child and he started crying. On examination, the child appears normal. Which one of the following would be most appropriate at this point? (check one) A. Reassurance and no further evaluation B. Discharge with a home apnea monitor C. Hospital admission for observation D. Mandatory referral to child protective services E. Direct laryngoscopy to rule out a foreign body

C Some experts recommend inpatient observation for all children with apparent life-threatening events such as this. Hospital admission is not always necessary, however, for a short, self-correcting episode associated with feeding. Given the history of not breathing for 20-25 seconds, having a blue tongue and lips, and being limp, admitting the child for observation is appropriate. Although child abuse is a concern, referral to child protective services is not mandatory. Laryngoscopy would not be routine, but might be appropriate in some cases depending on the history and physical findings. Many groups recommend home apnea monitoring after discharge for patients with more severe or undiagnosed cases.

A 62-year-old male with a history of prostate cancer and well-controlled hypertension presents with severe osteoporosis. At 55 years of age he received prostate brachytherapy and androgen deprivation for his prostate cancer and has been disease-free since. He presently takes lisinopril (Prinivil, Zestril), 5 mg daily; alendronate (Fosamax), 70 mg weekly; calcium, 1000 mg daily; and vitamin D, 1200 units daily. He has never smoked, exercises five times a week, and maintains a healthy lifestyle. In spite of his lifestyle and the medications he takes, he continues to have severe osteoporosis on his yearly bone density tests. In addition to recommending fall precautions, which one of the following would you consider next to treat his osteoporosis? (check one) A. Testosterone B. Calcitonin C. Teriparatide (Forteo) D. Raloxifene (Evista) E. Zoledronic acid (Reclast)

C Teriparatide is indicated for the treatment of severe osteoporosis, for patients with multiple osteoporosis risk factors, or for patients with failure of bisphosphonate therapy (SOR B). Therapy with teriparatide is currently limited to 2 years and is contraindicated in patients with a history of bone malignancy, Paget disease, hypercalcemia, or previous treatment with skeletal radiation. Its route of administration (subcutaneous) and high cost should be considered when prescribing teriparatide therapy. Testosterone therapy is contraindicated in patients with a history of prostate cancer. Zoledronic acid is a parenterally administered bisphosphonate and would not be appropriate in a patient who has already failed bisphosphonate therapy. Likewise, raloxifene and calcitonin are not indicated in patients with severe osteoporosis who have failed bisphosphonate therapy.

An asymptomatic 24-year-old white female comes to your office for a refill of oral contraceptive pills. A speculum examination is normal with the exception of a slightly friable, well-demarcated, 1.4-cm raised lesion involving a portion of the cervix. All previous Papanicolaou (Pap) tests have been normal and she has no history of abnormal bleeding or leukorrhea. Which one of the following would be most appropriate at this point? (check one) A. A Pap test, including a scraping of the erosion, with routine follow-up unless the patient becomes symptomatic B. A Pap test with follow-up in 3 months if results are normal C. A Pap test and a colposcopically-directed biopsy D. A cone biopsy E. Topical antibiotic cream

C The finding of a red, raised, friable lesion on the cervix, or a well-demarcated cervical lesion, mandates a biopsy to exclude cervical carcinoma, and treatment for chronic cervicitis should not be started until the biopsy results are available. A Papanicolaou test by itself is insufficient if there is a grossly visible lesion, as false-negatives occur in 10%-50% of tests. Ref: Berek JS (ed): Berek & Novaks Gynecology, ed 14. Lippincott Williams & Wilkins, 2007, p 20.

A 42-year-old female brings you the results of a comprehensive metabolic profile obtained through a health screening program offered by her employer. She fasted for 8 hours prior to the test, and her blood glucose level was reported as 110 mg/dL. Her lipid values and her blood pressure were normal, but her BMI is 30.5 kg/m 2 . She currently views herself as relatively healthy and reports no symptoms consistent with diabetes mellitus during your review of systems. Additional testing reveals a hemoglobin A1c of 6.3%. Based on this data, which one of the following is most appropriate at this time? (check one) A. Order a C-peptide level B. Order an islet cell antibody level C. Recommend lifestyle modifications only D. Start low-dose glyburide (DiaBeta) daily E. Start low-dose insulin glargine (Lantus) daily

C The ADA recommends testing to detect type 2 diabetes mellitus in asymptomatic adults with a BMI ≥25 kg/m 2 and one or more additional risk factors. Risk factors include physical inactivity, hypertension, an HDL-cholesterol level <35 mg/dL, a triglyceride level >250 mg/dL, a history of cardiovascular disease, a hemoglobin A 1c≥5.7%, a history of gestational diabetes or delivery of an infant weighing >4 kg (9 lb), and a history of polycystic ovary syndrome. Diabetes mellitus can be diagnosed if the patient's fasting blood glucose level is ≥126 mg/dL on two separate occasions. It can also be diagnosed if a random blood glucose level is ≥200 mg/dL if classic symptoms of diabetes are present. A fasting blood glucose level of 100-125 mg/dL, a glucose level of 140-199 mg/dL 2 hours following a 75-g glucose load, or a hemoglobin A 1c of 5.7%-6.9% signifies impaired glucose tolerance. Patients meeting these criteria have a significantly higher risk of progression to diabetes and should be counseled about lifestyle modifications such as weight loss and exercise.

A 25-year-old white male truck driver complains of 1 day of throbbing rectal pain. Your examination shows a large, thrombosed external hemorrhoid. Which one of the following is the preferred initial treatment for this patient? (check one) A. Warm sitz baths, a high-residue diet, and NSAIDs B. Rubber band ligation of the hemorrhoid C. Elliptical excision of the thrombosed hemorrhoid D. Stool softeners and a topical analgesic/hydrocortisone cream (e.g., Anusol-HC)

C The appropriate management of a thrombosed hemorrhoid presenting within 48 hours of onset of symptoms is an elliptical excision of the hemorrhoid and overlying skin under local anesthesia (i.e., 0.5% bupivacaine hydrochloride [Marcaine] in 1:200,000 epinephrine) infiltrated slowly with a small (27 gauge) needle for patient comfort. Incision and clot removal may provide inadequate drainage with rehemorrhage and clot reaccumulation. Most thrombosed hemorrhoids contain multilocular clots which may not be accessible through a simple incision. Rubber band ligation is an excellent technique for management of internal hemorrhoids. Banding an external hemorrhoid would cause exquisite pain. When pain is already subsiding or more time has elapsed (in the absence of necrosis or ulceration), measures such as sitz baths, bulk laxatives, stool softeners, and local analgesia may all be helpful. Some local anesthetics carry the risk of sensitization, however counseling to avoid precipitating factors (e.g., prolonged standing/sitting, constipation, delay of defecation) is also appropriate. Ref: Rakel RE, Bope ET (eds): Conn's Current Therapy 2004. Saunders, 2004, pp 538-541. 2) Townsend CM Jr, Beauchamp RD, Evers BM, et al: Sabiston Textbook of Surgery: The Biological Basis of Modern Surgical Practice, ed 17. Elsevier Saunders, 2004, pp 1490-1494.

A 37-year-old gravida 3 para 2 at 33 weeks' gestation reports the onset of brisk vaginal bleeding. On examination the uterus is nontender and 32 cm above the symphysis. Pelvic examination reveals the presence of a large amount of bright red vaginal blood. This presentation is most consistent with: (check one) A. Threatened abortion B. Hemorrhagic cystitis C. Placenta previa D. Chorioamnionitis E. Abruptio placentae

C The classic clinical presentation of placenta previa is painless, bright red vaginal bleeding. This diagnosis must be considered in all patients beyond 24 weeks' gestation who present with bleeding. Threatened abortion is unlikely at this stage of pregnancy and hemorrhagic cystitis is not accompanied by brisk bleeding. Abruption of the placenta is the most common cause of intrapartum fetal death but is associated not only with brisk vaginal bleeding, but also with uterine tenderness that may be marked. Clinical signs of chorioamnionitis include purulent vaginal discharge, fever, tachycardia, and uterine tenderness. Ref: Rakel RE: Textbook of Family Practice, ed 6. WB Saunders Co, 2002, pp 518-556.

The parents of a young child ask your advice about the need for fluoride supplementation in order to prevent tooth decay. Which one of the following is true regarding current U.S. Preventive Services Task Force guidelines for fluoride supplementation? (check one) A. It is not recommended due to potential fluoride toxicity B. Dental fluoride varnish is too toxic for routine use C. Oral fluoride supplementation is recommended if the primary drinking water source is low in fluoride D. Fluoridated toothpaste provides adequate protection if used as soon as the child has teeth E. The need for fluoride supplementation is determined by serum fluoride levels

C The current (2004) recommendation of the U.S. Preventive Services Task Force (USPSTF) is that children over the age of 6 months receive oral fluoride supplementation if the primary drinking water source is deficient in fluoride. The USPSTF cites "fair" evidence (B recommendation) that such supplementation reduces the incidence of dental caries and concludes that the overall benefit outweighs the potential harm from dental fluorosis. Dental fluorosis is chiefly a cosmetic staining of the teeth, is uncommon with currently recommended fluoride intake, and has no other functional or physiologic consequences. Fluoridated toothpaste can cause fluorosis in children younger than 2 years of age, and is therefore not recommended in this age group. Fluoridated toothpaste by itself does not reliably prevent tooth decay. Fluoride varnish, applied by a dental or medical professional, is another treatment option to prevent caries. It provides longer-lasting protection than fluoride rinses, but since it is less concentrated, it may carry a lower risk of fluorosis than other forms of supplementation. Oral fluoride supplementation for children over the age of 6 months is based not only on age but on the concentration of fluoride in the primary source of drinking water, whether it be tap water or bottled water. Most municipal water supplies in the United States are adequately fluoridated, but concentrations vary. Fluoride concentrations in bottled water vary widely. If the concentration is >0.6 ppm no supplementation is needed, and may result in fluorosis if given. Lower concentrations of fluoride may indicate the need for partial or full-dose supplementation.

A 3-year-old white female is brought to the emergency department with an acute onset of epistaxis. The child, who has a history of good health, is brought in by her recently-divorced mother, a registered nurse. The mother appears relatively unconcerned about the child's illness, but otherwise is friendly and interacts appropriately with the health care team evaluating the child. The child's vital signs are normal, but she is bleeding mildly from both nostrils and there are areas of ecchymosis. Laboratory Findings Hemoglobin 12.3 g/dL (N 11.5-15.0) Hematocrit 36% (N 32-42) WBC count 4500/mm3 (N 6000-15,000) Platelets 235,000/mm3 (N >50,000) Prothrombin time 40.0 sec (N 11.0-15.0) Partial thromboplastin time 30 sec (N 24-36) INR 3.9 sec (N 2.0-3.0) ALT (SGPT) 18 IU/L (N 7-35) AST (SGOT) 16 IU/L (N 15-60) Bilirubin 0.8 mg/dL (N 0.3-1.2) You hospitalize the child for observation and further testing. Her bleeding subsides in several hours, no new skin lesions develop, and her PT/INR decreases to 32 sec/3.0, 23 sec/2.1, and 15 sec/1.4 on subsequent days. You suspect that the child's condition is due to: (check one) A. Acetaminophen overdose B. Antiphospholipid syndrome with lupus anticoagulant C. Munchausen syndrome by proxy D. Henoch-Schönlein purpura E. Traumatic injury (child abuse)

C The patient exhibits signs of a moderate bleeding diathesis. Her prothrombin time (PT) elevation, without evidence of hepatocellular damage or hepatic dysfunction, is highly suspicious for warfarin ingestion. The normalization of the PT under observation in a hospital setting is consistent with this suspicion. Although accidental poisoning is a possibility, the mother's affect is highly suspicious for Munchausen syndrome by proxy. The fact that her mother is a health-care worker and develops a close and appropriate relationship with the health-care team is consistent with this diagnosis. Acetaminophen toxicity of this degree would likely produce transaminase and bilirubin elevations, as well as mental status changes. Antiphospholipid syndrome produces a hypercoagulable state. Henoch-Schonlein purpura presents with purpura, joint pain, abdominal pain, and a normal PT. Traumatic injury would not result in PT elevations. Ref: Behrman RE, Kliegman RM, Jenson HB (eds): Nelson Textbook of Pediatrics, ed 17. Saunders, 2004, pp 73, 122, 130-131.

A 65-year-old Hispanic male with known metastatic lung cancer is hospitalized because of decreased appetite, lethargy, and confusion of 2 weeks' duration. Laboratory evaluation reveals the following: Serum calcium......................... 15.8 mg/dL (N 8.4-10.0) Serum phosphorus...................... 3.9 mg/dL (N 2.6-4.2) Serum creatinine. ...................... 1.1 mg/dL (N 0.7-1.3) Total serum protein..................... 5.0 g/dL (N 6.0-8.0) Albumin.............................. 3.1 g/dL (N 3.7-4.8) Which one of the following is the most appropriate INITIAL management? (check one) A. Calcitonin-salmon (Miacalcin) subcutaneously B. Pamidronate disodium (Aredia) by intravenous infusion C. Normal saline intravenously D. Furosemide intravenously

C The initial management of hypercalcemia of malignancy includes fluid replacement with normal saline to correct the volume depletion that is invariably present and to enhance renal calcium excretion. The use of loop diuretics such as furosemide should be restricted to patients in danger of fluid overload, since these drugs can aggravate volume depletion and are not very effective alone in promoting renal calcium excretion. Although intravenous pamidronate has become the mainstay of treatment for the hypercalcemia of malignancy, it is considered only after the hypercalcemic patient has been rendered euvolemic by saline repletion. The same is true for the other calcium-lowering agents listed. Ref: Behl D, Hendrickson AW, Moynihan TJ: Oncologic emergencies. Crit Care Clin 2010;26(1):181-205.

A 13-year-old white female reports a 6-month history of intermittent abdominal cramping, with each episode becoming progressively worse. Based on her history, there is no obvious relationship to eating, voiding, or defecating. She reports that she has not yet begun menstruating and is not sexually active. Her weight has been stable. She appears to be in mild emotional distress about being "the last girl in her class to have a period." She is in no physical discomfort and her vital signs are normal. Secondary sexual characteristics appear to be developing normally. She is in the 57th percentile for height and the 65th percentile for weight. A complete physical examination confirms your presumptive diagnosis. The therapeutic procedure of choice would be: (check one) A. Appendectomy B. Colonoscopy C. Hymenotomy D. Cystoscopy E. Paracentesis

C The key to making a diagnosis of imperforate hymen, aside from the obvious finding on physical examination, lies in the systematic drawing of inferences. One can speculate that this patient's recurrent crescendo abdominal cramping represented six menstrual sheddings, with no egress from the body. Her delay in menarche, despite normal growth parameters, offers another clue that structural amenorrhea is present. Amounts of retained blood vary among patients; up to 3000 mL have been reported. A large volume can accumulate without causing any permanent damage, and subsequent fertility is usually normal. Hymenotomy will relieve the pressure, and normal menses should ensue. Ref: Emans SJH, Laufer MR, Goldstein DP (eds): Pediatric and Adolescent Gynecology, ed 4. Lippincott Williams & Wilkins, 1998, pp 334-335. 2) Wall EM, Stone B, Klein BL: Imperforate hymen: A not-so-hidden diagnosis. Am J Emerg Med 2003;21(3):249-250. 3) Behrman RE, Kliegman RM, Jenson HB (eds): Nelson Textbook of Pediatrics, ed 17. Saunders, 2004, pp 1839, 1841.

A 35-year-old male consults you about vague chest pain he developed while sitting at his desk earlier in the day. The pain is right-sided and was sharp for a brief time when it began, but it rapidly subsided. There was no hemoptysis and the pain does not seem pleuritic. His physical examination, EKG, and oxygen saturation are unremarkable. A chest film shows a 10% right pneumothorax. Which one of the following should you do next? (check one) A. Admit the patient to the hospital for observation B. Admit the patient to the hospital for chest tube placement C. Obtain a repeat chest radiograph in 24-48 hours D. Obtain an expiratory chest radiograph

C The majority of patients presenting with spontaneous pneumothorax are tall, thin individuals under 40 years of age. Most do not have clinically apparent lung disease, and the chest pain is sometimes minimal at the time of onset and may resolve within 24 hours even if untreated. Patients with small pneumothoraces involving <15% of the hemithorax may have a normal physical examination, although tachycardia is occasionally noted. The diagnosis is confirmed by chest radiographs. When a pneumothorax is suspected but not seen on a standard chest film, an expiratory film may be obtained to confirm the diagnosis. Studies have found that an average of 30% of patients will have a recurrence within 6 months to 2 years. An initial pneumothorax of <20% may be monitored if the patient has few symptoms. Follow-up should include a chest radiograph to assess stability at 24-48 hours. Indications for treatment include progression, delayed expansion, or the development of symptoms. The majority of patients with spontaneous pneumothoraces, and perhaps almost all of them, will have subcutaneous bullae on a CT scan. Ref: Marx JA (ed): Rosen's Emergency Medicine: Concepts and Clinical Practice, ed 7. Mosby Elsevier, 2010, pp 942-943.

A 12-year-old male middle-school wrestler comes to your office complaining of a recurrent painful rash on his arm. There appear to be several dry vesicles. The most likely diagnosis is which one of the following? (check one) A. Molluscum contagiosum B. Human papillomavirus C. Herpes gladiatorum D. Tinea corporis E. Mat burn

C The most common infection transmitted person-to-person in wrestlers is herpes gladiatorum caused by the herpes simplex virus. Molluscum contagiosum causes keratinized plugs. Human papillomavirus causes warts. Tinea corporis is ringworm, which is manifested by round to oval raised areas with central clearing. Mat burn is an abrasion. Ref: Buescher ES: Infections associated with pediatric sport participation. Pediatr Clin North Am 2002;49(4):743-751.

A 55-year-old male who has a long history of marginally-controlled hypertension presents with gradually increasing shortness of breath and reduced exercise tolerance. His physical examination is normal except for a blood pressure of 140/90 mm Hg, bilateral basilar rales, and trace pitting edema. Which one of the following ancillary studies would be the preferred diagnostic tool for evaluating this patient? (check one) A. 12-lead electrocardiography B. Posteroanterior and lateral chest radiographs C. 2-dimensional echocardiography with Doppler D. Radionuclide ventriculography E. Cardiac MRI

C The most useful diagnostic tool for evaluating patients with heart failure is two-dimensional echocardiography with Doppler to assess left ventricular ejection fraction (LVEF), left ventricular size, ventricular compliance, wall thickness, and valve function. The test should be performed during the initial evaluation. Radionuclide ventriculography can be used to assess LVEF and volumes, and MRI or CT also may provide information in selected patients. Chest radiography (posteroanterior and lateral) and 12-lead electrocardiography should be performed in all patients presenting with heart failure, but should not be used as the primary basis for determining which abnormalities are responsible for the heart failure. Ref: Smith L: Practice guidelines: Diagnosis and management of chronic heart failure in adults. Am Fam Physician 2007;75(5):742-755.

You diagnose Trichomonas vaginitis in a 25-year-old white female, and treat her and her partner with metronidazole (Flagyl), 2 g in a single dose. She returns 1 week later and is still symptomatic, and a saline wet prep again shows Trichomonas. Which one of the following is the most appropriate treatment at this time? (check one) A. Metronidazole gel 0.75% (MetroGel) intravaginally for 5 days B. Metronidazole, 2 g orally, plus metronidazole gel 0.75% intravaginally for 5 days C. Metronidazole, 500 mg orally twice a day for 7 days D. Clindamycin cream (Cleocin) 2% intravaginally for 7 days E. Sulfadiazine (Microsulfon), 4 g orally in a single dose, plus pyrimethamine (Daraprim), 200 mg

C The preferred treatment for Trichomonas vaginitis is metronidazole, 2 g given in a single oral dose. Certain strains of Trichomonas vaginalis, however, have diminished sensitivity to metronidazole. Patients who fail initial treatment with metronidazole should be retreated with 500 mg orally twice a day for 7 days. If treatment fails again, the patient should be treated with 2 g daily for 3-5 days. Metronidazole gel and clindamycin cream are useful for treating bacterial vaginosis, but are not effective in the treatment of Trichomonas vaginitis. Sulfadiazine and pyrimethamine are used to treat toxoplasmosis. Ref: Sexually transmitted diseases treatment guidelines—2002. MMWR 2002;51(RR-6):43-45.

A moderately obese 50-year-old African-American female presents with colicky right upper quadrant pain that radiates to her right shoulder. Which one of the following is considered the best study to confirm the likely cause of the patient's symptoms? (check one) A. Plain abdominal radiography B. Oral cholecystography C. Abdominal ultrasonography D. A barium swallow E. Esophagogastroscopy

C The symptom complex presented is typical of cholelithiasis. Plain radiography of the abdomen may reveal radiopaque gallstones, but will not reveal radiolucent stones or biliary dilatation. Although rarely used, oral cholecystography is 98% accurate, but only when compliance is assured, the contrast agent is absorbed, and liver function is normal. Abdominal ultrasonography is considered the best study to confirm this diagnosis because of its high sensitivity and its accuracy in detecting gallstones. A barium swallow will identify some functional and structural esophageal abnormalities, but will not focus on the suspected organ in this case. The same is true of esophagogastroscopy. Ref: Bland KI, Sarr MG (ed): The Practice of General Surgery. Saunders, 2002, pp 667-670.

In evaluating an adult with anemia, which one of the following findings most reliably indicates a diagnosis of iron deficiency anemia? (check one) A. Low total iron-binding capacity B. Low serum iron C. Low serum ferritin D. Microcytosis E. Hypochromia

C The total iron-binding capacity is elevated, not decreased, in iron deficiency anemia. As an acute-phase reactant, serum iron may be decreased in response to inflammation even when total body stores of iron are not decreased. Microcytosis and hypochromia are both features of iron deficiency anemia occurring late in its development, but both can also be seen in the thalassemias. Serum ferritin is also an acute-phase reactant but is normal or elevated in the face of an inflammatory process. A low serum ferritin, however, is diagnostic for iron deficiency even in its early stages. Ref: Braunwald E, Fauci AS, Kasper DL, et al (eds): Harrison's Principles of Internal Medicine, ed 15. McGraw-Hill, 2001, pp 662-663.

The use of automated external defibrillators by lay persons in out-of-hospital settings: (check one) A. Has been frustrated by liability concerns B. Has been hampered by an unwillingness to place the devices in public areas C. Has been shown to contribute to significant gains in full neurologic and functional recovery D. Has been eclipsed by the widespread use of internal cardiac defibrillators in high-risk patients

C The use of automated external defibrillators (AEDs) by lay persons, trained and otherwise, has been quite successful, with up to 40% of those treated recovering full neurologic and functional capacity. At present, 45 states have passed Good Samaritan laws covering the use of AEDs by well-intentioned lay persons. There are initiatives for widespread placement of AEDs, to include commercial airlines and other public facilities. Implantable cardioverter defibrillators (ICDs) are useful in known at-risk patients, but the use of AEDs is for the population at large. Ref: Marenco JP, Wang PJ, Link MS, et al: Improving survival from sudden cardiac arrest: The role of the automated external defibrillator. JAMA 2001;285(9):1193-1200. 2) Hallstrom AP, Ornato JP, Wesifeldt M, et al: Public-access defibrillation and survival after out-of-hospital cardiac arrest. The Public Access Defibrillation Trial Investigators. N Engl J Med 2004;351(7):637-646.

Your hospital administrator asks you to develop a community screening program for melanoma. Which one of the following is true concerning screening for this disease? (check one) A. Screening for melanoma is not indicated since the disease is rare B. Screening for melanoma is not indicated since screening takes too much time C. No definite clinical evidence has shown that screening for melanoma reduces mortality D. Because of sunbathing, female patients are the most important population to screen

C There have been no randomized, controlled trials or other definitive data to indicate that screening for melanoma reduces mortality. There are, however, factors which indicate that screening would be beneficial, including the increasing prevalence of the disease and the fact that screening is time-effective and safe. If screening is performed, populations at greatest risk should be considered. Men, especially those over age 50, have the highest incidence of melanoma. Ref: Geller AC: Screening for melanoma. Dermatol Clin 2002;20(4):629-640.

A 25-year-old medical student reads about the benefits of moderate alcohol consumption on lipid levels and begins to drink 5 ounces of red wine a day, adding 100 calories to his diet. Assuming that his diet and exercise levels stay the same, what effect will the additional 3000 calories a month have on his body weight over the next 10 years? (check one) A. They will have essentially no effect B. His weight will increase by about 25 kg C. His weight will increase slightly then stabilize D. His normal caloric expenditure will decrease

C There is not a direct relation between daily calorie consumption and weight. An adult male consuming an extra 100 calories a day above his caloric need will not continue to gain weight indefinitely; rather, his weight will increase to a certain point and then become constant. Fat must be fed, and maintaining the newly created tissue requires an increase in caloric expenditure. An extra 100 calories a day will result in a weight gain of approximately 5 kg, which will then be maintained. Ref: Katan MB, Ludwig DS: Extra calories cause weight gain—But how much? JAMA 2010;303(1):65-66.

An 18-month-old white male has been brought into your office multiple times over the past year with a reported fever of over 101 degrees F (38 degrees C). The child's reported temperatures at home have usually been higher than those measured at the time of the office visit. The remainder of the history is usually unremarkable. The child has a sibling who is in good health, but another sibling died several years ago for unknown reasons. On two occasions you diagnosed acute otitis media and acute bronchitis. However, at most visits the child has not had any abnormal physical findings. Repeated laboratory studies have been within normal limits, including complete blood counts, erythrocyte sedimentation rates, blood cultures, chest radiographs, and urinalyses. Almost always, the mother has reported little reduction in fever with age-appropriate doses of acetaminophen or ibuprofen. At the last office visit the child's temperature was measured at 40.6 degrees C (105.1 degrees F). Although the examination was once again unrevealing, it was decided to hospitalize the child for close observation and evaluation by an infectious disease consultant. Closed-circuit television observation in the hospital showed the mother putting the thermometer into hot water before a nurse came to record the patient's temperature. During the hospitalization you make a diagnosis. Which one of the following is a strong indicator of the suspected final diagnosis? (check one) A. The child has seen no other health-care provider but you B. Both parents have been involved with each office visit C. The child is afebrile while staying at the day-care center D. The parents have resisted having painful or risky diagnostic tests performed on the child

C This is a characteristic presentation of factitious disorder by proxy, or what is commonly known as Munchausen syndrome by proxy. Warning signs for this disorder include the episodes of illness beginning only when the child is, or has recently been, with the parent; the parent taking the child to numerous caregivers, resulting in multiple diagnostic evaluations but neither cure nor definitive diagnosis; the other parent (usually the father) being notably uninvolved despite the ostensible health crises; the parent not being assured by normal test results and continually advocating for painful or risky diagnostic tests for the child; the child persistently failing to tolerate or respond to usual medical therapies; and another child in the family having an unexplained illness or childhood death. Ref: Hales RE, Yudofsky SC (ed): Textbook of Clinical Psychiatry, ed 4. American Psychiatric Publishing, 2003, pp 697-707.

A 73-year-old white female with a long history of rheumatoid arthritis has a normocytic normochromic anemia. Her hemoglobin level is 9.8 g/dL (N 12.0-16.0) with decreased serum iron, decreased total iron-binding capacity, and increased serum ferritin. Which one of the following is the most appropriate treatment for this patient? (check one) A. Oral iron B. Intramuscular iron dextran (DexFerrum, InFeD) C. Treatment of the rheumatoid arthritis D. Transfusion E. Folic acid

C This patient has anemia of chronic disease secondary to her rheumatoid arthritis. This anemia is usually mild, with hemoglobin levels of 9.0-11.0 g/dL, and is usually normocytic-normochromic, although it can be microcytic. Characteristically, serum iron and total iron-binding capacity are decreased and ferritin is increased. The best treatment of this anemia is to treat the underlying systemic disease. Neither iron nor folic acid is effective. Since the anemia is usually mild, transfusion is not necessary.

A 30-year-old female presents with concerns about vaginal bleeding. She states that her menstrual periods have occurred at regular intervals of 28-30 days for the past 15 years, but recently bleeding has also occurred for a day or two in the middle of her cycle. This bleeding has been heavy enough to require the use of multiple pads. Which one of the following terms best describes her bleeding pattern? (check one) A. Polymenorrhea B. Mid-cycle spotting C. Metrorrhagia D. Menometrorrhagia E. Acute emergent abnormal uterine bleeding

C This patient has metrorrhagia, or "bleeding intermenstrual," characterized by bleeding heavy enough to require the use of multiple pads; the heavy bleeding occurs between normal menstrual bleeding. It is important to evaluate metrorrhagia because potential causes include cervical disease, problems with IUDs, endometritis, polyps, submucous myomas, endometrial hyperplasia, and cancer. Mid-cycle spotting, as the term implies, refers to light spotting and is often caused by a decline in estrogen levels. Polymenorrhea is bleeding occurring at intervals of less than 21 days. Menometrorrhagia is heavy and/or prolonged bleeding occurring at irregular, noncyclic intervals. Acute emergent abnormal uterine bleeding is characterized by significant blood loss resulting in hypovolemia. Ref: Albers JR, Hull SK, Wesley RM: Abnormal uterine bleeding. Am Fam Physician 2004;69(8):1915-1926.

You see a 6-year-old male for the third time in 3 months with a persistently painful hand condition. He has been treated with oral amoxicillin, followed by oral trimethoprim/ sulfamethoxazole (Bactrim, Septra), with no improvement. A physical examination reveals retraction of the proximal nail fold, absence of the cuticle, and erythema and tenderness around the nail fold area. The thumb and second and third fingers are affected on both hands. The patient is otherwise healthy. First-line treatment for this condition includes: (check one) A. warm soaks three times a day B. avoidance of emollient lotions C. a topical corticosteroid cream D. an oral antifungal agent

C This patient has symptoms and signs consistent with chronic paronychia. This condition is often associated with chronic immersion in water, contact with soaps or detergents, use of certain systemic drugs (antiretrovirals, retinoids) and, as is most likely in a 6-year-old child, finger sucking. Findings on examination are similar to those of acute paronychia, with tenderness, erythema, swelling, and retraction of the proximal nail fold. Often the adjacent cuticle is absent. Chronic paronychia has usually been persistent for at least 6 weeks by the time of diagnosis. In addition to medication, basic treatment principles for the condition include avoidance of contact irritants, avoiding immersion of the hands in water, and use of an emollient. Topical corticosteroids have higher efficacy for treating chronic paronychia compared to oral antifungals (SOR B), particularly given the young age of the patient. A topical antifungal can also be tried in conjunction with the corticosteroid.

A 65-year-old white female comes to your office with evidence of a fecal impaction which you successfully treat. She relates a history of chronic laxative use for most of her adult years. After proper preparation, you perform sigmoidoscopy and note that the anal and rectal mucosa contain scattered areas of bluish-black discoloration. Which one of the following is the most likely explanation for the sigmoidoscopic findings? (check one) A. Endometriosis B. Collagenous colitis C. Melanosis coli D. Metastatic malignant melanoma E. Arteriovenous malformations

C This patient has typical findings of melanosis coli, the term used to describe black or brown discoloration of the mucosa of the colon. It results from the presence of dark pigment in large mononuclear cells or macrophages in the lamina propria of the mucosa. The coloration is usually most intense just inside the anal sphincter and is lighter higher up in the sigmoid colon. The condition is thought to result from fecal stasis and the use of anthracene cathartics such as cascara sagrada, senna, and danthron. Ectopic endometrial tissue (endometriosis) most commonly involves the serosal layer of those parts of the bowel adjacent to the uterus and fallopian tubes, particularly the rectosigmoid colon. Collagenous colitis does not cause mucosal pigmentary changes. Melanoma rarely metastasizes multicentrically to the bowel wall. Multiple arteriovenous malformations are more common in the proximal bowel, and would not appear as described. Ref: Feldman M, Friedman LS, Sleisenger MH (eds): Sleisenger & Fordtran's Gastrointestinal and Liver Disease, ed 7. WB Saunders Co, 2004, p 2305. 2) Hardman JG, Limbird LE, Gilman AG (eds): Goodman & Gilman's The Pharmacological Basis of Therapeutics, ed 10. McGraw-Hill, 2001, pp 1046-1047. 3) Kasper DL, Braunwald E, Fauci AS, et al (eds): Harrison's Principles of Internal Medicine, ed 16. McGraw-Hill, 2005, pp 231-233.

A 68-year-old African-American female is brought to your office by her daughter, who tells you that her mother has recently been exhibiting short-term memory loss and confusion. For example, she has difficulty remembering how to get dressed appropriately and sometimes forgets to turn off the oven after using it. These symptoms developed fairly abruptly. The patient's medical problems include type 2 diabetes mellitus, hypertension, hypercholesterolemia, and osteoarthritis. She had a stroke last year and has residual mild hemiparesis. A physical examination is normal except for mild hemiparesis. On cognitive testing she is able to recall only one of three words, and all the numbers are on one side on the clock-drawing test. Which one of the following types of dementia is most likely in this patient? (check one) A. Alzheimer's disease B. Dementia with Lewy bodies C. Vascular dementia D. Frontotemporal dementia E. Multisystem atrophy

C This patient's history and examination meet the criteria for vascular dementia published by the National Institute of Neurological Disorders and Stroke, and the Association Internationale pour la Neurosciences (NINDS-AIREN). Significant findings include cognitive decline from a previously higher level of functioning, manifested by impairment of memory and of two or more cognitive domains, and evidence of cerebrovascular disease by focal signs on neurologic examination, consistent with stroke. To fully meet the NINDS-AIREN criteria, she would need to have neuroimaging that demonstrates characteristic vascular dementia lesions. Ref: Aggarwal NT, DeCarli C: Vascular dementia: Emerging trends. Semin Neurol 2007;27(1):66-77.

A 31-year-old married white female complains of vaginal discharge, odor, and itching. Speculum examination reveals a homogeneous yellow discharge, vulvar and vaginal erythema, and a "strawberry" cervix. The most likely diagnosis is: (check one) A. Candidal vaginitis B. Bacterial vaginosis C. Trichomonal vaginitis D. Chlamydial infection E. Herpes simplex type 2

C Trichomonal vaginitis usually causes a yellowish discharge which sometimes has a frothy appearance. Colpitis macularis (strawberry cervix) is often present. Monilial vaginitis classically causes a cheesy, whitish exudate with associated vaginal itching and burning. There may be vaginal and vulvar erythema and edema, but colpitis macularis is not a feature. Bacterial vaginosis is characterized by a grayish discharge with few other physical signs or symptoms, if any. Chlamydia may cause a yellowish cervical discharge and symptoms of pelvic inflammatory disease or, alternatively, may be totally asymptomatic. Herpes simplex type 2 causes ulcerations on the vulva and vaginal mucosa which are exquisitely tender, often with marked surrounding erythema and edema. Ref: Scott JR, Di Saia PJ, Hammond CB, et al (eds): Danforth's Obstetrics and Gynecology, ed 8. Lippincott Williams & Wilkins, 1999, pp 585-586.

Patients with rheumatoid arthritis should be screened for tuberculosis before starting which one of the following medications? (check one) A. Gold B. Hydroxychloroquine (Plaquenil) C. Infliximab (Remicade) D. Methotrexate (Rheumatrex) E. Sulfasalazine (Azulfidine)

C Tumor necrosis factor inhibitors have been associated with an increased risk of infections, including tuberculosis. This class of agents includes monoclonal antibodies such as infliximab, adalimumab, certolizumab pegol, and golimumab. Patients should be screened for tuberculosis and hepatitis B and C before starting these drugs. The other drugs listed can have adverse effects, but do not increase the risk for tuberculosis. Ref: Scott DL, Wolfe F, Huizinga TW: Rheumatoid arthritis. Lancet 2010;376(9746):1094-1108.

A 21-year-old female complains of bulging veins in her right shoulder region, along with swelling and a "tingling" sensation in her right arm that has developed over the past 2 days. There were no unusual events other than her regular workouts with her swim team. Ultrasonography confirms an upper extremity deep-vein thrombosis of her right axillary vein. Which one of the following would be the most appropriate treatment? (check one) A. Intravenous heparin for 72 hours, followed by oral warfarin (Coumadin) for 3 months B. Low molecular weight heparin (LMWH) subcutaneously for 5 days only C. LMWH subcutaneously for at least 5 days, followed by oral warfarin for 3 months D. LMWH subcutaneously for at least 5 days, followed by oral warfarin indefinitely E. Oral warfarin for 3 months

C Upper extremity deep-vein thrombosis (UE-DVT) accounts for 4% of all cases of DVT. Catheter-related thromboses make up the majority of these cases. Occult cancer, use of oral contraceptives, and inheritable thrombophilia are other common explanations. Another proposed risk factor is the repetitive compression of the axillary-subclavian vein in athletes or laborers, which is the most likely cause of this patient's UE-DVT. Taken as a whole, UE-DVT is generally associated with fewer venous complications, including less chance for thromboembolism, postphlebitic syndrome, and recurrence compared to lower-extremity deep-vein thrombosis (LE-DVT). However, the rates of these complications are still high enough that most experts recommend treatment identical to that of LE-DVT. Specifically, heparin should be given for 5 days, and an oral vitamin-K antagonist for at least 3 months.

A 4-month-old white male in respiratory distress is brought to the emergency department. On examination, heart sounds include a grade 4/6 pansystolic murmur, best heard at the lower left sternal border. He is acyanotic. A chest radiograph shows an enlarged heart and increased pulmonary vascular markings, and an EKG shows combined ventricular hypertrophy. Of the following, the most likely diagnosis is: (check one) A. hypoplastic left heart syndrome (aortic valve atresia) B. transposition of the great vessels C. ventricular septal defect D. tetralogy of Fallot E. patent ductus arteriosus

C Ventricular septal defect causes overload of both ventricles, since the blood is shunted left to right. The murmur is harsh and holosystolic, generally heard best at the lower left sternal border. As the volume of the shunting increases, cardiac enlargement and increased pulmonary vascular markings can be seen on a chest radiograph. Hypoplastic left heart syndrome would be manifested by near-obliteration of the left ventricle on the EKG and chest radiograph, and the infant would be cyanotic. Transposition of the great vessels would cause AV conduction defects and single-sided hypertrophy on the EKG. The chest radiograph would show a straight shoulder on the left heart border where the aorta was directed to the right. Tetralogy of Fallot causes cyanosis and right ventricular enlargement. The murmur of patent ductus arteriosus is continuous, best heard below the left clavicle. The EKG shows left atrial and ventricular enlargement. Ref: Rudolph CD, Rudolph AM, Lister G, et al (eds): Rudolph's Pediatrics, ed 22. McGraw-Hill, 2011, pp 1807-1808.

A 67-year-old male who recently had a screening colonoscopy presents for follow-up. During the procedure, a mass was discovered in the sigmoid colon and a biopsy revealed a poorly differentiated adenocarcinoma. When you tell the patient you have the test results and can provide information about the prognosis, he says, "To be honest, I can tell that the news is not good, and I would rather not talk about it right now." Which one of the following would be the most appropriate next step? (check one) A. Relay the prognosis, but focus as much as possible on any positives B. Discuss the prognosis with the patient's wife, who is in the waiting room C. Ask the patient if he would like to talk again at another time D. Refer the patient to a support group

C When giving bad news to a patient, it is important to assess how much information the patient wants to know and tailor the discussion accordingly (SOR C). If the patient states that he does not want to know about the prognosis, the most appropriate response is to ask if he would like to talk again at another time. The physician may be regarded as rude, cruel, or uncaring if he persists in providing information that the patient is not ready to hear. In addition, the patient is more at risk for feelings of hopelessness, depression, or anxiety if he is not psychologically ready to hear a bad prognosis. Once the patient verbalizes a readiness to discuss the prognosis, specific information can be provided, focusing on both the positive and negative aspects of the situation. It is not appropriate to discuss the case with the patient's wife, unless he specifically requests she be a proxy to receive the information. Referrals to either a support group or an oncologist should be deferred until the information has been discussed with the patient. Ref: Ngo-Metzger Q, August KJ, Srinivasan M, et al: End-of-life care: Guidelines for patient-centered communication. Am Fam Physician 2008;77(2):167-174.

Which one of the following describes the McRoberts maneuver for managing shoulder dystocia? (check one) A. Suprapubic pressure B. Delivery of the posterior arm C. Maximal flexion and abduction of the maternal hips D. Rolling the mother to an "all-fours" position E. Rotation of the fetal head

C When the just-delivered fetal head retracts firmly against the perineum, shoulder dystocia is apparent. This is an obstetric emergency that requires appropriate assistance and a calm but timely approach to ensure a safe delivery. While all of the maneuvers described are steps in managing shoulder dystocia, the McRoberts maneuver by itself (maximal flexion and abduction of the maternal hips) relieves the impaction of the anterior shoulder against the maternal symphysis in a large percentage of cases, especially when combined with suprapubic pressure. Ref: Baxley EG, Gobbo RW: Shoulder dystocia. Am Fam Physician 2004;69(7):1707-1713.

A 72-year-old white male in otherwise good health complains of generalized pruritus that worsens in the winter. The itching is most intense after he bathes. He recently noticed a rash on his abdomen and legs as well. On examination you note poorly defined red, scaly plaques with fine fissures on the abdomen. No eruption is present at other pruritic sites. Which one of the following is the most likely cause of this problem? (check one) A. Stasis dermatitis B. Lichen simplex chronicus C. Xerosis D. Rosacea E. Candidiasis

C Xerosis is a pathologic dryness of the skin that is especially prominent in the elderly. It is probably caused by minor abnormalities in maturation of the epidermis that lead to decreased hydration of the superficial portion of the stratum corneum. Xerosis often intensifies in winter, because of the lower humidity and cold temperatures. Stasis dermatitis, due to chronic venous insufficiency, appears as a reddish-brown discoloration of the lower leg. Lichen simplex chronicus, the end result of habitual scratching or rubbing, usually presents as isolated hyperpigmented, edematous lesions, which become scaly and thickened in the center. Rosacea is most often seen on the face as an erythematous, acneiform eruption, which flushes easily and is surrounded by telangiectasia. Candidiasis is an opportunistic infection favoring areas that are warm, moist, and macerated, such as the perianal and inguinal folds, inframammary folds, axillae, interdigital areas, and corners of the mouth. Ref: Duthie EH Jr, Katz PR, Malone ML (eds): Practice of Geriatrics, ed 4. Saunders Elsevier, 2007, pp 536-537.

Which one of the following is most characteristic of patellofemoral pain syndrome in adolescent females? (check one) A. Posterior knee pain B. Pain exacerbated by walking on a flat surface C. Inadequate hip abductor strength D. A high rate of surgical intervention

C atellofemoral pain syndrome is a common overuse injury observed in adolescent girls. The condition is characterized by anterior knee pain associated with activity. The pain is exacerbated by going up or down stairs or running in hilly terrain. It is associated with inadequate hip abductor and core strength; therefore, a prescription for a rehabilitation program is recommended. Surgical intervention is rarely required.

You are writing a prescription for amoxicillin for a 6-year-old female with acute otitis media. Her mother has had an anaphylactic reaction to penicillin in the past and is concerned that she may have passed this trait down to her daughter. You reassure her that this is not usually the case but warn her about potential signs of an allergic reaction. Which one of the following is the most concerning early symptom of a dangerous drug reaction? A) Tachycardia and elevated blood pressure B) Small, bright, erythematous macules diffusely over the trunk C) Pruritus around the mouth and on the palms of the hands and soles of the feet D) Eczematous patches in the antecubital and popliteal fossae E) Diarrhea with blood on the tissue paper

C llergic reactions to medications have four primary mechanisms, referred to as Gell and Coombs classifications. The most frequent forms are type I reactions, which are immediate and mediated through IgE, and type IV reactions, which are delayed and mediated through T-cell hypersensitization. Severe type I reactions are often referred to as anaphylaxis and are the most likely to be life threatening with very little warning. Recognition of the early signs of anaphylaxis is the first step in preventing such catastrophes. Anaphylactic reactions result from a massive release of histamine and start with pruritus around the mouth, on the scalp, and on the palms and soles; flushing of the face and neck, with rhinitis and conjunctivitis; angioedema of the oral mucosa, especially of the pharynx and larynx; severe urticaria; dyspnea and bronchospasm (especially in known asthmatics); and hypotension. A delay in lifesaving therapy during this phase will result in full shock, hypotension, and death. Type IV reactions usually result in benign, diffuse erythematous macules on the trunk and proximal extremities, often referred to as a drug rash. These reactions infrequently become more severe and rarely are life threatening. In severe cases the lesions become painful and palpable, and may involve blistering, mucositis, and ecchymosis.

Which one of the following is recommended for the treatment of cough and cold symptoms in children younger than 2 years of age? (check one) A. Cough suppressants and humidified air B. Intranasal decongestants C. Nonsedating antihistamines and decongestants D. Nasal saline with bulb suction E. Antibiotics and hydration

D No medication available in the United States has been shown to effectively treat cough or cold symptoms in children younger than 2 years of age. However, many agents are commonly prescribed despite reports of numerous minor, and some serious, adverse effects. For this reason, the American Academy of Pediatrics and the American Academy of Family Physicians recommend using only nasal saline, bulb suction, humidified air, and good hydration in children younger than 2 years of age. Ref: Simasek M, Blandino DA: Treatment of the common cold. Am Fam Physician 2007;75(4):515-520.

An unconscious 22-year-old male is brought into the emergency department. His respiratory rate is 8/min, his pulse rate is 60 beats/min and regular, and his pupils are miotic. The most likely cause of his condition is: (check one) A. Organophosphate poisoning B. Scopolamine overdose C. Narcotics overdose D. Benzodiazepine overdose

C n cases of drug overdose, several critical physical findings must be evaluated. The most important is the size of the pupils. Tolerance rarely reduces the miotic effects of narcotic medications. A patient who is comatose, with decreased breathing, a slow pulse, and small pupils should be strongly suspected of having overdosed on a narcotic. Naloxone should be administered to reverse these effects. The response to treatment with naloxone is irregular. Cerebral infarction in the pontine angle, organophosphate poisoning, phenothiazine overdose, and treatment for glaucoma can also cause constricted pupils, but these associations are seen much less frequently than narcotics overdose. Ref: Longo DL, Fauci AS, Kasper DL, et al (eds): Harrison's Principles of Internal Medicine, ed 18. McGraw-Hill, 2012, p 3554.

A 35-year-old white male with known long QT syndrome has a brief episode of syncope requiring cardiopulmonary resuscitation. Which one of the following is most likely responsible for this episode? (check one) A. Sinus tachycardia B. Atrial flutter with third degree block C. Asystole D. Torsades de pointes

D Patients with long QT syndrome that have sudden arrhythmia death syndrome usually have either torsades de pointes or ventricular fibrillation. Sinus tachycardia would not explain the syncope, and atrial flutter and asystole are not usual in long QT syndrome. Ref: Meyer JS, Mehdirad A, Salem BI, et al: Sudden arrhythmia death syndrome: Importance of the long QT syndrome. Am Fam Physician 2003;68(3):483-488

A 60-year-old male has a drug-eluting stent placed in his right coronary artery. He will require treatment to prevent stent thrombosis, and once his initial treatment period is completed he will be placed on aspirin, 75-165 mg/day indefinitely. Which one of the following is the preferred initial regimen for preventing stent thrombosis in this situation? (check one) A. Aspirin/dipyridamole (Aggrenox) for 3 months B. Aspirin, 162-325 mg/day for 3 months C. Aspirin, 162-325 mg/day, plus clopidogrel (Plavix), both for 3 months D. Aspirin, 162-325 mg/day, plus clopidogrel, both for 12 months E. Warfarin (Coumadin) for 3 months

D A 60-year-old male has a drug-eluting stent placed in his right coronary artery. He will require treatment to prevent stent thrombosis, and once his initial treatment period is completed he will be placed on aspirin, 75-165 mg/day indefinitely. Which one of the following is the preferred initial regimen for preventing stent thrombosis in this situation? (check one) A. Aspirin/dipyridamole (Aggrenox) for 3 months B. Aspirin, 162-325 mg/day for 3 months C. Aspirin, 162-325 mg/day, plus clopidogrel (Plavix), both for 3 months D. Aspirin, 162-325 mg/day, plus clopidogrel, both for 12 months E. Warfarin (Coumadin) for 3 months Correct. In patients with a drug-eluting stent, combined therapy with clopidrogel and aspirin is recommended for 12 months because of the increased risk of late stent thrombosis. After this time, aspirin at a dosage of 75-165 mg/day is recommended. The minimum duration of combined therapy is 1 month for a bare metal stent, 3 months for a sirolimus-eluting stent, and 6 months for other drug-eluting stents.

A 35-year-old male with a previous history of kidney stones presents with symptoms consistent with a recurrence of this problem. The initial workup reveals elevated serum calcium. Which one of the following tests would be most appropriate at this point? (check one) A. Serum calcitonin B. 24-hour urine for calcium and phosphate C. Serum phosphate and magnesium D. Serum parathyroid hormone E. Spot urine for microalbumin

D A patient with a recurrent kidney stone and an elevated serum calcium level most likely has hyperparathyroidism, and a parathyroid hormone (PTH) level would be appropriate. Elevated PTH is caused by a single parathyroid adenoma in approximately 80% of cases. The resultant hypercalcemia is often discovered in asymptomatic persons having laboratory work for other reasons. An elevated PTH by immunoassay confirms the diagnosis. In the past, tests based on renal responses to elevated PTH were used to make the diagnosis. These included blood phosphate, chloride, and magnesium, as well as urinary or nephrogenous cyclic adenosine monophosphate. These tests are not specific for this problem, however, and are therefore not cost-effective. Serum calcitonin levels have no practical clinical use. Ref: Fauci AS, Braunwald E, Kasper DL, et al (eds): Harrison's Principles of Internal Medicine, ed 17. McGraw-Hill, 2008, pp 2380-2383.

Which one of the following provides the best evidence for a given therapeutic intervention? (check one) A. An individual randomized, controlled trial B. A prospective case-control study C. A systematic review of cohort studies D. A systematic review of randomized, controlled trials

D A systematic review is a literature review focused on a research question that tries to identify, appraise, select, and synthesize all high-quality research evidence relevant to that question. A randomized, controlled trial (RCT) involves a group of patients who are randomized into an experimental group and a control group. These groups are followed for the outcomes of interest. The process of randomization minimizes bias and is thus the individual study type that is most likely to provide accurate results about an intervention's effectiveness. A cohort study is a nonexperimental study design that follows a group of people (a cohort), and then looks at how events differ among people within the group. A study that examines a cohort of persons who differ in respect to exposure to some suspected risk factor such as smoking is useful for trying to ascertain whether exposure is likely to cause specified events such as lung cancer. This study design is less reliable due to inherent biases that may not be accounted for and may exist in the groupings of patients. Retrospective and prospective case-control studies compare people with a disease or specific diagnosis with people who do not have the disease. The groups are studied to find out if other characteristics are also different between the two groups. This type of study often overestimates the benefit of a trial and is of lower quality than a randomized, controlled trial. Ref: OCEBM Table of Evidence Working Group: The Oxford 2011 Table of Evidence. Oxford Centre for Evidence-Based Medicine, 2009. 2) Shaughnessy AF: Evaluating and understanding articles about treatment. Am Fam Physician 2009;79(8):668-670.

A painful thrombosed external hemorrhoid diagnosed within the first 24 hours after occurrence is ideally treated by: (check one) A. appropriate antibiotics B. office banding C. office cryotherapy D. thrombectomy under local anesthesia E. total hemorrhoidectomy

D A thrombosed external hemorrhoid is manifested by the sudden development of a painful, tender, perirectal lump. Because there is somatic innervation, the pain is intense, and increases with edema. Treatment involves excision of the acutely thrombosed tissue under local anesthesia, mild pain medication, and sitz baths. It is inappropriate to use procedures that would increase the pain, such as banding or cryotherapy. Total hemorrhoidectomy is inappropriate and unnecessary.

When added to compression therapy, which one of the following has been shown to be an effective adjunctive treatment for venous ulcers? (check one) A. Warfarin (Coumadin) B. Enoxaparin (Lovenox) C. Clopidogrel (Plavix) D. Pentoxifylline (Trental) E. Atorvastatin (Lipitor)

D Pentoxifylline is effective when used with compression therapy for venous ulcers, and may be useful as monotherapy in patients unable to tolerate compression therapy. Aspirin has also been shown to be effective. Other treatments that have been studied but have not been found to be effective include oral zinc and antibiotics (SOR A).

A 22-year-old competitive cross-country skier presents with a complaint of not being able to perform as well as she expects. She has been training hard, but says she seems to get short of breath more quickly than she should. She also coughs frequently while exercising. A review of systems is otherwise negative. Her family history is negative for cardiac or pulmonary diseases. Her physical examination is completely normal, and pulmonary function tests obtained before and after bronchodilator use are normal. After you discuss your findings with the patient, she acknowledges that her expectations may be too high, but can think of no other cause for her problem. Which one of the following would be the next reasonable step? (check one) A. An echocardiogram to look for cardiomyopathy or valvular dysfunction B. Counseling regarding competition stress and athlete burnout syndrome C. A sports medicine consultation to evaluate her training regimen D. A trial of inhaled albuterol (Proventil) for exercise-induced bronchospasm

D A trial of inhaled albuterol (Proventil) for exercise-induced bronchospasm

Which one of the following is preferred for chronic treatment of congestive heart failure due to left ventricular systolic dysfunction? (check one) A. Diuretics B. Digoxin C. Calcium channel blockers D. ACE inhibitors E. Hydralazine (Apresoline) plus isosorbide dinitrate (Isordil, Sorbitrate)

D ACE inhibitors are the preferred drugs for congestive heart failure due to left ventricular systolic dysfunction, because they are associated with the lowest mortality. The combination of hydralazine/isosorbide dinitrate is a reasonable alternative, and diuretics should be used cautiously. It is not known whether digoxin affects mortality, although it can help with symptoms. Ref: ACC/AHA Guidelines for the evaluation and management of chronic heart failure in the adult: Executive summary. Circulation 2001;104(24):2996-3007.

Which one of the following drug classes is preferred for treating hypertension in patients who also have diabetes mellitus? (check one) A. Centrally-acting sympatholytics B. Alpha-blocking agents C. Beta-blocking agents D. ACE inhibitors E. Calcium channel blockers

D ACE inhibitors have proven beneficial in patients who have either early or established diabetic renal disease. They are the preferred therapy in patients with diabetes and hypertension, according to guidelines from the American Diabetes Association, the National Kidney Foundation, the World Health Organization, and the JNC VII report. Ref: Konzem SL, Devore VS, Bauer DW: Controlling hypertension in patients with diabetes. Am Fam Physician 2002;66(7):1209-1214. 2) Chobanian AV, Bakris GL, Black HR, et al: Seventh report of the Joint National Committee on Prevention, Detection, Evaluation and Treatment of High Blood Pressure. Hypertension 2003;42(6):1206-1252.

The earliest presenting symptom in most older patients with open-angle glaucoma is: (check one) A. Unilateral eye pain B. Unilateral eye redness C. Unilateral visual loss D. Tunnel vision E. Double vision

D About 3% of persons over age 55 have glaucoma, making it a leading cause of vision impairment. Although it is usually asymptomatic, the most common presenting symptom is tunnel vision, a gradual loss of peripheral vision. Ref: Distelhorst JS, Hughes GM: Open-angle glaucoma. Am Fam Physician 2003;67(9):1937-1944.

A 72-year-old male with COPD presents to the emergency department with an acute exacerbation marked by increased sputum production and shortness of breath. His oxygen saturation is 88% on room air and he has diffuse inspiratory and expiratory wheezes bilaterally. In addition to oxygen and bronchodilators, which one of the following is most appropriate for this patient? (check one) A. No additional treatments B. Systemic corticosteroids only C. Inhaled corticosteroids only D. Systemic corticosteroids and antibiotics E. Inhaled corticosteroids and antibiotics

D Acute exacerbations of COPD are very common, with most caused by superimposed infections. Supplemental oxygen, antibiotics, and bronchodilators are used for management. Systemic corticosteroids, either oral or parenteral, have been shown to significantly reduce treatment failures and improve lung function and dyspnea over the first 72 hours, although there is an increased risk of adverse drug reactions.

A nursing-home resident is hospitalized, and shortly before she is to be discharged she develops a skin ulcer, which proves to be infected with methicillin-resistant Staphylococcus aureus (MRSA). Which one of the following is most important in terms of infection control when she returns to the nursing home? (check one) A. Surveillance cultures of nursing-home residents living near the patient B. Aggressive housekeeping in the patient's room C. Masks, gowns, and gloves for all those entering the patient's room D. Strict handwashing practices by all staff, visitors, and residents E. Isolation of the patient in a room by herself

D All staff, visitors, and nursing-home residents should observe strict handwashing practices in this situation. Barrier precautions for wounds and medical devices should also be initiated. Surveillance cultures are not warranted. Aggressive housekeeping practices play little, if any, role in preventing the spread of MRSA. Isolating the patient is not practical or cost-effective. Ref: Unwin BK, Porvaznik M, Spoelhof GD: Nursing home care: Part I. Principles and pitfalls of practice. Am Fam Physician 2010;81(10):1219-1227.

A 27-year-old male presents with what he thinks is a sinus infection. He has a 2-day history of right maxillary pain associated with nasal congestion and clear rhinorrhea. The only significant findings on examination are a low-grade fever and subjective tenderness with palpation over the right maxillary sinus. Which one of the following treatments is most supported by current evidence? (check one) A. Antihistamines B. Oral decongestants C. Topical vasoconstrictor sprays D. Oral analgesics E. Nasal lavage

D Although oral antibiotics are overwhelmingly prescribed as initial treatment in acute sinusitis, it has been shown that the majority of acute illnesses are viral in origin and that 98% of cases will resolve spontaneously. Analgesics are considered the mainstay of therapy for acute sinusitis, according to evidence-based recommendations (SOR A). Other treatments should be considered if symptoms are prolonged (>7 days) or severe (two or more localizing symptoms or signs of serious bacterial complications). There is little evidence of effectiveness for antihistamines, oral decongestants, or vasoconstrictor sprays. There is also little evidence of effectiveness for nasal lavage in acute sinusitis, although it has an emerging role in chronic sinusitis. Ref: Scheid DC, Hamm RM: Acute bacterial rhinosinusitis in adults: Part II. Treatment. Am Fam Physician 2004;70(9):1697-1704. 2) Ah-See KW, Evans AS: Sinusitis and its management. BMJ 2007;334(7589):358-361.

A 52-year-old male with stable coronary artery disease and controlled hypertension sees you for a routine visit and asks for advice regarding prevention of altitude illness for his upcoming trip to Bhutan to celebrate his anniversary. His medical chart indicates that he had a reaction to a sulfa drug in the past. Which one of the following would be most appropriate? (check one) A. Advise the patient to not make the trip B. Recommend ginkgo biloba C. Prescribe acetazolamide D. Prescribe dexamethasone

D Altitude illness is common, affecting 25%-85% of travelers to high altitudes. The most common manifestation is acute mountain sickness, heralded by malaise and headache. Risk factors include young age, residence at a low altitude, rapid ascent, strenuous physical exertion, and a previous history of altitude illness. However, activity restriction is not necessary for patients with coronary artery disease who are traveling to high altitudes (SOR C). Ginkgo biloba has been evaluated for both prevention and treatment of acute mountain sickness and high-altitude cerebral edema, and it is not recommended. Acetazolamide is an effective prophylactic agent (SOR B), but is contraindicated in patients with a sulfa allergy. If used, it should be started a minimum of one day before ascent and continued until the patient acclimatizes at the highest planned elevation. Dexamethasone is an effective prophylactic and treatment agent (SOR B), and it is not contraindicated for those with a sulfa allergy. It would be the best option for this patient.

A 75-year-old African-American male with no previous history of cardiac problems complains of shortness of breath and a feeling of general weakness. His symptoms have developed over the past 24 hours. On physical examination you find a regular pulse with a rate of 160 beats/min. You note rales to the base of the scapula bilaterally, moderate jugular venous distention, and hepatojugular reflux. His blood pressure is 90/55 mm Hg; when he sits up he becomes weak and diaphoretic and complains of precordial pressure. An EKG reveals atrial flutter with 2:1 block. Management at this time should include: (check one) A. intravenous digoxin B. intravenous verapamil (Calan, Isoptin) C. amiodarone (Cordarone) D. electrical cardioversion E. insertion of a pacemaker

D Atrial flutter is not ordinarily a serious arrhythmia, but this patient has heart failure manifested by rales, jugular venous distention, hepatojugular reflux, hypotension, and angina. Electrical cardioversion should be performed immediately. This is generally a very easy rhythm to convert. Digoxin and verapamil are appropriate in hemodynamically stable patients. A pacemaker for rapid atrial pacing may be beneficial if digitalis intoxication is the cause of atrial flutter, but this is unlikely in a patient with no previous history of cardiac problems. Amiodarone is not indicated in this clinical situation.

A 55-year-old white male smoker has had daily severe gastroesophageal reflux symptoms unrelieved by intensive medical therapy with proton pump inhibitors. A recent biopsy performed during upper endoscopy identified Barrett's esophagus. Which one of the following is true about this condition? (check one) A. It will regress after antireflux surgery B. It will regress following esophageal dilation C. It will regress after Helicobacter pylori treatment D. It is associated with an increased risk of adenocarcinoma

D Barrett's esophagus is an acquired intestinal metaplasia of the distal esophagus associated with longstanding gastroesophageal acid reflux, although a quarter of patients with Barrett's esophagus have no reflux symptoms. It is more common in white and Hispanic men over 50 with longstanding severe reflux symptoms, and possible risk factors include obesity and tobacco use. It is a risk factor for adenocarcinoma of the esophagus, with a rate of about one case in every 200 patients with Barrett's esophagus per year. Treatment is directed at reducing reflux, thus reducing symptoms. Neither medical nor surgical treatment has been shown to reduce the carcinoma risk. One reasonable screening suggestion is to perform esophagoduodenoscopy in all men over 50 with gastroesophageal reflux disease (GERD), but these recommendations are based only on expert opinion (level C evidence), and no outcomes-based guidelines are available. Ref: Heidelbaugh JJ, Nostrant TT, Kim C, et al: Management of gastroesophageal reflux disease. Am Fam Physician 2003;68(7):1311-1318. 2) Shaluta MD, Saad R: Barrett's esophagus. Am Fam Physician 2004;69(9):2113-2118.

Breastfeeding a full-term, healthy infant is contraindicated when which one of the following maternal conditions is present? (check one) A. Chronic hepatitis B infection B. Seropositive cytomegalovirus carrier state C. Current tobacco smoking D. Herpes simplex viral lesions on the breasts E. Undifferentiated fever

D Breastfeeding provides such optimal nutrition for an infant that the benefits still far outweigh the risks even when the mother smokes tobacco, tests positive for hepatitis B or C virus, or develops a simple undifferentiated fever. Maternal seropositivity to cytomegalovirus (CMV) is not considered a contraindication except when it has a recent onset or in mothers of low birthweight infants. When present, the CMV load can be substantially reduced by freezing and pasteurization of the milk. All patients who smoke should be strongly encouraged to discontinue use of tobacco, particularly in the presence of infants, but smoking is not a contraindication to breastfeeding. Mothers with active herpes simplex lesions on a breast should not feed their infant from the infected breast, but may do so from the other breast if it is not infected. Breastfeeding is also contraindicated in the presence of active maternal tuberculosis, and following administration or use of radioactive isotopes, chemotherapeutic agents, "recreational" drugs, or certain prescription drugs. Ref: American Academy of Pediatrics Section on Breastfeeding: Policy Statement. Breastfeeding and the use of human milk. Pediatrics 2005;115(2):496-506.

Which one of the following is recommended for the treatment of intravaginal genital warts in pregnant women? (check one) A. Imiquimod 5% cream (Aldara) B. Podofilox 0.5% solution (Condylox) C. Podophyllin 10%-25% in tincture of benzoin (Podofin) D. Cryotherapy with liquid nitrogen E. Interferon-alpha

D Genital warts can proliferate and fragment during pregnancy, and many specialists recommend that they be eliminated. Imiquimod, podophyllin, and podofilox are not recommended for use during pregnancy. For the treatment of vaginal warts, the Centers for Disease Control and Prevention (CDC) recommends the use of cryotherapy. Liquid nitrogen, rather than a cryoprobe, should be used to avoid possible vaginal perforation and subsequent fistula formation. An alternative is the use of trichloroacetic acid or bichloroacetic acid carefully applied to the lesions to avoid damage to adjacent tissue. Interferon is no longer recommended for routine use in treating genital warts, due to a high frequency of systemic adverse effects. Ref: Berek JS (ed): Novak's Gynecology, ed 13. Lippincott Williams & Wilkins, 2002, p 405. 2) Centers for Disease Control and Prevention: Sexually transmitted diseases treatment guidelines 2002. MMWR 2002;51(RR-6):53-57.

A 30-year-old previously healthy male comes to your office with a 1-year history of frequent abdominal pain, nonbloody diarrhea, and a 20-lb weight loss. He has no history of travel outside the United States, antibiotic use, or consumption of well water. His review of systems is notable for a chronic, intensely pruritic rash that is vesicular in nature. His review of systems is otherwise negative and he is on no medications. The most likely cause of his symptoms is: (check one) A. lactose intolerance B. irritable bowel syndrome C. collagenous colitis D. celiac sprue E. Crohn's disease

D Celiac sprue is an autoimmune disorder characterized by inflammation of the small bowel wall, blunting of the villi, and resultant malabsorption. Symptoms commonly include diarrhea, fatigue, weight loss, abdominal pain, and borborygmus; treatment consists of elimination of gluten proteins from the diet. Extraintestinal manifestations are less common but may include elevated transaminases, osteopenia, and iron deficiency anemia. Serum IgA tissue transglutaminase (TTG) antibodies are highly sensitive and specific for celiac sprue, and a small bowel biopsy showing villous atrophy is the gold standard for diagnosis. This patient's rash is consistent with dermatitis herpetiformis, which is pathognomonic for celiac sprue and responds well to a strict gluten-free diet. Lactose intolerance, irritable bowel syndrome, collagenous colitis, and Crohn's disease are in the differential diagnosis for celiac sprue. However, significant weight loss is not characteristic of irritable bowel syndrome or lactose intolerance. The diarrhea associated with Crohn's disease is typically bloody. Collagenous colitis does cause symptoms similar to those experienced by this patient, but it is not associated with dermatitis herpetiformis. Ref: Presutti RJ, Cangemi JR, Cassidy HD, Hill DA: Celiac disease. Am Fam Physician 2007;76(12):1795-1802.

Contraindications to use of the levonorgestrel intrauterine system (Mirena) include which one of the following? (check one) A. Nulliparity B. A previous history of deep vein thrombosis C. A previous history of endometriosis D. Current pelvic inflammatory disease E. Current breastfeeding

D Contraindications to insertion of the levonorgestrel intrauterine system (LNG-IUS) include uterine anomalies, postpartum endometritis, untreated cervicitis, and current pelvic inflammatory disease. Nulliparity may increase discomfort during insertion but is not a contraindication. Levonorgestrel is a synthetic progestin and is not associated with an increased risk of deep vein thrombosis. It also is not associated with any adverse effect on quantity or quality of milk in breastfeeding women, and has no adverse effects on the infant. The LNG-IUS is not contraindicated in patients with endometriosis, and there is some evidence that it may improve symptom scores in these women. Ref: Lockhat FB, Emembolu JO, Konje JC: The evaluation of the effectiveness of an intrauterine-administered progestogen (levonorgestrel) in the symptomatic treatment of endometriosis and in the staging of the disease. Hum Reprod 2004;19(1):179-184. 2) Paladine HL, Blenning CE, Judkins DZ, Mittal S: What are contraindications to IUDs? J Fam Pract 2006:55(8):726-729.

At the 18-month visit, which one of the following is the most specific sign of autism? (check one) A. Delayed or odd use of language B. Repetitive behaviors C. Stereotypic movements D. Delayed attainment of social skill milestones E. Self-injurious behaviors

D Delayed attainment of social skill milestones is the earliest and most specific sign of autism. Delayed or odd use of language is a common, but less specific, early sign of autism. Compared with social and language impairments, restricted interests and repetitive behaviors are less prominent and more variable in young children. Self-injurious behaviors are associated with autism, but not specific for it. For example, new-onset head banging may be the way an autistic child attempts to deal with pain from a dental abscess, headache, sinusitis, otitis media, or other source of pain. Ref: Carbone PS, Davis T: Primary care for children with autism. Am Fam Physician 2010;81(4):453-460, 461.

A 32-year-old farmer comes to your office because of an upper respiratory infection. While he is there he points out a lesion on his forearm that he first noted approximately 1 year ago. It is a 1-cm asymmetric nodule with an irregular border and variations in color from black to blue. The patient says that it itches and has been enlarging for the past 2 months. He says he is so busy that he is not sure when he can return to have it taken care of. In such cases the best approach would be to (check one) A. perform a punch biopsy and have the patient return if the biopsy indicates pathology B. perform a shave biopsy and recheck in 2 months for signs of recurrence C. use electrocautery to destroy the lesion and the surrounding tissue D. perform an elliptical excision as soon as possible E. freeze the site with liquid nitrogen

D Despite this individual's busy schedule, he has a potentially life-threatening problem that needs proper diagnosis and treatment. Though an excisional biopsy takes longer, it is the procedure of choice when melanoma is suspected. After removal and diagnosis, prompt referral is essential for further evaluation and therapy. A shave biopsy should never be done for suspected melanoma, as this is likely to transect the lesion and destroy evidence concerning its depth, thus making it difficult to assess the prognosis. A punch biopsy should be used only with discretion when the lesion is too large for complete excision, or if substantial disfigurement would occur. Since this may not actually retrieve cancerous tissue from an unsampled area of a large lesion that might be malignant, it would be safest to refer such patients. Neither cryotherapy nor electrocautery should be used for a suspected melanoma. Ref: Habif TP, Campbell JL Jr, Chapman MS, et al: Skin Disease: Diagnosis and Treatment, ed 2. Elsevier, 2005, pp 472-479.

A 38-year-old male who is a new patient reports mild intermittent jaundice without other associated symptoms for the past several years. His liver function tests are normal except for a total bilirubin of 1.3 mg/dL (N 0.3-1.0) and an indirect or unconjugated bilirubin of 1.0 mg/dL (N 0.2-0.8). His CBC is normal. His past medical and surgical history is unremarkable. Findings are similar on repeat laboratory testing. The most likely cause of these findings is: (check one) A. Hepatitis C B. Wilson's disease C. Sickle cell anemia D. Gilbert's syndrome E. Drug toxicity

D Gilbert's syndrome is the most common inherited disorder of bilirubin metabolism. In patients with a normal CBC and liver function tests, except for recurrent mildly elevated total and unconjugated hyperbilirubinemia, the most likely diagnosis is Gilbert's syndrome. Fasting, heavy physical exertion, sickle cell anemia, and drug toxicity can also cause hyperbilirubinemia. Ref: Feldman M, Scharschmidt BF, Sleisenger MH (eds): Sleisenger & Fordtran's Gastrointestinal and Liver Disease, ed 7. WB Saunders Co, 2002, pp 251-252.

A 17-year-old white female at 20 weeks gestation presents with a 2-day history of painful vesicular lesions on her labia. This is the first time she has ever had this problem. Her last sexual contact was 10 days ago. She has also had a low-grade fever, malaise, headache, and mild, diffuse abdominal pain. On examination she has vesicles and erythematous papules on the labia bilaterally. A few firm, tender inguinal nodes are also noted. Which one of the following tests is most sensitive for confirming the diagnosis? (check one) A. A Papanicolaou smear of the lesions B. Amniocentesis C. Serologic studies D. Viral polymerase chain reaction (PCR) testing E. A Tzanck test

D Diagnosis and appropriate treatment of genital herpes during pregnancy is particularly important because of the high mortality in neonates who contract herpes during delivery and then develop disseminated infection. In those who survive, there is a very high risk of serious neurologic sequelae. HSV is acquired by deposition of the virus on a break in the skin or mucous membranes during close physical contact with an infected person. Neonatal infection most commonly results from transmission via the birth canal, although transplacental transmission can occur. The risk of HSV infection in the neonate is higher during an episode of primary genital herpes than during a recurrent episode. DNA polymerase chain reaction testing is 95% sensitive as long as an ulcer is present, and has a specificity of 90%. The diagnosis is established by culturing the virus from an infected lesion. A Tzanck prep and Papanicolaou smear can detect cellular changes, but both have low sensitivity. Serologic diagnosis is mainly an epidemiologic tool and has limited clinical usefulness. Cultures of the virus by amniocentesis have shown both false-positive and false-negative results.

Which one of the following variables is the most important risk factor for being a victim of domestic abuse? (check one) A. Educational background B. Psychological problems C. Race D. Gender E. Socioeconomic status

D Domestic violence can affect children, intimate partners, and older adults. It is a serious medical problem that should be considered in the care of patients and families. There is great variation in the profiles of patients affected by domestic violence. Neither demographic factors nor psychological problems have been found to be consistent predictors of victimization or violence. Domestic violence cuts across all racial, socioeconomic, religious, and ethnic lines. The only consistent risk factor for being a victim of domestic violence is female gender. Ref: Neufeld B: SAFE questions: Overcoming barriers to the detection of domestic violence. Am Fam Physician 1996;53(8):2575-2580. 2) Screening for Family and Intimate Partner Violence, Topic Page. US Preventive Services Task Force, Agency for Healthcare Research and Quality, 2004. Available at http://www.ahrq.gov/clinic/uspstf/uspsfamv.htm.

A 23-year-old white male is brought to the emergency department with slurred speech, confusion, and ataxia. He works as an auto mechanic and has been known to consume alcohol heavily in the past, but denies recent alcohol intake. He appears intoxicated, but no odor of alcohol is noted on his breath. Abnormalities on the metabolic profile include a carbon dioxide content of 10 mmol/L (N 20-30). His blood alcohol level is <10 mg/dL (0.01%). A urinalysis shows calcium oxalate crystals and an RBC count of 10-20/hpf. Woods lamp examination of the urine shows fluorescence. His arterial pH is 7.25. Which one of the following would be most appropriate at this point? (check one) A. Immediate hemodialysis B. Gastric lavage C. Administration of activated charcoal D. Fomepizole (Antizol)

D Ethylene glycol poisoning should be suspected in patients with metabolic acidosis of unknown cause and subsequent renal failure, as rapid diagnosis and treatment will limit the toxicity and decrease both morbidity and mortality. This diagnosis should be considered in a patient who appears intoxicated but does not have an odor of alcohol, and has anion gap acidosis, hypocalcemia, urinary crystals, and nontoxic blood alcohol levels. Ethylene glycol is found in products such as engine coolant, de-icing solution, and carpet and fabric cleaners. Ingestion of 100 mL of ethylene glycol by an adult can result in toxicity. The American Academy of Clinical Toxicology criteria for treatment of ethylene glycol poisoning with an antidote include a plasma ethylene glycol concentration >20 mg/dL, a history of ingesting toxic amounts of ethylene glycol in the past few hours with an osmolal gap >10 mOsm/kg H O2 (N 5-10), and strong clinical suspicion of ethylene glycol poisoning, plus at least two of the following: arterial pH <7.3, serum bicarbonate <20 mmol/L, or urinary oxalate crystals. Until recently, ethylene glycol poisoning was treated with sodium bicarbonate, ethanol, and hemodialysis. Treatment with fomepizole (Antizol) has this specific indication, however, and should be initiated immediately when ethylene glycol poisoning is suspected. If ethylene glycol poisoning is treated early, hemodialysis may be avoided, but once severe acidosis and renal failure have occurred hemodialysis is necessary. Ethylene glycol is rapidly absorbed, and use of ipecac or gastric lavage is therefore not effective. Large amounts of activated charcoal will only bind to relatively small amounts of ethylene glycol, and the therapeutic window for accomplishing this is less than 1 hour.

Painful ingrown toenails that display granulation tissue and lateral nail fold hypertrophy are best treated by: (check one) A. Antibiotic therapy B. Cotton-wick elevation of the affected nail corner C. Removal of the entire nail D. Excision of the lateral nail plate combined with lateral matricectomy

D Excision of the lateral nail plate with lateral matricectomy yields the best results in the treatment of painful ingrown toenails that display granulation tissue and lateral nail fold hypertrophy. Antibiotic therapy and cotton-wick elevation are acceptable for very mildly inflamed ingrown toenails. Partial nail avulsion often leaves a spicule of nail that will grow and become an ingrown nail. Phenol produces irregular tissue destruction and significant inflammation and discharge after the matricectomy procedure. Ref: Zuber TJ: Ingrown toenail removal. Am Fam Physician 2002;65(12):2547-2550.

You are asked to perform a preoperative evaluation on a 55-year-old white female with type 2 diabetes mellitus prior to elective femoral-anterior tibial artery bypass surgery. She is unable to climb a flight of stairs or do heavy work around the house. She denies exertional chest pain, and is otherwise healthy. Based on current guidelines, which one of the following diagnostic studies would be appropriate prior to surgery because the results could alter the management of this patient? (check one) A. Pulmonary function studies B. Coronary angiography C. Carotid angiography D. A dipyridamole-thallium scan E. A hemoglobin A1c level

D Family physicians are often asked to perform a preoperative evaluation prior to noncardiac surgery. This requires an assessment of the perioperative cardiovascular risk of the procedure involved, the functional status of the patient, and clinical factors that can increase the risk, such as diabetes mellitus, stroke, renal insufficiency, compensated or prior heart failure, mild angina, or previous myocardial infarction. This patient is not undergoing emergency surgery, nor does she have an active cardiac condition; however, she is undergoing a high-risk procedure (>5% risk of perioperative myocardial infarction) with vascular surgery. As she cannot climb a flight of stairs or do heavy housework, her functional status is <4 METs, and she should be considered for further evaluation. The patient's diabetes is an additional clinical risk factor. With vascular surgery being planned, appropriate recommendations include proceeding with the surgery with heart rate control, or performing noninvasive testing if it will change the management of the patient. Coronary angiography is indicated if the noninvasive testing is abnormal. Pulmonary function studies are most useful in patients with underlying lung disease or those undergoing pulmonary resection. Hemoglobin A1c is a measure of long-term diabetic control and is not particularly useful perioperatively. Carotid angiography is not indicated in asymptomatic patients being considered for lower-extremity vascular procedures.

Which one of the following is the most reliable clinical symptom of uterine rupture? (check one) A. Sudden, tearing uterine pain B. Vaginal bleeding C. Loss of uterine tone D. Fetal distress

D Fetal distress has proven to be the most reliable clinical symptom of uterine rupture. The "classic" signs of uterine rupture such as sudden, tearing uterine pain, vaginal hemorrhage, and loss of uterine tone or cessation of uterine contractions are not reliable and are often absent. Pain and bleeding occur in as few as 10% of cases. Even ruptures monitored with an intrauterine pressure catheter fail to show loss of uterine tone. Signs of fetal distress are often the only manifestation of uterine rupture. Ref: Toppenberg KS, Block WA Jr: Uterine rupture: What family physicians need to know. Am Fam Physician 2002;66(5):823-828.

A patient at 40 weeks' gestation has had a fundal height 3-4 cm greater than expected relative to dates for the last several visits. Ultrasonography 2 days ago showed a fetus in the vertex position with an estimated fetal weight of 4200 g (9 lb 4oz). On examination today the patient's cervix is closed, long, posterior, and firm, with the vertex at -2 station. Her pregnancy has been otherwise uncomplicated. Appropriate management at this point would be: (check one) A. Cesarean section B. Induction of labor with oxytocin (Pitocin) C. Cervical ripening with prostaglandins D. Scheduling a routine prenatal visit in 1 week

D Fetal macrosomia at term is defined by various authorities as birth weight above 4000-4500 g. Ultrasonography, unfortunately, does not provide a particularly accurate estimate of fetal weight for large fetuses. The risk of difficult vaginal delivery and shoulder dystocia does increase with birth weight above 4000-4500 g. This has led to attempts to prevent shoulder dystocia and possible birth injury by either performing an elective cesarean section or inducing labor when the fetus is estimated to be macrosomic. However, no studies have shown a benefit to either intervention in otherwise uncomplicated pregnancies. Suspected macrosomia on its own is no longer considered an indication for induction or cesarean section. However, should this patient not spontaneously go into labor she will soon need to be managed as a post-dates pregnancy and thus a return visit should be scheduled in a week. Ref: Sanchez-Ramos L, Bernstein S, Kaunitz A: Expectant management versus labor induction for suspected fetal macrosomia: A systematic review. Obstet Gynecol 2002;100(5):997-1002.

A 60-year-old female presents with a 1-year history of episodes of urinary incontinence. She tells you that she will suddenly feel the need to urinate and can barely make it to the bathroom. She occasionally loses urine before reaching the toilet. Her only medication is hydrochlorothiazide, which she has been taking for many years for hypertension. On examination, her vaginal mucosa is pale and somewhat dry. Minimal prolapse of her vaginal and urethral areas is noted. Which one of the following would be most appropriate at this point? (check one) A. Urodynamic testing B. Referral for surgical evaluation C. Oral estrogen D. Oral anticholinergic therapy E. Stopping the hydrochlorothiazide

D First-line therapies for urge urinary incontinence include behavioral therapy, such as pelvic muscle contractions, and anticholinergic therapy. Oral estrogen is not indicated. Noninvasive treatments should be tried initially. Urodynamic testing is indicated preoperatively. Stopping the hydrochlorothiazide would not be helpful, as it would not address the issue of detrusor instability.

Which one of the following indicates that a patient has entered the second stage of labor? (check one) A. A small amount of bloody, mucous discharge from the cervix ("bloody show") B. Braxton Hicks contractions C. Spontaneous rupture of the chorioamnionic membranes D. Complete dilation of the uterine cervix E. Successful delivery of the placenta

D For many women, labor will be preceded by several hours, or even days, by "bloody show." So-called "false labor," or Braxton Hicks contractions, consists of weak, irregular, regional contractions that usually occur for weeks before the onset of actual labor and abate with time, analgesia, and sedation. Spontaneous chorioamnionic membrane rupture precedes the onset of labor in about 10% of pregnancies, and amniotic fluid leaks through the cervix and out the vagina. The second stage of labor is defined as the period from complete cervical dilation to complete delivery of the baby. When the cervix is completely dilated, the patient usually experiences the urge to push with contractions. The third stage of labor begins with the delivery of the baby and ends with the delivery of the placenta. Ref: Scott JR, Gibbs RS, Karlan BY, et al (eds): Danforth's Obstetrics and Gynecology, ed 9. Lippincott Williams & Wilkins, 2003, pp 35-46.

A 22-year-old male has acute low back pain without paresthesias or other neurologic signs. There is no lower extremity weakness. Which treatment has been shown to be of most benefit initially? (check one) A. Complete bed rest for 2 weeks B. Bed rest plus local injection of corticosteroids C. A low-back strengthening program D. Resumption of physical activity as tolerated

D For patients who have acute back pain without sciatic involvement, a return to normal activities as tolerated has been shown to be more beneficial than either bed rest or a basic exercise program. Bed rest for more than 2 or 3 days in patients with acute low back pain is ineffective and may be harmful. Patients should be instructed to remain active. Injections should be considered only if conservative therapy fails.

A 32-year-old African-American female presents with a 3-day history of fever, cough, and shortness of breath. She has been healthy otherwise, except for a sinus infection 2 months ago treated with amoxicillin. She does not appear toxic. A chest radiograph reveals an infiltrate in the right lower lobe, consistent with pneumonia. Which one of the following would be the best choice for antibiotic treatment? (check one) A. High-dose amoxicillin B. Azithromycin (Zithromax) C. Doxycycline D. Levofloxacin (Levaquin) E. Cefuroxime axetil (Ceftin)

D For previously healthy patients with community-acquired pneumonia and no risk factors for drug resistance, a macrolide such as azithromycin is the preferred treatment (SOR A). Doxycycline is also acceptable (SOR C). Patients who have been treated with antibiotics within the previous 3 months should be treated with a respiratory fluoroquinolone (moxifloxacin, gemifloxacin, or levofloxacin) (SOR A). A β-lactam plus a macrolide is also an alternative (SOR A). The antibiotic chosen should be from a different class than the one used for the previous infection. These alternative treatments are also recommended for those with comorbidities such as chronic heart, lung, liver, or renal disease; diabetes mellitus; alcoholism; malignancies; asplenia; immunosuppressing conditions or use of immunosuppressing drugs; or other risk factors for drug-resistant Streptococcus pneumoniae infection (SOR A). Ref: Mandell LA, Wunderink RG, Anzueto A, et al: Infectious Diseases Society of America/American Thoracic Society consensus guidelines on the management of community-acquired pneumonia in adults. Clin Infect Dis 2007;44(Suppl 2):S27-S72.

A 52-year-old patient is concerned about a biopsy result from a recent screening colonoscopy. Which one of the following types of colon polyp is most likely to become malignant? (check one) A. Hyperplastic polyp B. Hamartomatous polyp C. Tubular adenoma D. Villous adenoma E. Tubulovillous adenoma

D Hamartomatous (or juvenile) polyps and hyperplastic polyps are benign lesions and are not considered to be premalignant. Adenomas, on the other hand, have the potential to become malignant. Sessile adenomas and lesions >1.0 cm have a higher risk for becoming malignant. Of the three types of adenomas (tubular, tubulovillous, and villous), villous adenomas are the most likely to develop into an adenocarcinoma.

A 53-year-old white female visits your office for her annual examination. During the last year she has stopped having menstrual periods and has had moderately severe sleep disturbance. She has been waking up at night with sweats. She denies other problems or complaints. She has a previous history of depression and her family history is significant for osteoporosis, heart disease, and Alzheimer's disease in older members of her family. There is no family history of breast cancer. The patient is concerned about her future and current health and wants to know the benefits and risks of hormone replacement therapy (HRT). Which one of the following statements about HRT is correct? (check one) A. It protects against coronary heart disease B. It slows progression of Alzheimer's disease C. It improves symptoms of depression D. It improves vaginal dryness E. It improves urinary incontinence

D Hormone replacement therapy (HRT) improves the urogenital symptoms of menopause, such as vaginal dryness and dyspareunia. However, recent research regarding HRT has not shown a benefit for reducing coronary events, slowing the progression of Alzheimer's disease, improving depression, or improving urinary incontinence. Ref: Executive Committee of the International Position Paper on Women's Health and Menopause, Wenger NK, chair: International position paper on women's health and menopause: A comprehensive approach, ch. 13. National Heart, Lung, and Blood Institute, Office of Research on Women's Health, National Institutes of Health, and Giovanni Lorenzini Medical Science Foundation, 2002, pp 1-31. 2) Scott JR, Gibbs RS, Karlan BY, et al (eds): Danforth's Obstetrics and Gynecology, ed 9. Lippincott Williams & Wilkins, 2003, pp 732-736.

A 24-year-old female with a past history of asthma presents to the emergency department with an asthma exacerbation. Treatment with an inhaled bronchodilator and ipratropium (Atrovent) does not lead to significant improvement, and she is admitted to the hospital for ongoing management. On examination she is afebrile, her respiratory rate is 24/min, her pulse rate is 92 beats/min, and oxygen saturation is 92% on room air. She has diffuse bilateral inspiratory and expiratory wheezes with mild intercostal retractions. Which one of the following should be considered in the acute management of this patient? (check one) A. Chest physical therapy B. Inhaled fluticasone/salmeterol (Advair) C. Oral azithromycin (Zithromax) D. Oral prednisone E. Oral theophylline

D Hospital management of acute exacerbations of asthma should include inhaled short-acting bronchodilators in all patients. Systemic corticosteroids are recommended for all patients admitted to the hospital. The efficacy of oral prednisone has been shown to be equivalent to that of intravenous methylprednisolone (SOR A). Oxygen should also be considered in most patients. Antibiotics are not recommended in the treatment of asthma exacerbations unless there is a comorbid infection. Inhaled ipratropium bromide is recommended for treatment in the emergency department, but not in the hospital (SOR A). Chest physical therapy and methylxanthines are not recommended in the treatment of acute asthma exacerbations. Ref: National Asthma Education and Prevention Program: Expert panel report 3: Guidelines for the diagnosis and management of asthma. National Heart, Lung, and Blood Institute, 2007, pp 373-417. Available at http://www.nhlbi.nih.gov/guidelines/asthma/asthgdln.pdf.

A 72-year-old male with class III congestive heart failure (CHF) due to systolic dysfunction asks if he can take ibuprofen for his "aches and pains." Appropriate counseling regarding NSAID use and heart failure should include which one of the following? (check one) A. NSAIDs are a good choice for pain relief, as they decrease systemic vascular resistance B. NSAIDs are a good choice for pain relief, as they augment the effect of his diuretic C. High-dose aspirin (325 mg/day) is preferable to other NSAIDs for patients taking ACE inhibitors D. NSAIDs, including high-dose aspirin, should be avoided in CHF patients because they can cause fluid retention

D If possible, NSAIDs should be avoided in patients with heart failure. They cause sodium and water retention, as well as an increase in systemic vascular resistance which may lead to cardiac decompensation. Patients with heart failure who take NSAIDs have a tenfold increased risk of hospitalization for exacerbation of their CHF. NSAIDs alone in patients with normal ventricular function have not been associated with initial episodes of heart failure. NSAIDs, including high-dose aspirin (325 mg/day), may decrease or negate entirely the beneficial unloading effects of ACE inhibition. They have been shown to have a negative impact on the long-term morbidity and mortality benefits that ACE inhibitors provide. Sulindac and low-dose aspirin (81 mg/day) are less likely to cause these negative effects. Ref: Bleumink GS, Feenstra J, Sturkenboom MC, et al: Nonsteroidal anti-inflammatory drugs and heart failure. Drugs 2003;63(6):525-534. 2) Amabile CM, Spencer AP: Keeping your patient with heart failure safe: A review of potentially dangerous medications. Arch Intern Med 2004;164(7):709-720. 3) Burton R, Kaplan N: NSAIDs effects in hypertension and heart failure. UpToDate 1/8/04, version 12.1.

A 65-year-old female who is morbidly obese presents to your office with intertrigo in the axilla. On examination you detect small, reddish-brown macules that are coalescing into larger patches with sharp borders. You suspect cutaneous erythrasma complicating the intertrigo. What would be the most appropriate topical treatment for this condition? (check one) A. Cornstarch B. A mild corticosteroid lotion C. A high-potency corticosteroid lotion D. Erythromycin

D Intertrigo is inflammation of skinfolds caused by skin-on-skin friction and is common on opposing cutaneous or mucocutaneous surfaces. Secondary cutaneous bacterial and fungal infections are common complications. Cutaneous erythrasma may complicate intertrigo of interweb areas, intergluteal and crural folds, axillae, or inframammary regions. Erythrasma is caused by Corynebacterium minutissimum and presents as small reddish-brown macules that may coalesce into larger patches with sharp borders. Intertrigo complicated by erythrasma is treated with topical or oral erythromycin.

A 12-month-old white female whom you have seen regularly for all of her scheduled well child care is found to have a hemoglobin level of 9.0 g/dL (N for age 10.5-13.5). She started whole milk at 9 months of age. She appears healthy otherwise and has no family history of anemia. A CBC reveals a mild microcytic, hypochromic anemia with RBC poikilocytosis, but is otherwise normal. The RBC distribution width is also elevated. Of the following, the most appropriate next step would be to: (check one) A. order tests for serum iron and total iron-binding capacity B. order a serum ferritin level C. order hemoglobin electrophoresis D. prescribe oral iron E. perform stool guaiac testing

D Iron deficiency is almost certainly the diagnosis in this child. The patient's response to a therapeutic trial of iron would be most helpful in establishing the diagnosis. Additional tests might be necessary if there is no response. Ref: Kliegman RM, Stanton BF, Geme JW III, et al (eds): Nelson Textbook of Pediatrics, ed 19. Elsevier Saunders, 2011, pp 1655-1658.

Early palliative care in patients with a terminal disease, including symptom management, psychosocial support, and assistance with decision making, has been shown to: (check one) A. shorten the time to death B. increase aggressive end-of-life care C. increase health care costs D. decrease depressive symptoms E. reduce the need for hospice

D It has been shown that palliative care offered early in the course of a terminal disease has many benefits. Palliative care leads to improvement in a patient's quality of life and mood, and patients who receive palliative care often have fewer symptoms of depression than those who do not receive palliative care. In addition, palliative care reduces aggressive end-of-life care and thus reduces health care costs. Palliative care does not reduce the need for hospice, but in fact enables patients to enter hospice care earlier and perhaps for longer. Palliative care has been shown to extend survival times in terminal patients (SOR B).

A 58-year-old white male comes to your office for follow-up after a recent bout of acute bronchitis. He reports having a productive cough for several months. He gets breathless with exertion and notes that every time he gets a cold it "goes into my chest and lingers for months." He has been smoking for 30 years. A physical examination is negative except for scattered rhonchi. A chest radiograph done 4 months ago at an urgent care visit was negative except for hyperinflation and flattened diaphragms. Which one of the following would be best for making the diagnosis? (check one) A. A chest radiograph B. CT of the chest C. Peak flow measurement D. Spirometry E. A BNP level

D It is important to distinguish between COPD and asthma because of the differences in treatment. Patients with COPD are usually in their sixties when the diagnosis is made. Symptoms of chronic cough (sometimes for months or years), dyspnea, or sputum production are often not reported because the patient may attribute them to smoking, aging, or poor physical condition. Spirometry is the best test for the diagnosis of COPD. The pressure of outflow obstruction that is not fully reversible is demonstrated by postbronchodilator spirometry showing an FEV /FVC ratio of 70% or less. Ref: Price DB, Yawn BP, Jones RCM: Improving the differential diagnosis of chronic obstructive pulmonary disease in primary care. Mayo Clin Proc 2010;85(12):1122-1129. 2) Dewar M, Curry RW Jr: Chronic obstructive pulmonary disease: Diagnostic considerations. Am Fam Physician 2006;73(4):669-676.

A 36-year-old female makes an appointment because her husband of 12 years was just diagnosed with hepatitis C when he tried to become a blood donor for the first time. He recalls multiple blood transfusions following a motorcycle crash in 1988. His wife denies past liver disease, blood transfusions, and intravenous drug use. She has had no other sexual partners. The couple has three children. Which one of the following is the best advice about testing the wife and their three children? (check one) A. No testing is required in the absence of jaundice or gastrointestinal symptoms B. No testing is required if her husband has normal liver enzyme levels C. No testing is required because tests have low sensitivity D. She should be offered testing because sexual transmission is possible E. All family members should be tested because of possible household fecal-oral spread

D Key risk factors for hepatitis C infection are long-term hemodialysis, intravenous drug use, blood transfusion or organ transplantation prior to 1992, and receipt of clotting factors before 1987. Sexual transmission is very low but possible, and the likelihood increases with multiple partners. The lifetime transmission risk of hepatitis C in a monogamous relationship is less than 1%, but the patient should be offered testing because she may choose to confirm that her test is negative. If the mother is seronegative, the children are at no risk. Maternal-fetal transmission is rare except in the setting of co-infection with HIV. Hepatitis C is insidious, and symptoms do not correlate with the extent of the disease. Normal liver enzyme levels do not indicate lack of infectivity. There is no risk to household contacts. Current HCV antibody tests are more than 99% sensitive and specific and are recommended for screening at-risk populations. Ref: Lauer GM, Walker BD: Hepatitis C virus infection. N Engl J Med 2001;345(1):41-52. 2) Ward RP, Kugelmas M, Libsch KD: Management of hepatitis C: Evaluating suitability for drug therapy. Am Fam Physician 2004;69(6):1429-1436. 3) Rakel RE, Bope ET (eds): Conn's Current Therapy 2005. Saunders, 2005, pp 608-609.

Of the following, which one causes the most deaths in the United States? (check one) A. Colorectal cancer B. Breast cancer C. Prostate cancer D. Lung cancer

D Lung cancer is the leading cause of cancer-related deaths in the United States. In 2006, lung cancer caused more deaths than colorectal, breast, and prostate cancers combined.

A 54-year-old female has pain and swelling of the right knee. Examination of the synovial fluid reveals a leukocyte count of 5000/mm3 , and crystals that appear as short, blunt rods, rhomboids, and cuboids when viewed under polarized light. The most likely diagnosis is: (check one) A. gonococcal arthritis B. tuberculous arthritis C. rheumatoid arthritis D. pseudogout E. gout

D Microscopic examination of synovial fluid in a patient suffering an acute attack of pseudogout shows large numbers of polymorphonuclear leukocytes. Calcium pyrophosphate dihydrate crystals are frequently found extracellularly and in polymorphonuclear leukocytes. When viewed with polarized light, the crystals appear as short, blunt rods, rhomboids, and cuboids. The diagnosis is made by finding typical crystals under compensated polarized light and is supported by radiographic evidence of chondrocalcinosis.

A 28-year-old male recreational runner has a midshaft posteromedial tibial stress fracture. Although he can walk without pain, he cannot run without pain. The most appropriate treatment at this point includes which one of the following? (check one) A. A short leg walking cast B. A non-weight-bearing short leg cast C. A non-weight-bearing long leg cast D. An air stirrup leg brace (Aircast) E. Low-intensity ultrasonic pulse therapy

D Midshaft posteromedial tibial stress fractures are common and are considered low risk. Management consists of relative rest from running and avoiding other activities that cause pain. Once usual daily activities are pain free, low-impact exercise can be initiated and followed by a gradual return to previous levels of running. A pneumatic stirrup leg brace has been found to be helpful during treatment (SOR C). Non-weight bearing is not necessary, as this patient can walk without pain. Casting is not recommended. Ultrasonic pulse therapy has helped fracture healing in some instances, but has not been shown to be beneficial in stress fractures. Ref: Harrast MA, Colonno D: Stress fractures in runners. Clin Sports Med 2010;29(3):399-418.

Which one of the following sleep problems in children is most likely to occur during the second half of the night? (check one) A. Confusional arousals B. Sleepwalking C. Sleep terrors D. Nightmares

D Nightmares occur in the second half of the night, when rapid eye movement (REM) sleep is most prominent. Parasomnias, including sleepwalking, confusional arousal, and sleep terrors, are disorders of arousal from non-REM (NREM) sleep. These are more common in children than adults because children spend more time in deep NREM sleep. Such disorders usually occur within 1-2 hours after sleep onset, and coincide with the transition from the first period of slow-wave sleep. Ref: Moore M, Allison D, Rosen CL: A review of pediatric nonrespiratory sleep disorders. Chest 2006;130(4):1252-1262.

A 35-year-old male presents with acute low back pain after he spent a weekend building a storage shed in his backyard. He has no neurologic symptoms, and the pain does not radiate into either leg. Which one of the following has been shown to be a useful treatment in this situation? A) Bed rest B) Acupuncture C) Lumbar traction D) Cyclobenzaprine (Flexeril) E) Methylprednisolone (Medrol) (check one) A. Bed rest B. Acupuncture C. Lumbar traction D. Cyclobenzaprine (Flexeril) E. Methylprednisolone (Medrol)

D Nonbenzodiazepine muscle relaxants such as cyclobenzaprine are beneficial for the relief of acute low back pain for the first 7-14 days after the onset of symptoms. Patient education, physical therapy, and the application of ice or heat may also help. Unsupported treatment options for acute low back pain include oral corticosteroids, acupuncture, lumbar support, massage, chiropractic spinal manipulation, and traction. Bed rest for acute low back pain is inadvisable. Ref: Casazza BA: Diagnosis and treatment of acute low back pain. Am Fam Physician 2012;85(4):343-350.

A 68-year-old Mexican American female is brought to your office by her son with a complaint of headaches. The patient speaks English adequately, but diverts her eyes to look at her son when answering your questions. Which one of the following is the most likely reason for this patient not making eye contact? (check one) A. Her son is overly controlling B. She is a victim of abuse C. She is being untruthful D. She is showing respect to you E. She is depressed

D Nonverbal communication is important for identifying issues that a patient may be hiding or be unwilling to divulge. Some nonverbal clues, however, are culturally biased. Many older or less-educated Mexican-Americans consider direct eye contact to be disrespectful. Because a physician is held in high regard, these patients will often either look down or look at another, more equal person in the room while being interviewed. Many Americans, on the other hand, may consider a lack of eye contact to be negative, implying that a patient is unsure of the information they are providing, has poor self-esteem, or is hiding something. Ref: Purnell LD, Paulanka BJ: Guide to Culturally Competent Health Care. FA Davis Company, 2005, pp 339-351.

A 24-year-old female at 36 weeks' gestation plans to breastfeed her infant. She has a history of bipolar disorder, but currently is doing well without medication, and also has a history of frequent urinary tract infections. She asks you about medications that she may need to take after delivery, and how they may affect her newborn. Which one of the following would be contraindicated if she breastfeeds her infant? (check one) A. Amoxicillin B. Macrodantin (Macrobid) C. Valproic acid (Depakote) D. Lithium

D Of the drugs listed, the only maternal medication that affects the infant is lithium. Breastfed infants of women taking lithium can have blood lithium concentrations that are 30%-50% of therapeutic levels. Ref: Ressel G: AAP updates statement for transfer of drugs and other chemicals into breast milk. Am Fam Physician 2002;65(5):979-980.

Which one of the following seafood poisonings requires more than just supportive treatment? (check one) A. Ciguatera B. Neurotoxic shellfish C. Paralytic shellfish D. Scombroid fish

D Only symptomatic treatment is indicated for ciguatera poisoning, as there is no specific treatment. The same is true for shellfish poisoning, although potential respiratory distress or failure must be kept in mind. Scombroid poisoning is a pseudoallergic condition resulting from consumption of improperly stored scombroid fish such as tuna, mackerel, wahoo, and bonito. Nonscombroid varieties such as mahi-mahi, amberjack, sardines, and herring can also cause this problem. The poisoning is due to high levels of histamine and saurine resulting from bacterial catabolism of histidine. Symptoms occur within minutes to hours, and include flushing of the skin, oral paresthesias, pruritus, urticaria, nausea, vomiting, diarrhea, vertigo, headache, bronchospasm, dysphagia, tachycardia, and hypotension. Therapy should be the same as for allergic reactions and anaphylaxis, and will usually lead to resolution of symptoms within several hours.

Hepatitis C screening is routinely recommended in which one of the following? (check one) A. Pregnant women B. Nonsexual household contacts of hepatitis C-positive persons C. Health care workers D. Persons with a history of illicit intravenous drug use

D Patients should be routinely screened for hepatitis C if they have a history of any of the following: intravenous drug abuse no matter how long or how often, receiving clotting factor produced before 1987, persistent alanine aminotransferase elevations, or recent needle stick with HCV-positive blood. Ref: Spach DH: Role of the primary care physician in the diagnosis and management of hepatitis C virus infection. J Am Board Fam Pract 1999;12(6):497-500. 2) Kasper DL, Braunwald E, Fauci AS, et al (eds): Harrison's Principles of Internal Medicine, ed 16. McGraw-Hill, 2005, pp 1822-1838.

A 16-year-old sexually active nulliparous white female complains of pelvic pain and vaginal discharge. On examination she is found to have a temperature of 39.8 degrees C (102.0 degrees F), pain with movement of the cervix, and tenderness and a mass in the right adnexa. According to CDC guidelines, which one of the following treatments would be appropriate? (check one) A. Outpatient treatment with penicillin G procaine (Wycillin) intramuscularly; probenecid (Benemid) orally; plus doxycycline (Vibramycin) orally for 14 days and reexamination in 3 days B. Outpatient treatment with ceftriaxone (Rocephin) intramuscularly; probenecid orally; plus doxycycline twice a day for 14 days and reexamination in 1 week C. Outpatient treatment with cefoxitin (Mefoxin) intramuscularly; plus doxycycline twice a day for 14 days and reexamination in 10 days D. Hospitalization for treatment with cefoxitin intravenously and doxycycline orally or intravenously, then doxycycline orally twice a day to complete 14 days of treatment

D Patients with PID and tubo-ovarian abscess and high fever should be hospitalized and treated for at least 24 hours with intravenous antibiotics. Amoxicillin and penicillin G procaine are no longer recommended because of the increasing prevalence of penicillinase-producing and chromosomally-mediated resistant Neisseria gonorrhoeae. If cefoxitin is used intramuscularly for outpatient treatment, it should be combined with probenecid. If ceftriaxone is used for outpatient treatment, probenecid is not required. Reexamination should be done within 3 days of initiation of therapy. Ref: Sexually transmitted diseases treatment guidelines—2002. MMWR 2002;51(RR-6):49-50.

Of the following, the greatest risk for developing colon cancer is associated with a personal history of: (check one) A. tobacco use B. ulcerative colitis C. villous adenoma D. familial adenomatous polyposis E. colon cancer in a first degree relative

D People with familial adenomatous polyposis typically develop hundreds or thousands of polyps in their colon and rectum, usually in their teens or early adulthood. Cancer usually develops in one or more of these polyps as early as age 20. By age 40, almost all people with this disorder will have developed cancer if preventive colectomy is not performed. The approximate lifetime risk of colon cancer in the general population of the United States is 6%. Most case-control studies of cigarette exposure and adenomas have found an elevated risk for smokers. Tobacco use raises the risk of colon cancer by approximately 50%. Patients with ulcerative colitis are at increased risk for colon cancer. The anatomic extent and duration of the disease correlate with the degree of risk. In one meta-analysis, investigators found that the risk of colon cancer was 2% in the first 10 years after ulcerative colitis develops, 8% during the first 20 years, and 18% during the first 30 years. The evidence is still evolving regarding the level of future risk of colon cancer associated with having had an adenomatous polyp removed in the past, but it may approach a doubling of the baseline risk of colon cancer. Studies suggest a clear association with a history of multiple polyps or a single large (>1 cm) polyp. The data is less clear for single small adenomas. Of the three types of adenomas (tubular, tubulovillous, and villous), villous adenomas are most likely to develop into adenocarcinomas. Having a family history of a first degree relative with colon cancer raises the risk approximately two-to threefold. If that relative was younger than age 50 at the time of diagnosis the risk is three-to fourfold higher.

A 74-year-old white male complains of pain in the right calf that recurs on a regular basis. He smokes 1 pack of cigarettes per day and is hypertensive. He has a history of a previous heart attack but is otherwise in fair health. Which one of the following findings would support a diagnostic impression of peripheral vascular disease? (check one) A. Pain during rest and exercise and the presence of swelling and soreness behind the knee and in the calf B. Pain that begins immediately upon walking and is unrelieved by rest C. Doppler waveform analysis showing accentuated waveforms at a point of decreased blood flow D. Treadmill arterial flow studies showing a 20-mm Hg decrease in ankle systolic blood pressure immediately following exercise E. An ankle-brachial index of 1.15

D Peripheral vascular disease (PVD) is a clinical manifestation of atherosclerotic disease and is caused by occlusion of the arteries to the legs. Patients with significant arterial occlusive disease will have a prominent decrease in the ankle-brachial index from baseline following exercise, and usually a 20-mm Hg or greater decrease in systolic blood pressure. Pain during rest and exercise and the presence of swelling and soreness behind the knee and in the calf is found in those with Baker's cysts. Peripheral nerve pain commonly begins immediately upon walking and is unrelieved by rest. Doppler waveform analysis is useful in the diagnosis of PVD and will reveal attenuated waveforms at a point of decreased blood flow. Employment of the ankle-brachial index is encouraged in daily practice as a simple means to diagnose the presence of PVD. Generally, ankle-brachial indices in the range of 0.91-1.30 are thought to be normal. Ref: Hiatt WR: Medical treatment of peripheral arterial disease and claudication. N Engl J Med 2001;344(21):1608-1621.

A 57-year-old African-American female has a partial resection of the colon for cancer. The surgical specimen has clean margins, and there is no lymph node involvement. There is no evidence of metastasis. You recommend periodic colonoscopy for surveillance, and also plan to monitor which one of the following tumor markers for recurrence? (check one) A. Prostate-specific antigen (PSA) B. Cancer antigen 27.29 (CA 27-29) C. Cancer antigen 125 (CA-125) D. Carcinoembryonic antigen (CEA) E. Alpha-fetoprotein

D Prostate-specific antigen (PSA) is a marker that is used to screen for prostate cancer. It is elevated in more than 70% of organ-confined prostate cancers. Alpha-tetoprotein is a marker for hepatocellular carcinoma and nonseminomatous germ cell tumor, and is elevated in 80% of hepatocellular carcinomas. CA-125 is a marker for ovarian cancer. Although it is elevated in 85% of ovarian cancers, it is elevated in only 50% of early-stage ovarian cancers. Carcinoembryonic antigen (CEA) is a marker for colon, esophageal, and hepatic cancers. It is expressed in normal mucosal cells and is overexpressed in adenocarcinoma, especially colon cancer. Though not specific for colon cancer, levels above 10 ng/mL are rarely due to benign disease. CEA levels typically return to normal within 4-6 weeks after successful surgical resection. CEA elevation occurs in nearly half of patients with a normal preoperative CEA level that have cancer recurrence. Cancer antigen 27.29 (CA 27-29) is a tumor marker for breast cancer. It is elevated in about 33% of early-stage breast cancers and about 67% of late-stage breast cancers. Some tumor markers, such as CEA, alpha-fetoprotein, and CA-125, may be more helpful in monitoring response to therapy than in detecting the primary tumor. Ref: Perkins GL, Slater ED, Sanders GK, et al: Serum tumor markers. Am Fam Physician 2003;68(6):1075-1082.

A 79-year-old male has psychosis secondary to dementia associated with Parkinson's disease. After exhausting all other options you decide to prescribe an antipsychotic agent. Which one of the following would be the best choice in this situation? (check one) A. Haloperidol B. Olanzapine (Zyprexa) C. Risperidone (Risperdal) D. Quetiapine (Seroquel) E. Thioridazine

D Quetiapine is an atypical antipsychotic that has no clinically significant effect on the dopamine D2 receptor, which is responsible for the parkinsonian side effects of antipsychotic medications. Because of this, it is considered the antipsychotic of choice in patients with dementia associated with Parkinson's disease, although its use has not been studied extensively in this clinical situation. The other atypical antipsychotics listed, olanzapine and risperidone, have some D2 receptor effect. Thioridazine and haloperidol are typical antipsychotics and have more side effects, including parkinsonian side effects; they are not recommended in this clinical situation. Ref: Rayner AV, OBrien JG, Schoenbachler B: Behavior disorders of dementia: Recognition and treatment. Am Fam Physician 2006;73(4):647-652, 653-654. 2 ) Reilly TH, Kirk MA: Atypical antipsychotics and newer antidepressants. Emerg Med Clin North Am 2007;25(2):477-497.

Patients with Wolff-Parkinson-White syndrome who have episodic symptomatic supraventricular tachycardia or atrial fibrillation benefit most from: (check one) A. Episodic intravenous digoxin B. Long-term oral digitalis C. Episodic beta-blockers D. Radiofrequency catheter ablation of bypass tracts

D Radiofrequency catheter ablation of bypass tracts is possible in over 90% of patients and is safer and more cost effective than surgery, with a similar success rate. Intravenous and oral digoxin can shorten the refractory period of the accessory pathway, and increase the ventricular rate, causing ventricular fibrillation. Beta-blockers will not control the ventricular response during atrial fibrillation when conduction proceeds over the bypass tract. Ref: Kasper DL, Braunwald E, Fauci AS, et al (eds): Harrison's Principles of Internal Medicine, ed 16. McGraw-Hill, 2005, pp 1347-1351.

An 88-year-old male has been hospitalized for the past 3 days after being found on the floor of his home by a neighbor and transported to the hospital by ambulance. He was cachectic and dehydrated at the time of admission, with a serum albumin level of 1.9 g/dL (N 3.5-4.7). He has received intravenous fluids and is now euvolemic. He began nasogastric tube feeding 2 days ago and has now developed nausea, vomiting, hypotension and delirium. Which one of the following is the most classic electrolyte abnormality with this condition? (check one) A. Hypocalcemia B. Hypercalcemia C. Hyperkalemia D. Hypophosphatemia E. Hyperphosphatemia

D Refeeding syndrome can be defined as the potentially fatal shifts in fluids and electrolytes that may occur in malnourished patients receiving artificial refeeding (whether enterally or parenterally). These shifts result from hormonal and metabolic changes and may cause serious clinical complications. The hallmark biochemical feature of refeeding syndrome is hypophosphatemia. However, the syndrome is complex and may also include abnormal sodium and fluid balance; changes in glucose, protein, and fat metabolism; thiamine deficiency; hypokalemia; and hypomagnesemia. Ref: Mehanna HM, Moledina J, Travis J: Refeeding syndrome: What it is, and how to prevent and treat it. BMJ 2008;336(7659):1495-1498.

An 8-year-old male is brought to the emergency department with an acute asthma attack that began 48 hours earlier. His mother initiated his asthma action plan when the attack began, starting oral prednisolone plus albuterol (Proventil, Ventolin) by metered-dose inhaler with a spacer every 3-4 hours. In the emergency department the child is alert, with a respiratory rate of 30 beats/min and an oxygen saturation of 94% on room air. He is audibly wheezing. Peak flow is 40% of the predicted value. (check one) A. Continue the current albuterol treatment but switch to a nebulizer B. Administer high-dose albuterol via nebulizer every 20 minutes for 1 hour C. Administer intravenous corticosteroids within the first hour D. Administer magnesium sulfate intravenously E. Prescribe high-dose mucolytics and chest physiotherapy

D Repeated doses of a short-acting β2-agonist and correction of hypoxia are the main elements of initial emergency department treatment for acute asthma exacerbations in children. Nebulizer treatments are no better than a metered-dose inhaler with a spacer (SOR A). High-dose nebulized albuterol every 20 minutes for 1 hour has not been shown to be beneficial. In children already receiving standard treatment with albuterol and corticosteroids the addition of intravenous magnesium sulfate has been shown to improve lung function and reduce the need for hospitalization (SOR A). Oral administration of corticosteroids is as effective as the intravenous route for reducing the need for hospital admission (SOR A). Mucolytics and chest physiotherapy have not been shown to be effective in children with acute asthma attacks.

A 12-year-old male presents with left hip pain. He is overweight and recently started playing tennis to lose weight. He says the pain started gradually after his last tennis game, but he does not recall any injury. He is walking with a limp. On examination he is afebrile and has limited internal rotation of the left hip. What is the most likely cause of the hip pain? (check one) A. Septic arthritis B. Juvenile rheumatoid arthritis C. Transient synovitis D. Slipped capital femoral epiphysis E. Legg-Calvé-Perthes disease

D Slipped capital femoral epiphysis is the most common hip disorder in this patient's age group. It usually occurs between the ages of 8 and 15 and is more common in boys and overweight or obese children. It presents with limping and pain, and limited internal rotation of the hip is noted on physical examination. Septic arthritis would typically present with a fever. Juvenile rheumatoid arthritis, transient synovitis, and Legg-Calvé-Perthes disease are more common in younger children. Ref: Peck D: Slipped capital femoral epiphysis: Diagnosis and management. Am Fam Physician 2010;82(3):258-262.

An asymptomatic 35-year-old female asks about having a thyroid test performed because hypothyroidism runs in her family. You order the tests, which show a TSH level of 7.6μU/mL (N 0.4-5.1) and a free T4 level within the normal range. Which one of the following is most likely in this patient? (check one) A. A euthyroid state B. Primary hyperthyroidism C. Secondary hyperthyroidism D. Subclinical hypothyroidism E. Overt hypothyroidism

D Subclinical hypothyroidism is defined as slightly elevated TSH (approximately 5-10 mIU/L) and normal levels of thyroid hormone (free T4 or free T3 ) in an asymptomatic patient. There is a low rate of progression to overt hypothyroidism manifested by symptoms, TSH levels >10 mIU/L, or reduced levels of thyroid hormone. Recent studies have shown that there is an increased risk for cardiovascular morbidity and mortality in those with subclinical hypothyroidism. However, treatment with thyroid replacement hormone did not seem to affect this risk. The decision about whether to recommend thyroid replacement therapy to patients like the one described here should be individualized. An alternative to treating the patient with medication at this time would be to retest her TSH annually, or sooner if she becomes symptomatic. Ref: Hennessey JV, Scherger JE: Evaluating and treating the patient with hypothyroid disease. J Fam Pract 2007;56(8):S31-S39. 2) Vaidya B, Pearce SHS: Management of hypothyroidism in adults. BMJ 2008;337:284-289.

You see a 16-year-old white female for a preparticipation evaluation for sports, and she asks for advice about the treatment of acne. She has a few inflammatory papules on her face. No nodules are noted. She says she has not tried any over-the-counter acne treatments. Which one of the following would be considered first-line therapy for this condition? (check one) A. Oral tetracycline B. Oral isotretinoin (Accutane) C. Topical sulfacetamide (Sulamyd) D. Topical benzoyl peroxide

D The American Academy of Dermatology grades acne as mild, moderate, and severe. Mild acne is limited to a few to several papules and pustules without any nodules. Patients with moderate acne have several to many papules and pustules with a few to several nodules. Patients with severe acne have many or extensive papules, pustules, and nodules. The patient has mild acne according to the American Academy of Dermatology classification scheme. Topical treatments including benzoyl peroxide, retinoids, and topical antibiotics are useful first-line agents in mild acne. Topical sulfacetamide is not considered first-line therapy for mild acne. Oral antibiotics are used in mild acne when there is inadequate response to topical agents and as first-line therapy in more severe acne. Caution must be used to avoid tetracycline in pregnant females. Oral isotretinoin is used in severe nodular acne, but also must be used with extreme caution in females who may become pregnant. Special registration is required by physicians who use isotretinoin, because of its teratogenicity. Ref: Feldman S, Careccia RE, Barham KL, et al: Diagnosis and treatment of acne. Am Fam Physician 2004;69(9):2123-2136.

The Mini-Mental State Examination (MMSE) tests for: (check one) A. Mood B. Behavior C. Intelligence quotient D. Cognitive function E. Functional impairment

D The MMSE is most commonly used in clinical settings. It is considered valuable because it assesses a broad range of cognitive abilities (i.e., memory, language, spatial ability, set shifting) in a simple and straightforward manner. In addition, the wide use of the MMSE in epidemiologic studies has yielded cutoff scores that facilitate the identification of patients with cognitive dysfunction. Ref: Geldmacher DS, Gordon B: Dementia in the elderly: Is it Alzheimer's disease? The AD Letter 2000;1(1):1-4. 2) Cassel CK, Leipzig RM, Cohen HJ, et al (eds): Geriatric Medicine: An Evidence-Based Approach, ed 4. Springer, 2003, p 206.

For the prevention of ischemic stroke in patients at low risk for gastrointestinal bleeding, the U.S. Preventive Services Task Force recommends aspirin for? (check one) A. Men age 45-79 B. Men age 55-79 C. Women age 45-79 D. Women age 55-79 E. No one, regardless of sex or age

D The U.S. Preventive Services Task Force has concluded that the net benefit of daily aspirin is substantial in women 55-79 years of age for whom the benefit of ischemic stroke prevention exceeds the harm of an increased risk for gastrointestinal bleeding (SOR A). Aspirin use is recommended in men 45-79 years of age for prevention of myocardial infarction when the potential benefit outweighs the potential harm of gastrointestinal hemorrhage (SOR A).

A rural community college has requested your guidance in offering a preventive health program to its students. The most appropriate plan would include which one of the following? (check one) A. Mammograms for female students B. Lead poisoning screening for all students C. Stool occult blood kits for students D. Smoking cessation programs E. An annual routine physical examination for all students

D The U.S. Preventive Services Task Force recommends a routine physical examination every 3-5 years for young adults until the age of 40. Mammograms are not recommended until age 40. Lead screening is recommended for at-risk individuals between 6 months and 6 years of age. Colorectal cancer screening for average-risk individuals is recommended at age 50. Counseling on tobacco use and other substance abuse is recommended as part of all routine preventive care. Ref: U.S. Preventive Services Task Force: Guide to Clinical Preventive Services, ed 3. AHRQ Publications Clearinghouse, 2002.

The Valsalva maneuver will typically cause the intensity of a systolic murmur to increase in patients with which one of the following conditions? (check one) A. Aortic stenosis B. Rheumatic mitral insufficiency C. Valvular pulmonic stenosis D. Hypertrophic obstructive cardiomyopathy

D The Valsalva maneuver decreases venous return to the heart, thereby decreasing cardiac output. This causes most murmurs to decrease in length and intensity. The murmur of hypertrophic obstructive cardiomyopathy, however, increases in loudness. The murmur of mitral valve prolapse becomes longer, and may also become louder. Ref: Bonow RO, Mann DL, Zipes DP, Libby P (eds): Braunwald's Heart Disease: A Textbook of Cardiovascular Medicine, ed 9. Elsevier Saunders, 2011, pp 117-118.

A 60-year-old female has been on conjugated equine estrogens/medroxyprogesterone (Prempro) since she went through menopause at age 52. She still has her uterus and ovaries. She is having no side effects that she is aware of and is experiencing no vaginal bleeding. She is worried about the health effects of her hormone replacement therapy and asks your advice about risks versus benefits. Which one of the following would be accurate advice regarding these risks and benefits? (check one) A. The incidence of stroke is decreased B. The incidence of myocardial infarction is decreased C. The incidence of pulmonary embolus is decreased D. The incidence of breast cancer is increased E. The incidence of colorectal cancer is increased

D The Women's Health Initiative Randomized Controlled Trial concluded that the health risks of hormone replacement therapy with combined estrogen plus progestin exceeded the benefits. Absolute risk reductions per 10,000 person-years attributable to estrogen plus progestin were 6 fewer colorectal cancers and 5 fewer hip fractures. However, absolute excess risks per 10,000 person-years included 7 more coronary heart disease events, 8 more strokes, 8 more pulmonary emboli, and 8 more invasive breast cancers. Ref: Rossouw JE, Anderson GL, Prentice RL, et al: Risks and benefits of estrogen plus progestin in healthy postmenopausal women: Principal results from the Women's Health Initiative randomized controlled trial. JAMA 2002;288(3):321-333.

You are consulted for medical management of a 45-year-old male, previously unknown to you, who is hospitalized in the psychiatric unit with paranoid schizophrenia. His fasting blood glucose level is 180 mg/dL. Which one of the following medications is the most likely cause of the hyperglycemia? (check one) A. Alprazolam (Xanax) B. Haloperidol (Haldol) C. Chlorpromazine (Thorazine) D. Olanzapine (Zyprexa) E. Thiothixene (Navane)

D The atypical antipsychotics include clozapine, olanzapine, risperidone, ziprasidone, quetiapine, and aripiprazole. As a class, they have fewer extrapyramidal side effects than the classical antipsychotics haloperidol, thiothixene, chlorpromazine, and others. Some of the atypical agents, notably olanzapine and clozapine, have been associated with hyperglycemia and the development of type 2 diabetes mellitus. Neither benzodiazepines like alprazolam nor the classical antipsychotics have been associated with hyperglycemia. Ref: Aripiprazole (Abilify) for schizophrenia. Med Lett Drugs Ther 2003;45:15-16. 2) Leslie DL, Rosenheck RA: Incidence of newly diagnosed diabetes attributable to atypical antipsychotic medications. Am J Psychiatry 2004;161(9):1709-1711.

The FDA has imposed a black box warning on all thiazolidinediones, such as pioglitazone (Actos). This warning addresses a contraindication to the prescription of these drugs in patients with: (check one) A. renal insufficiency B. dementia C. exposure to radiocontrast media D. heart failure E. respiratory failure

D The black box warning for thiazolidinediones specifically addresses heart failure. These agents are also contraindicated in patients with type 1 diabetes mellitus or hepatic disease, and in premenopausal anovulatory women.

Which cardiac arrhythmia has been reported with high-dose methadone use? (check one) A. Atrial fibrillation B. Paroxysmal supraventricular tachycardia C. Third degree heart block D. Torsades de pointes E. Multifocal atrial tachycardia

D The cardiac toxicity of methadone is primarily related to QT prolongation and torsades de pointes. Ref: Death, narcotic overdose, and serious cardiac arrhythmias. FDA Alert, 2006.

You have provided care for a 27-year-old married, monogamous female for several years. One year ago, she had abnormal cervical cytology that was interpreted as "atypical squamous cells of undetermined significance" (ASC-US). She had repeat cervical cytologic examinations 6 months ago and again last week, both reported as negative. Which one of the following would be the most appropriate next step? (check one) A. Repeat cervical cytology again in 4-6 months B. Screening for human papillomavirus C. Colposcopic examination with a biopsy and endocervical curettage D. Resuming a routine screening protocol E. Cervical culture for herpesvirus

D The cervical cytology category of atypical squamous cells of undetermined significance (ASC-US) is one that is poorly reproducible, having been shown to be frequently downgraded to negative or upgraded to a low- or high-grade squamous intraepithelial lesion on review. Recommended management strategies for women with ASC-US include repeat cytology at 4-6 months, immediate colposcopy, and reflex DNA testing for oncogenic HPV types. Should two repeat cytologic examinations at 4- to 6-month intervals prove negative, the patient can safely return to routine cytologic screenings. Should any repeat examination detect ASC-US or more significant cytology, colposcopy is indicated. Ref: Apgar BS, Brotzman G: Management of cervical cytologic abnormalities. Am Fam Physician 2004;70(10):1905-1916.

A male infant is delivered at 41 weeks gestation by spontaneous vaginal delivery. The amniotic fluid is meconium stained. Apgar scores are 7 at 1 minute and 7 at 5 minutes. The baby is noted to have respiratory distress from birth and is hypoxic by pulse oximetry. Respiration improves with supplemental oxygen, as does the hypoxia, but does not return to normal. Which one of the following would most likely be seen on a chest radiograph? (check one) A. A normal heart and lungs B. Fluid in the pulmonary fissures C. Homogeneous opaque infiltrates with air bronchograms D. Patchy atelectasis

D The chest radiograph of a child with meconium aspiration syndrome will show patchy atelectasis or consolidation. If the child has a normal chest film and respiratory distress, a noncardiopulmonary source should be considered (i.e., a neurologic or metabolic etiology). The chest film of a child with transient tachypnea of the newborn will show a wet silhouette around the heart, diffuse parenchymal infiltrates, or intralobar fluid accumulation. Homogeneous opaque infiltrates with air bronchograms on a chest radiograph are seen with hyaline membrane disease. Ref: Hermansen CL, Lorah KN: Respiratory distress in the newborn. Am Fam Physician 2007;76(7):987-994.

One of your patients has been diagnosed with monoclonal gammopathy of undetermined significance (MGUS). Which one of the following is used to determine whether his condition has progressed to multiple myeloma? (check one) A. The length of time since the diagnosis of MGUS was made B. The level of M protein C. The percentage of plasma cells in bone marrow D. Evidence of end-organ damage

D The diagnosis of multiple myeloma is based on evidence of myeloma-related end-organ impairment in the presence of M protein, monoclonal plasma cells, or both. This evidence may include hypercalcemia, renal failure, anemia, or skeletal lesions. Monoclonal gammopathy of undetermined significance does not progress steadily to multiple myeloma. There is a stable 1% annual risk of progression. Ref: Landgren O, Waxman AJ: Multiple myeloma precursor disease. JAMA 2010;304(21):2397-2404.

A 4-year-old white female is brought to your office by her mother, who reports that the child recently developed a foul-smelling vaginal discharge. After an appropriate history and general examination, you determine that a genital examination is necessary. Which one of the following positions is most likely to allow for visualization of the child's vagina and cervix without instrumentation? (check one) A. Supine in the mother's lap B. The left lateral decubitus position on an examination table C. Trendelenburg's position on an examination table D. The knee-chest position on an examination table E. Supine with the knees spread apart on an examination table

D The knee-chest position has been found to allow for visualization of the vagina and cervix of a prepubertal child after 2 years of age without instrumentation. The vagina is filled with air when the child is in the knee-chest position, facilitating inspection. An assistant holds the child's buttocks apart and the child is asked to relax her abdominal muscles and take a few deep breaths. With these preliminary steps, the vaginal orifice opens and the short vaginal canal fills with air. A bright light will help to illuminate the prepubertal child's vagina and cervix. Inspection of genitalia (where examination of the vaginal canal and cervix are not indicated) during a general physical examination need not be in the knee-chest position. In the young child (usually less than 2 years of age), examination is best done with the child lying supine in the mother's lap. For the older prepubertal child, examination is best done with the child lying supine with the knees spread apart on the examination table. The other positions listed are not helpful or recommended when examining the genital area of a prepubertal child. Ref: Emans SJ, Goldstein DP: The gynecologic examination of the prepubertal child with vulvovaginitis: Use of the knee-chest position. Pediatrics 1980;65(4):758-760. 2) Stenchever MA, Droegemueller W, Herbst AL, et al: Comprehensive Gynecology, ed 4. Mosby, 2001, pp 270-274. 3) Behrman RE, Kliegman RM, Jenson HB (eds): Nelson Textbook of Pediatrics, ed 17. Saunders, 2004, pp 1827-1828.

A 42-year-old male with a history of intravenous drug use asks to be tested for hepatitis C. The hepatitis C virus (HCV) antibody enzyme immunoassay and recombinant immunoblot assay are both reported as positive. The quantitative HCV RNA polymerase chain reaction test is negative. These test results are most consistent with: (check one) A. very early HCV infection B. current active HCV infection C. a false-positive antibody test D. past infection with HCV that is now resolved

D The most widely used initial assay for detecting hepatitis C virus (HCV) antibody is the enzyme immunoassay. A positive enzyme immunoassay should be followed by a confirmatory test such as the recombinant immunoblot assay. If negative, it indicates a false-positive antibody test. If positive, the quantitative HCV RNA polymerase chain reaction is used to measure the amount of virus in the blood to distinguish active from resolved HCV infection. In this case, the results of the test indicate that the patient had a past infection with HCV that is now resolved. Ref: Wilkins T, Malcolm JK, Raina D, Schade RR: Hepatitis C: Diagnosis and treatment. Am Fam Physician 2010;81(11):1351- 1357.

A 79-year-old white male with a previous history of prostate cancer has a lumbar spine film suggesting osteopenia. Subsequent bone density studies show a T score of -2.7. Which one of the following would be appropriate therapy? (check one) A. Testosterone B. Calcitonin nasal spray (Micalcin) C. Raloxifene (Evista) D. Alendronate (Fosamax)

D The only approved treatments for male osteoporosis are alendronate and recombinant parathyroid hormone. Several drugs have been tested in clinical trials, and more pharmacologic treatments should become available in the future as male osteoporosis is increasingly recognized. Testosterone should not be used in this patient because of his history of prostate cancer. Ref: Becker C: Clinical evaluation for osteoporosis. Clin Geriatr Med 2003;19(2):299-320.

You evaluate an 80-year-old white male who is a heavily medicated chronic schizophrenic. You note constant, involuntary chewing motions and repetitive movements of his legs. Which one of the following is the most likely diagnosis? (check one) A. Neuroleptic malignant syndrome B. Acute dystonia C. Huntington's disease D. Tardive dyskinesia E. Oculogyric crisis

D The patient has classic signs of tardive dyskinesia. Repetitive movement of the mouth and legs is caused by antipsychotic agents such as phenothiazines and haloperidol. Neuroleptic malignant syndrome consists of fever, autonomic dysfunction, and movement disorder. Acute dystonia involves twisting of the neck, trunk, and limbs into uncomfortable positions. Huntington's disease causes choreic movements, which are flowing, not repetitive. Oculogyric crisis involves the eyes. Ref: Sadock BJ, Sadock VA (eds): Kaplan & Sadock's Comprehensive Textbook of Psychiatry, ed 7. Lippincott Williams & Wilkins, 2000, pp 295-296. 2) Ross GW, Bowen JD: The diagnosis and differential diagnosis of dementia. Med Clin North Am 2002;86(3):455-476.

An 83-year-old female is admitted to the hospital with an exacerbation of her COPD. On the second hospital day she is clinically improved but is quite disoriented, experiencing visual hallucinations, agitation, and problems with recent memory and attention span. She is noted by the nursing staff to periodically fall asleep during conversation. Her previous medical history is notable for emphysema and hypertension, but there is no history of psychiatric problems. Her blood pressure is 140/82 mm Hg, pulse 88 beats/min, and oxygen saturation 98% on 2 L of nasal O2. Which one of the following does this patient most likely have? (check one) A. Dementia B. Acute depression C. Mania D. Delirium E. Schizophrenia

D The primary distinguishing feature of delirium is a course that is typically acute, with rapid deterioration over hours or days, rather than months as with dementia. Also, the severity of delirium tends to fluctuate over the course of hours, with patients appearing quite normal at times and wildly agitated with hallucinations at others. Frequently, extreme changes in psychomotor activity are noted with delirium; although this may also be seen with dementia, it is typically not seen until the latter stages. Bipolar disorders are characterized by the occurrence of mania, which is manifested by a full-blown disturbance of mood together with elation and irritability. Its onset is generally in the third or fourth decade of life. Schizophrenia, while often including hallucinations and delusions, usually starts in late adolescence or early adulthood, with a prodromal phase showing a gradual deterioration in function. Ref: Gleason OC: Delirium. Am Fam Physician 2003;67(5):1027-1034.

A 67-year-old Hispanic male comes to your office with severe periumbilical abdominal pain, vomiting, and diarrhea, which began suddenly several hours ago. His temperature is 37.0°C (98.6°F), blood pressure 110/76 mm Hg, and respirations 28/min. His abdomen is slightly distended, soft, and diffusely tender; bowel sounds are normal. Other findings include clear lungs, a rapid and irregularly irregular heartbeat, and a pale left forearm and hand with no palpable left brachial pulse. Right arm and lower extremity pulses are normal. Testing reveals the presence of blood in both his stool and his urine. His hemoglobin level is 16.4 g/dL (N 13.0-18.0) and his WBC count is 25,300/mm3 (N 4300-10,800). The diagnostic imaging procedure most likely to produce a specific diagnosis of the abdominal pain is: (check one) A. intravenous pyelography (IVP) B. sonography of the abdominal aorta C. a barium enema D. celiac and mesenteric arteriography E. contrast venography

D The sudden onset of severe abdominal pain, vomiting, and diarrhea in a patient with a cardiac source of emboli and evidence of a separate embolic event makes superior mesenteric artery embolization likely. In this case, evidence of a brachial artery embolus and a cardiac rhythm indicating atrial fibrillation suggest the diagnosis. Some patients may have a surprisingly normal abdominal examination in spite of severe pain. Microscopic hematuria and blood in the stool may both occur with embolization, and severe leukocytosis is present in more than two-thirds of patients with this problem. Diagnostic confirmation by angiography is recommended. Immediate embolectomy with removal of the propagated clot can then be accomplished and a decision made regarding whether or not the intestine should be resected. A second procedure may be scheduled to reevaluate intestinal viability.

A 23-year-old female sees you with a complaint of intermittent irregular heartbeats that occur once every week or two, but do not cause her to feel lightheaded or fatigued. They last only a few seconds and resolve spontaneously. She has never passed out, had chest pain, or had difficulty with exertion. She is otherwise healthy, and a physical examination is normal. Which one of the following cardiac studies should be ordered initially? (check one) A. 24-hour ambulatory EKG monitoring (Holter monitor) B. 30-day continuous closed-loop event recording C. Echocardiography D. An EKG E. Electrophysiologic studies

D The symptom of an increased or abnormal sensation of one's heartbeat is referred to as palpitations. This condition is common to primary care, but is often benign. Commonly, these sensations have their basis in anxiety or panic. However, about 50% of those who complain of palpitations will be found to have a diagnosable cardiac condition. It is recommended to start the evaluation for cardiac causes with an EKG, which will assess the baseline rhythm and screen for signs of chamber enlargement, previous myocardial infarction, conduction disturbances, and a prolonged QT interval. Ref: Abbott AV: Diagnostic approach to palpitations. Am Fam Physician 2005;71(4):743-750.

Which one of the following is the most common cause of bacterial diarrhea? (check one) A. Listeria monocytogenes B. Escherichia coli O157:H7 C. Shigella dysenteriae D. Campylobacter jejuni E. Salmonella enterica

D The treatment of acute and significant diarrhea often requires a specific diagnosis. Epidemiologic studies have shown that Campylobacter infections are the leading cause of bacterial diarrhea in the U.S. Ref: Kasper DL, Braunwald E, Fauci AS, et al (eds): Harrison's Principles of Internal Medicine, ed 16. McGraw-Hill, 2005, pp 907-909.

Over the past year, a 27-year-old female has had marked feelings of anxiety, tension, and irritability during the week preceding most menstrual cycles, accompanied by extreme fatigue and insomnia. She has regularly missed several days of work each month because of fatigue. She has no previous history of any health or mental problems, and within a few days of the onset of her period she is back to normal. Which one of the following is true concerning this condition? (check one) A. Neither biologic nor psychological factors play a part in this condition B. This condition is a variation of a depressive disorder C. Oral contraceptive pills are consistently effective in the treatment of this condition D. This problem can be effectively treated with serotonergic antidepressants E. Alprazolam (Xanax) is an effective first-line agent for treatment of this condition

D Women with premenstrual dysphoric disorder (PMDD) experience a cluster of mood, cognitive, and physical symptoms that recur in the luteal phase of the menstrual cycle and remit in the follicular phase. Multiple rigorously conducted, placebo-controlled, randomized trials have consistently shown the value of SSRIs, especially if administered during the luteal phase of the menstrual cycle. Among women whose mothers have been affected by PMS, 70% have PMS themselves, compared with 37% of women whose mothers have not been affected. Because many patients with PMDD do not have depressive symptoms, this disorder should not be considered as simply a depressive variant. Some studies have shown that symptoms actually worsen with the administration of oral contraceptive pills. Because of the potential for drug dependence, high-potency benzodiazepines such as alprazolam should be used only as second-line drug therapy if an optimal response is not achieved with an SSRI. Ref: Johnson SR: Premenstrual syndrome therapy. Clin Obstet Gynecol 1998;41(2):405-421. 2) Bhatia SC, Bhatia SK: Diagnosis and treatment of premenstrual dysphoric disorder. Am Fam Physician 2002;66(7):1239-1248, 1253-1254.

Which one of the following has the best evidence that it is safe for use in pregnancy? (check one) A. Alprazolam (Xanax) B. Lithium C. Bupropion (Wellbutrin) D. Fluoxetine (Prozac) E. Paroxetine (Paxil)

D The use of psychiatric medications during pregnancy should always involve consideration of the potential risks to the fetus in comparison to the well-being of the mother. Lithium is known to be teratogenic. Benzodiazepines such as alprazolam are controversial due to a possible link to cleft lip/palate. Studies have shown no significant risk of congenital anomalies from SSRI use in pregnancy, except for paroxetine. Paroxetine is a category D medication and should be avoided in pregnant women (SOR B). There is concern about an increased risk of congenital cardiac malformations from first-trimester exposure. Bupropion has not been studied extensively for use in pregnancy, and in one published study of 136 patients it was linked to an increased risk of spontaneous abortion. Ref: ACOG Committee on Practice Bulletins: ACOG Practice Bulletin: Clinical management guidelines for obstetricians-gynecologists number 92, April 2008 (replaces practice bulletin number 87, November 2007). Use of psychiatric medications during pregnancy and lactation. Obstet Gynecol 2008;111(4):1001-1020.

A slender 22-year-old female is concerned about a recent weight loss of 10 lb, frequent mild abdominal pain, and significant diarrhea of 2 months' duration. Her physical examination is unremarkable, and laboratory studies reveal only a moderate microcytic, hypochromic anemia. Based on this presentation, which one of the following is the most likely diagnosis? (check one) A. Irritable bowel syndrome B. Villous adenoma C. Infectious colitis D. Celiac disease E. Ulcerative colitis

D This constellation of symptoms strongly suggests celiac disease, a surprisingly common disease with a prevalence of 1:13 in the U.S. Half the adults in the U.S. with celiac disease or gluten-sensitive enteropathy present with anemia or osteoporosis, without gastrointestinal symptoms. Individuals with more significant mucosal involvement present with watery diarrhea, weight loss, and vitamin and mineral deficiencies. Ref: Goldman L, Ausiello D (eds): Cecil Textbook of Medicine, ed 22. Saunders, 2004, pp 852-854. 2) Kasper DL, Braunwald E, Fauci AS, et al (eds): Harrison's Principles of Internal Medicine, ed 16. McGraw-Hill, 2005, pp 1770-1772.

An 81-year-old male with type 2 diabetes mellitus has a hemoglobin A 1c of 10.9%. He is already on the maximum dosage of glipizide (Glucotrol). His other medical problems include mild renal insufficiency and moderate ischemic cardiomyopathy. Which one of the following would be the most appropriate change in this patient's diabetes regimen? (check one) A. Add metformin (Glucophage) B. Add sitagliptin (Januvia) C. Add pioglitazone (Actos) D. Initiate insulin therapy

D This geriatric diabetic patient should be treated with insulin. Metformin is contraindicated in patients with renal insufficiency. Sitagliptin should not be added to a sulfonylurea drug initially, the dosage should be lowered in patients with renal insufficiency, and given alone it would probably not result in reasonable diabetic control. Pioglitazone can cause fluid retention and therefore would not be a good choice for a patient with cardiomyopathy.

You examine an 11-month-old male who has had several paroxysms of abdominal pain in the last 2 hours. The episodes last 1-2 minutes; the infant screams, turns pale, and doubles up. Afterward, he seems normal. A physical examination is normal except for a possible fullness in the right upper quadrant of the abdomen. The most likely diagnosis is: (check one) A. pyloric stenosis B. choledochal cyst C. Meckel's diverticulum D. intussusception E. intestinal malrotation

D This is a classic presentation for intussusception, which usually occurs in children under the age of 2 years and is characterized by paroxysms of colicky abdominal pain. A mass is palpable in about two-thirds of patients. Pyloric stenosis presents with a palpable mass, but usually develops between 4 and 6 weeks of age. A choledochal cyst presents with the classic triad of right upper quadrant pain, jaundice, and a palpable mass. Meckel's diverticulum usually presents in this age group with painless lower gastrointestinal bleeding. Intestinal malrotation usually presents within the first 4 weeks of life and is characterized by bilious vomiting. Ref: Kliegman RM, Stanton BF, Geme JW III, et al (eds): Nelson Textbook of Pediatrics, ed 19. Elsevier Saunders, 2011, pp 1287-1289.

You have recently begun caring for a 25-year-old white female who has multiple complaints. You have seen her 3 times for walk-in office visits over the past month. She has shown appreciation for your work during the encounters, but has been critical of your care when talking to the office staff. At times she has been kind and charming, and at other times she has been rude and verbally abusive to your staff. She has a string of multiple relationships in the past, none of which has lasted very long. During times of intense stress, she has sometimes engaged in self-mutilation. She frequently changes jobs and living arrangements. Which one of the following strategies would be most appropriate in the care of this patient? (check one) A. Strive to develop a close relationship with the patient B. Ignore verbal attacks on staff members C. Prescribe lorazepam (Ativan) D. Schedule frequent office visits for follow-up E. Provide detailed, technical explanations for any therapies provided

D This patient demonstrates features of borderline personality disorder. These patients often demonstrate instability in interpersonal relationships and self-image, and may be impulsive. They can present with a wide range of symptoms, including depression, anger, paranoia, extreme dependency, self-mutilation, and alternating idealization and devaluation of their physicians. Their lives are often chaotic. Treatment strategies include maintaining a caring but somewhat detached professional stance. A close personal relationship is typically not therapeutic for these patients. Angry outbursts will often have to be tolerated, but limit-setting is necessary with respect to appropriate behaviors. SSRIs, atypical antipsychotics, and mood stabilizers may be of help at times, but anxiolytics are often abused and may be associated with self-mutilating behaviors. These patients tend to respond best to clear, simple, non-technical explanations related to their medical care. Ref: Ward RK: Assessment and management of personality disorders. Am Fam Physician 2004;70(8):1505-1512.

A 12-year-old Hispanic female develops fever, knee pain with swelling, diffuse abdominal pain, and a palpable purpuric rash. A CBC and platelet count are normal. Her long-term prognosis depends on the severity of involvement of the: (check one) A. gastrointestinal tract B. heart C. liver D. kidneys E. lungs

D This patient has Henoch-Schönlein purpura. This condition is associated with a palpable purpuric rash, without thrombocytopenia. Other diagnostic criteria include bowel angina (diffuse abdominal pain or bowel ischemia), age ≤20, renal involvement, and a biopsy showing predominant immunoglobulin A deposition. The long-term prognosis depends on the severity of renal involvement. Almost all children with Henoch-Schönlein purpura have a spontaneous resolution, but 5% may develop end-stage renal disease. Therefore, patients with renal involvement require careful monitoring (SOR A). Ref: Reamy BV, Lindsay TJ: Henoch-Schönlein purpura. Am Fam Physician 2009;80(7):697-704.

A 12-year-old male is brought to your office with an animal bite. After talking with the patient, you learn that he was bitten on his left hand as he attempted to pet a stray cat a little over 24 hours ago. He says that the bite was very painful, and that it bled for a few minutes. His parents cared for the bite by rinsing it and covering it with a bandage. His chart indicates that he received a tetanus shot last year. On examination, the patient is afebrile with stable vital signs. The site is warm and tender to light palpation, with surrounding erythema measuring approximately 3 cm in diameter. Which one of the following is the most likely infectious agent in this situation? (check one) A. Candida albicans B. Capnocytophaga canimorsus C. Methicillin-resistant Staphylococcus aureus (MRSA) D. Pasteurella multocida E. Streptococcus pneumoniae

D asteurella species are isolated from up to 50% of dog bite wounds and up to 75% of cat bite wounds, and the hand is considered a high-risk area for infection (SOR A). Although much more rare, Capnocytophaga canimorsus, a fastidious gram-negative rod, can cause bacteremia and fatal sepsis after animal bites, especially in asplenic patients or those with underlying hepatic disease. Anaerobes isolated from dog and cat bite wounds include Bacteroides, Fusobacterium, Porphyromonas, Prevotella, Propionibacterium and Peptostreptococcus. In addition to animal oral flora, human skin flora are also important pathogens, but are less commonly isolated. These can include streptococci and staphylococci, including methicillin-resistant Staphylococcus aureus (MRSA). Coverage for MRSA may be especially important if the patient has risk factors for colonization with community-acquired MRSA. Pets can also become colonized with MRSA and transmit it via bites and scratches. Cat bites that become infected with Pasteurella multocida can be complicated by cellulitis, which may form around the wound within 24 hours and is often accompanied by redness, tenderness, and warmth. The use of prophylactic antibiotics is associated with a statistically significant reduction in the rate of infection in hand bites (SOR A). If infection develops and is left untreated, the most common complications are tenosynovitis and abscess formation; however, local complications can include septic arthritis and osteomyelitis. Fever, regional adenopathy, and lymphangitis are also seen.

A 65-year-old male presents for a follow-up visit for severe depression. His symptoms have included crying episodes, difficulty maintaining sleep, and decreased appetite. He has suicidal ideations and states that he has a gun in his home. He also thinks his wife is having an affair, but she is present and is adamant that this is not true. His symptoms have not been relieved by maximum doses of sertraline (Zoloft), venlafaxine (Effexor), or citalopram (Celexa). He currently is taking duloxetine (Cymbalta), which also has failed to relieve his symptoms. Which one of the following would most likely provide the quickest relief of his symptoms? (check one) A. Counseling B. Bupropion (Wellbutrin) C. Stopping duloxetine and starting an MAO inhibitor D. Electroconvulsive therapy

D This patient has psychotic depression with suicidal ideations and has not responded to maximum doses of several antidepressants. He is more likely to respond to electroconvulsive therapy than to counseling or a change in medication. Ref: Lisanby SH: Electroconvulsive therapy for depression. N Engl J Med 2007;359(19):1939-1945.

A 45-year-old female with no significant past medical history presents to your office with 2 weeks of worsening pain in her right arm. For the past 2 months she has worked on a plastics manufacturing assembly line. A physical examination reveals no swelling and a normal range of motion. She has normal strength in the upper extremity but she experiences increased pain with extension of her right wrist against resistance. Palpation reveals marked tenderness over the lateral epicondyle of the right arm. Which one of the following is most likely to improve the patient's long-term outcome? (check one) A. Physical therapy B. Regular physical activity using her hands and arms C. Use of an inelastic, nonarticular proximal forearm strap D. Modifying her work routines

D This patient has signs and symptoms of lateral epicondylitis, also known as tennis elbow, or alternatively as lateral epicondylalgia to reflect the noninflammatory nature of the condition. This is an overuse tendinopathy of the common extensor tendon origin of the lateral elbow. Conservative care that includes offloading the involved tendons is the key to improving outcomes at 1 year, which would mean modifying this patient's work. Physical therapy can improve pain and function in the short term, but has not been shown to improve long-term outcomes at 1 year in randomized trials (SOR A). The evidence is weaker for bracing, with some studies showing improved pain and function at 3-6 weeks (SOR B). Recent randomized, controlled trials have made it clear that while corticosteroid injections reduce acute pain for up to 6 weeks, their use increases rates of poor long-term outcomes (SOR A).

A 60-year-old male is referred to you by his employer for management of his hypertension. He has been without primary care for several years due to a lapse in insurance coverage. During a recent employee health evaluation, he was noted to have a blood pressure of 170/95 mm Hg. He has a 20-year history of hypertension and suffered a small lacunar stroke 10 years ago. He has no other health problems and does not smoke or drink alcohol. A review of systems is negative except for minor residual weakness in his right upper extremity resulting from his remote stroke. His blood pressure is 168/98 mm Hg when initially measured by your nurse, and you obtain a similar reading during your examination. In addition to counseling him regarding lifestyle modifications, which one of the following is the most appropriate treatment for his hypertension? (check one) A. An angiotensin receptor blocker B. A β-blocker C. A calcium channel blocker D. A thiazide diuretic/ACE inhibitor combination E. No medication

D This patient has stage 2 hypertension, and his history of stroke is a compelling indication to use specific classes of antihypertensives. For patients with a history of previous stroke, JNC-7 recommends using combination therapy with a diuretic and an ACE inhibitor to treat the hypertension, as this combination has been clinically shown to reduce the risk of recurrent stroke. Other classes of drugs have not been shown to be of benefit for secondary stroke prevention. Although blood pressure should not be lowered quickly in the setting of acute ischemic stroke, this patient is not having an acute stroke, so treatment of his hypertension is warranted.

A 16-year-old female presents with a complaint of pelvic cramps with her menses over the past 2 years. She describes her periods as heavy, and says they occur once a month and last for 7 days, with no spotting in between. She has never been sexually active and does not expect this to change in the foreseeable future. An abdominal examination is normal. Which one of the following would be the most appropriate next step? (check one) A. A pelvic examination B. Ultrasonography C. A TSH level D. Naproxen prior to and during menses

D This patient is experiencing primary dysmenorrhea, a common finding in adolescents, with estimates of prevalence ranging from 20% to 90%. Because symptoms started at a rather young age and she has pain only during menses, endometriosis or other significant pelvic pathology is unlikely. An infection is doubtful, considering that she is not sexually active and that symptoms have been present for 2 years. In the absence of red flags, a pelvic examination, laboratory evaluation, and pelvic ultrasonography are not necessary at this time. However, they can be ordered if she does not respond to simple treatment. NSAIDs such as naproxen have a slight effect on platelet function, but because they inhibit prostaglandin synthesis they actually decrease the volume of menstrual flow and lessen the discomfort of pelvic cramping. Acetaminophen would have no effect on prostaglandins. Ref: French L: Dysmenorrhea. Am Fam Physician 2005;71(2):285-291.

You see a 23-year-old gravida 1 para 0 for her prenatal checkup at 38 weeks gestation. She complains of severe headaches and epigastric pain. She has had an uneventful pregnancy to date and had a normal prenatal examination 2 weeks ago. Her blood pressure is 140/100 mm Hg. A urinalysis shows 2+ protein; she has gained 5 lb in the last week, and has 2+ pitting edema of her legs. The most appropriate management at this point would be: (check one) A. Strict bed rest at home and reexamination within 48 hours B. Admitting the patient to the hospital for bed rest and frequent monitoring of blood pressure, weight, and proteinuria C. Admitting the patient to the hospital for bed rest and monitoring, and beginning hydralazine (Apresoline) to maintain blood pressure below 140/90 mm Hg D. Admitting the patient to the hospital, treating with parenteral magnesium sulfate, and planning prompt delivery either vaginally or by cesarean section

D This patient manifests a rapid onset of preeclampsia at term. The symptoms of epigastric pain and headache categorize her preeclampsia as severe. These symptoms indicate that the process is well advanced and that convulsions are imminent. Treatment should focus on rapid control of symptoms and delivery of the infant. Ref: Cunningham FG, Gant NF, Leveno KJ, et al: Williams Obstetrics, ed 21. McGraw-Hill, 2001, pp 569-571, 591-592.

An 8-year-old male is brought to your office for evaluation of recurrent headaches. His mother explains that the headaches occur at least twice a week and often require him to miss school. The patient says he sometimes feels nauseated and that being in a dark room helps. His mother states that she had migraines as a child. The child's only other medical issue is constipation. A head CT ordered by another physician was negative. Which one of the following would be best for preventing these episodes? (check one) A. Sumatriptan (Imitrex) B. Ibuprofen C. Carbamazepine (Tegretol) D. Propranolol (Inderal) E. Amitriptyline

D This patient most likely is suffering from recurrent migraine headaches; at the described frequency and intensity, he meets the criteria for prophylactic medication. Ibuprofen or acetaminophen could still be used as rescue medications, but a daily agent is indicated and propranolol is the best choice for this patient (SOR B). Sumatriptan is not approved for children under the age of 12 years. Carbamazepine has significant side effects and requires monitoring. Amitriptyline is a commonly used agent, but it could worsen his constipation.

Which one of the following is an effective screening method for ovarian cancer in elderly females at average risk? (check one) A. Annual CA-125 assays B. Annual pelvic ultrasonography C. Annual Papanicolaou (Pap) tests and pelvic examinations D. No currently available method

D Two large European trials studied the use of CA-125 and CA-125 with transvaginal ultrasonography (TVU) as screening methods for ovarian cancer. TVU has been reviewed separately. None of these methods is effective as a screening test. No major organization recommends screening women at average risk. The American Cancer Society does not recommend routine screening; the American College of Obstetricians and Gynecologists recommends against population-based screening; an NIH consensus conference recommended obtaining a family history and performing annual pelvic examinations. The U.S. Preventive Services Task Force graded routine screening for ovarian cancer as a "D," meaning that there is fair evidence to recommend excluding ovarian cancer screening as a part of the periodic health examination. This recommendation reflects both a lack of benefit from screening and the fact that a significant number of women have to undergo exploratory surgery to find a single case. Ref: Wender RC, Smith R, Harper D: Cancer screening. Prim Care 2002;29(3):697-725.

You see a 78-year-old male in the hospital the day after his hip-replacement surgery. He has not voided in the past 12 hours. A urethral catheter is placed and 500 mL of urine is removed from his bladder. Which one of the following is most likely to improve the success rate of a voiding trial? (check one) A. Using a specialized catheter coudé instead of a standard catheter B. Leaving the catheter in place for at least 2 weeks C. Immediately removing the catheter to prevent a urinary tract infection D. Starting tamsulosin (Flomax), 0.4 mg daily, at the time of catheter insertion E. Starting antibiotic prophylaxis at the time of catheter insertion

D Urinary retention is a common problem in hospitalized patients, especially following certain types of surgery. Starting an a-blocker at the time of insertion of the urethral catheter has been shown to increase the success of a voiding trial (SOR A). Voiding trial success rates have not been shown to be improved by leaving the catheter in for 2 weeks, immediate removal of the catheter, using a specialized catheter, or antibiotic prophylaxis.

Which one of the following is associated with vacuum-assisted delivery? (check one) A. Lower fetal risk than with forceps delivery B. More maternal soft-tissue trauma than forceps delivery C. A reduced likelihood of severe perineal laceration compared to spontaneous delivery D. An increased incidence of shoulder dystocia

D Vacuum-assisted delivery is associated with higher rates of neonatal cephalhematoma and retinal hemorrhage compared with forceps delivery. A systematic review of 10 trials found that vacuum-assisted deliveries are associated with less maternal soft-tissue trauma when compared to forceps delivery. Compared with spontaneous vaginal delivery, the likelihood of a severe perineal laceration is increased in women who have vacuum-assisted delivery without episiotomy, and the odds are even higher in vacuum-assisted delivery with episiotomy. Operative vaginal delivery is a risk factor for shoulder dystocia, which is more common with vacuum-assisted delivery than with forceps delivery.

Which one of the following is true regarding medical errors? (check one) A. Malpractice litigation is more common when physicians disclose errors to patients B. The use of the word "error" should be avoided when disclosing mistakes to patients C. Physicians in private practice are more likely to disclose errors to patients than physicians employed by institutions or health care organizations D. Patients prefer to receive apologies and explanations when an error has been made E. It is ethically defensible to only disclose an error if the patient is aware there is a problem

D When a medical error has been made, patients prefer that their physician disclose the error and offer an explanation of events. Withholding that information from a patient is not ethical and is counter to standards set forth by various organizations such as the Joint Commission on Accreditation of Health Care Organizations. Using the word "error" is acceptable and does not lead to an increase in litigation. In fact, there is no evidence that malpractice litigation rates increase when an error is admitted, and rates often decrease. Private-practice physicians are less likely to admit errors to patients. It is surmised that these physicians have less access to training in disclosure than those employed by hospitals or health care organizations.

An asymptomatic 55-year-old male visits a health fair, where he has a panel of blood tests done. He brings the results to you because he is concerned about the TSH level of 12.0 µU/mL (N 0.45-4.5). His free T4 level is normal. Which one of the following is most likely to be associated with this finding? (check one) A. Atrial fibrillation B. Reduced bone density C. Systolic heart failure D. Elevated LDL cholesterol E. Type 2 diabetes mellitus

D With subclinical thyroid dysfunction, TSH is either below or above the normal range, free T3 or T4 levels are normal, and the patient has no symptoms of thyroid disease. Subclinical hypothyroidism (TSH >10 µU/mL) is likely to progress to overt hypothyroidism, and is associated with increased LDL cholesterol. Subclinical hyperthyroidism (TSH <0.1 µU/mL) is associated with the development of atrial fibrillation, decreased bone density, and cardiac dysfunction. Neither type of subclinical thyroid dysfunction is associated with diabetes mellitus. There is insufficient evidence of benefit to warrant early treatment of either condition. Ref: Wilson GR, Curry RW Jr: Subclinical thyroid disease. Am Fam Physician 2005;72(8):1517-1524.

Which one of the following is an indication for a second dose of pneumococcal polysaccharide vaccine (Pneumovax 23) in children? (check one) A. Cerebrospinal fluid leak B. Cyanotic congenital heart disease C. Type 1 diabetes mellitus D. Sickle cell disease E. Chronic bronchopulmonary dysplasia

D atients with chronic illness, diabetes mellitus, cerebrospinal fluid leaks, chronic bronchopulmonary dysplasia, cyanotic congenital heart disease, or cochlear implants should receive one dose of pneumococcal polysaccharide vaccine after 2 years of age, and at least 2 months after the last dose of pneumococcal conjugate vaccine (Prevnar 13). Revaccination with polysaccharide vaccine is not recommended for these patients. Individuals with sickle cell disease, those with anatomic or functional asplenia, immunocompromised persons with renal failure or leukemia, and HIV-infected persons should receive polysaccharide vaccine on this schedule and should be revaccinated at least 5 years after the first dose. Ref: Zimmerman RK, Middleton DB: Vaccines for persons at high risk, 2007. J Fam Pract 2007;56(2 Suppl Vaccines):S38-S46, C4-C5.

A 36-year-old male complains of clear rhinorrhea, nasal congestion, and watery, itchy eyes for several months. Tests in the past have suggested that he has an allergy to dust mites. Which one of the following is most likely to provide the most relief from his symptoms? (check one) A. Oral antihistamines B. An oral leukotriene-receptor antagonist C. Intranasal antihistamines D. Intranasal corticosteroids E. Furnace filters and mite-proof bedding covers

D his patient has classic symptoms of allergic rhinitis. Intranasal corticosteroids are considered the mainstay of treatment for mild to moderate cases. In multiple studies, intranasal corticosteroid sprays have proven to be more efficacious than the other options listed, even for ocular symptoms. Air filtration systems and bedding covers have not been shown to reduce symptoms. Ref: Sur DK, Scandale S: Treatment of allergic rhinitis. Am Fam Physician

Your community recently experienced an outbreak of infectious diarrheal illness due to the protozoan Cryptosporidium, a chlorine-resistant organism. A reporter from the local newspaper asks you if there are other chlorine-resistant fecal organisms that could contaminate public drinking water. You would tell the reporter that such organisms include: (check one) A. Escherichia coli B. Vibrio cholerae C. Campylobacter jejuni D. Giardia lamblia E. Rotavirus

D rganisms that can persist in water environments and survive disinfection, especially chlorination, are most likely to cause disease outbreaks related to drinking water. Cryptosporidium oocysts and Giardia cysts are resistant to chlorine and are important causes of gastroenteritis from drinking water. Entamoeba histolytica and hepatitis A virus are also relatively chlorine resistant. The other organisms listed are chlorine sensitive. Ref: Balbus JM, Lang ME: Is the water safe for my baby? Pediatr Clin North Am 2001;48(5):1129-1152.

Which one of the following medications should be discontinued in a patient with diabetic gastroparesis? (check one) A. Exenatide (Byetta) B. Benazepril (Lotensin) C. Metformin (Glucophage) D. Hydrochlorothiazide E. Prochlorperazine maleate

Delayed gastric emptying may be caused or exacerbated by medications for diabetes, including amylin analogues (e.g., pramlintide) and glucagon-like peptide 1 (e.g., exenatide). Delayed gastric emptying has a direct effect on glucose metabolism, in addition to being a means of reducing the severity of postprandial hyperglycemia. In a clinical trial of exenatide, nausea occurred in 57% of patients and vomiting occurred in 19%, which led to the cessation of treatment in about one-third of patients. The other medications listed do not cause delayed gastric emptying. Ref: Camilleri M: Diabetic gastroparesis. N Engl J Med 2007;356(8):820-829.

You are preparing to evaluate a patient in the emergency department. A BPN level was ordered by the physician from the previous shift who handed the patient over to you. The level is reported as 459 pg/mL. You have not yet interviewed or examined the patient. Based on the information you have at this point, which one of the following is true regarding this patient? (check one) A. The patient has heart failure B. The patient has systolic heart failure C. The patient has acute heart failure D. The patient does not have heart failure E. The patient's diagnosis is uncertain

E According to the 2010 American Heart Association scientific statement regarding acute heart failure syndrome, levels of natriuretic peptides such as BNP lack the specificity necessary to function as absolute indicators of acute heart failure syndrome even when they exceed established thresholds for the diagnosis. BNP levels vary with age, sex, body habitus, renal function, and abruptness of symptom onset. Elevated BNP levels also have been associated with renal failure (because of reduced clearance), pulmonary embolism, pulmonary hypertension, and chronic hypoxia. BNP measures are not a substitute for a comprehensive assessment for signs and symptoms of heart failure, and a laboratory test by itself cannot be used to determine the diagnosis or management of heart failure. Clinical evaluation and follow-up are essential to assure proper care for patients with heart failure or any other cardiac problem.

A newborn male has a skin eruption on his forehead, nose, and cheeks. The lesions are mostly closed comedones with a few open comedones, papules, and pustules. No significant erythema is seen. Which one of the following is the most likely diagnosis? (check one) A. Erythema toxicum neonatorum B. Localized superficial Candida infection C. Herpes simplex D. Milia E. Acne neonatorum

E Acne neonatorum occurs in up to 20% of newborns. It typically consists of closed comedones on the forehead, nose, and cheeks, and is thought to result from stimulation of sebaceous glands by maternal and infant androgens. Parents should be counseled that lesions usually resolve spontaneously within 4 months without scarring. Findings in erythema toxicum neonatorum include papules, pustules, and erythema. Candida and herpes lesions usually present with vesiculopustular lesions in the neonatal period. Milia consists of 1- to 2-mm pearly keratin plugs without erythema, and may occur on the trunk and limbs. Ref: O'Connor NR, McLaughlin MR, Ham P: Newborn skin: Part I. Common rashes. Am Fam Physician 2008;77(1):47-52.

A 28-year-old female consults you because of fatigue, arthralgias that are worse in the morning, and painful, swollen finger joints. She is a high-school teacher. Her erythrocyte sedimentation rate is 60 mm/hr and a test for rheumatoid factor is strongly positive. The best choice for initial therapy would be: (check one) A. prednisone B. aspirin C. naproxen (Naprosyn) D. rituximab (Rituxan) E. methotrexate (Rheumatrex)

E Aspirin was once the best initial therapy for rheumatoid arthritis and then NSAIDs became the preferred treatment. Now, however, disease-modifying drugs such as methotrexate are the best choice for initial therapy. Aspirin and NSAIDs are no longer considered first-line treatment because of concerns about their limited effectiveness, inability to modify the long-term course of the disease, and gastrointestinal and cardiotoxic effects. Glucocorticoids such as prednisone are often useful, but have significant side effects. Biologic agents such as rituximab are expensive and have significantly more side effects than methotrexate. Ref: Scott DL, Wolfe F, Huizinga TWJ: Rheumatoid arthritis. Lancet 2010;376(9746):1094-1108.

A 73-year-old male with COPD presents to the emergency department with increasing dyspnea. Examination reveals no sign of jugular venous distention. A chest examination reveals decreased breath sounds and scattered rhonchi, and the heart sounds are very distant but no gallop or murmur is noted. There is +1 edema of the lower extremities. Chest radiographs reveal cardiomegaly but no pleural effusion. The patient's B-type natriuretic peptide level is 850 pg/mL (N <100) and his serum creatinine level is 0.8 mg/dL (N 0.6-1.5). Which one of the following would be the most appropriate initial management? (check one) A. Intravenous heparin B. Tiotropium (Spiriva) C. Levalbuterol (Xopenex) via nebulizer D. Prednisone, 20 mg twice daily for 1 week E. Furosemide (Lasix), 40 mg intravenously

E B-type natriuretic peptide (BNP) is secreted in the ventricles and is sensitive to changes in left ventricular function. Concentrations correlate with end-diastolic pressure, which in turn correlates with dyspnea and congestive heart failure. BNP levels can be useful when trying to determine whether dyspnea is due to cardiac, pulmonary, or deconditioning etiologies. A value of less than 100 pg/mL excludes congestive heart failure as the cause for dyspnea. If it is greater than 400 pg/mL, the likelihood of congestive heart failure is 95%. Patients with values of 100-400 pg/mL need further investigation. There are some pulmonary problems that may elevate BNP, such as lung cancer, cor pulmonale, and pulmonary embolus. However, these patients do not have the same extent of elevation that those with acute left ventricular dysfunction will have. If these problems can be ruled out, then individuals with levels between 100-400 pg/mL most likely have congestive heart failure. Initial therapy should be a loop diuretic. It should be noted that BNP is partially excreted by the kidneys, so levels are inversely proportional to creatinine clearance. Ref: Maisel AS, Zoorob R: B-type natriuretic peptide in congestive heart failure: Diagnosis and management. CME Bulletin 2004;3(3). 2) Maisel AS, Mehra MR: Understanding B-type natriuretic peptide and its role in diagnosing and monitoring congestive heart failure. Clin Cornerstone 2005;7(Suppl 1):S7-S17.

The most common cause of abnormal vaginal discharge in a sexually active 19-year-old female is (check one) A. Candida albicans B. Trichomonas vaginalis C. Staphylococcus D. Group B Streptococcus E. Bacterial vaginosis

E Bacterial vaginosis (BV) is the most common cause of acute vaginitis, accounting for up to 50% of cases in some populations. It is usually caused by a shift in normal vaginal flora. BV is considerably more common as a cause of vaginal discharge than C. albicans and T. vaginalis. Ref: Eckert LO: Acute vulvovaginitis. N Engl J Med 2006;355(12):1244-1252.

Which one of the following is the leading cause of death in women? (check one) A. Breast cancer B. Lung cancer C. Ovarian cancer D. Osteoporosis E. Cardiovascular disease

E Cardiovascular disease is the leading cause of death among women. According to the CDC, 29.3% of deaths in females in the U.S. in 2001 were due to cardiovascular disease and 21.6% were due to cancer, with most resulting from lung cancer. Breast cancer is the third most common cause of cancer death in women, and ovarian cancer is the fifth most common. Ref: Anderson RN, Smith BL: Deaths: Leading causes for 2001. NatlVital Stat Rep 2003 Nov 7;52(9). Available at www.cdc.gov/nchs/data/nvsr/nvsr52/nvsr52_09.pdf. 2) Centers for Disease Control: CDC Office of Women's Health. Leading causes of death females—United States, 2001. Available at www.cdc.gov/od/spotlight/nwhw/lcod.htm. 3) American Cancer Society: Cancer Facts and Figures 2002. Publication number 02-250M-No. 5008.02. 4) Centers for Disease Control and Prevention: CDC recommendations regarding selected conditions affecting women's health. MMWR 2000;49(RR-2):1-73.

Which one of the following is most predictive of increased perioperative cardiovascular events associated with noncardiac surgery in the elderly? (check one) A. An age of 80 years B. Left bundle-branch block C. Atrial fibrillation with a rate of 80 beats/min D. A history of previous stroke E. Renal insufficiency (creatinine 2.0 mg/dL)

E Clinical predictors of increased perioperative cardiovascular risk for elderly patients include major risk factors such as unstable coronary syndrome (acute or recent myocardial infarction, unstable angina), decompensated congestive heart failure, significant arrhythmia (high-grade AV block, symptomatic ventricular arrhythmia, supraventricular arrhythmias with uncontrolled ventricular rate), and severe valvular disease. Intermediate predictors are mild angina, previous myocardial infarction, compensated congestive heart failure, diabetes mellitus, and renal insufficiency. Minor predictors are advanced age, an abnormal EKG, left ventricular hypertrophy, left bundle-branch block, ST and T-wave abnormalities, rhythm other than sinus, low functional capacity, history of stroke, and uncontrolled hypertension. Ref: Schroeder BM: Updated guidelines for perioperative cardiovascular evaluation for noncardiac surgery. Am Fam Physician 2002;66(6):1096-1107.

Which one of the following is contraindicated in the second and third trimesters of pregnancy? (check one) A. Amoxicillin B. Azithromycin (Zithromax) C. Ceftriaxone (Rocephin) D. Ciprofloxacin (Cipro) E. Doxycycline

E Doxycycline is contraindicated in the second and third trimesters of pregnancy due to the risk of permanent discoloration of tooth enamel in the fetus. Cephalosporins such as ceftriaxone are usually considered safe to use during pregnancy. The use of ciprofloxacin during pregnancy does not appear to increase the risk of major congenital malformation, nor does the use of amoxicillin. Animal studies using rats and mice treated with daily doses of azithromycin up to maternally toxic levels revealed no impairment of fertility or harm to the fetus. Ref: Briggs GG, Freeman RK, Yaffe SJ (eds): Drugs in Pregnancy and Lactation, ed 8. Lippincott Williams & Wilkins, 2008, pp 6, 158, 292-293, 361, 364, 579.

An 83-year-old female presents to your office as a new patient. She recently moved to the area to be closer to her family. A history reveals that she has been in excellent health, has no complaints, and is on no medications except occasional acetaminophen for knee pain. She has never been in the hospital and has not had any operations. She says that she feels well. The examination is normal, with expected age-related changes, except that her blood pressure on three different readings averages 175/70 mm Hg. These readings are confirmed on a subsequent follow-up visit. In addition to lifestyle changes, which one of the following would be most appropriate for the initial management of this patient's hypertension? (check one) A. An alpha-blocker B. An ACE inhibitor C. A beta-blocker D. An angiotensin receptor blocker E. A thiazide diuretic

E Clinical trials support the treatment of systolic hypertension in the older person with a systolic blood pressure of at least 160 mm Hg. (Systolic hypertension is defined as systolic blood pressure of at least 140 mm Hg and a diastolic blood pressure of less than 90 mm Hg.) The studies most strongly support the use of thiazide diuretics and long-acting calcium channel blockers as first-line therapy. Alpha-blockers are not recommended. ACE inhibitors, beta-blockers, and angiotensin receptor blockers are used when certain compelling indications are present, e.g., in a patient with diabetes or who has had a myocardial infarction. Ref: Chaudhry SI, Krumholz HM, Foody JM: Systolic hypertension in older persons. JAMA 2004;292(9):1074-1080.

You have been asked to see a 75-year-old female who has just had hip surgery to correct a fractured femoral neck. She has a 2-year history of diabetes mellitus treated with pioglitazone (Actos), 30 mg daily, and metformin (Glucophage), 1000 mg twice daily. She is now fully alert and has been able to eat her evening meal. A physical examination is normal except for her being mildly overweight and having a bandage on her left hip. A CBC and chemistry profile done earlier today were normal except for a serum glucose level of 200 mg/dL. Her hemoglobin A1c at an office visit 2 weeks ago was 6.8%. Which one of the following would be the best management of this patient's diabetes at this time? (check one) A. Stop her usual medications and begin a sliding-scale insulin regimen B. Stop the metformin only C. Initiate an insulin drip to maintain glucose levels of 80-120 mg/dL D. Decrease the dosage of pioglitazone E. Continue with her usual medication regimen

E Current evidence indicates that traditional sliding-scale insulin as the only means of controlling glucose in hospitalized patients is inadequate. For patients in a surgical intensive-care unit, using an insulin drip to maintain tight glucose control decreases the risk of sepsis but has no mortality benefit. Metformin should be stopped if the serum creatinine level is ≥1.5 mg/dL in men or ≥1.4 mg/dL in women, or if an imaging procedure requiring contrast is needed. In patients who have not had their hemoglobin A 1c measured in the past 30 days, this could be done to provide a better indication of glucose control. If adequate control has been demonstrated and no contraindications are noted, the patient's usual medication regimen should be continued

A 4-year-old Hispanic female has been discovered to have a congenital hearing loss. Her mother is an 18-year-old migrant farm worker who is currently at 8 weeks' gestation with her second pregnancy. The mother has been found to have cervical dysplasia on her current Papanicolaou (Pap) smear and has also tested positive for Chlamydia. The most likely cause of this child's hearing loss is: (check one) A. Human parvovirus B19 B. Varicella zoster virus C. Herpes simplex virus D. Toxoplasmosis E. Cytomegalovirus

E Cytomegalovirus (CMV) is the most common congenital infection and occurs in up to 2.2% of newborns. It is the leading cause of congenital hearing loss. The virus is transmitted by contact with infected blood, urine, or saliva, or by sexual contact. Risk factors for CMV include low socioeconomic status, birth outside North America, first pregnancy prior to age 15, a history of cervical dysplasia, and a history of sexually transmitted diseases. Infection can be primary or a reactivation of a previous infection. While the greatest risk of infection is during the third trimester, those occurring in the first trimester are the most dangerous to the fetus. Ref: Choby BA: Pregnancy Care. AAFP Self-Assessment monograph series, 2003, no 292, pp 40-43. 2) Rudolph CD, Rudolph AM (eds): Rudolph's Pediatrics, ed 21. McGraw-Hill, 2003, pp 1031-1035.

A 55-year-old female presents to an urgent-care facility with a complaint of weakness of several weeks' duration. She has no other symptoms. She has been healthy except for a history of hypertension that has been difficult to control despite the use of hydrochlorothiazide, 25 mg daily; lisinopril (Prinivil, Zestril), 40 mg daily; amlodipine (Norvasc), 10 mg daily; and doxazosin (Cardura), 8 mg daily. On examination her blood pressure is 164/102 mm Hg, with the optic fundi showing grade 2 changes. She has normal pulses, a normal cardiac examination, and no abdominal bruits. A CBC is normal and a blood chemistry panel is also normal except for a serum potassium level of 3.1 mmol/L (N 3.5-5.5). Which one of the following would be best for confirming the most likely diagnosis in this patient? (check one) A. Magnetic resonance angiography of the renal arteries B. A renal biopsy C. 24-hour urine for metanephrines D. Early morning fasting cortisol E. A plasma aldosterone/renin ratio

E Difficult-to-control hypertension has many possible causes, including nonadherence or the use of alcohol, NSAIDs, certain antidepressants, or sympathomimetics. Secondary hypertension can be caused by relatively common problems such as chronic kidney disease, obstructive sleep apnea, or primary hyperaldosteronism, as in the case described here. As many as 20% of patients referred to specialists for poorly controlled hypertension have primary hyperaldosteronism. It is more common in women and often is asymptomatic. A significant number of these individuals will not be hypokalemic. Screening can be done with a morning plasma aldosterone/renin ratio. If the ratio is 20 or more and the aldosterone level is >15 ng/dL, then primary hyperaldosteronism is likely and referral for confirmatory testing should be considered. Ref: Viera AJ, Hinderliter AL: Education and management of the patient with difficult-to-control or resistant hypertension. Am Fam Physician 2009;79(10):863-

An 85-year-old white male with terminal pancreatic cancer is expected to survive for another 2 weeks. His pain has been satisfactorily controlled with sustained-release morphine. He has now developed a disturbed self-image, hopelessness, and anhedonia, and has told family members that he has thought about suicide. Psychomotor retardation is also noted. His family is supportive. His daughter feels he is depressed, while his son feels this is more of a grieving process. Which one of the following would be most appropriate for managing this problem? (check one) A. Reassurance B. Alprazolam (Xanax) C. Trazodone (Desyrel) D. Olanzapine (Zyprexa) E. Methylphenidate (Ritalin)

E Distinguishing between preparatory grief and depression in a dying patient is not always simple. Initially one should evaluate for unresolved physical symptoms and treat any that are present. If the patient remains in distress, mood should be evaluated. If it waxes and wanes with time and if self-esteem is normal, this is likely preparatory grief. The patient may have fleeting thoughts of suicide and likely will express worry about separation from loved ones. This usually responds to counseling. In patients with anhedonia, persistent dysphoria, disturbed self-image, hopelessness, poor sense of self-worth, rumination about death and suicide, or an active desire for early death, depression is the problem. For patients who are expected to live only a few days, psychostimulants such as methylphenidate should be used. For those who are expected to survive longer, SSRIs are a good choice. Ref: Periyakoil VS, Hallenbeck J: Identifying and managing preparatory grief and depression at the end of life. Am Fam Physician 2002;65(5):883-890, 897-898.

Which one of the following is the most effective drug for the treatment of alcohol dependence? (check one) A. Disulfiram (Antabuse) B. Diazepam (Valium) C. Amitriptyline (Elavil) D. Fluoxetine (Prozac) E. Naltrexone (ReVia)

E Drug therapy should be considered for all patients with alcohol dependence who do not have medical contraindications to the use of the drug and who are willing to take it. Of the several drugs studied for the treatment of dependence, the evidence of efficacy is strongest for naltrexone and acamprosate. Naltrexone is currently available in the U.S.; acamprosate and tiapride are currently available in Europe but not in the U.S. Ref: Swift RM: Drug therapy for alcohol dependence. N Engl J Med 1999;340(19):1482-1490. 2) Kasper DL, Braunwald E, Fauci AS, et al (eds): Harrison's Principles of Internal Medicine, ed 16. McGraw-Hill, 2005, pp 1810, 2562-2564.

A 34-year-old white male presents with a history and findings that satisfy DSM-IV criteria for bipolar disorder. Which one of the following treatment options is the most effective for long-term management of the majority of patients with this disorder? (check one) A. Electroconvulsive therapy (ECT) B. Tricyclic antidepressants C. SSRIs D. Monoamine oxidase (MAO) inhibitors E. Lithium

E Electroconvulsive therapy (ECT) is as effective as medication for the acute treatment of the severe depression and/or mania of bipolar disorder. However, ECT should be reserved for patients with severe mood syndromes who may be unable to wait for mood-stabilizing drugs to take effect. Neuroleptic (antipsychotic) drugs are effective in acute mania, but are not recommended for long-term use because of side effects. Bipolar depression generally responds to tricyclic antidepressants, SSRIs, and MAO inhibitors, but when used as long-term therapy these drugs may induce episodes of mania. Anticonvulsants, such as carbamazepine, valproic acid, and benzodiazepines, have been useful adjuncts combined with lithium in patients with breakthrough episodes of mania and/or depression. Lithium is the classic mood stabilizer. It has been shown to have antimanic efficacy, prophylactic efficacy in bipolar disorder, and some efficacy in prophylaxis against bipolar depression. Lithium remains the drug of choice for long-term treatment of the majority of patients with bipolar illness. Ref: Belmaker RH: Bipolar disorder. N Engl J Med 2004;351(5):476-486.

Which one of the following is true about end-of-life care? (check one) A. Physicians underestimate life expectancies B. Most physicians are comfortable with their level of education in palliative care and pain control C. Most patients who qualify for hospice care receive services early in the course of their illness D. Most terminal patients want their lives prolonged as much as possible E. Most terminal patients express a desire for a sense of control

E End-of-life issues are a challenge to primary care physicians because of concerns about a lack of education in pain control and palliative care. Trying to determine the prognosis of patients is difficult, and even with established criteria, the estimated prognosis is right only 50% of the time. There is a tendency for most physicians to overestimate life expectancy in a terminal patient. Most patients who are appropriate candidates for hospice care do not receive referrals until late in their illness, if at all. Patients at the end of life have five main areas of concern: control of pain and other symptoms; avoiding a prolongation of the dying process; having a sense of control; relieving burdens on family and loved ones; and strengthening relationships with family and friends. Ref: Standridge JB, Zylstra RG, Miller KE, et al: Caring for Elderly Individuals. AAFP Home Study Self-Assessment monograph series, 2004, no 294, pp 53-55.

Which one of the following is true concerning falls in the elderly? (check one) A. Treating depression with SSRIs reduces the risk of falling B. Patients tend to fall less often immediately after coming home from the hospital C. Ambulatory blood pressure monitoring should be ordered for all patients who fall D. Arthritis and vision impairment are not associated with an increased risk of falling E. Reducing the number of medications a patient takes reduces the risk of falling

E Falling is one of the most common adverse events associated with drugs. The elderly frequently take many medications; reducing these medications also reduces the risk of falling. SSRIs, tricyclic antidepressants, benzodiazepines, and anticonvulsants have the strongest association with falls in the elderly. The highest risk for falling occurs immediately after hospital stays and lasts for about a month. Ambulatory blood pressure monitoring is associated with so many false-negative and false-positive results that it cannot be recommended for all patients who fall. Arthritis and vision problems are both strongly associated with an increased risk of falls. Ref: Tinetti ME: Preventing falls in elderly persons. N Engl J Med 2003;348(1):42-49.Q

A 72-year-old male presents to your clinic in atrial fibrillation with a rate of 132 beats/min. He has hypertension, but no history of congestive heart failure or structural heart disease. He is otherwise healthy and active. The best INITIAL approach to his atrial fibrillation would be: (check one) A. Rhythm control with antiarrythmics and warfarin (Coumadin) only if he cannot be consistently maintained in sinus rhythm B. Rhythm control with antiarrythmics and warfarin regardless of maintenance of sinus rhythm C. Ventricular rate control with digoxin, and warfarin for anticoagulation D. Ventricular rate control with digoxin, and aspirin for anticoagulation E. Ventricular rate control with a calcium channel blocker or beta-blocker, and warfarin for anticoagulation

E Five recent randomized, controlled trials have indicated that in most patients with atrial fibrillation, an initial approach of rate control is best. Patients who were stratified to the rhythm control arm of the trials did NOT have a morbidity or mortality benefit and were more likely to suffer from adverse drug effects and increased hospitalizations. The most efficacious drugs for rate control are calcium channel blockers and beta-blockers. Digoxin is less effective for rate control and should be reserved as an add-on option for those not controlled with a beta-blocker or calcium channel blocker, or for patients with significant left ventricular systolic dysfunction. In patients 65 years of age or older or with one or more risk factors for stroke, the best choice for anticoagulation to prevent thromboembolic disease is warfarin. Of note, in patients who are successfully rhythm controlled and maintained in sinus rhythm, the thromboembolic rate is equivalent to those managed with a rate control strategy. Thus, the data suggest that patients who choose a rhythm control strategy should be maintained on anticoagulation regardless of whether they are consistently in sinus rhythm. Ref: Snow V, Weiss K, LeFevre M, et al: Management of newly detected atrial fibrillation: A clinical practice guideline from the AAFP and the ACP. Ann Intern Med 2003;139(12):1009-1017. 2) McNamara RL, Tamariz L, Segal JB, et al: Management of atrial fibrillation: Review of the evidence for the role of pharmacologic therapy, electrical cardioversion and echo. Ann Intern Med 2003;139(12):1018-1033.

The only antidepressant approved by the Food and Drug Administration for the treatment of depression in children 8-17 years of age is (check one) A. venlafaxine (Effexor) B. amitriptyline C. lithium D. paroxetine (Paxil) E. fluoxetine (Prozac)

E Fluoxetine is the only SSRI approved by the FDA for the treatment of depression in children 8-17 years of age, although all of the antidepressants may be used off-label . The FDA has warned against the use of paroxetine in this age range because of a possible increased risk of suicidal thinking and suicide attempts associated with the drug. Tricyclic antidepressants have more side effects and can be lethal in overdose. In children and adolescents, there is limited or no evidence evaluating the use of lithium, monoamine oxidase inhibitors, St. Johns wort, or venlafaxine. Ref: Bhatia SK, Bhatia SC: Childhood and adolescent depression. Am Fam Physician 2007;75(1):73-80.

You are initiating treatment for a patient being admitted to the hospital with a new diagnosis of pulmonary embolus. Low molecular weight heparin and warfarin (Coumadin) are started immediately. When can the low molecular weight heparin be stopped? (check one) A. When the INR is ≥2.0 B. When the INR is ≥2.0 for 24 hours C. After 4 days, if the INR is ≥³2.0 D. After 4 days, if the INR has been ≥2.0 for 24 hours E. After 5 days, if the INR has been ≥2.0 for 24 hours

E For patients with a pulmonary embolus, American College of Chest Physicians guidelines recommend initial treatment with low molecular weight heparin (LMWH), unfractionated heparin, or fondaparinux for at least 5 days, and then can be stopped if the INR has been ≥2.0 for at least 24 hours (SOR C). Warfarin reduces the activity of coagulation factors II, VII, IX, and X produced in the liver. Coagulation factors produced prior to initiating warfarin remain active for their usual several-day lifespan, which is why LMWH and warfarin must be given concomitantly for at least 5 days. The INR may reach levels >2.0 before coagulation factors II and X have reached their new plateau levels, accounting for the need for an additional 24 hours of combined therapy before stopping LMWH. Ref: Kearon C, Kahn S, Agnelli G, et al: Antithrombotic therapy for venous thromboembolic disease: American College of Chest Physicians evidence-based clinical practice guidelines (8th edition). Chest 2008;133(6):454S-545S.

A 28-year-old white male comes to your office complaining of pain in the right wrist since falling 2 weeks ago. On examination, he is tender in the anatomic snuffbox. A radiograph reveals a nondisplaced fracture of the distal one-third of the carpal navicular bone (scaphoid). Which one of the following is the most appropriate management at this time? (check one) A. A bone scan B. Physical therapy referral C. A Velcro wrist splint D. A short arm cast E. A thumb spica cast

E Fracture of the scaphoid should be suspected in every "sprained wrist" presenting with tenderness in the anatomic snuffbox. Radiographs may be negative initially. The scaphoid circulation enters the bone for the most part through the distal half. Fractures through the proximal third tend to cause loss of circulation and are slower to heal, and should be referred to an orthopedist because of the risk of nonunion and avascular necrosis. Fractures through the middle or distal one-third can be handled by the family physician in consultation with an orthopedist. The fracture is treated with a thumb spica cast for 10-12 weeks. A wrist splint does not provide adequate immobilization. A bone scan is unnecessary, and physical therapy is inappropriate. If there is still no evidence of union after 10 weeks of immobilization, the patient should be referred to an orthopedist for further care.

A 45-year-old female presents with a 3-month history of hoarseness that is not improving. She works as a high-school teacher. The most appropriate management at this time would be: (check one) A. voice therapy B. azithromycin (Zithromax) C. a trial of inhaled corticosteroids D. a trial of a proton pump inhibitor E. laryngoscopy

E Hoarseness most commonly affects teachers and older adults. The cause is usually benign, but extended symptoms or certain risk factors should prompt evaluation; specifically, laryngoscopy is recommended when hoarseness does not resolve within 3 months or when a serious underlying cause is suspected (SOR C). The American Academy of Otolaryngology/Head and Neck Surgery Foundation guidelines state that antireflux medications should not be prescribed for patients with hoarseness without reflux symptoms (SOR C). Antibiotics should not be used, as the condition is usually caused by acute laryngitis or an upper respiratory infection, and these are most likely to be viral. Inhaled corticosteroids are a common cause of hoarseness. Voice therapy should be reserved for patients who have undergone laryngoscopy first (SOR A). Ref: Schwartz SR, Cohen SM, Dailey SH, Rosenfeld RM, et al: Clinical practice guideline: Hoarseness (dysphonia). Otolaryngol Head Neck Surg 2009;141(3 suppl 2):S1-S31. 2) Huntzinger A: Guidelines for the diagnosis and management of hoarseness. Am Fam Physician 2010;81(10):1292-1296.

A 13-year-old male is found to have hypertrophic cardiomyopathy. His father also had hypertrophic cardiomyopathy, and died suddenly at age 38 following a game of tennis. The boy's mother asks you for advice regarding his condition. What advice should you give her? (check one) A. He may participate in noncontact sports B. He should receive lifelong treatment with beta-blockers C. His condition usually decreases lifespan D. His hypertrophy will regress with age E. His siblings should undergo echocardiography

E Hypertrophic cardiomyopathy is an autosomal dominant condition and close relatives of affected individuals should be screened. The hypertrophy usually stays the same or worsens with age. This patient should not participate in strenuous sports, even those considered noncontact. Beta-blockers have not been shown to alter the progress of the disease. The mortality rate is believed to be about 1%, with some series estimating 5%. Thus, in most cases lifespan is normal. Ref: Maron BJ: Hypertrophic cardiomyopathy. JAMA 2002;287(10):1308-1320.

A 36-year-old white female presents to the emergency department with palpitations. Her pulse rate is 180 beats/min. An EKG reveals a regular tachycardia with a narrow complex QRS and no apparent P waves. The patient fails to respond to carotid massage or to two doses of intravenous adenosine (Adenocard), 6 mg and 12 mg. The most appropriate next step would be to administer intravenous (check one) A. amiodarone (Cordarone) B. digoxin (Lanoxin) C. flecainide (Tambocor) D. propafenone (Rhythmol) E. verapamil (Calan)

E If supraventricular tachycardia is refractory to adenosine or rapidly recurs, the tachycardia can usually be terminated by the administration of intravenous verapamil or a β-blocker. If that fails, intravenous propafenone or flecainide may be necessary. It is also important to look for and treat possible contributing causes such as hypovolemia, hypoxia, or electrolyte disturbances. Electrical cardioversion may be necessary if these measures fail to terminate the tachyarrhythmia. Ref: Delacretaz E: Supraventricular tachycardia. N Engl J Med 2006;354(10):1039-1051.

A previously healthy gravida 1 para 1 who is 3 weeks post partum complains of bilateral nipple pain with breastfeeding. When she first started breastfeeding she had some soreness that went away after repositioning with feeding. The current pain began gradually 3 days ago. It has been worsening, inhibiting feeding, and is present between feedings. Examination of the breast is notable for erythema and cracking of the areola. The most likely cause is: (check one) A. engorgement B. mastitis C. improper latch-on D. eczema flare E. Candida infection

E In breastfeeding women, bilateral nipple pain with and between feedings after initial soreness has resolved is usually due to Candida. Pain from engorgement typically resolves after feeding. Mastitis is usually unilateral and is associated with systemic symptoms and wedge-shaped erythema of the breast tissue. Improper latch-on is painful only during feedings. Eczema isolated to the nipple, while a reasonable part of the differential, would be much more unusual.

Which one of the following is effective for single-dose prophylaxis against Lyme disease after an Ixodes scapularis tick bite? (check one) A. Azithromycin (Zithromax) B. Amoxicillin C. Cefuroxime (Ceftin) D. Trimethoprim/sulfamethoxazole (Bactrim, Septra) E. Doxycycline

E In controlled studies, it has been shown that a single 200-mg dose of doxycycline given within 72 hours after an Ixodes scapularis tick bite can prevent the development of Lyme disease.

The treatment of choice for a 4-month-old infant with suspected pertussis is: (check one) A. Supportive care (respiratory, fluids) only B. Ceftriaxone (Rocephin) C. Ampicillin D. Gentamicin (Garamycin) E. Erythromycin

E In spite of widespread vaccination of infants, pertussis occurs endemically in 3- to 5-year cycles in the U.S. It appears to be more common within populations not routinely immunized, such as Mennonite communities, but can occur widely. Infants younger than 6 months are affected most severely, although pertussis occurs in all age groups. The diagnosis is made by nasopharyngeal culture, but because the disease is uncommon and the organism is fastidious, laboratory personnel should be advised of the physician's suspicion of pertussis. Treatment includes respiratory and nutritional supportive care, particularly for infants younger than 6 months. Antibiotic therapy is most effective in shortening the illness when given early, during the upper respiratory phase, but is indicated at any stage to reduce the spread of disease to others. The drug of choice is erythromycin, 40-50 mg/kg/day divided into four doses, for 14 days. Also effective are azithromycin and clarithromycin, which may be better tolerated with improved compliance. Resistance to these agents is rare. Penicillins and cephalosporins are ineffective. Gentamicin is potentially very toxic, and is not indicated. Ref: Pickering LK (ed): 2003 Red Book: Report of the Committee on Infectious Diseases, ed 26. American Academy of Pediatrics, 2003, pp 472-475.

An 18-year-old male presents with a sore throat, adenopathy, and fatigue. He has no evidence of airway compromise. A heterophil antibody test is positive for infectious mononucleosis. Appropriate management includes which one of the following? (check one) A. A corticosteroid B. An antihistamine C. An antiviral agent D. Strict bed rest E. Avoidance of contact sports

E Infectious mononucleosis presents most commonly with a sore throat, fatigue, myalgias, and lymphadenopathy, and is most prevalent between 10 and 30 years of age. Both an atypical lymphocytosis and a positive heterophil antibody test support the diagnosis, although false-negative heterophil testing is common early in the disease course. The cornerstone of treatment for mononucleosis is supportive, including hydration, NSAIDs, and throat sprays or lozenges. In general, corticosteroids do not have a significant effect on the clinical course of infectious mononucleosis, and they should not be used routinely unless the patient has evidence of acute airway obstruction. Antihistamines are also not recommended as routine treatment for mononucleosis. The use of acyclovir has shown no consistent or significant benefit, and antiviral drugs are not recommended. There is also no evidence to support bed rest as an effective management strategy for mononucleosis. Given the evidence from other disease states, bed rest may actually be harmful. Although most patients will not have a palpably enlarged spleen on examination, it is likely that all, or nearly all, patients with mononucleosis have splenomegaly. This was demonstrated in a small study in which 100% of patients hospitalized for mononucleosis had an enlarged spleen by ultrasound examination, whereas only 17% of patients with splenomegaly have a palpable spleen. Patients should be advised to avoid contact- or collision-type activities for 3-4 weeks because of the increased risk of rupture. Ref: Ebell MH: Epstein-Barr virus infectious mononucleosis. Am Fam Physician 2004;70(7):1279-1287. 2) Dickens KP, Nye AM, Gilchrist V, et al: Clinical Inquiries. Should you use steroids to treat infectious mononucleosis? J Fam Pract 2008;57(11):754-755.

A 24-year-old white female presents to the office with a 6-month history of abdominal pain. A physical examination, including pelvic and rectal examinations, is normal. Which one of the following would indicate a need for further evaluation? (check one) A. Relief of symptoms with defecation B. Changes in stool consistency from loose and watery to constipation C. Passage of mucus with bowel movements D. Abdominal bloating E. Worsening of symptoms at night

E Irritable bowel syndrome (IBS) is a benign, chronic symptom complex of altered bowel habits and abdominal pain. It is the most common functional disorder of the gastrointestinal tract. The presence of nocturnal symptoms is a red flag which should alert the physician to an alternate diagnosis and may require further evaluation. The other symptoms listed are Rome I and II criteria for diagnosing irritable bowel syndrome. Ref: Viera AJ, Hoag S, Shaughnessy J: Management of irritable bowel syndrome. Am Fam Physician 2002;66(10):1867-1874, 1880.

Of the following cardiovascular parameters, which one increases with normal aging? (check one) A. Maximum heart rate B. Heart rate variability C. Ejection fraction D. Arterial wall elasticity E. Blood pressure

E It can be difficult to determine the point at which changes of normal aging are more appropriately considered disease processes. Although the direction of expected change is generally well understood, variables such as the level of fitness and overall health of an individual affect the degree of change. As the body ages, the measured left ventricular ejection fraction, heart rate variability, and maximum heart rate trend downward, the walls of the major aorta and major arteries stiffen, and the vasodilator capacity of most smaller vessels is reduced (SOR A). The arterial wall changes increase peripheral resistance and result in an increase in blood pressure. Positive adaptive changes have been shown in older adults who engage in regular aerobic exercise, however, and these changes can be measured after only 3 months of moderate-intensity exercise (SOR A). Ref: American College of Sports Medicine, Chodzko-Zajko WJ, Proctor DN, et al: American College of Sports Medicine position stand: Exercise and physical activity for older adults. Med Sci Sports Exerc 2009;41(7):1510-1530.

A 65-year-old white male comes to your office with a 0.5-cm nodule that has developed on his right forearm over the past 4 weeks. The lesion is dome shaped and has a central plug. You schedule a biopsy but he does not return to your office for 1 year. At that time the lesion appears to have healed spontaneously. The most likely diagnosis is (check one) A. benign lentigo B. lentigo maligna C. basal cell carcinoma D. squamous cell carcinoma E. keratoacanthoma

E Keratoacanthoma grows rapidly and may heal within 6 months to a year. Squamous cell carcinoma may appear grossly and histologically similar to keratoacanthoma but does not heal spontaneously. The other lesions do not resemble keratoacanthoma. Ref: Habif TP, Campbell JL Jr, Chapman MS, et al: Skin Disease: Diagnosis and Treatment, ed 2. Elsevier, 2005, pp 404-407.

Late decelerations on fetal monitoring are thought to indicate which one of the following? (check one) A. Fetal head compression B. Umbilical cord compression C. Fetal sleep D. Uterine hypotonus E. Uteroplacental insufficiency

E Late decelerations are thought to be associated with uteroplacental insufficiency and fetal hypoxia due to decreased blood flow in the placenta. This pattern is a warning sign and is associated with increasing fetal compromise, worsening fetal acidosis, fetal central nervous system depression, and/or direct myocardial hypoxia. Early decelerations are thought to result from vagus nerve response to fetal head compression, and are not associated with increased fetal mortality or morbidity. Variable decelerations are thought to be due to acute, intermittent compression of the umbilical cord between fetal parts and the contracting uterus. Ref: Cunningham FG, Leveno KJ, Bloom SL, et al: Williams Obstetrics, ed 22. McGraw-Hill, 2005, pp 452-453.

A 45-year-old white male consults you because of a painless, circular, 1-cm white spot inside his mouth, which he noticed 3 days ago. You are treating him with propranolol (Inderal) for hypertension, and you know him to be a heavy alcohol user. After a careful physical examination, your tentative diagnosis is leukoplakia of the buccal mucosa. You elect to observe the lesion for 2 weeks. On the patients return, the lesion is still present and unchanged in appearance. The best course of management at this time is to (check one) A. reassure the patient and continue to observe B. discontinue propranolol C. treat with oral nystatin D. order a fluorescent antinuclear antibody test E. perform a biopsy of the lesion

E Leukoplakia is a white keratotic lesion seen on mucous membranes. Irritation from various mechanical and chemical stimuli, including alcohol, favors development of the lesion. Leukoplakia can occur in any area of the mouth and usually exhibits benign hyperkeratosis on biopsy. On long-term follow-up, 2%-6% of these lesions will have undergone malignant transformation into squamous cell carcinoma. Oral nystatin would not be appropriate treatment, as this lesion is not typical of oral candidiasis. Candidal lesions are usually multiple and spread quickly when left untreated. A fluorescent antinuclear antibody test is also not indicated, as the oral lesions of lupus erythematosus are typically irregular, erosive, and necrotic. An idiosyncratic reaction to propranolol is unlikely in this patient. Ref: Gonsalves WC, Chi AC, Neville BW: Common oral lesions: Part II. Masses and neoplasia. Am Fam Physician 2007;75(4):509-512. 2) Goldman L, Ausiello D (eds): Cecil Medicine, ed 23. Saunders, 2008, pp 1451-1452.

Question 7 of 10 Which one of the following is associated with ulcerative colitis rather than Crohn's disease? (check one) A. The absence of rectal involvement B. Transmural involvement of the colon C. Segmental noncontinuous distribution of inflammation D. Fistula formation E. An increased risk of carcinoma of the colon

E Long-standing ulcerative colitis (UC) is associated with an increased risk of colon cancer. The greater the duration and anatomic extent of involvement, the greater the risk. Initial colonoscopy for patients with pancolitis of 8-10 years duration (regardless of the patient's age) should be followed up with surveillance examinations every 1-2 years, even if the disease is in remission. All of the other options listed are features typically associated with Crohn's disease. Virtually all patients with UC have rectal involvement, even if that is the only area affected. In Crohn's disease, rectal involvement is variable. Noncontinuous and transmural inflammation are also more common with Crohn's disease. Transmural inflammation can lead to eventual fistula formation, which is not seen in UC. Ref: Langan RC, Gotsch PB, Krafczyk MA, et al: Ulcerative colitis: Diagnosis and treatment. Am Fam Physician 2007;76(9):1323-1330, 1331.

A 60-year-old African-American female has a history of hypertension that has been well controlled with hydrochlorothiazide. However, she has developed an allergy to the medication. Successful monotherapy for her hypertension would be most likely with which one of the following? (check one) A. Lisinopril (Prinivil, Zestril) B. Hydralazine (Apresoline) C. Clonidine (Catapres) D. Atenolol (Tenormin) E. Diltiazem (Cardizem)

E Monotherapy for hypertension in African-American patients is more likely to consist of diuretics or calcium channel blockers than β-blockers or ACE inhibitors. It has been suggested that hypertension in African-Americans is not as angiotensin II-dependent as it appears to be in Caucasians. Ref: The Seventh Report of the Joint National Committee on Prevention, Detection, Evaluation, and Treatment of High Blood Pressure. National High Blood Pressure Education Program, 2004, NIH Publication No. 04-5230, p 39. 2) Rao S, Cherukuri M, Mayo HG: Clinical inquiries. What is the best treatment for hypertension in African-Americans? J Fam Pract 2007;56(2):149-151.

A 47-year-old gravida 3 para 3 is seen for a physical examination. She has had a total abdominal hysterectomy for benign uterine fibroids. Which one of the following is the recommended interval for Papanicolaou (Pap) screening in this patient? (check one) A. Every 5 years B. Every 3 years C. Every 2 years D. Annually E. Routine screening is not necessary

E Most American women who have undergone hysterectomy are not at risk of cervical cancer, as they underwent the procedure for benign disease and no longer have a cervix. U.S. Preventive Services Task Force recommendations issued in 1996 stated that routine Papanicolaou (Pap) screening is unnecessary for these women. Nevertheless, data from the Behavioral Risk Factor Surveillance System (1992-2002) indicated that in the previous 3 years, some 69% of women with a previous history of hysterectomy for benign causes had undergone screening. Ref: U.S. Preventive Services Task Force: Recommendations and rationale: Screening for cervical cancer. U.S. Preventive Services Task Force, U.S. Department of Health and Human Services, 2002. 2) Sirovich BE, Welch HG: Cervical cancer screening among women without a cervix. JAMA 2004;291(24):2990.

A 63-year-old white male has been diagnosed with myasthenia gravis and is experiencing progressive muscle weakness despite maximum pharmacotherapy. Which one of the following surgical options would be most likely to improve his condition? (check one) A. Thyroidectomy B. Radioactive thyroid ablation C. Adrenalectomy D. Removal of a pituitary microadenoma E. Thymectomy

E Myasthenia gravis is a neuromuscular illness with an underlying immune-related cause. Corticosteroids and anticholinesterase medications such as oral pyridostigmine can be helpful, but thymectomy may be appropriate for patients with generalized disease not responding to medication. Thymectomy increases the remission rate and improves the clinical course. Ref: Goldman L, Schafer AI (eds): Goldman's Cecil Medicine, ed 24. Elsevier Saunders, 2011, pp 2418-2422.

The most common cause of fainting is: (check one) A. Cardiac dysrhythmia B. Medications C. Orthostatic hypotension D. Psychiatric disorders E. Vasovagal syncope

E Neurally mediated syncope (also termed neurocardiogenic or vasovagal syncope) comprises the largest group of disorders causing syncope. These disorders result from reflex-mediated changes in vascular tone or heart rate. Ref: Kapoor WN: Syncope. N Engl J Med 2000;343(25):1856-1862. 2) Kasper DL, Braunwald E, Fauci AS, et al (eds): Harrison's Principles of Internal Medicine, ed 16. McGraw-Hill, 2005, pp 126-128.

Which one of the following procedures carries the highest risk for postoperative deep venous thrombosis? (check one) A. Abdominal hysterectomy B. Coronary artery bypass graft C. Transurethral prostatectomy D. Lumbar laminectomy E. Total knee replacement

E Neurosurgical procedures, particularly those with penetration of the brain or meninges, and orthopedic surgeries, especially those of the hip, have been linked with the highest incidence of venous thromboembolic events. The risk is due to immobilization, venous injury and stasis, and impairment of natural anticoagulants. For total knee replacement, hip fracture surgery, and total hip replacement, the prevalence of DVT is 40%-80%, and the prevalence of pulmonary embolism is 2%-30%. Other orthopedic procedures, such as elective spine procedures, have a much lower rate, approximately 5%. The prevalence of DVT after a coronary artery bypass graft is approximately 5%, after transurethral prostatectomy <5%, and after abdominal hysterectomy approximately 16%. Ref: Geerts WH, Heit JA, Clagett GP, et al: Prevention of venous thromboembolism. Chest 2001;119(1 Suppl):132S-175S. 2) Kaboli P, Henderson MC, White RH: DVT prophylaxis and anticoagulation in the surgical patient. Med Clin North Am 2003;87(1):77-110.

Which one of the following is true concerning the treatment of tobacco use? (check one) A. Tobacco withdrawal symptoms abate in 3 days B. Physicians' advice to patients to stop smoking is ineffectual C. Of all the products available for smoking cessation, only bupropion (Wellbutrin) is consistently effective D. Nicotine replacement therapy is dangerous for patients with stable angina E. Nicotine causes physical dependence

E Nicotine causes both physical dependence and tolerance. Withdrawal from nicotine can last several weeks or months. Physicians' advice to stop smoking increases the rate of stopping smoking by about 30%. Bupropion is no more or less effective than other products for smoking cessation. Nicotine replacement therapy is safe in patients with stable angina. Ref: Rigotti NA: Treatment of tobacco use and dependence. N Engl J Med 2002;346(7):506-512.

A 55-year-old male sees you for a follow-up visit for hypercholesterolemia and hypertension. He is in good health, does not smoke, and drinks alcohol infrequently. His medications include a multiple vitamin daily; aspirin, 81 mg daily; lisinopril (Prinivil, Zestril), 10 mg daily; and lovastatin (Mevacor), 20 mg daily. His vital signs are within normal limits except for a BMI of 33.4 kg/m2 . At today's visit his ALT (SGPT) level is 55 IU/L (N 10-45) and his AST (SGOT) level is 44 IU/L (N 10-37). The remainder of the liver panel is normal. Which one of the following is the most likely cause of the elevation in liver enzymes? (check one) A. A side effect of lovastatin B. Gallbladder disease C. Hepatitis A D. Alcoholic liver disease E. Metabolic syndrome

E Non-alcoholic fatty liver disease (NAFLD) is the most common cause of abnormal liver tests in the developed world. Its prevalence increases with age, body mass index, and triglyceride concentrations, and in patients with diabetes mellitus, hypertension, or insulin resistance. There is a significant overlap between metabolic syndrome and diabetes mellitus, and NAFLD is regarded as the liver manifestation of insulin resistance. Statin therapy is considered safe in such individuals and can improve liver enzyme levels and reduce cardiovascular morbidity in patients with mild to moderately abnormal liver tests that are potentially attributable to NAFLD.

An 82-year-old resident of a local nursing home is brought to your clinic with fever, difficulty breathing, and a cough productive of purulent sputum. The patient is found to have an oxygen saturation of 86% on room air and a chest radiograph shows a new infiltrate. A decision is made to hospitalize the patient. Which one of the following intravenous antibiotic regimens would be most appropriate for this patient? (check one) A. Levofloxacin (Levaquin) B. Ceftriaxone (Rocephin) and azithromycin (Zithromax) C. Ceftazidime (Fortaz, Tazicef) and levofloxacin D. Ceftazidime and vancomycin E. Ceftazidime, levofloxacin, and vancomycin

E Nursing home-acquired pneumonia should be suspected in patients with a new infiltrate on a chest radiograph if it is associated with a fever, leukocytosis, purulent sputum, or hypoxia. Nursing-home patients who are hospitalized for pneumonia should be started on intravenous antimicrobial therapy, with empiric coverage for methicillin-resistant Staphylococcus aureus (MRSA) and Pseudomonas aeruginosa. The 2005 American Thoracic Society/Infectious Diseases Society of America guideline recommends combination therapy consisting of an antipseudomonal cephalosporin such as cefepime or ceftazidime, an antipseudomonal carbapenem such as imipenem or meropenem, or an extended-spectrum β-lactam/β-lactamase inhibitor such as piperacillin/tazobactam, PLUS an antipseudomonal fluoroquinolone such as levofloxacin or ciprofloxacin, or an aminoglycoside such as gentamicin, tobramycin, or amikacin, PLUS an anti-MRSA agent (vancomycin or linezolid). Ceftriaxone and azithromycin or levofloxacin alone would be reasonable treatment options for a patient with nursing home-acquired pneumonia who does not require hospitalization. Ref: American Thoracic Society; Infectious Diseases Society of America: Guidelines for the management of adults with hospital-acquired, ventilator-associated, and healthcare-associated pneumonia. Am J Respir Crit Care Med 2005;171(4):388-416. 2) Mills K, Nelson AC, Winslow BT, Springer KL: Treatment of nursing home-acquired pneumonia. Am Fam Physician 2009;79(11):976-992.

A 39-year-old male with a BMI of 41 kg/m2 is interested in weight loss. His medical history includes adequately controlled type 2 diabetes mellitus, well-controlled hypertension, hyperlipidemia, and obstructive sleep apnea. He has no history of coronary artery disease or COPD. Which one of the following is likely to be most effective for long-term weight loss in this patient? (check one) A. A very low calorie diet B. Increased physical activity C. Frequent, long-term weight-loss counseling D. Pharmacotherapy E. Bariatric surgery

E Obesity increases the risk of a variety of medical conditions, including type 2 diabetes mellitus, hypertension, hyperlipidemia, pulmonary disease, coronary artery disease, gallstones, fatty liver disease, obstructive sleep apnea, GERD, osteoarthritis, and a variety of forms of cancer. A weight loss of at least 10% for greater than 1 year leads to statistically significant improvement in lipid ratios, blood glucose homeostasis, and coronary artery disease risk reduction. The AAFP recommends screening for obesity and intensive counseling (more than 1 session per month for more than 3 months) with behavior modification for obese patients. Counseling is ineffective by itself and must be combined with lifestyle modification. Dietary modification, increased physical activity, and behavior modification are effective for maintaining modest weight loss for greater than 1 year (SOR B). However, there are few large, randomized, controlled trials with subjects maintaining weight reductions of 10% for over 1 year, even when combining therapy, exercise, and dietary restriction. Long-term pharmacotherapy can lead to weight loss, but regaining some weight is typical. Bariatric surgery leads to the most effective weight reduction and long-term maintenance in patients who are morbidly obese (SOR A). Gastric bypass is effective, with a mean weight loss of 71.2% at 3 years; with laparoscopic gastric banding the mean weight loss is 55.2% at 3 years. In one study, 94% of gastric bypass patients maintained at least a 20% weight loss at 6 years. Bariatric surgery has also been shown to significantly reduce fasting blood glucose, with resolution of diabetes mellitus in 31%-77% of lap band patients and 72%-100% of gastric bypass patients. Bariatric surgery is a safe and effective means for long-term weight loss and should be considered in adults with a BMI >40, or >35 with obesity-related comorbidities.

An 18-year-old single white female at 30 weeks' gestation presents to the hospital with uterine contractions 10 minutes apart. Her previous pregnancy 18 months ago resulted in a preterm birth at 29 weeks' gestation. The most accurate test to determine whether this patient will need hospitalization and tocolysis would be: (check one) A. Serum corticotropin-releasing hormone B. Maternal serum alpha-fetoprotein C. Serum human chorionic gonadotropin (hCG) D. Salivary estriol concentration E. Vaginal fetal fibronectin

E Of the biochemical markers listed, the most clinically useful test to differentiate women who are at high risk for impending preterm delivery from those who are not is the fetal fibronectin in cervical or vaginal secretions. In symptomatic women, this is most accurate in predicting spontaneous preterm delivery within 7-10 days. It is less accurate in those who are asymptomatic. If the fetal fibronectin is negative, it may be possible to avoid interventions such as hospitalization, tocolysis, and corticosteroid administration. Ref: Gabbe SG, Niebyl JR, Simpson JL (eds): Obstetrics: Normal and Problem Pregnancies, ed 4. Churchill Livingstone, 2002, pp 769-770. 2) Lockwood CJ: Prediction of preterm labor and delivery. UpToDate. April 2004.

A 75-year-old white female presents with severe pain of the carpometacarpal joint at the base of her thumb. Examination of her hands also reveals hypertrophic changes of the distal interphalangeal and proximal interphalangeal joints of her fingers. These findings are most consistent with (check one) A. rheumatoid arthritis B. gout C. systemic lupus erythematosus D. scleroderma E. osteoarthritis

E Osteoarthritis causes changes predominantly in the proximal interphalangeal (PIP) and distal interphalangeal (DIP) joints of the hands known as Bouchard's and Heberden's nodes respectively, and the carpometacarpal joints of the thumbs. While rheumatoid arthritis commonly causes subluxations in the metacarpophalangeal joints, this patient's hypertrophic changes are most likely due to osteoarthritis. The other choices are less likely to cause this presentation. Ref: Klippel JH, Stone JH, Crofford LJ, et al (eds): Primer on the Rheumatic Diseases, ed 13. Springer, 2008, pp 224-228.

Which one of the following is a risk factor for acute pancreatitis? (check one) A. Gastroesophageal reflux disease B. Intravenous drug abuse C. Angiotensin receptor blocker use D. Pyelonephritis E. Gallstones

E Pancreatitis is most closely associated with gallstones, extreme hypertrigliceridemia, and excessive alcohol use. Gastroesophageal reflux disease, pyelonephritis, drug abuse (other than alcohol), and angiotensin receptor blocker use are not risk factors for the development of pancreatitis. Ref: Whitcomb DC: Acute pancreatitis. N Engl J Med 2006;354(20):2142-2150.

A 68-year-old female has an average blood pressure of 150/70 mm Hg despite appropriate lifestyle modification efforts. Her only other medical problems are osteoporosis and mild depression. The most appropriate treatment at this time would be (check one) A. lisinopril (Prinivil, Zestril) B. clonidine (Catapres) C. propranolol (Inderal) D. amlodipine (Norvasc) E. hydrochlorothiazide

E Randomized, placebo-controlled trials have shown that isolated systolic hypertension in the elderly responds best to diuretics and to a lesser extent, β-blockers. Diuretics are preferred, although long-acting dihydropyridine calcium channel blockers may also be used. In the case described, β-blockers or clonidine may worsen the depression. Thiazide diuretics may also improve osteoporosis, and would be the most cost-effective and useful agent in this instance. Ref: The Seventh Report of the Joint National Committee on Prevention, Detection, Evaluation, and Treatment of High Blood Pressure. National High Blood Pressure Education Program, 2004, NIH Publication No. 04-5230, pp 29-30, 44-46.

A 16-year-old white female is brought to your office because she has been "passing out." She tells you that on several occasions while playing in the high-school band at the end of the half-time show she has "blacked out." She describes feeling lightheaded with spots before her eyes and tunnel vision just prior to falling. Friends in the band have told her that she appears to be pale and sweaty when these episodes occur. No seizure activity has ever been observed. In each instance she regains consciousness almost immediately; there is no postictal state. She has been seen in the emergency department for this on two occasions with normal vital signs, physical findings, and neurologic findings. A CBC, a metabolic profile, and an EKG are also normal. Which one of the following tests is most likely to yield the correct diagnosis? (check one) A. A sleep-deprived EEG B. 24-hour Holter monitoring C. A pulmonary/cardiac stress test D. An echocardiogram E. Tilt table testing

E Reflex syncope is a strong diagnostic consideration for episodes of syncope associated with a characteristic precipitating factor. The major categories of syncope include carotid sinus hypersensitivity, and neurally mediated and situational syncopes. The most common and benign forms of syncope are neurally mediated or vasovagal types with sudden hypotension, frequently accompanied by bradycardia. Other terms for this include neurocardiogenic, vasomotor, neurovascular, or vasodepressive syncope. Most patients are young and otherwise healthy. The mechanism of the syncope seems to be a period of high sympathetic tone (often induced by pain or fear), followed by sudden sympathetic withdrawal, which then triggers a paradoxical vasodilatation and hypotension. Attacks occur with upright posture, often accompanied by a feeling of warmth or cold sweating, lightheadedness, yawning, or dimming of vision. If the patient does not lie down quickly he or she will fall, with the horizontal position allowing a rapid restoration of central profusion. Recovery is rapid, with no focal neurologic sense of confusion or headache. The event can be duplicated with tilt testing, demonstrating hypotension and bradycardia. Ref: Weimer LH, Williams O: Syncope and orthostatic intolerance. Med Clin North Am 2003;87(4):835-865.

Which one of the following treatments for diabetes mellitus reduces insulin resistance? (check one) A. Acarbose (Precose) B. Sitagliptin (Januvia) C. Repaglinide (Prandin) D. Exenatide (Byetta) E. Pioglitazone (Actos)

E Repaglinide and nateglinide are nonsulfonylureas that act on a portion of the sulfonylurea receptor to stimulate insulin secretion. Pioglitazone is a thiazolidinedione, which reduces insulin resistance. It is believed that the mechanism for this is activation of PPAR-Y, a receptor that affects several insulin-responsive genes. Acarbose is a competitive inhibitor of α-glucosidases, enzymes that break down complex carbohydrates into monosaccharides. This delays the absorption of carbohydrates such as starch, sucrose, and maltose, but does not affect the absorption of glucose. Sitagliptin is a DPP-IV inhibitor, and this class of drugs inhibits the enzyme responsible for the breakdown of the incretins GLP-1 and GIP. Exenatide is an incretin mimetic that stimulates insulin secretion in a glucose-dependent fashion, slows gastric emptying, and may promote satiety.

Which one of the following treatments is most appropriate for a patient with uncomplicated acute bronchitis? (check one) A. Amoxicillin B. Amoxicillin/clavulanate (Augmentin) C. Azithromycin (Zithromax) D. Doxycycline E. Supportive care only

E Respiratory viruses appear to be the most common cause of acute bronchitis; however, the organism responsible is rarely identified in clinical practice because viral cultures and serologic assays are not routinely performed. Fewer than 10% of patients will have a bacterial infection diagnosed as the cause of bronchitis. For this reason, for patients with a putative diagnosis of acute bronchitis, routine treatment with antibiotics is not justified and should not be offered. Antitussive agents are occasionally useful and can be offered as therapy for short-term symptomatic relief of coughing. Ref: Braman SS: Chronic cough due to acute bronchitis: ACCP evidence-based clinical practice guidelines. Chest 2006;129(1 Suppl):95S-103S.

A case of meningococcal meningitis has just been confirmed at a day-care center. The susceptibility of the microorganism is not yet known. At this point, you should do which one of the following for the day-care center contacts? (check one) A. Culture their nasopharyngeal secretions B. Administer meningococcal vaccine C. Prescribe sulfadiazine D. Prescribe chloramphenicol (Chloromycetin) E. Prescribe rifampin (Rifadin)

E Rifampin, in the absence of major contraindications, is the drug of choice for preventing the spread of meningococcal disease when the susceptibility of the organism is not known. In this situation, meningococcal vaccines are of no value because their protective effects take a few days to develop, and because they do not protect against group B meningococci, the most prevalent strain for meningococcal disease. Sulfadiazine is the drug of choice if the meningococcus is known to be susceptible to it. Chloramphenicol and penicillin, which are effective in treating the disease, are ineffective in eliminating nasopharyngeal carriers of meningococci, possibly because they do not appear in high concentrations in saliva. Culturing contacts for meningococcal carriage in the nasopharynx has no value for identifying those at risk for meningococcal disease. Ref: Pickering LK (ed): 2003 Red Book: Report of the Committee on Infectious Diseases, ed 26. American Academy of Pediatrics, 2003, pp 123-137, 430-436.

A 27-year-old white female has a 10-year history of significant premenstrual dysphoria. Her condition has significantly worsened in the past 3 years, to the point that it is endangering her marriage of 5 years. Her symptoms are worse for the 10 days prior to her menstrual period and are gone by day 2 of her period. She has tried several measures without success, including birth control pills, various herbal preparations, and counseling at a woman's health center. Which one of the following is most likely to improve her symptoms? (check one) A. Reduction of caffeine and refined sugar intake B. Alprazolam (Xanax) C. Bupropion (Wellbutrin) D. Progesterone for 2 weeks starting at about the time of ovulation E. Fluoxetine (Prozac, Serafem) for the last 2 weeks of the menstrual cycle

E Several randomized trials have shown that they are superior to placebo for this condition. Fluoxetine and sertraline have been studied the most. There have been no controlled trials to support anecdotal reports of benefit from the reduction of caffeine or refined sugar. Studies using alprazolam have shown it to be effective for premenstrual anxiety only. Progesterone has not been proven more effective than placebo in clinical trials, and bupropion is less effective than agents that primarily boost serotonergic activity. Treatment during the luteal phase alone has been shown to be more effective than continuous treatment for this condition. Ref: Grady-Weliky T: Premenstrual dysphoric disorder. N Engl J Med 2003;348(5):433-438.

A 25-year-old white female comes to your office complaining of abdominal pain. She requests that you hospitalize her and do whatever is necessary to get rid of the pain that has been present for a number of years. She has difficulty describing the pain. She is a single parent, and becomes defensive when asked about her previous marriage, stating only that her former husband is an alcoholic, "just like my father." Her previous medical history includes an appendectomy, a cholecystectomy, and a hysterectomy. On physical examination she appears healthy and a CBC, erythrocyte sedimentation rate, serum amylase level, serum electrolyte levels, and multiple chemical profile are all normal. Management of this patient should include which one of the following? (check one) A. Long-term use of antidepressants B. Referral to a surgeon for exploratory laparotomy C. Informing her that her problems are psychogenic and that there is nothing to worry about D. Hospitalization as requested, then consultation with a psychiatrist E. Scheduling frequent, regular office visits

E Somatoform disorder is often encountered in family practice. Studies have documented that 5% of patients meet the criteria for somatization disorder, while another 4% have borderline somatization disorder. Most of these patients are female and have a low socioeconomic status. They have a high utilization of medical services, usually reflected by a thick medical chart, and are often single parents. As a rule, physicians tend to be less satisfied with the care rendered to these patients as opposed to those without the disorder. Patients with multiple unexplained physical complaints have been described as functionally disabled, spending an average of one week per month in bed. Many of these patients seek and are ultimately granted surgical procedures, and it is not uncommon for them to have multiple procedures, especially involving the pelvic area. Often there are associated psychiatric symptoms such as anxiety, depression, suicidal threats, alcohol or drug abuse, interpersonal or occupational difficulties, and antisocial behavior. A background of a dysfunctional family unit in which one or both parents abused alcohol or drugs or were somatically preoccupied is also quite common. Unfortunately, these individuals tend to marry alcohol abusers, and thus continue the pattern of dysfunctional family life. Treatment of somatoform disorder should be by one primary physician where an established relationship and regular visits can curtail the dramatic symptoms that many times lead to hospitalization. The family physician is in a position to monitor family dynamics and provide direction on such issues as alcoholism and child abuse. Each office visit should be accompanied by a physical examination, and the temptation to tell the patient that the problem is not physical should be avoided. Knowing the patient well helps to avoid unnecessary hospitalization, diagnostic procedures, surgery, and laboratory tests. These should be done only if clearly indicated. Psychotropic medications should be avoided except when clearly indicated, as medications reinforce the sick role, may be abused, and may be used for suicidal gestures. Following these recommendations significantly decreases the cost of care for the patient. Ref: Rakel RE: Textbook of Family Practice, ed 6. WB Saunders Co, 2002, pp 1497-1498. 2) Hales RE, Yudofsky SC (ed): Textbook of Clinical Psychiatry, ed 4. American Psychiatric Publishing, 2003, pp 659-673.

A 45-year-old male asks you to evaluate his cardiovascular health status. He is currently asymptomatic, but wants to do everything he can to prevent heart disease and to understand his potential cardiovascular risk. His Framingham score indicates that he is at low risk (10-year risk <6%), and his physical examination is normal. He asks which laboratory and imaging tests he should have, and you recommend a lipid profile. According to the American College of Cardiology Foundation and the American Heart Association, which one of the following should also be recommended for this patient at this time? (check one) A. Lipoprotein and apolipoprotein levels B. A C-reactive protein level C. Measurement of cardiac calcium D. An ankle-brachial index E. No further testing

E The American College of Cardiology Foundation/American Heart Association guidelines for early cardiovascular assessment do not recommend lipoprotein and apolipoprotein levels. A C-reactive protein level can help to determine the need for statin therapy in men 50 and older and women 60 and older whose LDL-cholesterol levels are <130 mg/dL and who are not on lipid-lowering medication, hormone therapy, or immunosuppressive therapy, and who do not have clinical coronary heart disease, diabetes mellitus, chronic kidney disease, severe inflammatory disease, or contraindications to statins. A C-reactive protein level may also be reasonable in younger patients with intermediate, but not low, cardiovascular risk. Measurement of cardiac calcium levels is reasonable in patients whose cardiovascular risk is intermediate (10-year risk 10%-20%) or low-to-intermediate (10-year risk 6%-10%). An ankle-brachial index is reasonable for intermediate-risk, but not low-risk, patients. At this point in time, the patient described here does not meet any recommended criteria for further testing.

A 9-month-old male is seen for a routine well-baby examination. There have been no health problems and developmental milestones are normal. Review of the growth chart shows that length, weight, and head circumference have continued to remain at the 75th percentile. The examination is normal with the exception of the anterior fontanelle being closed. Proper management at this time would include: (check one) A. A CT scan of the head B. MRI of the head C. A CBC, a metabolic profile, and thyroid studies D. Referral to a neurologist E. Serial measurement of head circumference

E The anterior fontanelle in the newborn is normally 0.6-3.6 cm, with the mean size being 2.1 cm. It may actually enlarge the first few months, but the medial age of closure is 13.8 months. The anterior fontanelle closes at 3 months in 1% of cases, and by 1 year, 38% are closed. While early closure of the anterior fontanelle may be normal, the head circumference must be carefully monitored. The patient needs to be monitored for craniosynostosis (premature closure of one or more sutures) and for abnormal brain development. When craniosynostosis is suspected, a skull radiograph is useful for initial evaluation. If craniosynostosis is seen on the film, a CT scan should be obtained. Ref: Kiesler J, Ricer R: The abnormal fontanel. Am Fam Physician 2003;67(12):2547-2552.

The daily intake of vitamins and minerals recommended by the Food and Nutrition Board varies according to sex, age, and condition. The recommended daily allowance of vitamin D is greatest for which one of the following? (check one) A. A 15-year-old nonpregnant female B. A 25-year-old pregnant female C. A 35-year-old lactating female D. A 55-year-old female E. A 75-year-old female

E The current Dietary Reference Intake (DRI - which has replaced RDA's) recommendation for vitamin D is 200 IU/day for all women between the ages of 9 and 50 years; pregnancy or lactation does not affect the recommendation. The DRI doubles to 400 IU daily for women age 51-70 and triples to 600 IU daily for women over the age of 70. The maximum daily oral intake of vitamin D thought to be safe is 2000 IU/day for all females over the age of 12 months. Ref: Dietary reference intakes: Recommended intakes for individuals, vitamins. National Academy of Sciences, 2004.

A 15-month-old male is brought to your office 3 hours after the onset of an increased respiratory rate and wheezing. He has an occasional cough and no rhinorrhea. His immunizations are up to date and he attends day care regularly. His temperature is 38.2°C (100.8°F), respiratory rate 42/min, and pulse rate 118 beats/min. The child is sitting quietly on his mother's lap. His oxygen saturation is 94% on room air. On examination you note inspiratory crackles in the left lower lung field. The child appears to be well hydrated and the remainder of the examination, including an HEENT examination, is normal. Nebulized albuterol (AccuNeb) is administered and no improvement is noted. Which one of the following would be most appropriate in the management of this patient? (check one) A. Laboratory evaluation B. Inpatient monitoring, with no antibiotics at this time C. Hospitalization and intravenous ceftriaxone (Rocephin) D. Close outpatient follow-up, with no antibiotics at this time E. Oral high-dose amoxicillin (90 mg/kg/day), with close outpatient follow-up

E The diagnosis of community-acquired pneumonia is mostly based on the history and physical examination. Pneumonia should be suspected in any child with fever, cyanosis, and any abnormal respiratory finding in the history or physical examination. Children under 2 years of age who are in day care are at higher risk for developing community-acquired pneumonia. Laboratory tests are rarely helpful in differentiating viral versus bacterial etiologies and should not be routinely performed. Outpatient antibiotics are appropriate if the child does not have a toxic appearance, hypoxemia, signs of respiratory distress, or dehydration. Streptococcus pneumoniae is one of the most common etiologies in this age group, and high-dose amoxicillin is the drug of choice. Ref: Ostapchuk M, Roberts D, Haddy R: Community-acquired pneumonia in infants and children. Am Fam Physician 2004;70(5):899-908. 2) Kliegman RM, Stanton BF, Geme JW III, et al (eds): Nelson Textbook of Pediatrics, ed 19. Elsevier Saunders, 2011, pp 1474-1479.

Which one of the following would most likely be found in a patient with giardiasis? (check one) A. Fecal leukocytosis B. Mucus in the stool C. Eosinophilia D. Hematochezia E. Foul-smelling flatus

E The diagnosis of giardiasis is suggested by its most characteristic symptoms: foul-smelling, soft, or loose stools; foul-smelling flatus; belching; marked abdominal distention; and the virtual absence of mucus or blood in the stool. Stools are usually mushy between exacerbations, though constipation may occur. If eosinophilia occurs, it is more likely to be related to some other concomitant cause rather than to giardiasis.

The mother of a 16-year-old male brings him to your office stating that she wants to find out if he has Crohn's disease. She says that both she and the child's aunt were diagnosed with this condition by another physician with "blood tests." The son tells you that for the past several years his stool is intermittently loose and he has up to three bowel movements in a day. He says he does not have fever, pain, hematochezia, weight loss, or any extraintestinal symptoms. A physical examination is normal. Which one of the following would be the most appropriate preliminary testing? (check one) A. A plain radiograph of the abdomen B. CT of the abdomen and pelvis C. An inflammatory bowel disease serologic panel D. Colonoscopy with a biopsy E. A CBC, serum chemistry panel, and erythrocyte sedimentation rate

E The diagnosis of inflammatory bowel disease (IBD) can be elusive but relies primarily on the patient history, laboratory findings, and endoscopy (or double-contrast radiographs if endoscopy is not available). Endoscopy is usually reserved for patients with more severe symptoms or in whom preliminary testing shows the potential for significant inflammation. It is recommended that this preliminary evaluation include a WBC count, platelet count, potassium level, and erythrocyte sedimentation rate. Patients who have minimal symptoms and normal preliminary testing likely do not have a significant case of IBD. Plain radiographs and CT of the abdomen may help rule out other etiologies but are not considered adequate to diagnose or exclude IBD. Panels of serologic blood tests have recently been developed and are being assessed as to their place in evaluating patients who may have IBD. However, this testing is expensive, lacks sufficient predictive value, and has yet to prove its utility compared to standard testing. Ref: Langan RC, Gotsch PB, Krafczyk MA, Skillinge DD: Ulcerative colitis: Diagnosis and treatment. Am Fam Physician 2007;76(9):1323-1330. 2) Benor S, Russell GH, Silver M, et al: Shortcomings of the Inflammatory Bowel Disease Serology 7 panel. Pediatrics 2010;125(6):1230-1236.

A 3-year-old toilet-trained female is brought to your office by her mother, who has noted a red rash on the child's perineum for the last 5 days. The rash is pruritic and has been spreading. The mother has treated the area for 3 days with nystatin cream with no obvious improvement. The child has not used any other recent medications and has no significant past medical history. Your examination reveals a homogeneous, beefy red rash surrounding the vulva and anus. The most likely etiologic agent is: (check one) A. Malassezia furfur B. Escherichia coli C. Haemophilus influenzae D. Staphylococcus aureus E. group A Streptococcus pyogenes

E The epidemiology of group A streptococcal disease of the perineum is similar to that of group A streptococcal pharyngitis, and the two often coexist. It is theorized that either auto-inoculation from mouth to hand to perineum occurs, or that the bacteria is transmitted through the gastrointestinal tract. In one study, the average age of patients with this disease varied from 1 to 11 years, with a mean of 5 years. Girls and boys were almost equally affected. The incidence is estimated to be about 1 in 200 pediatric visits and peaks in March, April, and May in North America. The condition usually presents with itching and a beefy redness around the anus and/or vulva and will not clear with medications used to treat candidal infections.

The Centers for Disease Control and Prevention (CDC) recommends antenatal screening for group B streptococcal disease by: (check one) A. Culturing the urine at 20 weeks' gestation B. Obtaining cultures from the rectum and vaginal introitus at 20 weeks' gestation C. Obtaining a culture from the cervix at 35-37 weeks' gestation D. Obtaining cultures from the cervix and rectum at 35-37 weeks' gestation E. Obtaining cultures from the rectum and vaginal introitus at 35-37 weeks' gestation

E The gastrointestinal tract is the most likely reservoir of group B Streptococcus with secondary spread to the genital tract. Cultures from the vaginal introitus and the rectum are the most sensitive for detecting colonization. No speculum examination is necessary. The closest time to delivery that cultures can be performed and allow time for results to be available is 35-37 weeks' gestation. Culture-positive women are then treated during labor. Other criteria for the use of chemoprophylaxis during delivery continue to apply. Ref: Centers for Disease Control and Prevention: Prevention of perinatal group B streptococcal disease: A public health perspective. MMWR 1996;45(RR-7):1-24. 2) Hospital-based policies for prevention of perinatal group B streptococcal disease—United States, 1999. MMWR 2000;49(41):936-940. 3) Adoption of perinatal group B streptococcal disease prevention recommendations by prenatal-care providers—Connecticut and Minnesota, 1998. MMWR 2000;49(11):228-231. 4) Pickering LK (ed): 2003 Red Book: Report of the Committee on Infectious Diseases, ed 26. American Academy of Pediatrics, 2003, pp 584-591.

Women who use low-dose estrogen oral contraceptives have a 50% lower risk of cancer of the: (check one) A. breast B. cervix C. head and neck D. lung E. ovary

E Women who use low-dose estrogen oral contraceptives have at least a 50% lower risk of subsequent epithelial ovarian cancer than women who have never used them. Epidemiologic data also suggests other potential long-term benefits of oral contraceptives, including a reduced risk of postmenopausal fractures, as well as reductions in the risk of endometrial and colorectal cancers. Oral contraceptives do not reduce the risk of carcinoma of the breast, cervix, lung, or head and neck. Ref: Kaunitz AM: Hormonal contraception in women of older reproductive age. N Engl J Med 2008;358(12):1262-1270.

Which one of the following decreases pain from infiltration of local anesthetics? (check one) A. Cooling the anesthetic solution B. Using a 22-gauge needle rather than a 30-gauge needle C. Infiltrating quickly D. Infiltrating through surrounding intact skin E. Adding sodium bicarbonate to the mixture

E The pain from infiltration of local anesthetics can be decreased by using a warm solution, using small needles, and performing the infiltration slowly.It is also helpful to add sodium bicarbonate to neutralize the anesthetic since they are shipped at an acidic pH to prolong shelf life. An exception to this tip is bupivicaine (Marciane, Sensorcaine) as it will precipitate in the presence of sodium bicarbonate. It also helps to inject the agent through the edges of the wound (assuming the wound is not contaminated) and to pretreat the wound with topical anesthetics. Ref: Singer AJ, Hollander JE, Quinn JV: Evaluation and management of traumatic lacerations. N Engl J Med 1997;337(16):1142-1148. 2) Marx JA (ed): Rosen's Emergency Medicine: Concepts and Clinical Practice, ed 6. Mosby Elsevier, 2006, pp 2930-2933.

A 44-year-old African-American female reports diffuse aching, especially in her upper legs and shoulders. The aching has increased, and she now has trouble going up and down stairs because of weakness. She has no visual symptoms, and a neurologic examination is normal except for proximal muscle weakness. Laboratory tests reveal elevated levels of serum creatine kinase and aldolase. Her symptoms improve significantly when she is treated with corticosteroids. Which one of the following is the most likely diagnosis? (check one) A. Duchenne's muscular dystrophy B. Myasthenia gravis C. Amyotrophic lateral sclerosis D. Aseptic necrosis of the femoral head E. Polymyositis

E The patient described has an inflammatory myopathy of the polymyositis/dermatomyositis group. Proximal muscle involvement and elevation of serum muscle enzymes such as creatine kinase and aldolase are characteristic. Corticosteroids are the accepted treatment of choice. It is extremely unlikely that Duchenne's muscular dystrophy would present after age 30. In amyotrophic lateral sclerosis, an abnormal neurologic examination with findings of upper motor neuron dysfunction is characteristic. Patients with myasthenia gravis characteristically have optic involvement, often presenting as diplopia. The predominant symptom of aseptic necrosis of the femoral head is pain rather than proximal muscle weakness. Ref: Fauci AS, Braunwald E, Kasper DL, et al (eds): Harrison's Principles of Internal Medicine, ed 17. McGraw-Hill, 2008, pp 2696-2703.

A 34-year-old white female comes to the office for a Papanicolaou (Pap) test. On a review-of-system checklist, she checks "yes" to depressive symptoms, insomnia, and anxiety. On questioning, she admits to feeling depressed for about 4 months, after a recent job change. She is not suicidal. With probing, she admits that she repeatedly checks her locks and constantly worries about cleanliness; she has been this way "all of her life," but finds it very time-consuming. Which one of the following drugs is the best choice for this patient? (check one) A. Risperidone (Risperdal) B. Clorazepate (Tranxene) C. Clonazepam (Klonopin) D. Imipramine (Tofranil) E. Fluoxetine (Prozac)

E The patient most likely has obsessive-compulsive disorder (OCD) with a depressive episode. SSRIs are most frequently used. Risperidone and clonazepam are considered second-line drugs and are used as augmentation drugs when there is a partial response to an SSRI. There is no evidence that clorazepate or imipramine is effective in OCD. Ref: Hales RE, Yudofsky SC: Textbook of Clinical Psychiatry, ed 4. American Psychiatric Publishing, Inc., 2003, p 590.

A 17-year-old white female visits you for a physical examination prior to entering college. During the review of systems her only complaint is cyclic lower abdominal cramps around the onset of menstruation. She reports that pain has been present to some degree with most of her periods since about 6 months after menarche. The pain is often severe enough for her to miss school. Each episode lasts 24-48 hours and is somewhat relieved by rest and acetaminophen. Her menstrual history is otherwise normal. She denies ever being sexually active and tells you that she has received little empathy from her mother, who had similar symptoms as an adolescent that improved after her first pregnancy. Pelvic and rectal examinations are within normal limits. Which one of the following management choices would be appropriate at this time? (check one) A. Referral for hysterosalpingography B. Referral for psychological counseling C. Danazol (Danocrine) D. Acetaminophen/hydrocodone (Vicodin HP) E. Naproxen sodium (Anaprox)

E The patient's history is typical of primary dysmenorrhea, defined as severe cramping pain in the lower abdomen that occurs during menses; it may also occur prior to the onset of menses in the absence of associated pelvic pathology. Although many women complain of pain beginning with the first cycle, symptoms usually begin at the onset of ovulation around 6-12 months after menarche. Symptoms typically last 48 hours or less, but sometimes may last up to 72 hours. It is common to find daughters with dysmenorrhea whose mothers had the same symptoms. Additionally, the symptoms of primary dysmenorrhea often resolve after the first pregnancy. In this patient, who has no history suggesting an emotional disorder, there is no need for psychological counseling at this time. Further evaluation could include ultrasonography to rule out causes of dysmenorrhea such as uterine leiomyomata, adnexal masses, and endometrial polyps. However, a trial of symptomatic therapy is most reasonable before other invasive studies, such as a laparoscopic examination or a hysterosalpingogram, are ordered. It is not reasonable to begin danazol without a diagnosis of endometriosis, which is by definition secondary dysmenorrhea. Since neither inhibits prostaglandin synthetase, acetaminophen (which she had already tried without complete relief) combined with a narcotic is not an appropriate management strategy. Multiple placebo-controlled studies have shown that NSAIDs such as naproxen, at the onset of symptoms, provide significant relief of primary dysmenorrhea compared to placebo. Ref: Howard FM (ed): Pelvic Pain: Diagnosis and Management. Lippincott, Williams and Wilkins, 2000, pp 100-107. 2) Scott JR, Gibbs RS, Karlan BY, et al (eds): Danforth's Obstetrics and Gynecology, ed 9. Lippincott Williams & Wilkins, 2003, pp 536-537.

With regard to the cardiovascular system, activation of the sympathetic branch of the autonomic nervous system will cause a decrease in which one of the following? (check one) A. Heart rate B. Coronary flow rate C. Metabolic demand D. Contractility of cardiac myocytes E. The P-R interval

E The sympathetic nervous system acts as a positive chronotropic (increases heart rate) and inotropic (increases contractility) agent. This additional work by the heart will increase metabolic demand and coronary flow rate. The increased heart rate will decrease the time intervals between electrical events shown on an EKG. Ref: Fauci AS, Braunwald E, Kasper DL, et al (eds): Harrison's Principles of Internal Medicine, ed 17. McGraw-Hill, 2008, pp 1370-1373.

Which one of the following is an absolute contraindication to electroconvulsive therapy (ECT)? (check one) A. Age >80 years B. A cardiac pacemaker C. An implantable cardioverter-defibrillator D. Pregnancy E. There are no absolute contraindications to ECT

E There are no absolute contraindications to electroconvulsive therapy (ECT), but factors that have been associated with reduced efficacy include a prolonged episode, lack of response to medication, and coexisting psychiatric diagnoses such as a personality disorder. Persons who may be at increased risk for complications include those with unstable cardiac disease such as ischemia or arrhythmias, cerebrovascular disease such as recent cerebral hemorrhage or stroke, or increased intracranial pressure. ECT can be used safely in elderly patients and in persons with cardiac pacemakers or implantable cardioverter-defibrillators. ECT also can be used safely during pregnancy, with proper precautions and in consultation with an obstetrician. Ref: Lisanby SH: Electroconvulsive therapy for depression. N Engl J Med 2007;357(19):1939-1945.

Patients with which one of the following conditions are at increased risk for complications from electroconvulsive therapy for depression? (check one) A. Pregnancy B. Seizure disorder C. Cardiac pacemaker implantation D. Depression unresponsive to oral medications E. Recent cerebral hemorrhage

E There are no absolute contraindications to electroconvulsive therapy (ECT), but more complications are seen in patients with a history of recent cerebral hemorrhage, stroke, or increased intracranial pressure. The efficacy of ECT may be reduced in patients who have not responded to oral antidepressants. Ref: Lisanby SH: Electroconvulsive therapy for depression. N Engl J Med 2007;357(19):1939-1945.

A 6-month old Hispanic female has had itching and irritability for 4-5 weeks. There is a family history of atopy and asthma. Physical examination reveals an excoriated dry rash bilaterally over the antecubital and popliteal fossae, as well as some involvement of the face. In addition to maintenance therapy with an emollient, which of the following topical medications would be approporate first-line treatment for flare-ups in this patient. (check one) A. A calcineurin inhibitor such as pimecrolimus (Elidel) B. An anesthetic C. An antihistamine D. An antibiotic E. A corticosteroid

E This child has atopic dermatitis (ecxema). It is manifested by a pruritic rash on the face and/or flexural surfaces of the arms and/or legs, expecially in children. There often is a family history of atopy or allergies. In addition to the regular use of emollients, the mainstay of maintenance therapy, topical corticosteroids have been shown to be the best first-line treatment for flare-ups of atopic dermatitis. Topical cancineurin inhibitors should be second-line treatment for flare-ups, but are not recommended for use in children under 2 years of age. Antibiotics should be reserved for the treatment of acutely infected lesions. There is no evidence to support the use of topical anesthetics in the treatment of this disorder. Ref: Buys LM. Treatment options for atopic dermatitis. Am Fam Physician. 2007 Feb 15;75(4):523-8. Review. PubMed PMID: 17323714.

A 23-month-old child is brought to your office with a 2-day history of a fever to 102°F (39°C), cough, wheezing, and mildly labored breathing. He has no prior history of similar episodes and there is no improvement with administration of an aerosolized bronchodilator. Which one of the following is now indicated? (check one) A. Bronchodilator aerosol treatment every 6 hours B. Corticosteroids C. An antibiotic D. A decongestant E. Supportive care only

E This child has typical findings of bronchiolitis. The initial infection usually occurs by the age of 2 years. It is caused by respiratory syncytial virus (RSV). Bronchodilator treatment may be tried once and discontinued if there is no improvement. Treatment usually consists of supportive care only, including oxygen and intravenous fluids if indicated (SOR B). Corticosteroids, antibiotics, and decongestants are of no benefit. RSV infection may recur, since an infection does not provide immunity. Up to 10% of infected children will have wheezing past age 5, and bronchiolitis may predispose them to asthma.

A 12-year-old Hispanic female is brought to your office because of the recent onset of a white vaginal discharge. She is otherwise asymptomatic and has never menstruated. She denies sexual activity and a general examination reveals no abnormalities. You note the presence of breast buds and scant pubic hair. Microscopic examination of the vaginal discharge shows sheets of vaginal epithelial cells. Which one of the following is most likely in this setting? (check one) A. Pinworm (Enterobius vermicularis) infestation B. Sexual abuse C. Vaginal foreign body D. Trichomoniasis E. Physiologic secretions

E This child is entering puberty. In the 6- to 12-month period before menarche, girls often develop a physiologic vaginal discharge secondary to the increase in circulating estrogens. The gray-white discharge is non-irritating. When physiologic discharge is examined with the microscope, sheets of vaginal epithelial cells are seen. The only treatment necessary is reassurance of both parents and child that this is a normal process that will subside with time. The other conditions listed are pathologic and have other associated symptoms and findings not seen in this case. Pinworms normally cause perianal and vulvar pruritus and irritation. The findings in sexual abuse range from an inflamed vulvovaginal area, to evidence of sexually transmitted diseases, to evidence of local trauma. Trichomoniasis would cause vulvovaginal irritation and microscopic examination of the discharge would show Trichomonas organisms. A vaginal foreign body would usually present with a foul and/or bloody vaginal discharge. Ref: Stenchever MA, Droegemueller W, Herbst AL, et al: Comprehensive Gynecology, ed 4. Mosby, 2001, pp 274-277. 2) Behrman RE, Kliegman RM, Jenson HB (eds): Nelson Textbook of Pediatrics, ed 17. Saunders, 2004, pp 663-667.

An overweight 13-year-old male presents with a 3-week history of right lower thigh pain. He first noticed the pain when jumping while playing basketball, but now it is present even when he is just walking. On examination he can bear his full weight without an obvious limp. There is no localized tenderness, and the patella tracks normally without subluxation. Internal rotation of the hip is limited on the right side compared to the left. Based on the examination alone, which one of the following is the most likely diagnosis? (check one) A. Avascular necrosis of the femoral head (Legg-Calvé-Perthes disease) B. Osteosarcoma C. Meralgia paresthetica D. Pauciarticular juvenile rheumatoid arthritis E. Slipped capital femoral epiphysis

E This is a classic presentation for slipped capital femoral epiphysis (SCFE) in an adolescent male who has probably had a recent growth spurt. Pain with activity is the most common presenting symptom, as opposed to the nighttime pain that is typical of malignancy. Obese males are affected more often. The pain is typically in the anterior thigh, but in a high percentage of patients the pain may be referred to the knee, lower leg, or foot. Limited internal rotation of the hip, especially with the hip in 90°; flexion, is a reliable and specific finding for SCFE and should be looked for in all adolescents with hip, thigh, or knee pain. Meralgia paresthetica is pain in the thigh related to entrapment of the lateral femoral cutaneous nerve, often attributed to excessively tight clothing. Legg-Calvé-Perthes disease (avascular or aseptic necrosis of the femoral head) is more likely to occur between the ages of 4 and 8 years. Juvenile rheumatoid arthritis typically is associated with other constitutional symptoms including stiffness, fever, and pain in at least one other joint, with the pain not necessarily associated with activity. Ref: Griffin LY (ed): Essentials of Musculoskeletal Care, ed 3. American Academy of Orthopaedic Surgeons, American Academy of Pediatrics, 2005, pp 941-943.

A 66-year-old male smoker is being evaluated for a persistent cough and difficulty breathing. Spirometry confirms a fixed obstructive pathology with an FEV1 of about 50% of predicted for habitus and age. His oxygen saturation is 89%-90% on room air. Which one of the following would be most effective to prevent worsening of this patient's condition? (check one) A. A combined inhaled corticosteroid and long-acting β-agonist B. A long-acting anticholinergic agent C. Long-term oral corticosteroids D. Oxygen therapy E. Smoking cessation

E This patient has moderate to severe COPD. Smoking cessation is the single most important therapeutic intervention in patients with this condition and should be the priority of care. No existing medications have been shown to modify the long-term decline in lung function that is typical of COPD, but smoking cessation does prevent this decline. Long-term use of oxygen in COPD patients who also have chronic, severe hypoxia (<88% saturation) can improve quality of life and prolong survival; however, oxygen cannot prevent further decline in lung function. Long-term use of oral corticosteroids is discouraged because of an unfavorable risk-to-benefit ratio. Ref: Global strategy for the diagnosis, management, and prevention of chronic obstructive pulmonary disease. Medical Communication Resources, 2006, pp 8-15. 2) Rabe KF, Beghe B, Luppi F, et al: Update in chronic obstructive pulmonary disease 2006. Am J Respir Crit Care Med 2007;175(12):1222-1232.

A 44-year-old female who suffers from obstructive sleep apnea complains of gradual swelling in her legs over the last several weeks. Her vital signs include a BMI of 44.1 kg/m2 , a respiratory rate of 12/min, a blood pressure of 120/78 mm Hg, and an O 2 saturation of 86% on room air. An EKG and a chest radiograph are normal. Pulmonary function testing shows a restrictive pattern with no signs of abnormal diffusion. Abnormal blood tests include only a significantly elevated bicarbonate level. Which one of the following treatments is most likely to reduce this patient's mortality rate? (check one) A. ACE inhibitors B. Routine use of nebulized albuterol (AccuNeb) C. High-dose diuretic therapy D. Continuous oxygen therapy E. Continuous or bilevel positive airway pressure (CPAP or Bi-PAP)

E This patient has obesity-hypoventilation syndrome, often referred to as Pickwickian syndrome. These patients are obese (BMI >30 kg/m 2 ), have sleep apnea, and suffer from chronic daytime hypoxia andcarbon dioxide retention. They are at increased risk for significant respiratory failure and death compared to patients with otherwise similar demographics. Treatment consists of nighttime positive airway pressure in the form of continuous (CPAP) or bi-level (BiPAP) devices, as indicated by sleep testing. The more hours per day that patients can use this therapy, the less carbon dioxide retention and less daytime hypoxia will ensue. Several small studies suggest that the increased mortality risk from obesity-hypoventilation syndrome can be decreased by adhering to this therapy. The use of daytime oxygen can improve oxygenation, but is not considered adequate to restore the chronic low respiratory drive that is characteristic of this condition.

A 72-year-old African-American male with New York Heart Association Class III heart failure sees you for follow-up. He has shortness of breath with minimal exertion. The patient is adherent to his medication regimen. His current medications include lisinopril (Prinivil, Zestril), 40 mg twice daily; carvedilol (Coreg), 25 mg twice daily; and furosemide (Lasix), 80 mg daily. His blood pressure is 100/60 mm Hg, and his pulse rate is 68 beats/min and regular. Findings include a few scattered bibasilar rales on examination of the lungs, an S3 gallop on examination of the heart, and no edema on examination of the legs. An EKG reveals a left bundle branch block, and echocardiography reveals an ejection fraction of 25%, but no other abnormalities. Which one of the following would be most appropriate at this time? (check one) A. Increase the lisinopril dosage to 80 mg twice daily B. Increase the carvedilol dosage to 50 mg twice daily C. Increase the furosemide dosage to 160 mg daily D. Refer for coronary angiography E. Refer for cardiac resynchronization therapy

E This patient is already receiving maximal medical therapy. The 2002 joint guidelines of the American College of Cardiology, the American Heart Association (AHA), and the North American Society of Pacing and Electrophysiology endorse the use of cardiac resynchronization therapy (CRT) in patients with medically refractory, symptomatic, New York Heart Association (NYHA) class III or IV disease with a QRS interval of at least 130 msec, a left ventricular end-diastolic diameter of at least 55 mm, and a left ventricular ejection fraction (LVEF) ≤30%. Using a pacemaker-like device, CRT aims to get both ventricles contracting simultaneously, overcoming the delayed contraction of the left ventricle caused by the left bundle-branch block. These guidelines were refined by an April 2005 AHA Science Advisory, which stated that optimal candidates for CRT have a dilated cardiomyopathy on an ischemic or nonischemic basis, an LVEF ≤0.35, a QRS complex ≥120 msec, and sinus rhythm, and are NYHA functional class III or IV despite maximal medical therapy for heart failure. Ref: Jarcho JA: Biventricular pacing. N Engl J Med 2006;355(3):288-294.

A 7-year-old female with a history of asthma is brought to your office for a routine follow-up visit. She has a history of exercise-induced asthma, but also has had exacerbations in the past that were unrelated to exercise. In the past month, she has premedicated herself with albuterol (Proventil, Ventolin) with a spacer before recess 5 days/week as usual. She has also needed her albuterol to treat symptoms (wheezing and/or shortness of breath) once or twice per week and had one exacerbation requiring medical treatment in the past year. She has had no nighttime symptoms. Albuterol as needed is her only medication. After reinforcing asthma education, which one of the following would be most appropriate? (check one) A. Referral to an asthma specialist B. Addition of a low-dose inhaled corticosteroid C. Addition of a long-acting β-agonist D. Elimination of premedication with albuterol, restricting use to an as-needed basis E. No changes to her regimen

E This patient's asthma is well-controlled according to the 2007 NHLBI asthma guidelines. The "rule of twos" is useful in assessing asthma control: in children under the age of 12, asthma is NOT well-controlled if they have had symptoms or used a β-agonist for symptom relief more than twice per week, had two or more nocturnal awakenings due to asthma symptoms in the past month, or had two or more exacerbations requiring systemic corticosteroids in the past year. For individuals over 12 years of age, there must be more than two nocturnal awakenings per month to classify their asthma as not well controlled. Exercise-induced asthma is considered separately. A β-agonist used as premedication before exercise is not a factor when assessing asthma control. Since this patient does not exceed the rule of twos, her asthma is categorized as well-controlled and no changes to her therapy are indicated. Asthma education should be reinforced at every visit.

A 40-year-old female presents with a complaint of fatigue. She says she is also concerned because she has gained about 10 lb over the last several months. Physical examination reveals no enlargement or other abnormalities of the thyroid gland. Laboratory testing reveals a TSH level of 0.03 μU/mL (N 0.4-4.0) and a free T4 level of 1.0 μg/dL (N 1.5-5.5). Which one of the following is the most likely cause of her problem? (check one) A. Malnutrition B. Graves' disease C. Goiter D. Hashimoto's thyroiditis E. Pituitary failure

E This patient's symptoms and laboratory findings suggest a significant lack of TSH despite low levels of circulating thyroid hormone. This is diagnostic of secondary hypothyroidism. Such findings should prompt a workup for a pituitary or hypothalamic deficiency that is causing a lack of TSH production. Primary hypothyroidism, such as Hashimoto's thyroiditis, would be evidenced by an elevated TSH and low (or normal) T4 . Graves' disease is a cause of hyperthyroidism, which would be expected to increase T4 levels, although low TSH with a normal T4 level may be present. Some nonthyroid conditions such as malnutrition may suppress T4 . In such cases the TSH would be elevated or normal. This patient has gained weight, which does not coincide with malnutrition. The patient does not have the thyroid gland enlargement seen with goiter.

An 83-year-old male has a long history of COPD. His resting oxygen saturation is 86% on room air. Treatment includes oral bronchodilators, inhaled corticosteroids, inhaled beta-agonists, inhaled cholinergics, and home oxygen. Which one of his treatments has been shown to prolong survival in cases such as this? (check one) A. Oral bronchodilators B. Inhaled corticosteroids C. Inhaled beta-agonists D. Inhaled cholinergics E. Home oxygen

E Treatment of hypoxemia is critical in the management of COPD and trials have shown a reduction in mortality with the use of oxygen for 15 or more hours daily. Inhaled beta-adrenergic agonists and cholinergic agents, either alone or in combination, provide symptomatic relief but do not prolong survival. Theophylline can be used for symptoms inadequately relieved by bronchodilators. Inhaled corticosteroids do not appear to alter the rate of decline in lung function in COPD. However, some evidence shows that these agents alleviate symptoms and reduce disease exacerbation. Pulmonary rehabilitation improves quality of life and reduces hospitalizations. Ref: Sin DD, McAlister FA, Man SF, et al: Contemporary management of chronic obstructive pulmonary disease. JAMA 2003;290(17):2301-2312.

A 40-year-old male with HIV infection presents to the emergency department with a 5-day history of progressive cough and dyspnea on exertion. A chest radiograph shows bilateral diffuse interstitial infiltrates. Arterial blood gas levels show an increased alveolar-arterial gradient and a pO2 of 60 mm Hg. His CBC is normal but his CD4 count is 150/mm3 . In addition to trimethoprim/sulfamethoxazole (Bactrim, Septra), which one of the following medications should be prescribed? (check one) A. Pentamidine (Pentam) B. Dapsone C. Atovaquone (Mepron) D. Clindamycin (Cleocin) and primaquine E. Corticosteroids

E Trimethoprim/sulfamethoxazole is the treatment of choice for acute Pneumocystis pneumonia. Adjunctive corticosteroids should also be started in any patient whose initial pO2 on room air is <70 mm Hg. Three prospective trials have shown that there is a decrease in mortality and frequency of respiratory failure when corticosteroids are used in addition to antibiotics. All of the other medications listed are effective therapy for Pneumocystis pneumonia, but they do not need to be given with trimethoprim/sulfamethoxazole. Ref: Mandell GL, Bennett JE, Dolin R (eds): Mandell, Douglas, and Bennett's Principles and Practice of Infectious Diseases, ed 7. Churchill Livingstone, 2009, pp 1858-1874.

A positive spot urine test for homovanillic acid (HMA) and vanillylmandelic acid (VMA) is a marker for which one of the following? (check one) A. Hepatoblastoma B. Wilms' tumor C. Lymphoma D. Malignant teratoma E. Neuroblastoma

E Tumor markers are useful in determining the diagnosis and sometimes the prognosis of certain tumors. They can aid in assessing response to therapy and detecting tumor recurrence. Serum neuron-specific enolase (NSE) testing, as well as spot urine testing for homovanillic acid (HVA) and vanillylmandelic acid (VMA), should be obtained if neuroblastoma or pheochromocytoma is suspected; both should be collected before surgical intervention. Quantitative beta-human chorionic gonadotropin (hCG) levels can be elevated in liver tumors and germ cells tumors. Alpha-fetoprotein is excreted by many malignant teratomas and by liver and germ cell tumors. Ref: Golden CB, Feusner JH: Malignant abdominal masses in children: Quick guide to evaluation and diagnosis. Pediatr Clin North Am 2002;49:1369-1392.

A 53-year-old female presents to the emergency department following a fall. She is found to have an ankle fracture and a blood pressure of 160/100 mm Hg. She tells the emergency department physician that she is not aware of any previous medical problems. A focused cardiovascular examination is otherwise normal. You are the patient's regular physician, and the emergency physician calls your office for further information about the blood pressure elevation. You confirm that this is a new problem. Which one of the following would you ask the emergency physician to do? (check one) A. Administer a dose of intravenous labetalol (Trandate) and ask the patient to follow up in your office within the week B. Administer nifedipine (Adalat, Procardia), 10 mg; discharge the patient once the blood pressure falls to 140/90 mm Hg; and ask the patient to follow up with you tomorrow C. Prescribe an appropriate antihypertensive agent and have the patient follow up with you in a month D. Order an EKG and chest radiograph, and ask the patient to see you in a week if the results are normal E. Perform no further evaluation of the hypertension, but ask the patient to follow up with you within a month

E Uncomplicated hypertension is frequently detected in the emergency department. Many times this is a chronic condition, but it also may result from an acutely painful situation. Hypertensive emergencies, defined as severe blood pressure elevations to >180/120 mm Hg complicated by evidence of impending or worsening target organ dysfunction, warrant emergent treatment. There is no evidence, however, to suggest that treatment of an isolated blood pressure elevation in the emergency department is linked to a reduction in overall risk. In fact, the aggressive reduction of blood pressure with either intravenous or oral agents is not without potential risk. The appropriate management for the patient in this scenario is simply to discharge her and ask her to follow up with you in the near future.

When an interpreter is needed for a patient with limited English proficiency, which one of the following should be AVOIDED when possible? (check one) A. Using mostly short sentences, with frequent pauses B. Using diagrams and pictures C. Addressing the patient in the second person (i.e., "you") D. Maintaining eye contact with the patient when speaking E. Using an educated adult family member who is bilingual

E Using trained, qualified interpreters for patients with limited English proficiency leads to fewer hospitalizations, less reliance on testing, a higher likelihood of making the correct diagnosis and providing appropriate treatment, and better patient understanding of conditions and therapies. Although the patient may request that a family member interpret, there are many pitfalls in using untrained interpreters: a lack of understanding of medical terminology, concerns about confidentiality, and unconscious editing by the interpreter of what the patient has said. Additionally, the patient may be reluctant to divulge sensitive or potentially embarrassing information to a friend or family member. The other principles listed are important practices when working with interpreters. Pictures and diagrams can help strengthen the patient's understanding of his or her health care.

A 75-year-old male has not seen a physician in 25 years and presents with advanced Parkinson's disease. The best initial treatment would be: (check one) A. Referral to a neurosurgeon for thalamotomy B. Amantadine (Symmetrel) C. Benztropine (Cogentin) D. Pramipexole (Mirapex) E. Carbidopa/levodopa (Sinemet)

E While anticholinergics such as benztropine and amantadine may provide some improvement of symptoms, these effects wane within a few months. Such medications are not a good option in this patient with advanced disease. Dopamine agonists provide some improvement in motor complications, but are mainly used to delay the introduction of levodopa in younger patients, to avoid levodopa-related adverse reactions. Carbidopa/levodopa is better for initial therapy in older patients, and those who present with more severe symptoms. Slow-release versions of this combination may decrease motor fluctuations. Stereotactic thalamotomy is used to ameliorate tremors that have become disabling. This procedure has been replaced by other surgical options such as pallidotomy and high-frequency, deep-brain stimulation of specific nuclei. Ref: Siderowf A, Stern M: Update on Parkinson disease. Ann Intern Med 2003;138(8):651-658. 2) Goldman L, Ausiello D (eds): Cecil Textbook of Medicine, ed 22. Saunders, 2004, pp 2306-2310.

A 32-year-old gravida 3 para 2 is in labor at term following an uncomplicated prenatal course. As you deliver the fetal head it retracts against the perineum. Downward traction fails to free the anterior shoulder. The most appropriate course of action would be to: (check one) A. Apply increasingly strong downward traction to the fetal head B. Have an assistant apply fundal pressure C. Deliberately fracture the clavicle of the fetus D. Begin an intravenous nitroglycerin drip E. Place the mother's thighs on her abdomen

E While there are several risk factors for shoulder dystocia, most cases occur in fetuses of normal birth weight and are not anticipated. Once it does occur, excessive force should not be applied to the fetal head or neck and fundal pressure should be avoided, as these manuevers are unlikely to free the fetus and can injure both mother and infant. Up to 40% of shoulder dystocia cases can be successfully treated with the McRoberts maneuver, in which the maternal hips are flexed and abducted, placing the thighs up on the abdomen. Adding suprapubic pressure can resolve about half of all shoulder dystocias. Additional maneuvers include internal rotation, removal of the posterior arm, and rolling the patient over into the all-fours position. As a last resort, one can deliberately fracture the fetal clavicle, perform a cesarean section with the vertex being pushed back into the birth canal, or have the surgeon rotate the infant transabdominally with vaginal extraction performed by another physician. General anesthesia or nitroglycerin, orally or intravenously, may be used to achieve musculoskeletal or uterine relaxation. Intentional division of the cartilage of the symphysis under local anesthesia has been used in developing countries, but should be used only if all other maneuvers have failed and a cesarean delivery is not feasible. Ref: Rodis JF: Shoulder dystocia. UpToDate. Dec 2003. 2) Baxley EG, Gobbo RW: Shoulder dystocia. Am Fam Physician 2004;69(7):1707-1714.

A 52-year-old Hispanic female with diabetes mellitus and stage 3 chronic kidney disease sees you for follow-up after tests show an estimated glomerular filtration rate of 56 mL/min. Which one of the following medications should she avoid to prevent further deterioration in renal function? (check one) A. Lisinopril (Prinivil, Zestril) B. Folic acid C. Low-dose aspirin D. Candesartan (Atacand) E. Ibuprofen

E atients with chronic kidney disease (CKD) and those at risk for CKD because of conditions such as hypertension and diabetes have an increased risk of deterioration in renal function from NSAID use. NSAIDs induce renal injury by acutely reducing renal blood flow and, in some patients, by causing interstitial nephritis. Because many of these drugs are available over the counter, patients often assume they are safe for anyone. Physicians should counsel all patients with CKD, as well as those at increased risk for CKD, to avoid NSAIDs. ACE inhibitors and angiotensin II receptor blockers are renoprotective and their use is recommended in all diabetics. The use of low-dose aspirin and folic acid is recommended in all patients with diabetes, due to the vasculoprotective properties of these drugs. High-dose aspirin should be avoided because it acts as an NSAID. Ref: Fauci AS, Braunwald E, Kasper DL, et al (eds): Harrison's Principles of Internal Medicine, ed 17. McGraw-Hill, 2008, p 1771.

A 72-year-old white male develops a rapidly growing epithelial tumor just in front of his right ear. He states that it began as a firm red papule about 6 weeks ago. It is now 1.5 cm in diameter and has a horny plug in the center. The most likely diagnosis is: (check one) A. Bowen's disease B. Basal cell carcinoma C. Keratoacanthoma D. Kaposi's sarcoma E. Seborrheic keratosis

Keratoacanthoma is a relatively common lesion in the elderly, but is difficult to distinguish from squamous cell carcinoma. However, it is easily distinguished from Bowen's disease, basal cell carcinoma, Kaposi's sarcoma, and seborrheic keratosis. Most keratoacanthomas undergo a benign self-healing course but may leave a large, unsightly scar. Treatment is almost always preferred, both for cosmetic reasons and to prevent the rare case of malignant transformation. Proper treatment for a lesion with this appearance is excisional biopsy in order to distinguish between keratoacanthoma and squamous cell carcinoma. Ref: Habif TP: Clinical Dermatology: A Color Guide to Diagnosis and Therapy, ed 5. Mosby Elsevier, 2010, pp 790-791.

A 3-year-old male presents with a 3-day history of fever and refusal to eat. Today his parents noted some sores just inside his lips. No one else in the family is ill, and he has no significant past medical history. He is up-to-date on his immunizations and has no known allergies. On examination, positive findings include a temperature of 38.9°C (102.0°F) rectally, irritability, and ulcers on the oral buccal mucosa, soft palate, tongue, and lips. He also has cervical lymphadenopathy. The remainder of the physical examination is normal. The child is alert and has no skin lesions or meningeal signs. Which one of the following would be the most appropriate treatment? (check one) A. Ceftriaxone (Rocephin) intramuscularly B. Nystatin oral suspension C. Amoxicillin suspension D. Acyclovir (Zovirax) suspension E. Methotrexate (Trexall)

The history and physical findings in this patient are consistent with gingivostomatitis due to a primary or initial infection with herpes simplex virus type 1 (HSV-1). There are no additional findings to suggest other diagnoses such as aphthous ulcers, Behçet's syndrome, or herpangina (coxsackievirus). After a primary HSV-1 infection with oral involvement, the virus invades the neurons and replicates in the trigeminal sensory ganglion, leading to recurrent herpes labialis and erythema multiforme, among other things. Although some clinicians might choose to use oral anesthetics for symptomatic care, it is not a specific therapy. Antibiotics are not useful for the treatment of herpetic gingivostomatitis and could confuse the clinical picture should this child develop erythema multiforme, which occurs with HSV-1 infections. An orally applied corticosteroid is not specific treatment, but some might try it for symptomatic relief. An immunosuppressant is sometimes used for the treatment of Behçet's syndrome, but this patient's findings are not consistent with that diagnosis. Therefore, the only specific treatment listed is acyclovir suspension, which has been shown to lead to earlier resolution of fever, oral lesions, and difficulties with eating and drinking. It also reduces viral shedding from 5 days to 1 day (SOR B). Ref: Usatine RP, Tinitigan R: Nongenital herpes simplex virus. Am Fam Physician 2010;82(9):1075-1082.

An asymptomatic 3-year-old male presents for a routine check-up. On examination you notice a systolic heart murmur. It is heard best in the lower precordium and has a low, short tone similar to a plucked string or kazoo. It does not radiate to the axillae or the back and seems to decrease with inspiration. The remainder of the examination is normal. Which one of the following is the most likely diagnosis? (check one) A. Eisenmenger's syndrome B. Mitral stenosis C. Peripheral pulmonic stenosis D. Still's murmur E. Venous hum

There are several benign murmurs of childhood that have no association with physiologic or anatomic abnormalities. Of these, Still's murmur best fits the murmur described. The cause of Still's murmur is unknown, but it may be due to vibrations in the chordae tendinae, semilunar valves, or ventricular wall. A venous hum consists of a continuous low-pitched murmur caused by the collapse of the jugular veins and their subsequent fluttering, and it worsens with inspiration or diastole. The murmur of physiologic peripheral pulmonic stenosis (PPPS) is caused by physiologic changes in the newborns pulmonary vessels. PPPS is a systolic murmur heard loudest in the axillae bilaterally that usually disappears by 9 months of age. Mitral stenosis causes a diastolic murmur, and Eisenmenger's syndrome involves multiple abnormalities of the heart that cause significant signs and symptoms, including shortness of breath, cyanosis, and organomegaly, which should become apparent from a routine history and examination. Ref: Rudolph CD, Rudolph AM (eds): Rudolph's Pediatrics, ed 21. McGraw-Hill, 2003, pp 1755-1757.

A 3-day-old female developed a rash 1 day ago that has continued to progress and spread. The infant was born at term after an uncomplicated pregnancy and delivery to a healthy mother following excellent prenatal care. The infant was discharged 2 days ago in good health. She does not appear to be irritable or in distress, and she is afebrile and feeding well. On examination, abnormal findings are confined to the skin, including her face, trunk, and proximal extremities, which have macules, papules, and pustules that are all 2-3 mm in diameter. Her palms and soles are spared. A stain of a pustular smear shows numerous eosinophils. Which one of the following is the most likely diagnosis? (check one) A. Staphylococcal pyoderma B. Herpes simplex C. Acne neonatorum D. Erythema toxicum neonatorum E. Rocky Mountain spotted fever

This infant has a typical presentation of erythema toxicum neonatorum. Staphylococcal pyoderma is vesicular and the stain of the vesicle content shows polymorphonuclear leukocytes and clusters of gram-positive bacteria. Because the mother is healthy and the infant shows no evidence of being otherwise ill, systemic infections such as herpes are unlikely. Acne neonatorum consists of closed comedones on the forehead, nose, and cheeks. Rocky Mountain spotted fever is a tickborne disease that does not need to be considered in a child who is not at risk. Ref: O'Connor NR, McLaughlin MR, Ham P: Newborn skin: Part I. Common rashes. Am Fam Physician 2008;77(1):49-52.


Kaugnay na mga set ng pag-aaral

CITI IRB (Conflicts of Interest & Basics of Info Security), Internet-Based research SBE CITI, Citi Questions

View Set

chapter 25 (hard Rock - led zeppelin)

View Set

Disaster Planning Adaptive Quizzing

View Set

Quiz I Review Firewalls and Networking

View Set

Sociology final terms and definitions

View Set

Organizational Behavior Final Exam

View Set

NU472 Week 6 HESI Case Study Evolve Elsevier: Intimate Partner Violence and PTSD - 29 Questions

View Set